Cardinal and Ordinal Numbers
Cardinal and Ordinal Numbers
MONOGRAFIE MATEMATYCZNE
TOM 34
WACLAW SIERPINSKI
WARSZAWA 1958
PANSTWOWE WYDAWNICTWO NAUKOWE
Pe
CARDINAL AND ORDINAL NUMBERS
POUL
6S) Ri Aa A RecA
ag tee Nie eee
MONOGRAFIE MATEMATYCZNE
KOMITET REDAKCYJNY
TOM 34
Pi AUN'S.T)-W-.0,W.EO W Y D°AIWN
IGG DW O72 NGA.
Gl ko Oawee
WACLAW SIERPINSKI
WARSZAWA 1958
PRINTED IN POLAND
https://archive.org/details/cardinalordinalnO00Ounse
FOREWORD
This book is the product of long years of work. In the year 1909
appeared a lithographic edition of my lectures on the Theory of Sets
delivered at the University of Lw6w, and in 1912 the first edition of my
»An Outline of the Set Theory“ was published in Warsaw. The second
edition of that book appeared in 1923 and the third (divided into two
parts, having VITI+ 237 and VII+4 232 pages respectively) in 1928.
I lectured on the Theory of Sets repeatedly at the University of Warsaw
in the years 1920-1939. This work, however, does not comprise the whole
of the Theory of Sets or even the General Theory of Sets, but only deals
with certain important branches of it, namely the cardinal and the ordinal
numbers. I finished the manuscript of this book in Polish at the end
-of 1952. The process of translating it into English and the preparations
for printing occupied the next few years. During that period only very
slight changes and additions could be made. That is why it is only occasio-
nally that I was able to introduce results that were published later than
1952; in particular, I was unable to make use of those which are contained
in the work of Heinz Bachmann, Transfinite Zahlen, 1955.
W. Sierpinski
Pe
Pte. cr rT ar
f My ; Ake sage pene
3haa 4Deeks ie) Loa
Sy Peal
haan Sots eee
Os
¢ ' -
od.
7? fr, ere
tet nh”
CHAPTER I
1. Sets. From given objects we can form sets. For example we can
form, sets of letters {a, b,c¢,}, fe; 4,7, 9,h}, (0, f}-
From natural numbers (7. e. positive integers) we can form, for
example, the set of all natural numbers smaller than 10, the set of all
odd numbers, the set of all natural numbers between 100 and 1000, the
set of all squares, ete.
Here are some other examples of sets: the set of all the words used
in this book, the set of all rational numbers, the set of all points on a
straight line, the set of all spheres in the space.
and to denote that the object p is not an element of the set A we write
pea.
Thus, for instance, if A = {a, b, c}, then we have acA, DeA, ceA,
but d¢A, if d is none of the objects a, b,c.
4, Set consisting of one element. According to our notation {a} de-
notes the set consisting of a single element, namely the object a. Indeed,
it could be maintained that no set exists where there is only one element,
6 I. Sets and elementary set operations
{a} € {a},
ae {a},
these formulae prove that {a}! Aa. Thus, the set containing only the single
element a must be distinguished from that element itself.
Therefore, if A,¢A,, then A,~A,. More generally, let us assume
that if, for a natural n >1,A,¢«A,¢ A,...€A,, then the objects A,,
A,,..., A, are all distinct from one another (Zermelo [5], p. 31).
{a,b,c}= {e,
b, a} = {a,
¢, b} = {a, b,c, a}.
> & 3} for k= 2,3,...,n. Thus we have a, = {a}, d= (a, {a,3} a fa, {ay},
{a {a,}}} ete.
2. Prove that if a, b, c and d are any objects (different from one another or not),
then in order to have
Proof. It follows from (7.1) that {c} « {{a}, {a, b3\; therefore {c} = {a} or {e}=
= {a, b}. The latter equality can hold only when a = b, because if a 4 b, the set {a, b}
would have two different elements, while the set {c} has only one element. But if a= b,
we have {a, b} = {a, a} = {a}. Thus in any case we have the equality {c} = {a}, which
gives c= a.
If a ¥ 6, formula (7.1) gives {a, b} = {e, d} (since fa, bse {{c}, {e, d3\), the equality
{a, b} = {c} being impossible in view of a 4 b. Since, as we have already proved, a = ¢,
we have {a, b} = {a, d}, whence, in view of a 4 b, we find that b= d.
If a=b, formula (7.1) gives {{a3} = {{c}, {e, d}\, whence {c, d} = {a}, therefore
c=d=—a=b, whence again b = d.
Thus in any case formula (7.1) implies equality (7.2). Conversely, it is obvious
that equality (7.2) imples formula (7.1). The equivalence of formulae (7.1) and (7.2)
has therefore been proved.
1) Such sets play a part in certain reasonings; see e.g. Neumann [1].
8 I. Sets and elementary set operations
EXERCISES. 1. Prove that the equality A = B holds for the sets A and B if
and only if the two formulae Ac B and Bc A hold.
2. Prove that the formula p «A holds for an object p anda set A if and only
if {p}.c A.
3. Prove that for the sets 4, B and OC the formula 4c BC Oc A is equivalent to
the formula A = B=0O.
4, Let » denote a natural number >1. Prove that for the sets A,, A.,..., A, the
formula A,c A,c...c A, cA, is equivalent to the formula A, = A,=...=
5. Find all the subsets of the set {a,b} containing two different elements.
Answer: 0, {a}, {b}, {a, b} (since the empty set is a subset of every set).
6. Find all the subsets of the set {a,b,c} containing three different elements.
Answer. 0, {a}, {b}, {ce}, {a, b}, {a, C}, {b, Ch, ta, b, Ch.
§ 8. Subset of a set i)
7. Let k and n be two natural numbers, k < », How many different subsets of k
‘elements does a set of n elements contain?
Answer.
TON Mask n!}
k} kY(n—k)!’
since to find them it is sufficient to take into consideration all combinations of 7 ele-
ments of the set, taken k at a time.
8. 'dow many different subsets does a set of n elements contain (n being a natural
number)?
Answer. 2” (we include here the empty subset, of course).
9. Let Z(a), for real « > 0, denote the set of all natural numbers n = 2”?~'(2q—1),
where p and q are natural numbers such that p < qx. Prove that for 0 <a < y the
set Z(x) is always a proper subset of the set Z (y).
Proof. Let ~ and y be two real numbers >0, such that w < y. If ne Z(a), then
n = 2”-*(2q—1), where p and q are natural numbers such that p < ga and, since x < y,
we have p < qy, whence it follows that n « Z(y). Each element of the set 7 (x) belongs
therefore to the set Z(y), whence Z(x)c Z(y). Since 0 << a4 < y, there exists, as we
know, a rational positive number w such that « < w < y. The rational positive num-
ber w can be represented in the form w = r/s, where r and s are natural numbers. Thus
we shall have « < 1r/s < y, therefore r > sx and r < sy. Let n = 2"—*(2s—1). Since
y < sy, we shall have ne Z(y).
Suppose that » « Z(x). According to the definition of the set Z(x) we should then
have n = 2”~*(2q—1), where » and q are natural numbers such that p < qa. There-
fore we should have Rog i} = ea es 1), where p, q, 7 and s are natural numbers,
whence, as we know, it follows that p =r and q = s. This, however, is contrary to the
inequalities 7 > sw and p < qx. Thus n € Z(x), and since n « Z(y), we have Z(x) 4 Z(y).
Thus the set Z(x) is a proper subset of the set Z(y), q. e. d.
10. Give an example of a non-empty set whose all elements are subsets of that set.
Answer. The set Z = {A}, where dA denotes the empty set. Similarly, the set
9h |A, {A}} and the sets
The sets of the family F are called cept nen: of the sum SN. If @ is
a family of families of sets and
Yi MF,
FeD
10 I. Sets and elementary set operations
Se oe
Aev FeD A€F
i CaO aYe,
yeas
XEU(A)
or in the form
ASX.
XCA
Every set A is also the sum of all its one-element subsets (but gene-
rally it is not the sum of all its elements); this can be written in the form
A= >, {p}
peA
From the definition of the sum of sets and the relation C it follows
immediately that every swm of sets contains each of its components.
If n is a natural number and family F' consists of n sets A,, As, ..., An;
then the sum of all sets of the family F is denoted by A,+4,.+...+A,.
For every set A we have, as can easily be verified, the formulae
A+0=0+A=A and A+A=A. It follows from the last of these
formulae that in writing down a sum of sets there is no need to repeat
identical components.
It is also easy to see that if A and B are sets such that BCA, then
A+B= A. We can express this by saying that any component of a sum
of sets absorbs any other of its components contained in it. Conversely,
if d+B= A, then BCA.
It follows from the definition of the sum of sets that for any sets A
and B we have A+B= B+ A, and for any sets 4, B and CO we have
(A+B)+C=A+(B+0C).
The sum of sets, therefore, depends neither on the order of the
components nor on the way of joining them.
10. Difference of sets. The set of all those elements of the set A
which do not belong to the set B, is denoted by A—B and called the
difference of the sets A and B. ;
‘As can easily be seen, 4—B= 0 if and only if we have ACB,
and A—B= B—A if, and only if, A = B.
§ 10. Difference of sets Tt
Not all the properties of the algebraic sum of numbers are valid for
ets. H. g. if the sets A and C have an element in common, then
A+(B—C€)#(A+B)—C,
and if C—A #0, then
(A—B)+CA4A—(B—C).
(A—B)—O = (A—O)—(B_-0Q),
(BOB AeA] OG(A=B)-3(B <0)
11. Product of sets. If / is a given family of sets, then by the product
of all the sets of the family Ff we understand the set P containing all
those and only those elements which are contained in each set of the
family F. We then write
p= [[a.
A€F
y= »'P,
he,
[l4=[][T-4,
AEev FeD A€F
the product of sets it follows that for every two sets A and B we have
the formula 4B — BA, and for every three sets A, B and C we have the
formula (AB)C = A(BC).
It can easily be seen that if # is an arbitrary family of sets and B
an arbitrary set, then
BY A=) BA.
AeF A€F
pep a,
A€F
of the set of all integers (or, more generally, of all complex numbers)
into disjoint components, the two numbers a and 6 being regarded as
belonging to the same component if and only if there exists an integer
k such that a—b = km.
EXERCISES. 1. Prove that for any two sets A and B we have the following
decomposition of the set A into two disjoint components (which can be empty sets):
A =(A—B)+AB.
2. Prove that for any sets A, B,O we have the following decomposition of their
sum into four disjoint components:
and give an example of sets A, B, C, D for which the components of the right-hand
side are not disjoint.
Proof. The right-hand side of our formula is, of course, contained in the left-hand
side. In order to prove the formula it suffices to show that its left-hand side is contained
in the right-hand side. Therefore let p<« 4+B+O+D. Thus the element p belongs
to at least one of the sets A, B, VU, D. Obviously, if we perform a cyclic permutation
of the letters A, B, CO, D, our formula remains unchanged; therefore, we can suppose,
without loss of generality, that p « A. If we had p « ABCD, p would belong to the last
component of the right-hand side of our formula. Suppose further that p ¢ ABOD.
The element p, therefore, does not belong to at least one of the sets B, U and D (since
it belongs to A). If the first of the sets B, OC, D to which p does not belong is B, then
p¢«A—B; if Cis the set in question, then p< B—(; finally, if Dis that set, then peC0—D.
Thus, in any case, p belongs to at least one of the five components of the right- |
hand side, therefore p is an element of the right-hand side of our formula, q. e. d.
An example of the sets A, B, 0, D for which the components of the right-hand
side of our formula are not disjoint -is given by the sets A = 0 = {p}, B= D= 0. The
element p belongs here to the first and to the third component of the right-hand
side of our formula.
4. Prove that for any sets A and B we have A= A(A+B).
5. Prove that if A, B, O, D are sets such that 4+B=0O+D, then there exist
sets P,Q,
&, S such that A= P+Q, B=R+8, C=P+IR, D=Q+8.
Proof. As can easily be verified, we can assume that P = AO, Q = AD, R= BO,
S = BD, since we have, for instance, P+Q = A(O0+D) = A(A+B)=A.
6. Prove that if A, B, 0, D are sets such that A+ B= C+D and AB=CD=0O,
then there exists only one system of sets P, Y, Rk, S such that A =9P+Q, B= R+S8,
C=P+k, D=@Q-+S8, and moreover, we must have PY = RS = 0.
Proof. Let A, B,C, D be sets such that A+B=CO+D and AB=OD=0.
According to exercise 5 there. exist sets P,Q, R and S such that 4 = P+Q, B= R+8,
C= P+kh, D=Q+8. Now let P,Q, Rk, S be any sets satisfying these four equalities.
Hence we have Yc D, kc OC, therefore QR c CD = 0 and thus QR = 0, whence ACO =
= (P+Q)(P+8) = P+P(Q+8)+Qk =P. Therefore AO =P. Similarly, we shall
find AD=Q, BC = Kk, BD=S8. Thus the sets P, Q, RB, S are determined by the sets
§$ 12. Disjoint sums 15
These are called De Morgan’s formulae for sets. The first of them
states that the complement of a product of sets is the sum of their comple-
ments, the second — that the complement of a sum of sets is the product
of their complements. x
We shall prove the first of these formulae. Let us suppose that
peO]] A. Therefore p ¢ ITA, and there exists a set A, of the family F
A€F €
such that p ¢ A,, whence, in view of the fact that the sets of the family 7
are subsets of the set W, it follows that pe W—A,=(CA,, and,
since A,«F, therefore p« >) CA. Thus we have
A€F
CA Mee iene:
A€F AeF
On the other hand, let us suppose that p « » OA. Thus there exists
A€F
a set A,¢«F such that peCA, therefore p¢A, and naturally p¢// A,
A€F
C[[a=) c4,
A€F AeF
qié5 de
16 I. Sets and elementary set operations
EXERCISE. Prove that if the sets A and B are subsets of the set W, with respect
to which the complements are taken, then
and, since ab, a b) ~(b, a), for in case of (a, b) = (b, a) we should
have {{a}, {a, b}}= {{b}, (0, a}}, whence, as we have proved in section 7
(exercise 2), it would follow that a = b, contrary to our assumption that
Geer.
Thus, unlike the set {a,b}, the arrangement (a,b) depends on the
order in which the elements a and b occur in it. We call it an ordered pair.
In the ordered pair (a, b), a is called the first, b — the second element of
the pair.
By means of ordered pais we can define ordered triplets as the
arrangements ((a, Ay), Ms) , ordered quadruplets as the arrangements
(a1, Ml), M3), 44) and, generally, ordered sequences of n terms, n being an
arbitrary natural number.
If the set f(a) contains only one element dD, 7. e. if f(a) = {b}, then
we write b = y(a). In this way with each element a of the set A is as-
sociated an element g(a) of the set B. However, the formulae a,« A,
a,€ A, a, ~ 4, do not necessarily imply the formula y(a,) + y(a,). If to
each two different elements a, and a, of the set A always correspond two
different elements y(a,) and y(a,), then the function » is called invertible
in the set A.
We might also consider functions f(a) for which the values of variable «
do not form a set, e. g. functions whose variable can be any object. (As we
know, the concept of the set of all objects leads to an antinomy (Sier-
pinski [58], p. 50 and 113). For instance, f(7) and g(a#) will be such func-
tions if f(#) = w for every object «, while g(x) = 2? if 2 is a real number,
and g(a) = 0 if the object x is not a real number. U(X) will also be such
a function, where if X is a set, U(X) is the set of all subsets of the set X,
and if X is an object, not a set, U(X) is the empty set.
5. Let A denote a given set, 7’ — the set of all subsets of the set A, P — the set
of all functions f(«) defined for 7 « A and assuming only the values 0 and 1. We can
establish a (1-1) correspondence between the elements of the sets 7 and P, relating
to each element X of the set 7, 7. e. to each subset X of the set A, a function f(x) de-
fined for «<A by the conditions f(z)=1 for we X and f(x) = 0 for we A—X. This
function is the so-called characteristic function of the subset X of the set A. If we de-
note it by fy, then a (1-1) correspondence between the elements of the sets 7 and P
can be established by the formula y(X) = fy for X«T (i. e. for Xc A). Clearly, the
functions g71, inverse to the function @ will be defined for the elements f of the set P
by the following condition: g 4(f) is the set of all elements w of the set A for which
Gp) ake
If we had 6 «(D—O), then b€ 0, thus b& O—D and we should have (a, b)«
« Ox (D—C), whence « « C. Therefore we should have w« 0 for w« A, and thus AcC.
But, in view of be D—O and b« B—A it could be proved in exactly the same way
that Cc A. Thus we should have A = OC, which is impossible. Therefore b € D—C and
thus 6 « C—D since if a « A, then the element (x, b) belongs to the left-hand side of
formula (19.1) and thus to the right-hand side of this formula. But then, just as before,
we come to the formula A = D, which is impossible. Thus the assumption that B—A + 0
leads to a contradiction. Therefore B—A = 0. Similarly we prove that D—C= 0.
Therefore we have A—B +4 0 and C-D + 0. Thus formula (19.1) gives Bx (A—B)=
= Dx(C—D), and since the sets B, D, A—B, O—D are non-empty, we must have
B=D and A—B = C-—D, and, since Bc A-and DcC, we have A = B+ (A—B)=
= D+(C—D) = OC, whence A = O, which is impossible. Therefore formula (19.1) does
not hold, q.e.d.
1) The subject of this chapter belongs to the so-called Algebra of Sets. The reader
who wishes to study the subject at a greater length is referred to Sierpinski [58], espe-
cially Chapters II and III. The reader will find there a large number of exercises.
CHAPTER II
EQUIVALENT SETS
This follows from the fact that, as we have proved in I.16, if the
function gy establishes a (1-1) correspondence between the elements of
the sets A and B, then its inverse function g establishes a (1-1)
correspondence between the elements of the sets B and A.
Ill. Transitivity. If A, B, C are sets such that A~B and B~C,
then AW~C.
If the function gy, establishes a (1-1) correspondence between the
elements of the sets A and B, and the function g, establishes a (1-1)
correspondence between the elements of the sets B and O, then, obviously,
the function ¢, defined by the condition p(x) = ¢,(y,(x)) establishes a (1-1)
correspondence between the elements of the sets A and C. The function
p (a) — gt (v2 (2)) for we C will be inverse to the function 9.
s= YAS,
é€Z
and similarly
APB(eys
&€Z
Then Sei ~ T.
Proof. Let p denote any element of the set S. Since S is a disjoint
sum of the sets A(&é), where « Z, there exists for the element p one,
and only one, element & of the set Z, such that p« A(é,).
Let (p)=¢2,(p). We have of course ¢;,(p)<« B(é,), and since
B(é) C 1, O(p) = vz,(p) « T. In the set S we have thus defined the func-
tion 3(p), whose values are elements of the set 7. The function @ is in-
vertible in the set S. Indeed, if pe S, ge S and pq, then, in case of
& = &,, we have z,(p) A @z,(¢), since the function g;, is invertible in
the set A(é,), and, in view of (p) = 9,(p) and 9(¢)= ¢z,(¢) ~ %,(P),
we have 0(p) # #(q); and in case of & ~ &, we have 0(p) = ¢z,(p) « B(&),
3(q) = ¢2,(q) ¢ B(Eq), and since, in view of & ~ &,, we have B(é,) B(é,) = 0,
therefore we must have #(p) 4 0(q). Finally, suppose that te 7. Thus
there exists an element of the set Z such that t« B(&,). Since the fune-
tion gq, establishes a (1-1) correspondence between the elements of the
sets A(é,) and B(é,), we have u = gz, (t) « A(&,)C S, and it follows from |
the definition of the function 0 that d(w)=t. Thus for such element t
of the set 7 there exists an element uw of the set S such that #(u) = t.
From the already proved properties of the function @ it follows that
this function establishes a (1-1) correspondence between the elements
of the sets S and 7. Thus we have proved Theorem 2.
It is easy to see how to weaken the assumptions of Theorem 2 in
order to get the statement S~T instead of the statement S ef~T.
The following theorems can easily be proved.
THEOREM 3. If A, B, A,, B, are seis such that A ef~A, and Bef~B,,
then A xBef~A, xB,.
THEOREM 4. If A,B, A,, B, are sets such that A~A, and B~B,,
then A xXB~A, xB,.
EXERCISES. Prove that if A -B~B—A, then A~B, but not necessarily vice versa.
Proot. We have, as we know, the decompositions 4 = (A—B)+AB and B=
= (B—A)+AB, and the equalities (A—B)-AB=0 and (B—A)-AB=0. According
to Theorem 1 (in which A and B should be replaced by A—B and AB respectively,
and A, and B, by B—A and AB respectively), and in view of 4—B~B—A and AB~AB,
we obtain (A—B)+AB~(B—A)+AB, and therefore A~B, q. e. d.
28 Il. Equivalent sets
On the other hand, let A denote the set of all integers, and B — the set of all
natural numbers. As already proved, A4~ B, but the relation A—B~B—A does not
hold since the set B—A is empty while the set 4 —B is not empty (having as elements 0
and all negative integers).
2. Prove that if d~A, and B~B,, then AB~ A,B, does not necessarily follow.
Proof. Let 4 = B= A, = {1}, B, = {2}. Then we have A~A,, B~B,, AB= {1},
A,B, =0, whence it follows that AB~A,B, does not hold.
THEOREM 5. If A, B, A,, B,, are sets such that A ef~A, and B ef~B,,
then A® ef~Aj?. :
Proof. It follows from the assumption of Aef~A, and Bef~B,
that we are able to establish a (1-1) correspondence gy between the ele-
ments of the sets A and A, and a (1-1) correspondence wy between the
elements of the sets B and Be
Let f « A’. According to the definition of the set A® (1.18) a function
f(wv) is defined for we B and such that f(a) « A for we B. To the function f
let us relate the function g = 0(f) defined for xe B in the following way:
§(e)= elt (y-())) for we B. Obviously, we have ge Af? and f(y) =
= 9 (9 (p( y))) for y e B, whence it follows that the function # establishes
a (1-1) correspondence between the elements of the sets A® and Ape
Thus we have A’ ef~A?, q.e. d.
Sunilarly we prove
THEOREM 6. If A,B, A,,B, are sets such that A~A, and B~B,,
then A?~A??,
Next let us prove
THEOREM 7. If A, B, C are sets such that BC = 0, then A8+©
ef ~A® Xx AS.
Proof. Let fe A?+¢. Therefore a function f(x) is defined for
ce B+C and such that f(x)« A for re B+C. By g=g(f) denote the
function g(x) defined for ce B by the equality g(#)=f(#), and by
h=~yp(f) denote the function h(w#) defined for «eC by the equality
h(x) = f(z) Obviously, we shall have g« A?, he A°, therefore
(g,h)« A®?x AC. Thus to each element f of the set A2+° corresponds an
element #(f) = (p(f), v(f)) of the set A?x A‘. We shall show that the
function ? establishes a (1-1) correspondence between the elements of
the sets A?+° and A?x AS. Indeed, let f « A?+°,f,<¢ A?*°,fAf,. Thus
there exists an element x7, « B+C (at least one) such that f(a) < f(a).
Since 7,« B+C, we have #,<B or w «CC. If a, ¢ B, then for g= g(f),
gi = plfi) we shall have g(%) = f(#o), 91(@o) = fi(@o); therefore g, 4 g and
thus, setting h = y(f), hi = y(f,), we shall have (g, h) 4 (g,, h,), therefore
O(f) AO(f,). Thus the function # is invertible in the set A®+C.
It remains to show that for each element 7 of the set A? AC there
exists an element f of the set A+© such that r= #(f).
§ 5. Various theorems on the equivalence of sets 29
Therefore, let r<¢ A? x AC. Thus r = (g, h) where g« A?, he AT. Now
let « denote an element of the set B+C. With regard to BC = 0 one,
and only one, of the two formulae #w¢« B and we C holds. If veB, let
f(x) = g(a); if we C, let f(v~) = h(a). The function f(x) will thus be defined
for «« B+C, while obviously f(v) « A. Thus we shall have f « A?+°. It
follows at once from the definition of the functions » and y that
g=¢(f), h=vlf),
therefore r= (g,h) = B(f), q. e. d.
It is easy to see that an immediate corollary of Theorem 7 is
THEOREM 7a. If A, B, C are sets such that BO = 0, then A®+*°C~A®X AS.
THEOREM 8. For any sets A, B,C we have (Ax B)° ef ~ACX BS.
Proof. Let fe(AxB)°. Thus f is a function f(x), defined for we C,
whose values are elements of the set A xB. For each x « C we thus have
f(x) = (u,v) where we A and ve B. Since the elements uw and v depend
on #, we can write u=g(x),v=h(z), while of course ge A°, he B°,
and thus (g,h)« ACx BS. Thus to each element / of the set (A x B)¢
have related an element (g, h) = #(f) of the set Acx BC.
Now we shall show that the function &? establishes a (1-1) corres-
pondence between the elements of the sets (Ax B)° and ACx BS.
Let fe(AxB)°,f,¢(AxB)°,f~Af,. Thus there exists an element
#,« C_such that 7(2%) © J,(@)- ihe OGY aA, 0, 8 (Fe (Gartlig) LO ere iC.
we shall then have f(x) = (g(x), h(a yey (onlay, h,(a)), therefore
(g (0); h (ao) i (91(%o) 5 hy(&»)) and thus either
g(#9) 7 g1(o) or h(%) A hy(Ho),
or both. If g(@,)#9,(v) then g+~9,, and if h(a.) ~ h(a), then h ~ hy.
Thus in any case (g, h)~(g,, h,), therefore #(f) 4 O(f,). Thus the fune-
tion # is invertible in the set (Ax B)¢.
It remains to show that for each element r of the set A° x BC there
exists at least one element f of the set (A xB)° such that r= #(f). Let
re A°x BS, and so r= (g,h) where ge A©, he BC. Thus g is a function
g(x) defined for x « O such that g(#) « A for we C, and h is a function h(x)
defined for «« C and such that h(w7)« B for we C. Let f(x) = (g(x), h(a))
for «eC; we shall have f(x)eAXxB for we C, therefore f<«(AxB)¢,
while +r = (g, h) = &(f) follows from the definition of the function #.
Thus we have proved Theorem 8.
It is easy to see that an immediate corollary of Theorem 8 is
THEOREM 8a. For any sets A, B, C we have (AX B)°~ACX
BS,
THEOREM 9. For any sets A, B, C we have (A®)° ef ~ A®XS,
Proof. Let f«(A)°. Thus j is a function f(x), defined for we C,
such that f(x) « A¥, v. e. f(x) is a function g, defined for y « B, such that
gxly) « A. ad let ge BXC; therefore = (y;#), where ye B, we C.
30 II. Equivalent sets
Let h(z) = gx(y): we shall have h(z)<« A for ze BXC, therefore h « A?*¢.
Thus to each element f of the set (A?)° we have related an element h=
= b(f) of the set A?*°. We shall show that the function ? establishes
a (1-1) correspondence between the elements of sets (A?)° and A®*C¢.
Let f « (A), f’ « (A%)°,f #f’. Thus there exists an element 2) « C such
that f(a) A f(x). But f(a) is a function g, where g,« A and, similarly,
f'(w) is a function gi, where g; « A. Therefore, since f(x.) 4 f(x), we have
Jxo F Jxo, and there exists an element y, of the set B, such that g..(Yo)~
F JxiYo). Let 2 = (Yo; Bo), h= AS), ho —O(f'). From the detinition of.
the function % it follows that h(Z) = Gx (Yo) and h'(2o) = Jxo(Yo), therefore:
h(Z) A h'(29), which proves that h ~ h’. Thus the function @ is invertible:
in the set (A)°. It remains to show that for each element A of the set.
A%xC there exists at least one element f of the set (A”)° such that the equal-
ity h=9#(f) holds.
Let h« A®*¢; thus fh is a function h(z), defined for z« BxC, such
that h(z)« A for ze BXC. For we C, let us denote by f(x) a function
gx(y) of the variable y, defined for y « B by the formula g,(y) = h(z) where
z= (y,«x). We shall have f(x) « A® for we C, therefore f « (A?)°, and it
follows at once from the definition of the function # that 0(f) = h.
Thus we have proved Theorem 9.
It is easy to see that an immediate corollary of Theorem 9 is
THEOREM 9a. For any sets A, B, C we have (A®)°~A®*XC.
As an example of a theorem which is of the same kind as Theorems 7,
8 and 9 but is difficult to prove we give without proof the following
theorem:
If A, B, C, D are sets such that AB= CD=0, Aei ~B, Cef ~D
and A+Bef ~C+D, then A ef ~C.
Proof. See Sierpinski [8], p. 1-6.
If the element y, exists and y,¢ B, then it follows from the defini-
tion of the function g that there exists one and only one element « of
the set A such that y(x#)= y,. We shall denote this element by 7,. If,
however, y, ¢ B,, then we shall say that the element x, does not exist.
Generally, let us suppose that for a given natural number k there
exists an element 7,. If x, « A,, then by yxz41 we shall denote that single
element of the set B for which w(y,z41) = %, and if #,¢ A, we shall
say that the element y;,; does not exist. Similarly, if there exists an ele-
ment y, and y,« B,, then by a, we shall denote that single element of
the set A for which y(a,) = y;,, and if y,¢ B,, we shall say that the ele-
ment 2, does not exist.
Clearly, for each element x, of the set A one and only one of the
following three cases occurs:
1° For every natural number & there exist elements a, and y;;
2° For a certain natural number k there exist elements y,, 7%, y2,
Lay «+5 Ye-15 He-1, but the element y, does not exist;
3° For a certain natural number k there exist elements ¥,, 2, Y2,
May +009 Ye-1) Uk-1y Yk, Dut the element x, does not exist.
If the first or the second case occurs, we shall take 3(x) = p(X),
and if the third case occurs, we take #(a) = y, 7. e. B(a%>) = yp(Xp).
Thus we have defined the function 0(#) defined for we A. We shall
show that it establishes a (1-1) correspondence between the elements
of the sets A and B.
Let us suppose that for the elements x, and ao of the set A we have
P(X) = B(x). Case 1° or case 2° cannot occur for a, and case 3° cannot
occur for 2%, since we should then have #(a) = (4%), B(%o).= po),
whence (2) = y~(#), Which would give a = p (yp(xo) and therefore
y(%) = ¥, and x2 = a. Still it is clearly always the same of the three
cases 1°, 2°, 3° that arises both for x and #,. Similarly we prove that
case 1° or case 2° cannot occur for x), and case 3° cannot occur for «7.
Thus we have either 3(xo) = 9(%), (x5) = (2), 4. & (2) = v(x),
which, in view of the function being invertible in the set A, gives % = a,
or else 3(%) = pao), O(x,) = p'(a,), 4. & pHa) = p(x), which gives
Ly = x. Thus we have proved that the function # is invertible in the set A.
It follows at once from the definition of the function # that its values
are the elements of the set B.
Now let y denote any element of the set B. Let 2) = p(Yo); # will
be an element of the set A, and, according to our notation, y, will be the
element y, corresponding to a, 1. €& Yo = %-
If there exists no element x, corresponding to x, then case 3° occurs,
and we have #(a%)) = y,. From y,= y, we know that for y, there exists
an element 2 = %, of the set A such that (x) = yp.
32 II. Equivalent sets
Suppose now that for x, the element «, exists. If for x, and thus
also for 2,, case 1° or case 2° occurs, then we have #(x%,) = 9(%,). But,
according to the definition of the element 7,, we have (#,) = y,. Thus
O(xv,) = y,, and, since y, = yy, we conclude that there exists an element
a” = #, of the set A such that 3(x) = yy. Finally, if for a case 3° occurs,
then we have #(x%)=y,, and, since y,= Y), we conclude that there
exists an element © =v, of the set A, such that #(7) = yy.
Thus we have proved that for each element y, of the set B there
exists an element x of the set A such that #(x) = y,. Since, as we have
proved above, the function # defined in the set A is invertible in this
set and its values are the elements of the set B, # establishes a (1-1) cor-
respondence between the elements of the sets A and B, q.e. d.
Thus Theorem 1 has been proved.
Now if we replace, in Theorem 1, the formulae Aef~B, and
Bef ~A, by the formulae A~B, and B~A, and in the first sentence of
the proof of that theorem the words “we are able to define the func-
tion gm and the function »“ by the words “there exists a function m and
a function y“, then, as can easily be seen, we should obtain the proof of
THEOREM 2 (of Cantor-Bernstein). If A, B, A,, B, are sets such that
A,C A, B,C B, A~B,, B~A,, then A~B.
The proof of the Cantor-Bernstein theorem given above is based on
an idea of J. Konig. Two more proofs of this theorem will be found in
my book (Sierpiiski [58], p. 145-148).
The theorem of Cantor and Bernstein can also be formulated in the
following way:
Two sets of which each is equivalent to a subset of the other are equi-
valent.
Now let us apply Theorem 1 to the case of B= A. Replacing B by C,
and taking into account the symmetry and transitivity of the relation
ef ~ we obtain
THEOREM 3. If A, B, C are sets such that AD BIC and Aef ~C,
then Aef ~B and Cef ~B.
Similarly, from Theorem 2 we obtain
THEOREM 4. If A, B,C are sets such that AD BOC and A~O,
then A~BW~C.
In other words: if the set A is equivalent to its subset OC, then each set
intermediate between C and A (1. e. containing C and contained in A) is
equivalent both to the set A and to the set C.
EXERCISES. 1. Prove that if X5 Y and Yef ~Y+Z, then X ef ~X-+ Z.
Proof. We have Yc ¥+(Z—-X)c Y+ Z, whence, since Yef ~Y+Z and in
virtue of Theorem 3, we find Y ef~Y+(Z—X).
§ 6. The Cantor-Bernstein theorem 33
for every odd index an index greater by 1, which will give the infinite
sequence:
Az, Ay, Ag, Az, Ag, Ug, +++ Mak, Aok—1y Aek4+2) Aak+1; +
Similarly, for every denumerable set there exist many infinite se-
quences composed of all its elements.
THEOREM 1. Hach subset of an effectively denumerable set is a finite
set or an effectively denumerable set.
Proof. Let A denote an effectively denumerable set. We are able
to arrange all the elements of the set in an infinite sequence
(1.2) Ors es ign s-:
Now let B denote a subset of A which is not a finite set. Let us re-
move from the sequence (1.2) all those terms which are not elements of
the set B (without changing the order of the remaining terms of the se-
quence). In this way what remains of the sequence (1.2) will be a well-
defined infinite sequence, which of course will be composed of all the
elements of the set B. Thus we are able to arrange the elements of the
set B in an infinite sequence, and therefore the set B is effectively denu-
merable, q.e. d.
In particular, every infinite set of natural numbers is effectively denu-
merable (as an infinite subset of the effectively denumerable set of all
natural numbers).
The concept of effective denumerability is understood differently
by E. Borel. In his book (Borel [3]) on p. 229-232, in § 66, entitled Les
ensembles dénombrables et les ensembles effectivement énumérables, he as-
serts that an infinite subset of an effectively denumerable set need not
be an effectively denumerable set, and gives several examples of this,
among them one which is somewhat like the following. Let H denote
the set of all natural numbers which appear infinitely many times as
denominators in the development of the number z into a continued arith-
metical fraction. If the set H is infinite, then, according to Borel, it
is not effectively denumerable (although it is a subset of the effectively
denumerable set of all natural numbers), because we do not know the
smallest number belonging to it. Of course, we are not able to calculate
this number, but we can define it in a unique manner as the smallest
natural number belonging to H. Thus, what Borel calls effective de-
numerability is the caleulability of every term of the sequence’). I have
1) In his note (Borel [2], p. 164) E. Borel calls effectively denumerable every set
given in the form u,, u,,..., where every term wu, is known if its index nm is given and
conversely (cf. also Sierpinski [57], p. 42-43). The question is what should be meant
by a “known term. We understand by it an object which is effectively defined; for
Borel “a known number“ means a calculable number.
3%
36 Il]. Denumerable and non-denumerable sets
} i) x99 4) #4; ay
in which we shall find every rational number and find it only once. This
proves that the set of all rational numbers is effectively denumerable,
Quiesd.
If we wanted to determine the place occupied in our sequence by
a given rational number (e.g. 22/7) it might require long calculations;
this, however, is of no importance, since our aim has been only to establish
a (1-1) correspondence between rational numbers and natural numbers,
and not to apply this correspondence in practice.
§2. Effective denumerability of the set of all rational numbers 34
da
From Theorem 1 and from the proof of III.1, Th. 1, we get some-
thing more than the information that every infinite set of rational numbers
is effectively denumerable. Namely, there obviously follows
THEOREM 2. In the set Z of all infinite sets of rational numbers we
are able to define a function y such that, for Pe Z, p(P) is an infinite se-
quence composed of all different elements of the set P.
In Theorem 2 we can of course replace the set Z by the set of all
infinite subsets of an arbitrary effectively denumerable set.
therefore y(6,) = y(d,) is impossible since g(6,) lies inside the interval 6,
while y(é,) lies inside the interval 6,.
Denote by T the set of all numbers y(6) where 6 « Z. Since the func-
tion @ is invertible in the set Z, the set Z is effectively equivalent to the
set 7. Thus the set 7 is infinite since the set Z is infinite. But the set T
is a subset of the set of all rational numbers, which, according to III.1,
Th. 1, is effectively denumerable. In view of the transitivity of the rela-
tion ef ~, it follows that the set Z is effectively denumerable. Thus we
have proved Theorem 1.
We might also prove Theorem 1 in another way, observing that if
we are given an interval 6, then each set of non-overlapping intervals,
lying in the interval 6, and whose lengths are not less than a given positive
number, is always finite. Therefore let us first take the intervals of the
set Z having lengths >1 and lying in the interval [—1, 1] (if there are
such intervals in the set Z), arranging them, for instance, from left to
right; similarly, the intervals of the set 4 having lengths >1/2, and
lying in the interval [—2, 2], which have not yet been taken into account;
next, similarly, the intervals having lengths > 1/3, lying in the interval
{[—3,3], etc. In this way we obtain a well-defined infinite sequence,
made up of all the intervals of the set Z.
EXERCISES. 1. Prove that if D is a denumerable set of points on a plane, then D
is the sum of two sets, of which one is finite on every straight line parallel to the axis
of abscissae, and the other is finite on every straight line parallel to the axis of ordinates.
Proof. Let D be a denumerable set of points on a plane. Let D, denote the set
of all the abscissae of the points of the set D, and D, — the set of all the ordinates of
the points of the set D. The sets D, and D, are of coursé finite or denumerable: thus
there exist finite or infinite sequences ,, 7, ..., and Y,, Y2,..., composed, respectively,
of all the elements of those sets. It can easily be seen that if the first of those sets is
finite, then each straight line parallel to the axis of abscissae contains a finite number
of points of the set D and, setting D = D+ 0, we shall have the desired decomposi-
tion of the set D. Similarly, if the sequence y,, yo, ... is finite, then D = 0+D will be
the desidered decomposition of the set D. Therefore let us assume that the sequences
24, Ge, --- aNd Yi, Ya, --- are infinite.
The set D is of course contained in the set of all points of the plane (x,, y,), where k
and J are natural numbers.
Let A denote the set of all the points (x,, y,) of the set D for which k <1 and
let B= D—A. We shall have, of course, D= A+B,
Now let y = b.be a given straight line parallel to the axis of abscissae. If b is none
of the terms of the sequence y,, yz, ..., then, ebviously, the straight line y = b contains
no point of the set B, and, therefore, no point of the set A. If, on the other hand, 0 is
one of the terms of the sequence ¥,, Yo, ..., for example b = y,, then, as follows at once
from the definition of the set A, the straight line y = b contains only the points (x,, b)
of the set A where k <1, and thus at most J points of the set A.
Similarly, let « = a denote a straight line parallel to the axis of ordinates. If a is
none of the terms of the sequence a,, #,..., then the straight line =a contains no
point of the set D, and, therefore, no point of the set B. If, on the other hand, a = z,,
§ 3. The infinite set of non-overlapping intervals 39
then, in virtue of the definition of the set B, the straight line x = a contains only the
points (a, y,) of the set B, where / < k, and thus less than & points of the set B.
Thus the desired proof is completed.
2. Prove that if f(a”) is a function having real values, defined in the set A composed
of real numbers, and increasing in the set A (i. ec. such that f(x,) < f(a.) for a <A,
w,¢ A, %, < #2), then the set of all points of the set A at which the function f(a) is not
continuous in the set A is finite or effectively denumerable.
Proof. We shall prove first that the set A, of all points of the set A at which the
function f(x) is not continuous from the right in the set A, is finite or effectively de-
numerable. By III.2, Th. 1, we are able to define the infinite sequence 7,, 72, ..., formed
of all rational numbers. Now let a « A. We shall show that there exists a rational num-
ber 7 such that f(a) < r and f(x) > r for xe A, x > a. Indeed, if there were no such
rational number z, then for each rational number r > f(a) there would exist an element
b<« A such that b > a and f(b) <r. Since the function f(x) is increasing in the set 4,
we should have f(a) < f(x) < f(b)<yr, for each element xe A such that a<a<b,
therefore f(a) < f(x) <r, which, in view of the arbitrariness of the rational number
7 >a (on which the number b > a depends) proves that the function f is continuous
from the right at the point a, in the set A, contrary to our assumption. Thus there exists
the first term 7, of the sequence 7,,7,,..., such that f(a) < 7, and f(x) > 7, for ve A,
x >a;letr(a)=7,. Now let us suppose that ae A,, a’ « A,, a 4 a’, for instance a < a’.
Thus we have f(a’) < r(a’) and, since a’>a, f(a’) > r(@), we have r(a) < r(a’). Thus
the function r(a) is determined for @ « A, and relates different terms of the sequence
1,.%2,... to different elements of the set A. Hence it immediately follows that the set A,
is finite or effectively denumerable.
Similarly we prove that the set A, of all points of the set A at which the function f
is not continuous from the left in the set A is finite or effectively denumerable. And
since the set of all points of the set A at which the function f/ is not continuous in the
set A is the sum of the sets A, and A,, it follows from [1.5, Th. 1, that this set (4,+ A.)
is finite or effectively denumerable, q. e. d.
Oe = Data —1
SRS eg a ata
Teas
-1
If n = 31—1, we shall replace everywhere in the above definition the upper in-
dices ’, ’’ and ’’” by ”’, ’” and ’, respectively, and if » = 31, we shall replace them respec-
tively by °”, %, and”.
It is easy to prove that the infinite sequences {u/}, {u/’}, {u/’’}, defined thus by
induction, satisfy the required conditions.
2. Find an infinite sequence {u,} of rational numbers which is not a sum of two
sequences, each of them composed of all different rational numbers.
Solution. Such is for instance the sequence {u,} where u,= 1, and u, = 0 for
SOs Be ep
Indeed, if we had u, = u,+ uj’ for n = 1, 2,..., where each of the sequences {u;,}
and, {u}’} is composed of all different rational numbers, then we should have u/-+ u}’ = 0
for n = 2,3,..., therefore u/’ = — us for n = 2, 3,... But the sequence ui, uj, ..., and
therefore also the sequence —u/, —uf,... are made up of all different rational numbers.
Thus the sequence w,’, u;’,... contains all rational numbers except the number —w{,
whence, in view of the property of the sequence {u/’}, we conclude that uj’/= —u/,
therefore u, = uj+ ui’ = 0, which is impossible since u, = 1.
magnitude of their imaginary parts), then (in the same way) all the roots
of the polynomial f,(7), next those of the polynomial f,(x), ete.
In the infinite sequence of algebraic numbers obtained in this way
let us retain only those terms which are different from any of the terms
preceding them. We shall obtain the well-defined infinite sequence
(5.2) pe er ane
composed of all and, also, of different algebraic numbers. That the
sequence is infinite follows from the remark that, in any case, this se-
quence will contain all rational numbers as roots of polynomials of the
first degree with integer coefficients.
Thus we are able to arrange all algebraic numbers in an infinite
sequence, which proves Theorem 1].
EXERCISES. 1. Prove that the set of all finite sequences whose terms are al-
gebraic numbers is effectively denumerable.
Proof. It is a consequence of Theorem 1 of III.5 and Theorem 2 of III.4.
2. Prove that the set of all straight lines in a plane each of which passes through
(at least) two different points with rational coordinates is effectively denumerable.
3. Prove that if & denotes the set of all points of a plane that have rational co-
ordinates, and D any straight line lying in the plane, then for the set D& only the fol-
lowing three cases are possible: 1° The set DR is empty, 2° The set DR consists of only
one point, 3° The set DR is denumerable.
4. Prove that the same applies to the case where #& denotes the set of all points
of a plane that have integer coordinates, and to the case where & denotes the set of
all points of a plane whose coordinates are algebraic numbers, but not to the case of &
denoting the set of all points of a plane that have natural coordinates (because then
the set DR may also have an arbitrary finite number of points).
where a”, af, af, ... is an infinite sequence (corresponding to the na-
tural number ») of natural numbers.
44 III. Denumerable and non-denumerable sets
Proof. As we proved in III.4 we are able to define an infinite sequence 0;, oy, ...
of all finite sequences of natural numbers. Now denote by s,, for natural n, the infinite
sequence arising from the finite sequence o, if, after the last term of that sequence, we
add the infinite sequence of numbers 1.
Now let us suppose that the function # mentioned in our exercise exists. For the
infinite sequence of the sequences s,, 8, ... just defined, let s = #(s,, 82, ...). For every
natural number nm there exist at most q terms of the sequence s,, equal to the correspond-
ing terms of the sequence s.
Let s = (k,, k,, ...). The finite sequence k,, k,, ...,-k q+ is one of the terms of the
infinite sequence o;, o2,...; thus there exists a natural number n such that (k,, ke,..., k, 41) =
= o,. According to the definition of the set s,, we shall then have s, = (k,, ky, ..., ky.4;
1,1,1,...). Thus each of the first g+1 terms of the sequence s, is equal to the cor-
responding term of the sequence s, contrary to the above-mentioned property of the
set s,. Thus the assumption of the existence of the function # mentioned in Exercise 2
deads to a contradiction. This completes the proof.
A= {P1) Pes Ee
§ 7. Sets containing denumerable subsets 47
and B= {q%, q,...}. Now let us remove from the infinite sequence p,, q,
Dey 2) Ps) Is, --- every term that is equal to one of the terms preceding it.
As can easily be seen, we shall obtain in this way an infinite sequence
composed of all different elements of the set A+B. Thus the set 4+B
is effectively denumerable.
Hence we obtain by induction
THEOREM 7. The sum of a finite number of effectively denumerabie
sets is an effectively denumerable set.
Obviously we also have
THEOREM 8. The swum of a fimte number of denumerable sets is a de-
numerable set.
THEOREM 9. Every set in which we are able to indicate an effectively
denumerable subset is effectively equivalent to a certain proper subset of
itself (which we are able to indicate).
Proof. Let us suppose that in the set A we are able to indicate an
effectively denumerable subset D= {p,, p2,...} of that set. Let B=
== {o5 Day Dey ---} Oud C= {p,, Ps, Ps, .>-}; these will be.effectively. de-
numerable sets, and we shall have A DB, A—B DC, whence,’according
to Theorem 5, A ef ~A—B. But, in view of BCA and B #0, we have
A ~A-—B; therefore the set A—B is a proper subset of the set A. Thus
the set A is effectively equivalent to its proper subset A—B, which we
are able to define (since we know the effectively denumerable set D).
Thus we have proved Theorem 9.
We also have, as can easily be seen,
THEOREM 10. A set containing a denumerable subset is equivalent to
a certain proper subset of itself.
Theorem 10 can be reversed: namely we have
THEOREM 11. A set which is equivalent to a certain proper subset of
itself contains a denumerable subset.
Proof. Let us suppose that a set A is equivalent to a certain proper
subset of itself B. Thus 4 DB, A—B ~0 and there exists a function
establishing a (1-1) correspondence between the elements of the sets A
and B. Since A—B #0, there exists an element p,<«4—B. We shall
have p,= 9(p,)¢«B, therefore p, & p,- Let us determine by induction
the infinite sequence p,, pz,..., setting pp, = p(pr-1) for n= 2,3,... We
easily prove by induction that p, « B for n = 2,3,... We shall show that
all terms of the infinite sequence p,, p.,... are different from one another.
Let us assume that this is not true, and let p denote the first of the terms
which are equal to one of the preceding terms, thus p, = p, where 7 > q;
therefore p, « B, and since p,¢ B, the equality g = 1 is impossible. Thus
48 III. Denumerable and non-denumerable sets
1) If anyone wished to assert that the function f(x) = x is an element of the set A,
he would have to prove first that the set # is empty, which we are not able to do (and
from the so-called axiom of- choice it follows that IF + 0).
2) The truth of the proposition stating that we are able to indicate an element
of the set A depends on the time-and on the person who makes it. Logic knows
various propositions of this kind, e.g. the propositions: “I am 75 years old“, “I am
in Paris“, “It is Friday to-day“.
3) According to A. A. Fraenkel (L’axiome du choix, Revue Philosophique de Teena sin
50 (1952), p. 444), this conjecture was formulated as early as 1904 by H. Lebesgue.
Cardinal and ordinal numbers 4
50 III. Denumerable and non-denumerable sets
is equivalent to the set obtained by joining one element to the set A. Thus the condition
stated in the exercise is necessary.
On the other hand, let us suppose that the set A is equivalent to the set B = A+ {p}
where p ¢ A. Thus the set B is equivalent to its proper subset A, and therefore, accord-
ing to the definition of sets infinite in the sense of Dedekind, it is infinite in the sense
of Dedekind. Such is also the set A, being equivalent to the set B. Thus the condition
stated in the exercise is sufficient.
2. Prove that the set B is infinite in the sense of Dedekind if and only if it is
equivalent to the set obtained by removing from the set B one of its elements.
Proof. If pe B, let A = B— {p}, whence B= A+ {p}.
If the set B is infinite in the sense of Dedekind, then, according to Theorem 1
of III.1 B contains a denumerable subset {p,, p., ...}. The set {p,, pz, -..}—{p} will of
course be a denumerable subset of the set A; according to Theorem 1, the latter is in-
finite in the sense of Dedekind and, according to Exercise 1, we have A+ {p}~4dA,
Therefore B~ B— {p}, which proves that the condition stated in Exercise 2 is necessary.
On the other hand, if for a certain p«B we have B—{p}~B, then the set B is
equivalent to its proper subset B— {p}, therefore it is infinite in the sense of Dedekind.
Thus the condition stated in Exercise 2 is sufficient.
3. Prove that a set infinite in the sense of Dedekind is equivalent to the set ob-
tained by adding to it or removing from it an arbitrary finite number of elements.
Proof. The proof is easily obtained from Exercises 1 and 2, by induction.
1) ,,De nombreux mathématiciens se sont lancés sur ses traces et se sont posé
une série de questions de méme genre. IIs se sont tellement familiarisés avec les nombres
transfinis qu’ils en sont arrivés 4 faire dépendre la théorie des nombres finis de celle
des nombres cardinaux de Cantor. A leurs yeux, pour enseigner l’arithmétique d’une
fagon vraiment logique, on devrait commencer par établir les propriétés générales des
nombres cardinaux transfinis, puis distinguer parmi eux une toute petite classe, celle
des nombres entiers ordinaires. Grace & ce détour on pourrait arriver 4 démontrer toutes
les propositions relatives 4 cette petite classe (c’est-4-dire toute notre arithmétique et
notre algébre) sans se servir d’aucun principe étranger 4 la logique“ (Poincaré [1]).
§ 9. Various definitions of finite sets pet
1. Sets of the power of the continuum and sets effectively of the power of
the continuum. A set which is equivalent to the set of all real numbers
is said to be of the power of the continuum and the set which is effectively
equivalent to the set of all real numbers is said to be effectively of the
power of the continuum.
According to Brouwer the set of all real numbers is not a finished
creation: its elements cannot be said to exist but only to be coming into
existence in the process of our defining them. Thus the set of all real
numbers is presented as a set in which we are able to indicate every time
only a denumerable subset, but for every subset of this kind it is always
possible to define the real numbers which do not belong to it (Fraenkel
[3], p. 47; see Theorem 1 of IV.2).
Lusin also questions the concept of the set of all real numbers since
we do not know the general law of defining every real number (Lusin [2],
p. 242). Kronecker and Brouwer go still further, rejecting the concept
of the set of all natural numbers (Bernays [2]).
A. Fraenkel, however, is of the opinion that a theory of sets which
did not ensure the existence of sets of the power of the continuum (so
essential for both Analysis and Geometry) would be a mere shadow and
would mean a suicidal renunciation of the teachings of Cantor for the
sake of intuitionism (Fraenkel [3]).
2.2, Cn ==
Out Oe a0.
er Date Seti:
The infinite sequence ¢,, ¢,,... will be well-defined by these condi-
tions and the numbers ¢,(n = 1,2,...) will be digits of the decimal system
other than the digit 9. Let
(2.3) (Vig Was noo) =, Oy OC
oGyan
Formula (2.3) gives the normal decimal expansion of the real num-
DOL LL == eae ae)
Now let » denote a given natural number. Formulae (2.2) and (2.3)
prove that the n-th digit of the expansion (2.3) is different from the n-th
digit of the expansion (2.1). Since every real number has only one normal
decimal expansion, it follows that «4A x,. Thus we have «~ 42, for n=
= 1,2,... Therefore the function y(a#,, 7, ...) satisfies Theorem 1, which
has thus been proved.
As the set of all real numbers is infinite (since all natural numbers
belong to it), therefore it follows from Theorem 1 (in the same way as
Theorem 2 of III.6 follows from Theorem 1 of III.6) that the set of all
real numbers is non-denumerable.
In view of Theorem 1 we likewise conclude that the set of all real
numbers may be called effectively non-denumerable *).
From Theorem 1 it follows that we can define a function /(#) which
relates to every effectively denumerable set H of real numbers a real
number f(#) such that f(#)¢ H. Let us observe that we cannot define
a function g(D) relating to every denumerable set D of real numbers
a real number g(D) such that g(D)¢ D. Neither can we define a func-
tion h(D) relating to every denumerable set D of real numbers a real
number h(D) such that h(D) « D.
€, = p(b,, Be, .:.) , Cn4t-== Y(C1, Cay +++). ny O14 Og, «--)
ATOR Ci 1 Sei.)
It follows immediately from these formulae and from the properties
of the function y that the terms of the infinite sequence ¢,, ¢,... are all
different from one another, and that the set O = {¢,, G&,...} is disjoint
with respect to the set B. Therefore we have X—BOC and the set C is
an effectively denumerable set, which we are able to define. Thus we
have proved Theorem 1. (
From Theorem 1 and Theorem 5 of III.7 we immediately have
THEOREM 2. If we remove from the set of all real numbers an effectively
denumerable subset, we shall obtain a set effectively of the power of the con-
tinuum.
From this theorem, in view of Theorem 1 of III.2 and Theorem 1
of III.5 we immediately infer the following two corollaries:
COROLLARY 1. The set of all. irrational numbers is effectively of the
power of the continuum.
COROLLARY 2. The set of all real transcendental +) numbers is effectively
of the power of the continuum.
No example of a transcendental number had been known up to the
middle of the past century. However, from IV.2, Theorem 1, and from
IIL.5, Theorem 1, it immediately follows that we are able to define
a transcendental number.
A transcendental number defined in this way would be very com-
plicated and hardly interesting. The proof of the existence of transcen-
dental numbers, based on the denumerability of the set of all algebraical
numbers, comes from G. Cantor?). In the same year, 1873, Hermite
proved that the number e (the base of natural logarithms) is transcendental,
thus giving a simple example of such numbers. About 10 years later
Lindemann proved that the number z is also transcendental, and, con-
sequently, that the squaring of a circle cannot be performed by means
of compasses and ruler (Lindemann [1]). In 1934 it was proved (by Gel-
fond and Schneider) that the numbers 2V? and e* are transcendental.
y=,ot
lying inside the interval (a, b); the proof that the correspondence is (1-1)
presents no difficulties.
Thus we have
THEOREM 1. The set of all real numbers lying inside the finite interval
(a,b) (where a and b are given real numbers, a <b), is effectively of the
power of the continuum.
By IV.3, Theorem 5, and the Corollary to it, Theorem 1 implies
THEOREM la. If a and b are real numbers, a <b, then both the set
of all real numbers x such that a< « <b and the set of all real numbers «x
such that a <<u“<b ave effectively of the power of the continuum.
EXERCISE. Give an example of the continuum of different increasing functions
mapping in a (1-1) way the set of all real numbers of the interval 0 < «<1 on the
set of all real numbers of the interval 0< y < 2.
Answer. The function
se err =e
The question arises whether we are able to define a set which can
be proved to be of the power of the continuum but cannot be proved
(at least now) to be effectively of the power of the continuum.
Here is an example of such a set.
Let us divide all real numbers into classes, including two numbers
in the same class if and only if their difference is a rational number.
Let A denote the set of all classes obtained in this way. Now we shail
define the set B as follows. If the set A is of the power of the continuum,
let B= A; otherwise let us denote by B the set of all real numbers.
Clearly, the set B defined in this way is of the power of the con-
tinuum but at the present state of science we are unable to prove that
it is effectively of the power of the continuum.
58 IV. Sets of the power of the continuum
this sum gives, as we know, a certain real number > 0 and <1.
Basing ourselves on the theorem known from Analysis stating that
every real number x such that 0 <a# <1 can be represented uniquely
in the form
HE es irc Tat yeeis Ae
60 IV. Sets of the power of the continuum
straight line, and if any two points of the set H are connected by line segments, then
no two segments have a common point which is not an end point.
Solution. A set of that kind is the set # of all points in the three-dimensional
space with the coordinates (¢, ¢, t?) where ¢t is an arbitrary real number. No three points
of the set # lie on the same straight line because if they did, there would exist four
different points of the set # lying in the same plane, which is impossible since the equa-
tion Ai?+ B+ Ct+D = 0, provided we do not have simultaneously A = B= O= D= 0,
has no more than th ee different roots. And if two segments whose end points are
points of the set H had a common point other than their common end point, then they
would lie in the same plane, containing four different points of the set #, which is im-
possible. Thus the set H has the required properties.
7. Prove that the set of all infinite sets of natural numbers is effectively of
the power of the continuum.
3 X= SA),
xeX
OLDER Oe
But the set (X x X)xZX is, as we know, effectively equivalent to the set
of all triplets (v, y,z) where wv, y,z2 are real numbers, and also to the
set of all points in the three-dimensional space. Thus we have
THEOREM 2b. The set of all points in the three-dimensional space is
effectively equivalent to the set of all points on a straight line.
Now, in the (1-1) correspondence between the points on a plane
and the points on a straight line, which we are able to establish accord-
ing to Theorem 2, let f,(7,y) denote the point on the straight line cor-
responding to ee point (#, y) on the plane, and let (p(t), p(t )) denote
the point on the plane to which corresponds the point t on the line. We
have of course for any real w and y
h(y) = 39(y)
(Lindenbaum [4], p. 27, Sierpinski [21], p. 133, footnote).
We shall further observe that by using the so-called Continuum
Hypothesis (see V.2) we can prove that there exists a real function of
two real variables f,(~2, y) such that for every real function f(#, y) of two
real variables there exists a real function of one real variable #(¢) in-
vertible and such that for any real « and y we have f(a, y) = f,(9 (x), 3(y))
(Sierpinski [25], p. 8, Sierpinski [24], p. 9).
2. Prove that if the function p(t) = (p(t), p(t)) is a (1-1) mapping of a straight
line on the whole plane, then the function q(é) = (p(t), (v(t), v(y@))) is a (1-1)
mapping of a straight line on the whole three-dimensional space.
§ 8. Cartesian product 65
Proof. Since the function p(t) is invertible in the set of all real numbers, we can
easily prove that the function q(t) is also invertible in the set of all real numbers. Thus
it remains to show that for every point (x, y, z) of the three-dimensional space there
exists a real number ¢ such that q(t) = (x, y, 2).
The properties of the function p(t) imply the existence of a real number uw such
that y = p(u), 2 =y(u) and of a real number ¢ such that « = y(t), wu= p(t). Hence we
have x = g(t), y= o(p(t)), 2 = y(y(t)), therefore g(t) = (x, y, 2), q.e. d.
3. Prove that we are able to define a function of three real variables f,(x, y, 2)
such that every real function f(x, y, 2) of three real variables is a function of one varia-
ble of the function f,, and that we are able to define that function of one variable if
we know the function f(x, y, 2).
Proof. In the (1-1) correspondence between the points of three-dimensional
space and the points on a straight line, which we are able to establish by Theorem 2b,
let f,(~, y, 2) denote the point of the line corresponding to the point (x,y,z) of the
space, and let p(t), w(t), (t) denote the point of the space to an Ae ae the
point ¢ of the straight line. It is easy to prove that, taking F(t) = f(y( t), ®(t)) for
real t, we shall have f(z, y,2) =F (f(x, y,2)) for any real a, y, 2.
4. Prove that we are able to define a real function f,(x, y) of two real variables
such that for every function f(x, y, 2) of three real variables we are able to define
a function g(x, y) of two real variables for which we have, with any real z, y, 2,
{8.4) f(a, Ys 2) = G(X fl¥x2))
Proof. Let f), y and y denote the same functions as in the proof of formula (8.3).
Let g(x,y) = f(x, p(y), p(y)). In view of (8.1) it is easy to verify that we shall have
identity (8.4).
This proves that functions of three real variables can be expressed as a composite
function of functions of two variables. Still, as has been proved by Bieberbach ({1],
p. 90), it is not every continuous function of three variables that can be expressed as
a composite function of a finite number of continuous functions of two real variables.
5. Prove that if the functions f,, y, y, have the same meaning as in the proof of
formula (8.3), then, for any real function of four real variables f(x, y, 2, t), and h(x, y) =
=f (v(x), p(x), p(y), ply)), we shall have identically
and taking k(x, y) = f(a, p(y), p(yly)). v(y(y))). we shall have identically
Proof. The proof follows immediately from Theorem 2 and Exercise 2 of II.6.
8. Prove that the set Z of all two-element subsets of the set X of all real numbers
is effectively of the power of the continuum.
Proof. To each two-element subset of the set X we can assign in a (1-1) manner
an ordered pair composed of two real numbers, its first element being the smaller of
the numbers constituting the elements of the given two-element subset of the set X
and its second element being the larger of those numbers. Thus the set Z is effectively
equivalent to a subset of the set X x X (consisting of those of its elements (x, y) where
a < y), and therefore, by Theorem I, it is effectively equivalent to a subset of the set X.
On the other hand, the set X is effectively equivalent to a certain subset of the set Z,
namely to the subset composed of all the pairs {~, 7+ 1} where wx is a real number.
Hence, by Theorem 1 of III.7, we conclude that the sets Z and X are effectively equi-
valent, q.e. d.
9. Prove that for every natural number n the set of n-element subsets of the set X
of all real numbers is effectively of the power of the continuum.
Proof. The proof is analogous to that applied in Exercise 8.
10. Prove that the set Q of all finite subsets of the set X of all real numbers ig
effectively of the power of the continuum.
Proof. For n= 1, 2,3,... let us denote by Q, the set of all n-element subsets
of the set X. We shall have of course 9 = Q,+9Q,+..., and from Exercises 8 and 9 it
easily follows that we can map the set Q on the set of all positive real numbers so that
the image of the set Q,, for n= 1, 2,..., will be the set of all real numbers X such
that n—1 <a<n. Thus the set Q is effectively equivalent to a subset of the set X.
Conversely, the set X is of course effectively equivalent to the set Q,, 7. e. to a subset
of the set Y. Hence, in virtue of Theorem 1 of II.6, we conclude that the sets Q and X
are effectively equivalent, q. e. d.
f(t) is a linear function for + <t<#, and f(—t)=/f(t) for 0 <t <1.
The curve in question is defined by the equations
p(t)=Eft) + 18%)+35f(3%)+
(10.2)
y(t) = =f(3t)+59(3%) + 3yACB)+
From the inequality 0 </f(t) <1 for real ¢ it follows, by (10.2),
that 0 <gy(t) <1, 0 <yp(t) <1 for 0 <t <1, and that the two series
(10.2) are uniformly convergent in the interval (0,1), 7. e. that the func-
tions g(t) and y(t) are continuous. It remains to show that the curve
(10.1) meets every point of the square 0 <x <1,0<y <1. Let (a, yp)
denote a given point of this square. The dyadic expansion of the numbers
# and y, can be given in the form
Stata 92 | 93 veng
(10.3) ot
Yn =
Yo 9 t sat 93 Se 008 )
If a, = 0, then we have
Digi, 1
Oe a a ae
§ 10. Continuous curve filling up a square 69
Therefore, if, for a given real wv, 0(s) = 4, then every infinite se-
quence n,n, ..., i.e. also the numbers | pa y(n, no, - 3 OR
§ 11. Set of all infinite sequences 71
j =1,2,..., are well-defined (by the number a). This proves that (for
@ given x) there exists at most one element s of the set S such that #(s) = a.
On the other hand, given a real number « for which y(a) =
= (M,, Me, ...), We Write 7; = y'(Mp 41.) Mgi—1.5) Mgi—1,)---) ANA $ = (%, Ly,...).
Introducing the notation (11.3), we shall have y(a;) = (n®, n®, Jie AOD t=
=1,2,..., and thus, according to the definition of the function 0, we
get (11.1). But (11.3) for 7 = p, and h = q, gives (11.2). Thus (11.1) gives
WAS) = NM Ms 2.0) = (since we have assumed that w(x) = (m,, Mme, 32)) ,
Thus for every real number x there exists an element s of the set S
such that 0(s) = a.
Therefore we have proved that the function ? establishes a (1-1)
correspondence between the elements of the set S and all real numbers;
we have thus proved Theorem 1.
EXERCISES. 1. Prove that the set of all infinite sequences of complex numbers
is effectively of the power-of the continuum.
Proof. By Theorem 1 we are able to define a function # establishing a (1-1) cor-
respondence between infinite sequences of real numbers and real numbers. Now let
S = (2, 2,...) denote an infinite sequence of complex numbers. Thus, for natural n,
we have z, =a,+0,% where a, and b, are real numbers. Let t(s) = (a, b,, de, bz, a3,
bs, ...). The proof that the function 7 establishes a (1-1) correspondence between all
infinite sequences of complex numbers and all real numbers presents no difficulties.
2. Prove that the set of all power series with complex coefficients
set B, of the set B of all infinite sequences of real numbers. In order to obtain the func-
tion t establishing a (1-1) correspondence between the elements of the sets A and B it
suffices to assign to every sequence (@,,4,,...,d,) belonging to A an infinite sequence
COA Gp Wn coon Gan Un Or cox)
belonging to B.
Note. We shall not obtain an invertible function +t defined in the set A, if
we write
T(r sag teeetoLey =" Class Og totes ouanak Or Ose Ogre)
for every sequence (a,, d.,...,4,) belonging to A, since we then have e.g. t(1, 0) =
= 1(1,0, 0) = (1, 0, 0, 0,...), the sequences (1,0) and (1,0,0) being different ele-
ments of the set A.
Every set of non-empty disjoint sets whose sum is the set A is called a partition
of the set A. .
6. Prove that if NV is the set of all natural numbers, then the set Z of all partitions
of the set N is effectively of the power of the continuum.
Proof. Let S denote the set of all increasing infinite sequences of natural numbers.
From IV.7, Exercise 1, it follows that X ef ~S where X is the set of all real numbers.
Now let s«S, therefore s = (n,, n,...) where 1, n,,... are natural numbers and n,<
<<... Let P, = {my +1,n,+1, ...}, 9, = N—P,. The set {P,,Q,} is a partition of
the set N (since 1 «Q,): we denote it by f(s). Obviously for s « S, s’« S, s #8’ we shall
have f(s) ~ f(s’). Thus the function f is a (1-1) mapping of the set 8 on a certain sub-
set Z, of the set Z, whence S ef ~Z, c Z, and since X ef ~S, we have also X ef ~Z, c Z.
Now let P denote any partition of the set V; thus N = » 2B, the set Hmaking up
EeP
the partition P being disjoint and non-empty. For each set # « P let us denote by #(£)
the least natural number belonging to H. Since the sets H « P are disjoint, we shall
have &(H) 4 O(H’) for He P, H’ « P, HEA LH’. Thus the sets He P can be arranged
in a finite or an infinite sequence H,, H,,... according to the magnitude of the natural
numbers #(H#) corresponding to them, and every set H,, as a non-empty set of natural
numbers, can be arranged in a finite or infinite sequence according to the magnitude
of the numbers belonging to it: H, = n®, ne, ...}. In this way we obtain a certain finite
or infinite sequence of finite or infinite sequences of natural numbers
nO 4 al not Mm nO dence
n+ Lis nO 4 li nO) + 1S 8S
which we are able to arrange, for instance by the diagonal method, in a single se-
quence (m,, M2, ...). Let g(P) = (m,, mr, ...).
§ 11. Set of all infinite sequences 73
Let
Tilt) = 2 2 tan—t00n 1) ’
=r
wanehe by <(12.2) Cives wai dn, (Oren. 1, 2...
Thus Theorem 1 has been proved.
It will be noted that the function f,(t) (n=1,2,...), satisfying
Theorem 1, can be obtained from every continuous curve (10.1) filling
up a square by writing /,(¢) = oy"st(t) for »— 1,,2,..., 0 <t< 14).
Ene Ena
(13.3) (| = (=) for n= Li 2 sere
. Ene
lim aero
of the set B corresponds to the subset S, of the set S and we have S, ef ~B,.
Owing to the transitivity of the relation ef ~ formulae A ef ~S, and
S,ef ~B, give Aef ~B,.
Now let us denote by A, the set of all functions (continuous of course)
such that f(#) =a for xe B where a is a given real number. We have
of course Bef ~A,. Formulae A ef ~B,C B and Bef ~A, CA give,
by Theorem 1 of I1.6, the formula A ef ~B, which proves Theorem 1.
EXERCISES. 1. Prove that the set of all continuous complex functions of one
complex variable is effectively of the power of the continuum.
2. Prove that the set of all continuous (at least with respect to each variable se-
parately) real functions of two real variables is effectively of the power of the continuum.
i AD oF ee ok
f(A) will be a natural number well-defined by the set A. Thus f(A) is
a function having natural values, defined for A « Z. We shall show that
the function f is invertible in the set Z.
Let us assume that A4<¢Z, A’e«Z, AAA’, f(A)=—f(A’). Tf A=
== {14 Noy =f Where, <n, <9... and ACs int, ne, 1.3) Where ny << Ne < 2.2,
then f(A) = 2%+2%4 ...4.9%+1, f(A’) = gn 4 ora eb gtme therefore,
in view of f(A) =f(A’),
OO p Orme OM ont poets
Since 7,, Me, .--5%m41 and ni, N3,'..., M441 are natural numbers satisfy-
ing the inequalities n,<,.<...< m1 and ni < Nz <<... << Mii, this
gives, as we know, n;= nj for i=1, 2,...,.m+1.
Thus sets A and A’ have at least m+1 common elements, contrary
to our assumption that Ae« Z, A’eZ and A~ A’. Thus the function f
is invertible in the set Z and maps this set on a certain subset of the set
of all natural numbers. Hence it immediately follows that the set Z is
effectively denumerable, q. e. d.
EXERCISES. 1. Define a set Z composed of the continuum of different infinite
sets of natural numbers such that of each two different sets belonging to Z one is always
a proper subset of the other.
Solution. By Theorem 1 of III.2 we are able to arrange all different rational
numbers in an infinite sequence 7,,7,,... Now, for a given real number z, let us de-
note by A(a) the set of all natural numbers » such that r, < x; it will of course be an
infinite set. Clearly, for real x and y where « < y, we shall have A(x) c A(y), because
ifn « A(x), then we have r, < x and, in view of x < y, we have r, < y, whence n « A(y).
On the other hand, if w < y, then, as we know, there exists a rational number r such
that «<r<_y. In view of the properties of the sequence 7, 7,,..., the number r is
one of its terms; thus there exists a natural number m such that r = 7,,. Thus we have
x<1, <Y, Whence it follows that m¢€ A(x) and me A(y). Therefore A(x) #4 A(y),
and since A(«#)c A(y), the set A(x) is a proper subset of the set A(y).
§ 14. Decomposition of a set of natural numbers 79
Now let us denote by Z the set of all sets A(x) where x is a real number. Obviously
the set Z satisfies the required conditions.
2. According to S. Hartman two infinite sequences of natural numbers ay, dp, ...
and b,, b,... are called distant if for every natural number ¢ the inequality |a,,—b,| <¢
holds only for a finite number of pairs (m,n) of natural numbers. Give an example of
a continuum of infinite sequences of natural numbers each two of which are distant.
Solution. For real « > 1, let us denote by S(#) an infinite sequence w,, Ue, ...,
where u, = 27"(2Ena-+ 1)? for n = 1, 2,... We shall show that for «> l,y>1,«##y,
the sequences S(x) and S(y) are distant.
Let & denote an arbitrary natural number. We need only show that if at least.
one of the natural numbers mand nis > k-+ 1/|a—y], then |2°"(2Ema-+ 1)?—2°*"(2Eny+
+ 1)?| > k. Suppose that at least one of the numbers m and n is >k-+1/|x—y|. If we
had 2”"(2Emaz+ 1) = 2”(2Eny+1), we should have m=n and Emax = Emy, there-
fore |na—ny| <1, whence m= n < 1/|x—y|, contrary to our assumption that at
least one of the numbers m and n is >k+1/|~—y|. Thus we have 2"(2Emz+1) #4
#~2"(aEny+1). But
|2°"(2Ema-+ 1)?—2*"(2E
ny + 1)>|
= |2"(2E ma + 1)—2°(2Eny + 1)|-|2"(2Ema+ 1)4 2"(Env+ 1)|.
The first factor is >1 (since we have proved it to be 40) and the second is of
course >2”+ 2" > m+n, therefore >k+1/|~—y| >k. The product of these two
factors, and thus also the left-hand side of the last equality, is >k, q. e. d.
CHAPTER V
1. Sets of less and greater power. Let A and B be two given sets.
If the set A is equivalent to (7. e. of the same power as) a certain subset
of the set B, but the set B is not equivalent to any subset of the
set A, then we shall say that the set A is of less power than the set B and
write
(1.1) i ASB.
Obviously for finite sets A and B this formula holds if and only if
the set A has fewer elements than the set B.
7 If formula (1.1) holds, we shall also say that the set B is of greater
power than the set A and write
BSA:
From the definition of formula (1.1) it immediately follows that
if A < B, then neither A~B nor B < A.
THEOREM 1. If A, B, A,, B, are sets such that Aes B, A~A, and
B~B,, then A, < B,.
Proof. From A < B it follows that the set A is equivalent to a cer-
tain subset B’ of the set B. Thus A~B’, and since A~A,, we have
A,~B’. In view of B~B,, the subset B’ of the set B is equivalent to
a certain subset Bj of the set B,. Thus B’~Bj, whence, in view of A,~B’,
we have A,~B;. Thus the set A, is equivalent to a certain subset of the
set B,.
If the set B, were equivalent to a certain subset A; of the set A,,
then we should have B,~Aj. In view of A,~A, the subset Aj of the set A,
would be equivalent to a certain subset A’ of the set A. Thus we should
have B,~Aj, Ai~A’, and since B~B,; this would imply (in view of
the transitivity of the relation ~) B~A’, and the set B would be
equivalent to a certain subset of the set A, contrary to our assumption
that 4 < B.
§ 1. Sets of less and greater power 81
Thus the set B, is not equivalent to any subset of the set A,, and
since we have proved above that the set A, is equivalent to a certain
subset of the set B,, we have Ay =a Bis q-ecd.
We shall write A = B instead of A~B and A < B instead of A < B
or A = B. Obviously if A < B, then the set A is equivalent to a certain
subset of the set B. We shall prove that the inverse is also true, 7. e. that
we have
THEOREM 2. Formula A <B holds for the sets A and B if and only
af the set A is equivalent to a certain subset of the set B.
Proof. In view of the remark made above it is sufficient to prove
that if the set A is equivalent to a certain subset B, of the set B, then
A <B. For this purpose we shall distinguish two cases:
1° The set B is equivalent to a certain subset A, of the set <A.
Thus we have A~B,C B and B~A,C A, whence, in virtue of the
Cantor-Bernstein theorem (Theorem 2 in II.6), 4~B, therefore A = B.
2° The set B is not equivalent to any subset of the set A. Since the
set A is equivalent to a certain subset of the set B, we have A < B.
Thus we have either A = B or A < B, therefore in any case A < B,
Gee. d:
Thus we have proved Theorem 2.
It will be noted that not only is it easy to deduce Theorem 2 from
the Cantor-Bernstein theorem, but also conversely the latter theorem can
easily be deduced from Theorem 2.
Indeed, let us assume that Theorem 2 is true and let A, B, A,, B,
be sets such that A~B,C B and B~A,C A. From Theorem 2 it fol-
lows that A <B and B <A. If we did not have 4 = B, we should have
A<Band B< A, which, as we know, is impossible. Therefore we must
have A= B, i. e. A~B, which shows the truth of the Cantor-Bernstein
theorem.
THEOREM 3. If A, B,C are sets such that A < Band B< C, then
A= C.
Proof. Since A < zB, the set A is equivalent to a certain subset B,
of the set B, and since B < C, the set B is equivalent to a certain subset Co
of the set C. Thus the subset B, of the set B is equivalent to a certain
subset OC; of the set C. Therefore we have A~C;C C.
If the set C were equivalent to a certain subset A, of the set A, then
the set B, being equivalent to the subset C, of the set C, would be equi-
valent to a certain subset A; of the set A, contrary to our assumption
Cardinal and ordinal numbers 6
82 V. Comparing the power of sets
that A < B. Thus the set C is not equivalent to any subset of the set 4
and since A~ OC; C Od, a 2 Ole que. a:
Similarly we can prove
THEOREM 3a. If A, B, C are sets such that A <B and B < O,.then
A <0:
It is likewise easy to prove that the following theorem holds:
THEOREM 3b. If A, B, CO are sets such that A <B and B <0, then
Teg 6h
2. Sets of greater power than finite sets and denumerable sets. Hypothesis:
of the continuum. We shall prove
THEOREM 1. Any infinite set is of greater power than any finite set.
Proof. Let A denote an infinite set, B a finite set. If the set B is
empty, then it is equivalent to the empty subset of the set A. If the set B
is not empty, then the number of its elements is a natural number n.
In II.2 we have proved that if the set A is infinite, then for every natural
number n it contains a subset of n elements. Thus the set B is any case
equivalent to a certain subset of the set A, but the set A, being infinite,
is not equivalent to any subset of the finite set B. Therefore we have
B< A. Thus we have proved Theorem 1.
The inverse theorem is of course also true: every set which is of
greater power than every finite set is an infinite set.
THEOREM 2. A set of the power of the continuum is of greater power
than any denumerable set.
Proof. Let A denote a set of the power of the continuum, 7. e.
A~X where X is the set of all real numbers, and let B denote a denume-
rable set. Thus the set B is equivalent to the set N of all natural numbers.
Therefore B~N C X, but the set X is not equivalent to any subset of
the set B, since, by Theorem 1a of III.1, every subset of a denumerable
set is finite or denumerable, and the set X, as we have proved in IV.2,
is non-denumerable, 7. ¢. not equivalent to any finite or denumerable
set. Therefore B < X, and since X~A, by Theorem 1 of V.1, we have
B< A. Thus we have proved Theorem 2.
Obviously, a set of greater power than a denumerable set is non-
denumerable. It will be noted, however, that without the aid of the axiom
of choice we are not able to prove that any non-denumerable set is
greater than any denumerable set. The proof of this theorem, based on
the axiom of choice, will be given in VI.7 (see Corollary 1, p. 114).
We do not know whether there exists a set which is of greater power
than a denumerable set and of less power than a set of the power of the
§ 2. Hypothesis of the continuum 83
continuum. The assumption that there are no such sets is called the
Continuum Hypothesis. So far the Hypothesis has neither been proved
nor disproved. K. Goddel has proved that the continuum hypothesis is
consistent with the generally accepted axioms of the Theory of Sets,
provided the latter are consistent with one another (Go6del [2] and [3]).
In the paper The present state of research into the foundations of mathe-
matics, written by A. Mostowski in collaboration with A. Grzegorezyk,
A. Jaskowski, J. Los, S. Mazur, H. Rasiowa and R. Sikorski, we find
the following statement:
“In 1939 Godel published a proof of the consistency of the Continuum
Hypothesis with the axiom of choice. That proof was closely bound with
the constructive trend. Namely Gédel defined a certain simple method
of constructing new sets from sets already known, and showed that if
the method was iterated an arbitrary transfinite number of times, we
should reach a class of sets (the so-called constructible sets) in which all
axioms of the Set Theory as well as the axiom of choice, the Conti-
nuum Hypothesis and some other hypotheses of the Set Theory are
satisfied’’.
It should be noted, however, that the same K. Gédel writes in his
paper [1] as follows: ,,Therefore one may on good reason suspect that
the role of the continuum problem in set theory will be this, that it will
finally lead to the discovery of new axioms which will make it possible
to disprove Cantor’s conjecture“. By ,,Cantor’s conjecture“ the author
means the continuum hypothesis.
Let us denote by Z the set of all those elements a of the set A, for
which a¢qg-(a) (leaving open the question whether or not Z = 0); thus
we shall have ZC A,, and also ZC A, therefore Z « B. Hence 9(Z) € A;.
Let a) = y(Z) and let us consider two cases:
1° a,eZ. From the definition of the set Z it follows that a) ¢ g(a)).
But, since a) = y(Z) « A,, we have g(a.) = Z. Therefore a, ¢ Z, contrary
to our assumption. .
2° a,¢ Z. Since, in view of a,—(Z), we have Z = g—(a,), it fol-
lows that a,¢gy—(a,), and, in view of a)= y(Z)« A,, and according to
the definition of the set Z, we have a, « Z, again contrary to our assump-
tion. Thus the assumption that B~A,C A leads in any case to a con-
tradiction. Therefore the set B is not equivalent to any subset of the
Seb:A, "q3,6. G.
Thus we have proved Theorem 1.
From Theorem 1 we can immediately infer the following
CoROLLARY. The set of all subsets of real numbers is of greater power
than the continuum.
It will be noted that some authors, .¢e. g. N. Lusin, do not consider
the set of all subsets of real numbers as adequately defined 1), since we
do not know any regular procedure of obtaining all sets of real numbers.
When we speak of it — says Lusin — we actually only imagine various
methods which can be conceived to define very complicated sets, but
we have no complete definition which would allow us to comprise them
all. But as regards man’s capacity for creating methods of defining we
enter the field of subjectivity. Thus the set of all linear sets is different
for different mathematicians and therefore it cannot be regarded as
a mathematical concept to be introduced in reasonings.
Similarly, R. Baire thinks it does not follow that being given a cer-
tain infinite set we are eo ipso given the set of all its subsets (Menger [1],
p., o14):
E. Kamke, on the other hand, points out that although from the
logical point of view nothing can be said against the rejection of the set
of all subsets of a set, still it is mathematics we want to deal with and
our object is not to take the most limited view but to give the widest
possible range to mathematical considerations. Because of the extra-
ordinary fertility of the theory of sets of points, dealing in particular
with the continuum and its parts, those sets have become so familiar
to us that any decree forbidding us to consider them in future is not to
be thought of. Therefore the set of all subsets of a given set should be
regarded as well-defined *).
The concept of definable linear sets has been dealt with in an exten-
sive paper by A. Tarski [6], p. 210-233.
1) The article of Kamke [1], § 13, footnote 88, gives further bibliography on this
problem (Borel, Brouwer, Holder, Weyl).
86 V. Comparing the power of sets
s= DA
AEeZ
s= 4,
A€Z
EXERCISE. Prove that for every set A there exists a set F(A) of sets, such that
1° A « F(A), 2° if Xe F(A), then U(X) « F(A) and 3° if @ is an arbitrary set such that
Ae«@® and formula X «® always implies formula U(X) «@, then F(A)c@.
Proof. It is easy to verify that
AXIOM OF CHOICE
2. The axiom of choice for a finite set of sets. From the truth of the
axiom of choice for the case where the set Z consists of only one set A
it follows immediately by induction that it is true for every finite set
of sets. (H.g. let us suppose that A, and A, are two non-empty sets
without common elements: thus there exists an object p, which is an
1) It is an axiom stating that for every object we can form a set consisting of that
object only (the author’s note).
§ 2. The axiom of choice for a finite set of sets 93
element of the set A,, and an object p, which is an element of the set A,;
the set {p,, p.} will of course contain one and only one element from each
of the sets A, and A,). We shall not dwell upon this case any longer, since
even. those authors who reject the axiom of choice in its general form,
do not question it in the case of a finite number of sets.
2a. The axiom of choice for an infinite sequence of sets. The next
simplest case is the one where the set Z consists of an infinite sequence
of sets
(2a.1) Tae Wap! a po
Are we able to choose one element from each of these sets? It is
impossible to take this question too literally: actually in order to choose
one element from each of infinitely many sets we should have to perform
infinitely many operations, for which human life is too short. But the
problem which is presented here can be understood as a search for a law
according to which to each of our sets would correspond a certain element
of that set'). In other words, we should define a function f(n) of a na-
tural variable such that f(n)« A, for every natural number n. Obviously
this question is closely connected with the following one: “Can we estab-
lish a law according to which to every given non-empty set A would
correspond a certain element of that set?”
We are unable to establish such a law, not only in a general way
for every non-empty set, but also for sets forming certain given sets
of sets, e. g. for all non-empty sets of real numbers.
1) Jt will be observed that as early as 1890 G. Peano [1], p. 210, wrote: ,,Mais
on ne peut pas appliquer une infinité de fois une loi arbitraire avec laquelle & une
classe on a fait correspondre un individu de cette classe“.
94 VI. Axiom of choice
1) This criticism, by the way, concerns only the possibility of choices (a priori)
and not the axiom of choice itself.
2) Cf. also Fraenkel [3], p. 86, who points out that such an attitude is in agreement
with the views of Hilbert.
96 VI. Axiom of choice
EXERCISES. 1. Prove with the aid of the axiom of choice that in every infinite
set A of real numbers there exists a denumerable subset # dense in the set A, 4%. e. every
interval containing inside at least one element of the set A contains also at least one
element of the set H.
Proof. According to the general principle of choice there exists a function 7 as-
sociating with each non-empty subset X of the set «4 a certain element t(X) of the set A.
Thus, in particular, for every interval d with rational end-points such that dd #0
t(dA) will be an element of the set A. The set H of all the elements t(dA) where d is
1) H. Lebesgue [4] writes: ,,J’ai dit ailleurs (II, 259) que je ne comprends pas ces
questions tout 4 fait comme M. Sierpinski, mais je suis plainement d’accord avec lui
sur le fait qu’il faut attacher une bien autre importance aux exemples qui n’utilisent
pas Vaxiome de Zermelo qu’a ceux qui lutilisent“.
§ 4. General principle of choice 97
an interval with rational end-points such that dA 40 will obviously satisfy the re-
quired, conditions.
2. Prove with the aid of the axiom of choice that the set F of all real functions,
defined in a given non-empty set A of real numbers increasing in the set A is of the
power of the continuum.
Proof. It suffices of course to assume that the set A is infinite. Let f(x) denote
a function belonging to the set F. It follows from III.3, Exercise 2, that the set A of
all the points of the set A at which the function f(x) is not continuous in the set A is
finite or effectively denumerable. Therefore we are able to, arrange all elements of the set As
in an infinite sequence a,, a2, d3,... (repeating, if necessary, the same term infinitely
many times if the set A, is finite). From Exercise 2 it follows (with the aid of the axiom
of choice) that there exists an infinite sequence b,, b,,..., dense in the set A. Now let
us associate with each function f belonging to the set # the infinite sequence
In this way different sequences will be associated with different functions belong-
ing to the set F’. Indeed, if the same sequence corresponded to the function f and to the
function g belonging to F’, then the function g would be non-continuous in the set A
at the same points as the function f and would assume at those points the same values
as the function f. We should also have g(b,) = f(0,) for n = 1, 2,... and the function g
would be equal to the function f at each point of a certain set dense in the set A, whence
it would be equal to the function f at all those points of the A at which the function f
(and hence also the function g) is continuous. Therefore, we should have f(x) = g(x)
for xe A.
Since there is a continuum of infinite sequences of real numbers (see IV.11), it
follows hence that the set & is of power <c (by ¢ we shall denote the power of the
continuum). On the other hand, the set F is of power >c since to this set belongs every
function f(x) defined for x « A by the formula f(z) = «+ ¢ where ¢ is an arbitrary real
number independent of x. Thus we have F =, qeaeside ;
Observe further that it would be easy to prove that the set of all increasing real
functions of a variable is effectively of the power of the continuum. For this purpose
we need only take the effectively denumerable sequence formed of all rational numbers
as the sequence ),, do, ...
5. Axiom of choice for finite sets. For a given natural number n let
us denote, after A. Mostowski, by [n] the following particular case of the
general principle of choice G:
[n] For every set Z of n-element sets A there exists a correspondence
such that to each set A belonging to Z corresponds a certain element of it, t(A).
Just as easily as we have proved above the equivalence of Theorem G
and the axiom of choice we can prove the equivalence of Theorem [n]
and the following particular case of the axiom of choice:
For every set Z of n-element disjoint sets there exists a set having one
element from each of the sets belonging to Z.
However, it is not for every set Z of n-element disjoint sets that
we are able to indicate a set having one element from each of the sets
Cardinal and ordinal numbers 7
98 VI. Axiom of choice
belonging to Z. For let 7 denote the set of all real functions defined
for 0 <«% <1 and assuming only the values 0 or 1. Two functions f(x)
and g(x) of the set 7 will be assigned to the same class if there exists
an integer k such that
It can be proved that if we could choose one and only one element from
each of the considered classes it would be possible to prove the existence
of linear sets non-measurable in the sense of Lebesgue without the aid of
the axiom of choice (Sierpinski [50]). Thus we cannot choose one and
only one element from each of the two-element sets forming a certain
set of sets. Still, it will be observed that we cannot give an effective
example of any pair of two functions of a real variable from which we
should not be able to choose one of those two functions.
(I have been asked whether such a pair of functions could not be
defined in the following way: the pair {sinw, cosx} if the last theorem of
Fermat is true, and the pair {tga, ctga} if the last theorem of Fermat is
false. From such a pair we are able to choose one function, eé. g. sing if
the theorem of Fermat is true, and tga if it is false).
THEOREM 1. [2]—[4]?).
Proof. Let Z denote any set of four-element sets, A = {a,, a, 3, 4}
denote any set belonging to Z, and A* — the set of all two-element
(not ordered) subsets of the set A:
A* = {{a1, Az}, {Ay, Ms} {M,, Hg}, {2,3}, 1d2, Aas, (45, a,}} :
(i
p
7%
100 VI. Axiom of choice
different p-element subsets. For each element of the set A let us denote
by q(a) the number of all those different p-element subsets X of the set A
for which t(X) =a. We shall have of course
DS: q(4) ,
acA
that subset. Thus we shall have 1(A) = 9(g$(A)) «A, which implies the
truth of the proposition [n], and therefore, in view of the truth of proposi-
tion [k] for k <n—1, also the truth of theorem 7’.
We have thus proved by induction the truth of theorem 17, for every
natural n.
Hence it immediately follows that if proposition [p] is true for every
prime number, then proposition [n] is true for every natural number.
As a further conclusion we shall prove
THEOREM 3. If n is a natural number and if proposition [p] is true
for every prime number p <n, then proposition [k] ts true for every compound
number k < 2n+1.
Proof. If proposition [p] is true for every prime number <n, then,
as we have proved, theorem IT, holds, 7. e. proposition [k] is true for
every natural number k <n. Now let k denote a compound number
<2n+1 and p — the smallest prime divisor of the number k. Thus we
have k= Up where | >2, therefore 2p <lp=k <2n+1, whence 2p <
<2n-+1, therefore 2p < 2n and so p <n; thus, in virtue of our assump-
tion, proposition [p] is true. According to the lemma, there exists a function.
Te associating with every k-element set A (belonging to a given family Z
of sets) a certain non-empty proper subset of that set, Q@ = OCA), There-
fore that subset has 1,2,... or k—1 elements. Since proposition [1] is
true for every natural number / <n, there exists for such J a function fp
associating with each l-element subset of the set A a certain element.
of that subset. If Q= L<n, then let +(A) = f,(Q), and if Ve nm, then,,
in view of @ <k—1l and
A = k < 2n+1, we shall have 1 < A_Q =1 <n,.
and we shall write +(A) = f,(A—Q). Thus we shall always have 1(A) « A..
Therefore proposition [k] is true for compound k < 2n+1, q.e.d.
Thus in particular, for n = 2, we obtain [2]—[4]. For n = 4 Theo-
rem 3 gives ([2]|[3])—([6][8][9]). But from Theorem 2 it follows that.
[6]—([2]-[8]). Therefore we have
THEOREM 4. ([2]+[3]) = [6].
In view of ([2]-[3])—(({6][8][9]) we also have [6]—[8][6]—[9].
For n= 6 it follows from Theorem 3 that [2][3][5] imply [k] for-
every compound k <12, and for n= 10 — that [2][8][5][7] imply [4],
for all compound k < 21.
We shall now prove
THEOREM 5. ([{2]-[5])—[8].
Proof. Let us assume that propositions [2] and [5] are true; in
virtue of Theorem 1 proposition [4] is also true. In view of [2], [5],. [4]
102 VI. Axiom of choice
([2]-[3]-[5]) [15] .
In order to prove this, it suffices to observe that if the set Q@ = g$(A)
had 7 elements, then the set A—Q would have eight of them, and, as
we know, ([{2][3])—[8]. °
We leave it to the reader to prove that
over that f, exists for |] = 44 andl = 40. If we take Q = gS)(A) where Q etl it (Al) —
= f,(Q) for 1= 40, 1 = 44 and for 1 < 36, except the numbers / = 19, 23, 29, 31, and
for those numbers respectively t(A) = f,,_,(A —Q) (we shall then have 63—1 = 44, 40,
34, 32), and for 36 < 1< 62 where 1 # 44 andI1#4 also 1(A) = f,,_,(A—Q), we shall .
always have t(A) «A, which proves that [p] for prime numbers p< 17 implies for-
mula [63].
3. Prove that ([2][3][5][17] [13]) [32] and ([2] [3] [5] [7] [11] [13] [17] [23])
>[64].
We shall also prove the following formula of A. Mostowski?):
/
([3]-[7]) >[9] -
Proof. The only difficulty arises in the case where the set 7 = g)(A) has four
elements. In view of [3] there exists a function y(X) associating with each three-element
subset X of the set A a certain element of that subset, and in view of [7] there exists
a function y(X) associating with each seven-element subset of the set A a certain ele-
ment of that set.
Let P denote the set of all two-element subsets of the set 7. Thus P = oo For
X «P let f(X) =y(A—X) and let § = {f(X)}y-p. If S = 1, let z(A)= 4(8); if &
let t(A) = y(A—S); if S = 3, let t(A) = o(S). If S = 4, then the set # of all the ele-
ments a«S for which there exist more than one set X «P such that f(X)=a ob-
viously has one or two elements (since f is then a function defined in a set of six elements
and assuming’four different values). If R = 1, let r(A) =«(R), and if R = 2, let r(A) =
=y(A—f).
If S = 5, then there exists one and only one element ae¢«S for which there are
more than one element X <P such that f(X) =a. Then let t(A) =a.
If S = 6, let t(A) = g(A—S).
Thus we have avoided the difficulty of the case 7 = 4. In case of T = 5 the set
A~—T would have four elements and we should deal with it just as we have done with
the set 7. In case of 7 = 6, 7 or 8, we should take respectively (A) = p(A—T), p(T)
or ((A—T).
Thus we have proved that [3] [7]—[9].
However, we do not know whether [3] [5][13]~[15] (Mostowski [3], p. 168).
Moreover let us observe the following. Let n denote an arbitrary positive integer.
From the assumption that Theorem [n] is true we can deduce, without the aid of the
axiom of choice, the existence of linear sets non-measurable in the sense of Lebesgue ”).
In connection with the axiom of choice for finite sets let it be ob-
served that A. Fraenkel has dealt with two problems: 1° whether the
axiom of choice for sets of two-element sets is more restricted than the
general principle of choice, and 2° whether the axiom of choice for finite
sets is more restricted than the general principle of choice. According to
A. Mostowski both works of Fraenkel quoted here have gaps, but their
general idea is correct. Mostowski makes use of it giving the correct solu-
tion of problem 2° in his paper [6] (p. 201-252), where he investigates
1) See Mostowski [3], p. 164, Theorem IX, and Sierpinski [67], p. 98.
2) See Sierpinski [50], p. 36. For m = 2 see also Sierpinski [13], p. 177.
104 VI. Axiom of choice
the relation between the principle of ordering and the axiom of choice,
the solution of problem 1° being a corollary to paper [6] of Mostowski.
THEOREM 6. Let n denote a given natural number.
Theorem T,,, stating that there exists a function f,(Z) associating with
every set Z of n elements a certain ordered set f,(Z) of the same elements,
is equivalent to the theorem stating that proposition [k] is true for every na-
tural number k <n.
Proof. Suppose that Theorem 7, is true; thus there exists a func-
tion f,. Let k denote a natural number <n. Let us associate with every
set Z of k elements a set g,(Z) of n elements (obviously different) which
are the elements of the set Z and ordered pairs (Z,7) where j is a natural
number <n—k. Thus the set f,(g,(Z)) is an ordered set whose elements
are all the elements of the set g,(Z). The set Z- fulgu( Z)) will be of course
an ordered set of the same elements as the set Z. Denoting in a general
way the first element of the ordered (finite) set A by p(A) and writing
t(Z)= p[Z-fa(gn(Z))|, we shall have a function associating with every
k-element set a certain element of that set. Thus we have Broyes that
T,—>(k] for k <n.
Now suppose that proposition [k] is true for natural k <n. Thus
for k <n there exists a function h,(A), associating with every k-element
set A a certain element h,(A) of that set. Now let Z denote an arbitrary
set of n elements. Let us denote by /,(Z) the sequence of n terms
Let [s,] denote the theorem stating that there exists a function
assigning to every denumerable set a certain element of that set. [s,] imp-
lies that there exists a function f assigning to every denumerable set
a well-ordered set (XIII.1) composed of the same elements as the given
denumerable set; the proof, however, is not easy — it is a particular
case of Zermelo’s theorem for denumerable sets. We are not able, howe-
ver, to deduce from [s,] that there exists a function assigning to every
denumerable set an infinite sequence composed of all elements of that
set. We can deduce the existence of such a function from [c], where [c]
is the theorem stating that there exists a correspondence associating
with every set of the power of the continuum a certain element belong-
ing to that set.
Indeed, let us assign to every denumerable set Z the set s(Z) of all
infinite sequences composed of all elements of the set Z. Thus if
Z = {A,, 4, ...}, then the set s(Z) is the set of all infinite sequences
An.) Angy «+, Where 1, %,... 18 any sequence composed of all natural
numbers.
Therefore the set s(Z) will be of the power of the continuum. From
[No] it follows that there exists a function g assigning to every set of the
power of the continuum a certain element of that set. Thus the function
1A) = g(s(Z)) assigns to every denumerable set Z a certain infinite
sequence, composed of all elements of that set.
Hence we conclude that [c] implies that the sum of a denumerable
infinity of denumerable sets is a denumerable set. We are not able to deduce
this last theorem from [s,]. We even cannot deduce from [s,] the theorem
stating that the sum of a denumerable infinity of denumerable well-
ordered sets is a denumerable set.
Ena
a (nae 2h)
¥or if the set X has more than one element, then there exist real numbers a and b such
that ae X, b« X, a < b, and it suffices to take as r a rational number lying between a
and. b. Now let 7, denote the first term of the sequence 7,, 72, ..., such that the set f(X)
of all numbers of the set A that are smaller than r, is a non-empty proper subset of the
set X. The function f/(X) satisfies the required conditions.
Note. We are not able to define the correspondence assigning to every set of
functions of a real variable containing more than one function a certain non-empty proper
subset of that set.
2. Establish the law according to which to every infinite set of real numbers X
corresponds a certain decomposition X — X,+X,1+... of the set X into an infinite
series of disjoint non-empty sets.
Solution. Let f denote the function satisfying the conditions of exercise 1. If X
is an infinite set of real numbers, then the sets {(X) and X —f(X) are both non-empty,
and at least one of them is infinite. If the set f(X) is infinite, let g(X) = f(X), other-
wise let g(X) = X—f(X). Thus, for every infinite set X of real numbers the set g(X)
is an infinite proper subset of the set X. Denoting by g” the n-th iteration of the func-
tion, 9g, we put X,— 9g(X)—g¢(X) for w= 2,3,..., and X,=.%. =(X,7 4,4...)
‘obviously we shall have X—g(X)c X,, the sets XY, (n =1,2,...) will be disjoint
and X = X,+X,+... will be the required decomposition of the set X into an infinite
series, defined by X, of disjoint non-empty sets.
It will be observed that we are not able to prove without the aid of the axiom of
choice that every infinite set (of arbitrary elements) is the sum of an infinite series of
‘disjoint non-empty sets (Corollary 2 of VI.7).
3. Define the function assigning to every infinite series A,+A,+... of different
non-empty sets an infinite series B,+B,+... of disjoint non-empty sets such that
A,+A,+...= B,+ B,+...1).
Solution. Let A,, A,,... denote a given infinite sequence of different non-empty
sets. Let us denote by C the set of all real numbers x giving the triadic expansion
co
t= 2 oe Bae
where x, = 0 or 1 forn=1, 2,... (GC, as we know, will be the so-called nowhere dense
‘perfect set of Cantor). For a given number we C denote by P(x) the set of all those
elements » of the set A = A,+A,+... for which with any natural n we have pe A,
Woo eandep (Ait a 1102).
Obviously we shall have P(x)P(a#’)=0 for we 0, w «CC, x Au’. Let Z denote
the set of all non-empty sets P(x), where x « 0. Each of the sets A, is the sum of certain
sets belonging to Z, namely the sum of all those sets P(x) for which we have x, = 1 in
the triadic expansion of the number z.
If the set Z had only a finite number of different sets then the sets A,, A,, ... could
not all be different from one another, since from a finite number of sets we can form only
a finite number of different sums. Thus the set Z is infinite. Therefore the set X of
1) This problem has been solved by K. Kuratowski (see Tarski [2], p. 94— 95).
‘The solution given here is different from that found by Kuratowski (cf. also Sierpiiski
[43], p. 60-61).
2) Cf. the definition of the characteristic function of sequences of sets given by
Marczewski (see Szpilrajn [2], p. 211).
108 VI. Axiom of choice
the sets B, (n= 1, 2,...) will be non-empty, disjoint, and we shall have
foo}
Uva SsRie B
xzeX n=1
In connection with Exercises 3 it will be observed that a set which is the sum of
a non-denumerable infinity of different non-empty sets need not be the sum of a non-
denumerable infinity of disjoint non-empty sets, e.g. the set of all natural numbers
which is the sum of a non-denumerable infinity of the power of the continuum of its
different non-empty subsets.
4. Prove that the axiom of choice is equivalent to the following
THEOREM T. If Z is an arbitrary set and f(p) an arbitrary function (of arbitrary
values), defined for p « Z, then there exists a set Z,c Z such that the function f is invertible
am the set Z, and f(Z,) =f (Z).
Proof. Let us suppose that the axiom of choice is true and let f denote a function
defined in the set Z. The sets f(q) for q«f(Z) form afamily F of disjoint non-empty
sets contained in Z. The axiom of choice implies the existence of a set Z,, containing
one and only one element from each set of the family #, and thus one and only one
element y(q) from each set {~1(q) for q « f(Z). Hence Z, is the set of all the elements 9(q)
where q«f(Z). Now if p,« Z,, p.<« Z,, and p, ~ pz, then it follows from the definition
of the set Z, that there exist elements q,<«f(Z) and q,<f(Z) such that p, = y(q,),
P2 = 9(qz2), Whence in virtue of p, = p,, we find q, ~ q,; according to the definition of
the function » it follows that o(q) «f—*(q:) and ~(q.) = f(q), whence g, = f(p(qm)) =
= f(p;) and g. = f(~(q2)) = f(p2), which in virtue q, # q, gives f(pi) # f(p2). There-.
fore we have f(p,) ~ f(p.) for pi « Z,, Po « Zz, Pi # P2; thus the function f is invertible
in the set Z,. From the definition of the set Z, it follows that Z, c Z (since y(q) « f-"q) c Z
for q«f(Z)). In order to prove that theorem T is true it remains to show that {(Z,) =
= f(Z), i. e. that for every element q<«f(Z) there exists an element p« Z, such that
f(p) = q. Obviously p = ¢(q) is such an element. Thus we have proved that the axiom
of choice implies theorem T.
Now let us suppose that theorem T is true and let # denote an arbitrary family
of disjoint non-empty sets. Let us denote by Z the sum of all sets of the family F. Since
the sets of the family F#' are disjoint, therefore there exists for every element p of the
set Z one and only one set of the family F that contains p; let us denote it by f(p). Thus
the function f(p) (whose values are sets e /) will be defined in the set Z. In virtue of
theorem T there exists a set Z,c Z such that the function f is invertible in the set Z,
and f(Z,) = f(Z). We shall show that the set Z, contains one and only one element
from each,set of the family 7. Let H denote any set of the family 7; thus we shall have
H « Z. Since the sets of the family F are non-empty, there exists an element p « H, and
it follows from the definition of the function f that f(p) = H. Thus, in view of pe Hc Z
we have H «f(Z) = f(Z,) and there exists an element q of the set Z, such that H = f(q),
which, in virtue of the definition of the function f, proves that q« #. Thus the set Z,
contains elements from each set of the family Ff. Finally, if the set Z, contained two
§ 6. Examples 109
different elements g, and qg, from a certain set EH of the family F, then we should have
q ¢€# and q,«H, whence, in virtue of the definition of the function f and in view of Ec F,
we find f(q:) = # and f(q.) = H, therefore f(q,) = f(q.), and m¢Z,, g@¢4,, G# oq,
contrary to the fact that the function f is invertible in the set Z,. Therefore the set Z,
contains one and only one element from each set of the family F. The axiom of choice
is true for the family / of sets. Thus we have proved that theorem T implies the truth
of the axiom of choice.
Therefore the axiom of choice and theorem T are equivalent.
way the set of all real numbers into disjoint pairs, then the set of those
pairs is of the power of the continuum (Sierpinski [53], p. 31).
The proof that this set is of power < the continuum presents no
difficulties. We are also able to prove without the aid of the axiom of
choice that if we decompose the set of all real numbers into disjoint,
non-empty finite subsets, then the set of those subsets is of power < the
continuum, but we cannot prove without the aid of the axiom of choice
that it is of the power of the continuum. .
EXERCISES. 1. Prove without the aid of the axiom of choice that if we de-
compose an arbitrary non-empty set A into disjoint non-empty subsets, then the set
of those subsets is of less power than the set U(A) of all subsets of the set A.
Proof!). Let us suppose that a non-empty set A has been decomposed into
disjoint non-empty sets; let Z denote the set of those subsets. Let us associate with
each subset T of the set Z the sum f(7) of all those subsets of the set A which are ele-
ments of the set 7. Since the (non-empty) sets forming the elements of the set Z are
disjoint, we obviously assign to different subsets of the set A different subsets of the
set Z, whence it immediately follows that U(Z) < U(A), and since, by Theorem 1
of V, we have Z < U(Z), it follows that Z < U(A), q. e. d.
2. Prove without the aid of the axiom of choice that if we decompose the set X
of all real numbers into disjoint non-empty finite subsets, then the set of those subsets.
is of power =e
Proof. The proof follows immediately from the observation that we are able to
define a function associating with every non-empty finite subset of the set X a certain
element of that subset, namely the least number belonging to it. This function maps
the set of all the components into which we have decomposed the set X upon a certain
subset of the set X. Therefore the set of those components is of power <X.
It will be observed, however, that we are unable to prove without the aid of the
axiom of choice that in the case in question the set of all the components is of the power
of the continuum (or that it is of power >X).
3. Prove without the aid of the axiom of choice that if we decompose the set of
all real numbers into a denumerable infinity of components, then at least one of them
must be an infinite set.
Proof. Let us suppose that the set X of all real numbers is the sum of an infinite
series of components X = X,+4X,+... and that each of the sets X, (k = 1, 2,...) is
finite. Therefore the real numbers forming the set X,, arranged according to magnitude,
give a finite sequence a, a, 5006 an (well defined by the number k). Thus we
should have
X= ie5) ) As,
@ il
), ae al, af2 ) Rane Ged,
(2
af3 i eet
and the set X would be denumerable, which, as we know from IV.2, is not true. Thus.
each of the sets X,, X,,... cannot be finite, therefore at least one of them is infinite,
(anerads
Thus we are able to prove without the aid of the axiom of choice that the set of all real
numbers is not the sum of a denumerable infinity of finite sets.
‘Beside the relations < and < for the power of sets, already intro-
duced by Cantor, A. Tarski has introduced the relations <+* and <x
presenting certain analogies to the relations < and < and defined as
follows (Tarski [5], p. 301).
For given sets A and B we write A <+ B if A is the empty set, or
if the set B can be decomposed into disjoint non-empty sets in such
a way that the set of those sets is equivalent to the set A.
With the aid of the axiom of choice it is easy to prove that the rela-
‘tions <* and < are identical. Without the aid of the axiom of choice
we can prove that if for two sets A and B we have A < B, then A <x B,
and that if A < « B, then U(A)< U(B) (which, as can easily be seen,
has been deduced in the proof of the last exercise 1). Cf. also VIII.4.
From Theorem 1 we can easily infer the following
COROLLARY 1. If f(a”) is a single-valued function defined in the set A
then f(A)y<aon.
Proof. For each element y of the set f(A) there exists at least one
element x of the set A such that f(x)= y. For y« f(A), let us denote
by g(y) the set of all elements wx of the set A such that f(x) = y. The sets
g(y), for yef(A), will be non-empty, and obviously we shall have
g(y)g(y’) = 0 for ye f(A), y’ e f(A), y #y’. It is also clear that we shall
have .
A= Sy 9(Y);5
ye f(A)
thus we have a decomposition of the set A into disjoint non-empty
subsets, the set of those subsets 7h equivalent to the set f(A). Thus
in virtue of Theorem 1 we have f(A) = qg. e. d.
Thus we have proved that the a of all values of a single-valued
function is not of greater power than the set of all its arguments.
It should be noted that without the aid of the axiom of choice we
are unable to prove, for every set A, that if f(x) is a single-valued func-
tion defined for w« A, then the set f(A) is not of greater power than
the set A (Sierpinski [31], p. 157).
Further, Theorem 1 has the following
COROLLARY 2. The set of all denumerable sets of real numbers is of
the power of the continuum.
Proof. From Theorem 1 of IV.11 it follows that the set S of all
infinite sequences of real numbers is of the power of the continuum.
The set 7 of all infinite sequences whose terms are different real numbers,
being a subset of the set S, is of power < the continuum. Now let us
112 VI. Axiom of choice
divide all infinite sequences forming the set 7 into classes, including
two sequences in the same class if and only if they differ only in the
order of their terms. Let Z denote the set of the classes obtained in this
way. By Theorem 1 we shall have Z < 7, therefore the set Z is of power <
the continuum.
Now let M denote the set of all denumerable sets of real numbers.
To each set A belonging to M corresponds a certain class of sequences
of real numbers belonging to Z, namely the class of those sequences s
for which A is the set of all the real numbers which are terms of the se-
quence s. Clearly, to different sets belonging to M there will correspond
different classes belonging to 7. Hence the conclusion that M <Z,
therefore the set M is of power < the continuum.
On the other hand, we know from Exercise 1 of IV.8 that the set NV
of all increasing infinite sequences of natural numbers is of the
power of the continuum. Now let us denote by P the set of all de-
numerable sets of natural numbers. Obviously P~WN since every in-
creasing infinite sequence of natural numbers determines a certain denu-
merable set of natural numbers (which are the terms of the sequence in
question), and, conversely, every denumerable set of natural numbers
can be arranged in an increasing infinite sequence of natural numbers (by
ordering the elements of this set according to their increasing magnitudes).
Therefore the set P is of the power of the continuum. But, of course,
PC M, whence it follows that P < M, 7%. e. that the set M is of power >
the continuum. Since we have already proved that the set M is of power
< the continuum, we conclude that the set M is of the power of the con-
tinuum, q. e. d.
However, it will be observed that we are unable to prove that the
set M is effectively of the power of the continuum. Thus we have here
an example of a set which can be proved with the aid of the axiom of
choice to be of the power of the continuum but cannot be proved to be
effectively of the power of the continuum.
Thus with the aid of the axiom of choice we can prove that the
set of all denumerable subsets of a set of the power of the continuum is
of the power of the continuum (but we cannot prove that it is effectively
of the power of the continuum). But, without referring to the axiom
of choice we are able to prove that the set of all finite subsets of a set
‘ of the power of the continuum is of the power of the continuum.
We can even prove that the set of all finite sets of real numbers is
effectively of the power of the continuum.
Indeed, it is easy to establish a (1-1) correspondence between finite
sets of real numbers and increasing finite sequences of real numbers (by
arranging the numbers of the finite set in a sequence according to their
§ 7. Applications of the axiom of choice 113
EXERCISE. Prove with the aid of the axiom of choice that if we [decompose
a set of the power of the continuum into a finite number of subsets, then at least one
of them will be of the power of the continuum.
Now let us suppose that the set A is non-denumerable and that the
set B is finite or denumerable. According to Corollary 3 of III.7, the
set A—B will be non-denumerable, and therefore, according to Theo-
rem 3, it will contain a denumerable subset. By Theorem 6 of III.7 we
shall have 4—B~A. Thus with the aid of the axiom of choice we are
able to prove
THEOREM 7. If the set A is non-denumerable and the set B is finite
or denumerable, then A—B~A.
We are not able to prove Theorems 6 and 7 without the aid of the
axiom of choice.
Now we shall prove with the aid of the axiom of choice
THEOREM 8. If
(7.1) Aig Ag nw Ot as Bnet.
are two infinite sequences of sets such that no two sets belonging to the same
sequence have common elements, and
(7.2) Ay ewey | Ol eas Lad. aces
then putting
(7.3) S=A,+A4,... -and T=B,+B,++...,
we shall have
(7.4) S~T.
In other words: If the corresponding components of two infinite series
of disjoint sets are equivalent, then the swms of those series are also equivalent.
Proof. Let n denote a given natural number. According to the as-
sumptions of our theorem we have A,~B,; thus there exists a (1-1)
correspondence between the elements of the sets A, and B,. Let us denote
by @, the set of all (1-1) correspondences between the elements of the
sets A, and B,; therefore the set ©, is non-empty. Moreover, it is clear
that for natural m and n, where m #1, the sets ©, and ®, have no ele-
ments in common.
In virtue of the axiom of choice for the infinite sequence of sets
@,,,,... there exists an infinite sequence
(7.5) is Pay
such’ that ¢, ¢O,; for w= 1,2) te euch thatvior v=) 2, -.. 48
a (1-1) correspondence between the elements of the sets A, and B,.
Now let s denote a given element of the set S. In view of (7.3) and
the observation that the sets A,, A,,... have no common elements, there
exists one and only one natural number n such that s is an element of
the set A,. Let y(s) = 9,(s); we shall have p(s) « B, and, in view of (7.3),
§ 7. Applications of the axiom of choice G17
Se GE), ti Bl),
&€E nee
then S~T.
Now we shall prove
THEOREM 10. The sum of an infinite series of disjoint non-empty
sets, finite or denumerable, is a denumerable set.
Proof. Let A,(m= 1, 2,...) denote an infinite sequence of disjoint
non-empty sets, finite or denumerable, and let S= A,+A,+... Let n
denote a given natural number. The set A,, being finite or denumerable,
is equivalent to a certain subset B, of the set
1 ee aie rate i Gna (oll nae
In virtue of Theorem 8 the set S= A,+A,+... will be equivalent
to the set 7 = B,+B,+..., which is a certain set of natural numbers.
Hence it follows that the set S is finite or denumerable. But the set S =
= (,+C,+..., being the sum of an infinite series of disjoint non-empty
sets, is an infinite set. Thus it is a denumerable set, q. e. d.
It would also be easy to prove
THEOREM 10a. The sum of an infinite series of denumerable sets is
a denumerable set.
We are not able to prove Theorems 10 and 10a without the aid of
the axiom of choice. But here is what H. Lebesgue has written on the
subject (Lebesgue [3], p. 260):
»ll est vrai qu’en apparence on fait parfois des choix sans loi. Mais
cest, ou bien parce qu’il importe peu qu’on ait fait tel choix ou tel autre,
pourvu qu’on en ait fait un, et qu’il est évident qu’on pourrait définir
logiquement un choix, “par un nombre fini de mots“, c’est, par exemple,
le cas du texte; ou bien c’est parceque le choix est imposé. Supposons,
par exemple, qwil s’agisse de prouver que la somme d@’une infinité dénom-
brable d’ensembles dénombrables est un ensemble dénombrable. Soit
ai, a;,... le premier ensemble F,, soit a3, a3, ... le second E,,... On peut
ranger tous les éléments dans la suite dénombrable
1 2 il 2 1
Qi, A15 Az, Ag, Mg, «..
118 VI. Axiom of choice
We shall show that we are able to prove without referring to the axiom
of choice that for every denumerable set A of infinite sequences (of arbitrary,
not necessarily different terms) there exists an infinite sequence containing
each term of each of the sequences forming the set A.
Indeed, since the set A of infinite sequences is denumerable, there
exists an infinite sequence A,, A,,..., containing all the sequences form-
ing the set A. Thus, for every natural number n, A, is a certain infinite
sequence A, = (a, aS, af, ...). For natural m= 2"-'(2k—1), let am =
= a. Obviously the infinite sequence (a,, a2, ...) will contain each term
of each sequence of the set A.
Further, it will be observed that although we have proved without
the aid of the axiom of choice the existence, for every denumerable set 4
of infinite sequences, of a sequence containing each term of each of those
sequences, we are not able to define the function associating with each
denumerable set A of infinite sequences a certain infinite sequence (defined
by the set A), containing each term of each sequence of the set 4. The
sequence (a, a, ...), whose existence we have proved, depends on the
way of arranging all the sequences forming the denumerable set A of
sequences in an infinite sequence A,, Ag, ...
After A. Tarski, every set which is the sum of a denumerable infinity
of denumerable sets will be called the set D,, and every set which is the
sum of a denumerable infinity of sets D, — the set D,. Without the aid
of the axiom of choice we are not able to prove that the set of all real
numbers is not a set D,. But we are able to prove without the aid of the
axiom of choice (although it is by no means easy) that the set of all sets ©
of real numbers is not a set D, (see XV.2). A. Tarski has remarked that-
without the aid of the axiom of choice we are not able to prove the exist-
ence of even one infinite set which is not a set D,. In this connection
A. Tarski asks whether the axiom stating that the set of all real numbers
is a set D,, or the axiom stating that every infinite set is a set D,, would
be proved to be consistent with the usually accepted axioms of the Theory
of Sets (excluding the axiom of choice) (Tarski [2]).
THEOREM 11. The sum of an infinite series of sets of the power of
the continuum tis a set equivalent to the continuum.
Proof. Let C, (n=1, 2,...) denote an infinite sequence of sets of
the power of the continuum, and let S = C,+0C,+... Writing A, = (C,,
and for n= 2,3,..., A, = C,—(C,+0C,+...+C,-1) we shall have S=
= A,+A,+..., the sets A, (n=1,2,...) being disjoint, and A,C C,
foray abe
Let » denote a given natural number. According to Theorem 2a
of IV.4, the set C,, being of the power of the continuum, is equivalent
120 VI. Axiom of choice
to the set D, of all real numbers w such that n—1 <a <n. Thus, since
A, C C,, the set A, will be equivalent to a certain subset B, of the set D,.
According to Theorem 8 the set S= A,+A,+... will be equivalent to
the set 7’= B,+B,+.... But, since B,C D, for n= 1,2,..., we have
TCD,+D,+..., and the set D,+D,+... is obviously the set of all positive
real numbers and zero. Therefore, since S~Z, the set S is equivalent
to a certain set of real numbers, whence we conclude that the set S is
of power <the continuum. On the other hand, in view of S0C,, and
observing that the set C, is of the power of the continuum, we conclude
that the set S is of power > the continuum. Therefore the set S is of the
power of the continuum, which proves the truth of the theorem.
EXERCISE. Prove the following
THEOREM lla. If the set Z is of the power of the continuum and the sets O(&) for
E<Z are of the power of the continuum, then the set
S => Ole)
&€Z
as of the power of the continuum
Proof. The proof of this theorem is a modification of the proof of Theorem 9.
We must base ourselves on Theorem 1 of IV.6 and on the fact that, for € « Z, the set
C(é), being of the power of the continuum, is equivalent to the set of all pairs (é, x)
where x is a real number.
THEOREM 12. If
(7.6) AccAaeusy ONO. obiissDo ose:
are two infinite sequences of sets such that
(7.7) Ae ige, OR. wis ly,
De veers
then
(7.8) Aa CA ASE. DCsBa oe
Proof. From (7.7) and from the axiom of choice it follows, as in the
proof of Theorem 8, that there exists an infinite sequence 9, go, ...,
where y, denotes a function establishing a (1-1) correspondence between
the elements of the sets A, and B,.
Now let a denote an element of the set A,x A,X ... Therefore we have
== Adio. Oop.) WHC. On.< Ap LOr ay = 1),.2.59.... Let
For a given natural n, let us denote by fF, the set of all infinite se-
quences obtained from the sequence 7,, 72, ... by replacing its n-th term
by an arbitrary element of the set C,. In view of (7.12) we shall have of
course
(7.15) Di AG OU ai) eee.
the sets-7,, 7,,.... being. disjomt, and -7,,C BOB. xX ..4 100. 7,15 2
Let Aj= A, and A, = A,—(A,4+4,4+...+An_1) for n= 2,3,...:
the sets A, (n= 1,2,...) will be disjoint and A; CA, for = 1, 2,...,
and also A,+A,+...= Aj+A3+... On the other hand, in view of (7.15)
and A;,C A,, for every natural number n there exists a subset of the
set T,, equivalent to the set A;, whence, in virtue of the axiom of choice,
we conclude that there exists an infinite sequence of sets T7, (n = 1, 2, ...)
such that 7, 07T,~A, (for n=1,2,...). Since both the sets Aj, Ag, ...,
and the sets Tj, 73,... are disjoint, we have according to Theorem 8
Aj+A2+...~+7{+73+..., hence, in view of 4,+A,+...= Aj+Az+... and
eT ics, GE, Tae BOB, 2, wey concludes thate4so5 Asse
= Bios, cus
Thus in order to prove formula (7.11) it suffices to show that the
sets S= A,+A,+... and P= B,xB,x... cannot be equivalent.
Assume that there exists a (1-1) correspondence gy between the
elements of the sets S and P. In view of y(S) = P and S§ = A,+A,+...
we have P = (8S) = 9(A,)+9(A,)+... and, taking P, = 9(A,) for n=
=. 25.05 We: Shall navel As ~P asor Gnesi Oe ane
(7.16) Pees ep oe
Let us consider all those sequences of the set P which belong to P,,
and denote by W,, the set of all the (different) n-th terms of those se-
quences. ae
Let us divide all the sequences belonging to P, into classes of se-
quences having the same n-th term; the set of those classes will of course
be equivalent to the set W,, whence, in virtue of Theorem 1 we conclude
that W, <P,, which in view of P,~A, and (7 10) gives
(7.18) U, = BeaWe a ee
§ 7. Applications of the axiom of choice 123
are non-empty. Thus it follows from the axiom of choice that there exists
an infinite sequence
(7.19) Uys Was ess
also obvious that the sets P,, P.,... are disjoint and contained in the
set BX Bix... Let Ay='A,, and A,= Aj,= Aig for’ 7". Le
sets A, (n= 1, 2,...) will be disjoint and we shall have A,+A,+...=
= Aj+Aj+... Further, for natural n, A; C A, and in view of a < B, =
= Jee there exists a set P/ C P, such that Aj;~P/,. In virtue of Theorem 8
we have A,+A,.+...=eAitAst.. ~Pit+P34+..CP,+P,4+...C Bx
x B,X ..., Whence ae BCBG desl.
EXERCISE. Prove that if A,, A,, ... is an infinite sequence of sets of ever greater
powers and A, ~ 0, then A,+A,+...< A,XA,X...
Proof. Let B, = A,,, for n = 1, 2,... In view of the assumption that Je < Ans
for n = 1, 2, ..., we shall have the itequalities (7.10), whence, in virtue of Theorem 13
Ae eee a a SNS ae
But the set A, not pene empty, the set A,x A; x... is obviously equivalent to
a certain subset of the set A, x A, xA,X ... Therefore we have A, x A,X... < A,X AgX «-
and thus A,+A,+...<A,x A,X ..., q.e.-d.
s=
>) A,
&€Z
and denoting by P the set of all functions f(&) defined for &« Z and such
that f(&) « Bg for EZ, we have 8S <P (Lermelo [2], p. 277) 2).
It will be observed that Theorem 14 immediately implies Theorem 1
of V.5. In order to show this we assume in Theorem 14 that A; = {&}
and B.= {0,1}. We shall then have S = Z, and P will be the set of all
functions defined for x « Z and assuming only the values 0 or 1. The set P
is equivalent to the set T of all subsets of the set Z, since in order to
establish a (1-1) correspondence between the elements of sets 7 and P it
suffices to associate with each subset of the set Z its characteristic func-
tion (1.16, Example 5).
Thus, in virtue of Theorem 14, we have Z < T, which gives Theo-
PELL OL INEO:
Now we shall prove with the aid of the axiom of choice
1) From Theorem 14 (for finite sets Z) it follows that Theorem 13 is also true for
finite sequences of sets. But also in this case we are not able to prove it without the
aid of the axiom of choice.
§ 7. Applications of the axiom of choice 125
Proof. Theorem T follows from Theorem 15. For if f(x) is a function of a real
variable having natural values and if we denote by X the set of all real numbers and
by X,, for n = 1, 2,..., the set of all those real numbers a for which f(x) = n, then we
shall have X = X,+ X,+... Thus according to Theorem 15 there exists such a natural
number m that the set X,, is of the power of the continuum. But, according to the de-
finition of the set X,,, we have f(x) = m for « « X,,, therefore f(x,) = f(a.) for x, « X,,,
m?
With the aid of the axiom of choice D. Konig (the son of J. Kénig)
has proved the following theorem (Konig [4], p. 114):
If there exists between the sets A and B a (m-n) correspondence f (where n
is a natural number >2), then there exists between the elements of the sets A
and Ba(1-1) correspondence g such that, for each element of the set A, g(a) ws
one of the n elements associated with the element a in the correspondence f.
The proof of this theorem, even for n = 2, is not easy; for n = 3 it
is difficult, and even for finite sets it is not trivial.
D. Konig’s theorem is obviously equivalent to the following theorem
on the Cartesian product of sets:
If AXB ts the Cartesian product of two sets and P is a set, contained
in tt, which is. intersected by every parallel to the axis of abscissae or to the
axis of ordinates at n points (where n is a given natural number >1),
then there exists a subset P, of the set P such that every parallel to the
axis of abscissae or to the axis of ordinates intersects the set P, exactly at
one point.
Using the axiom of choice we shall prove this theorem for n = 2.
Let p = (a, b)« P. It follows from the properties of the set P that
the parallel x = a (or y = b) meets the set P exactly at two points, p and q;
let us denote the point q by g(p) (or y(p)). Therefore we shall have
gp) = yw(p) = p for pe P. Every point of the set P determines a se-
quence, infinite in both directions, of points which also belong to P but
are not necessarily different:
1) The set Q(p) might be defined as the product of all sets Q to which the element p
belongs and such that if qg«@, then also y(q) «<Q and y(q) <Q.
128 VI. Axiom of choice
It can easily be proved that the set P satisfies the required conditions.
Thus, using the axiom of choice, we have proved the theorem of
D. Konig for n = 2. It will be observed that in the case of n = 2 K6nig’s
theorem could be proved by using proposition [2] instead of the axiom
of choice, but the proof would be somewhat longer than the one given
here (Sierpinski [40], p. 42-44).
As a corollary to D. K6énig’s theorem, proved (with the aid of the
axiom of choice) for » = 2, we shall now prove
THEOREM 1. If M,N, P,Q are sets such that MN = PQ=0, M~N,
P~Q, and M+N~P+Q, then M~P.
Proof. In view of M~WN there exists a (1-1) correspondence
between the elements of the sets M and N, and in view of P~Q there
exists a (1-1) correspondence py between the elements of the-sets P and Q;
finally, in view of M+N~P+49Q there exists a (1-1) correspondence 0
between the elements of the sets M+N and P+4Q.
; For ae M we shall distinguish four cases:
If d(a)e P and dp(a)« P, let K(a) = {8(a), dy(a)}.
If #(a)« P and Yp(a) <Q, let K(a) = {8(a), yp 0p (a)}.
If &(a)«Q and bp(a)e P, let K(a) = {y Ha), B—(a)}.
If B(a)eQ and dy(a)eQ, let K(a) = ty 8(a), p“(a)}.
It is easy to verify that in each of these four cases the set K(a) is
formed of two different elements of the set P, and that for each element D
of the set P there exist exactly two different elements of the set M such
that b« K(a). Thus between the elements of the sets M and P there
exists a (2-2) correspondence. In virtue of D. Koénig’s theorem for n = 2,
we conclude that M~P, q.e.d. Thus we have proved Theorem 1.
It will be observed that Theorem 1 can be proved without resorting
to the axiom of choice, but such the proof is much longer and more dif-
ficult than the one given here. It is even possible to prove that if we are
given the functions y, y, 3, establishing a (1-1) correspondence between
the sets M and V, P and@d, M+WN and P+Q, then we are able to define
a (1-1) correspondence between the sets M and P (Sierpinski [8], p. 1).
Theorem 1 is a particular case of a more general theorem of F. Bern-
stein (see Bernstein [1], p. 122, Hobson [1], p. 159-162, Konig [2], p. 462):
If n is a natural number and M; and P; («=1, 2,...,) are sets
such that, for 1<i<k<n, we have M;M,;=P,;P,=0, M;~M;,,
Pi~P,, while M,+M,+...4+M,~P,4+P2,+...+P,, then My~P,.
This theorem can easily be deduced from D. K6nig’s theorem (see
Konig [31]) but its first proof without the aid of the axiom of choice
was given by A. Tarski [12] (p. 77-92) as late as 1949. The proof in ques-
tion is long and difficult.
§ 8. The m-to-n correspondence 129
g*
CHAPTER VII
(1.4) S, = My+N 45
134 VII. Cardinal numbers
‘and N,, whose sum is the-set S,, will correspond the disjoint subsets M and WN, whose
sum is the set S. Thus we shall have M~M,, N~N,, whence. M= M,== ih N= V,= n,
and S= M+N, MN = 0, Geresd's
We shall now give various examples of sums of cardinal numbers.
From the definition of the sum of cardinal numbers and from Theorem 13 of III.7
immediately follow the formulae
Notn=ys, £4xfor natural n
and
Not No = No ?
then at least one of the cardinal numbers m,, m,, ..., m, is equal to c. We are not able
to prove this theorem without the aid of the axiom of choice (even for n = 2).
2. Prove without the aid of the axiom of choice that if m is a cardinal number
such that m+, =c, then m=c.
Proof. In view of m+, = c¢ and in virtue of Exercise 1, if X denotes the set of
all real numbers, then there exist sets M and WV such that ees m, Ne n,' UN =0
and X = M+ N, whence M = X—N. Since the set NV is denumerable, it follows from
the formula obtained and from Theorem 3 of IV.3 that Mu = Giese ti Cogn Onl
3. Prove that if m,n,p,q are cardinal numbers such that m+n=p-+q, then
there exist cardinal numbers ),, p.,4,,4., such that p=p,t+p.,, q=GQatq, m=
=Pitha, N= Pet q.-
Proof. Let M and N be two disjoint sets such that M =m, N =n; therefore
we shall have M+N —m-+n. In view of m+n=p-+q and in virtue of Exercise 1,
there exist two disjoint sets P and QY such that P=p, Q =g and M+N=P+9Q.
Hence we have M= MP+MQ, N= NP+NQ, P= MP+NP, Q= MQ+NQ, these
decompositions being disjoint. Taking MP = Pi> NP = p2, WQ =a, NQ =a, We
obviously obtain the cardinal numbers satisfying the required conditions.
3. Product of two cardinal numbers. Let m and n be two cardinal
numbers. Thus there exist sets, MZ and N such that M = m, N=n.
Let P= MXN and p= P. The cardinal number p does not depend on
the choice of the sets M and WN provided only that M=m and V=nt.
Indeed, let us suppose that also the sets M’ and N’ satisfy the condi-
tions I’ =m, N’=n. Therefore we shall have M’~M and N’~N.
But in virtue of Theorem 4 of II.5 these formulae imply the formula
M'xN'’~MxXN. Taking p’= M’XxWN’, we shall have p’=p, q.e. d.
136 VII. Cardinal numbers
mt = NM
(3.2) Mt NE ee ei -
Let M = {1,2,...,m}, let N denote a set such that N= n, and
let P= MXN. Thus we shall have mn= P. But of course P= P,+
+P.+...+Pm, where P, denotes for any given number k= 1, 2,...,m
the set of all pairs (k,) where ne N. We shall have of course P,= 1
for k= 1, 2,..., m, and the sets P, (k=1, 2,...) will be disjoint. Thus
the formula for P gives P= P, ay Fs Ee hey iB =nt+n-+...+n, and since
mn = P, we have formula (3.2), q. e. d.
The concept of the product of cardinal numbers can be generalized
by induction to an arbitrary finite number of factors, and such a product
obviously depends neither on the order of factors nor on the manner of
associating them.
Further it will be observed that if » denotes an arbitrary natural
number, and M,, M,,..., M, are sets such that M, = m, for k = 2 yore sfDs
then, taking P = M,x M,x...x M,, we shall have P= mm... tp
We shall now give various examples of the product of cardinal
numbers.
Formula (3.2) and Theorem 8 of III.7 immediately imply
NoNo = No.
tC OTe ial ee
(Ro ='C;
and Theorem 1a of IV.8 implies »
c=—C.
A+B+O0+ABO = A+B+O+AB+A0+B0.
A+B+0+D+AB0+
A0D+ ABD+ BCD =
= Ae pO Dp AB ACT ADOBC- BD+ OD-_ABROD .
(As regards the generalization of this formula, see my note [49], p. 18-22).
5. Prove that if m and n are two cardinal numbers, then every set P of power mn
is a disjoint sum of sets, the set of its components being of power m and each component
being a set of power n.
Proof. Let P denote a set such that P = mn, and M and N — sets such that
M =m, N =n. From the definition of the product of cardinal numbers it follows that
MxWN=r1mn, therefore MxN~P; thus there exists a function » which associates
in a (1-1) manner to each element (m,n) of the set MxWN a certain element y(m, n)
of the set P. For each element m of the set M we denote by 4A,, the set of all elements
y(m,n) where n « N; obviously we shall have
Pw ete
meM
where the sets A,, for me M, are disjoint and of power n. This decomposition of the
set P satisfies the required conditions.
It should be noted that we are not able to prove the inverse theorem (stating
that if P is a disjoint sum of sets, the set of its components being of power m and each
component being a set of power n, then the set P is of power mn) without resorting to
the axiom of choice (even in the case where m = Ny), n = 2). The proof of that theorem
for arbitrary cardinal numbers m and n, based on the axiom of choice, will be given in
Chapter XVII. For m = &, and finite n, and for m = n= &,, the proof has been given
in VI.7 (Theorem 10).
matt — mm":
Now let m,, m,, t be given cardinal numbers, and M,, M,, N — sets
such that Ny, tlt ye Mn, = iit, N =n. From the definitions of the product
and the power of cardinal numbers it follows that m;m; is the power
of the set My x M2’, and (m,m,)" is the power of the set (M,xM,)%. But
in virtue of Theorem 8a of II.5 we have (M,x M,).~MY~x MY. Hence
follows the equality
(m,m,)" = mpm
for any cardinal numbers m,, m,, 1.
Now let m,un,p be given cardinal numbers, and M, NV, P — sets
~guch that M = m, N= Tt, P= p. From the definitions of the product
and the power of cardinal numbers it follows that (m")? is the power
140 VII. Cardinal numbers
of the set (M¥%)?, and m™ is the power of the set M%*?. But, according to
Theorem 9a of II.5, (M%)P~MN*x?, Hence we have the equality
(ins )® —— Tie.
5. Power of the set of all subsets of a given set. Let 1 denote a given
set of power m, U — the set of all subsets of the set WM. Let P = {0, 1}.
Obviously we shall have U~P™; a (1-1) correspondence between the
elements of the sets U and P™” can be established by associating with
each subset of the set M its characteristic function. Since, according to
the definition of exponentiation, the power of the set P” is 2™ (as P= 2),
the set U is of power 2™. Hence we have
THEOREM 1. The power of the set of all subsets of a set of power m is 2™
Thus, in particular, the power of the set of all subsets of the set
of natural numbers is 2°°. But, as we have proved in IV.8, Exercise 5,
the set of all subsets of the set of all natural numbers is of the power of
the continuum. Hence we have the formula
(5.1) C= 2",
In IV.7 (see the Corollary to Theorem 1) we have proved that if NV
denotes the set of all natural numbers, then the set N% is of the power
of the continuum. Hence, in view of N = s, and in virtue of the defini-
tion of exponentiation, we have the formula
iN
Noo
== Cs
From the formulae ¢ = 2", s 98%) =), and from ,the formula proved
above for the exponentiation of a power it follows that ¢ °= (2%) =
= 2foro gf,
Thus we have the formula:
Coe Bee
EXERCISES. 1. Prove that ¢* = 2°.
Proof. In view of ¢ = 2°° and y,c =¢ we have c° = (2%9)* = 9% — 9°,
2. Prove that xf = 2°.
Proof. In view of ¢ =Ny)¢ and x5? =c we have N& = Nho* = (wX0)6 = cf = 2°.
3. Prove that if M is a set of the power m, then the set of all infinite sequences
whose terms are elements of the set M is of the power m*°.
Proof. The proof follows from the definition of exponentiation and from the
observation that infinite sequences of elements of the set M are in a (1-1) correse
pondence with functions of a natural variable whose values are elements of the set M,
4. Prove (without resorting to the axiom of choice) that a cardinal number m is
finite if and only if
ge 1 oe,
Proof. The necessity of our condition is obvious. It remains to prove its sufficiency.
Assume that m is not a finite cardinal number and let M denote a set of power m; thus M
will be an infinite set. Therefore for every natural number n there exists a subset of
the set M having n different elements. Denote by T,, the set of all subsets of the set M
that are composed of n elements. Thus we shall have 7, 4 0 for n= 1, 2,..., and
T, ~ T,, for different natural numbers k and n. Denoting generally by U(Z) the set
of all subsets of the set Z, we shall have of course 7, c U(M); thus T, « UU(M) for
nm =1,2,..., which proves that the set UU(M) contains a denumerable subset, there-
fore its power will remain unaltered (in virtue of Theorem 4 of III.7) if we join one
element to it. But according to Theorem 1 the power of the set UU (MM) is 22", and the
power of this set enlarged by one element is 22"-+4 1; thus we have 22"-+ 1 = 22", contrary
to our assumption. Therefore the cardinal number m must be finite, which proves the
sufficiency of our condition. Thus the required proof is completed.
Now we are not able to prove without the aid of the axiom of choice that the
cardinal number m is finite if and only if 2"11 4 2™. Neither can we prove without
the aid of the axiom of choice that if the cardinal number m is such that 22"t* 4 22™,
then m is a finite number.
5. Prove without the aid of the axiom of choice that if m is a cardinal number
m
2 p20" = 92 ata 2 on ip 5
q. e. d.
However, we are not able to prove without resorting to the axiom of choice that
if the cardinal number m is not finite and p = g2 then p? = p, but we can prove without
using the axiom of choice that in that case 2p =p. :
142 _VII. Cardinal numbers
6. Prove that the set F of all real functions of a-real variable is of the power 2°,
’ Proof. From the definition of the exponentiation of sets it follows that / = DG
where X is the set of all real numbers. Thus f = F = c*. But, according to Exercise 1,
* — 2°; thus we have f = 2°, q.e. d.
It follows hence that the set of all real functions of a real variable has the same
power as the set of all functions of a real variable, assuming only two different values,
e.g. 0 and I.
7. Prove that a set of power c has 2° different subsets of power c.
Proof. It suffices of course to prove that the set X of all real numbers has ve
different subsets of power c.
Let us denote by 7 the set of all subsets of the set X, and by F the set of all those
subsets of the set X which are of power c. Let X, denote the set of all positive real num-
bers; X, is, as we know, a set of power c; therefore the set F’, of all its different subsets
is of power 2°. Thus the set F, of all sets of the form (X—X,)-+A where A«F, will
be of the same power. But the set X—X,, as the set of all real numbers <0, is of
power c, whence we easily conclude that every set belonging to Ff, is of power c. Thus
we have F,cF'c 7. But F, = 2° and, as we know, f= 2°; therefore F,~7, whence,
according to Theorem 4 of II.6, we conclude that F~7, and finally that TD q.e. d.
8. Prove without the aid of the axiom of choice that 1+ 22%° = 22%
Proof. In virtue of Theorem 4 of III. 7 it suffices to prove that a set of the power 27°
contains a denumerable subset. The set of all sets of real numbers is a set of power 27°
and its denumerable subset is the set of the sets {{1}, {2* 43%, ae
9. Prove without the aid of the axiom of choice that the total number of two-
-element subsets of a set of power 27° is 29,
Proof. As a set of power 22° we take the set U(X) of all subsets of the set X
of all real numbers. Let 7 denote the set of all two-element subsets of the set U(X),
a. e. the set of all sets {4, B} where A and B are different sets of real numbers. The
number of sets {A4, B} where one (and only one) of the sets A and B is empty is the
same as the number of non-empty subsets of the set X, i. e. 2%. Since 2?°°+ 27% —
= 20, 2 — o2%0+1 — 98%, the proof that the set T is of power 22°° is obtained by prov-
ing that the set 7, of all sets {4, B}, where A and B are different non-empty sets of
real numbers, is of power 2. Let f({4, B}) = Ax (B—A)+Bx (A—B). FromI.17,
Exercise 10, it follows that the function f maps the set 7, on a subset of the set U(X x X);
therefore there exists a function h mapping the set 7, on a subset of the set U(X),
since, according to Theorem la of IV.8, we have X x X~X, whence U(X x X)~U(X).
On the other hand, we have X~X — {1}, whence U(X)~U(X— {1}), and the function
g(A) = {1}, A} maps the set U(X — {1}) on a subset of the set 7,. In virtue of the
Cantor-Bernstein Theorem we thus have 7T,~U(X), whence Te alas Ce Ib
10. Let us divide all real functions of a real variable that are not everywhere = 0
into pairs, assigning two functions to the same pair if and only if they differ only
in their signs. Let M denote the set of the pairs obtained in this way. Prove without
the aid of the axiom of choice that M = 2°.
Proof. Let X denote the set of all real numbers. With every set Ac X let us as-
sociate a function f, such that f,(~) = 1 for «¢ A, and f(x) = 2 for we A. Obviously,
for AcX, Bc X,A +B the pairs {f,(a), —f,(x)} and {f,(x), —f,(x)} will be different
from eachother. Hence it follows that the set U(X) is equivalent to a certain
subset of the set MZ. On the other hand, the set M is of course a subset of the set of all
§ 5. Power of the set of all subsets 143
two-element subsets of the set of all real functions of a real variable, therefore, in virtue
of Exercise 6, it is a subset of a set whose power is 22°. Therefore the set M is equivalent
to a certain subset of the set U(X). Thus it follows from the Cantor-Bernstein theorem
that M~U(X), and finally M = 2°, g.e. d. ;
11. Prove that there are 2? different ways of decomposing the set of all points
on a straight line into two disjoint sets congruent by translation.
Proof. Let A denote an arbitrary set of points of the segment J = (0< 2 < l].
Let B = J—A and denote by E(a) the set congruent to the set EH by translation by the
length a along the straight line. Let
+00 +00
9 = » Aen >» B2k-1).
=—0C0O K=— —OO
Obviously, the sets 7, and 7 ,(1) are disjoint and congruent by translation, while
their sum is the whole straight line, different sets T., corresponding to different subsets A
of the interval J. Thus there are 22°° such sets.
CHAPTER VIII
(1.1) - nae
From Theorem 6 of V.5 and Theorem 1 of VI.5 immediately follows
the inequality
(1.2) Sait
In order to deduce the required theorem from this lemma it suffices to denote
by Q an arbitrary set of power n, and by gy — the mapping of the set P on a subset
of the set Q (such a mapping exists in view of n> p), and to take f(x) = y(#) for re H
and f(x) =y (x) for « «Q—E.
The function f maps the set Y on the set V = y(F)+y (Q—B). It follows from
our lemma that MZ5>N2>P and N-~Q, therefore NV = n}),
3. Prove without the aid of the axiom of choice that if MW is an infinite set of
power m and » a natural number, then the set M,, of all m-element subsets of the set W
is of power > 1m.
Proof. If M is an infinite set and » a natural number, then we are able to prove
without the aid of the axiom of choice that there exists a sequence p,, P2, +++. Pa» Pa+1>
composed of n+ 1 different elements of the set M. Let
It can easily be verified that the function f is invertible in the set VM and its values
are elements of the set M,5 thus it maps in a (1-1) manner the set M on a subset of
the set M,, n? whence JM < M,.; therefore. in view of M =m, M, Ss Gis Cs Ol
Note 1. Obviously if » is a natural number >1, then the above theorem holds
also for finite sets MW, having more than n elements.
Note 2. It can be proved with the aid of the axiom of choice that if M is an in-
finite set of power m, then the sets MZ, are of power m for n= 1, 2,...
4. Prove that if m is a cardinal number >, then the cardinal number 2™ is si-
multaneously even and odd, i. e. there exist cardinal numbers n and p such that 2™ =
== 21 = 2p-e 1.
Proof. Let m denote a cardinal number =8,. In virtue of Theorem 1 there exists
a cardinal number p such that m = p-+-§\,. (In case of m = 8, we can assume that p = 0).
Hence, in view of the formula 8)+N8)=8), wé obtain m+, = (P+S,) +8) = pt
+ (No No) =P+N, =m, q.e. d.
It is also obvious that the converse is true: if, for a certain cardinal number m,
we have m+'S, =m, then m>k,.
6. Prove without the aid of the axiom of choice that if m is a cardinal number >c,
then: tt ¢ = mm.
7. Prove without the aid of the axiom of choice that if m and n are cardinal
numbers such that m>c>n, then m+n =m.
8. Prove without the aid of the axiom of choice that if m is a cardinal number
which is not finite, then for any natural n we have 2™ > nm.
Proof. Let m denote a cardinal number which is not finite, and n — a natural
number. Therefore we have m>vn?+n and there exists a cardinal number n> n?2
such that m—n-+». Since, for natural n, we have 2” >n+1 and also oS nt, we
obtain 2™ = 2"*" — 2".2" >n(n+ 1). Thus, in view of n>=n?, we have 2™>nn+
tn > nn+n?= n(n+n) = nm, 7. e. 2° > ni, Gs a GE
However, we are unable to prove without the aid of the axiom of choice that if m
is a cardinal number which is not finite, then 2™ > ,m, or even that 2" >y,.
a | fete and. ie 1 = it 1)
then
(2.2) sis, SM ity
1) A. Tarski [10] (p. 309) stated without giving a proof the following theorem:
it can be proved without the use of the axiom of choice that if m< 2“ and m is not
a finite cardinal number, then 2° < 2™
§ 2. Addition of inequalities 149
=a) as
neZ
The sets X, (n= 1, 2,...) being disjoint and non-empty, to different sets Z of
natural numbers correspond different subsets 7'(Z) of the set X. Thus, denoting by WV
the set of all natural DUETS, we shall have a set U(N), equivalent to a certain subset
of the set U(X), whence U(N
(N) < U(X).3
But in view of V = No, X =m, and by Theo-
rem 1 of VII.5 we have U(N) = 2°, U(X) = 2™. Therefore we have 2°< 2™, q.e. d.
(Sierpinski [43], p. 60).
3. Prove without the aid of the axiom of choice that if, for a cardinal number m,
we have 2™ < 2%, then m < y, (in other words, that a set whose number of subsets 18
less than the continuum must be finite).
Proof. Let us assume that the cardinal number m is not finite. Since, as we know,
m < 2™ and 2™ < 2 by assumption, we obtain m < 2“ and, in virtue of Exercise 2,
we are able to prove without the aid of the axiom of choice that 2°° < 2™, contrary to
our assumption. Thus the cardinal number m must be finite, therefore m < &p, q. e. d.
Now, we cannot prove without the aid of the axiom of choice that if, for a cardinal
number m, we have 2™ > 2%, then m > &,, while without the aid of the continuum-
hypothesis we cannot prove that if 2™ = 2%, then m=X,.
4, Prove {without ;the aid of the axiom of choice that for a cardinal number m
not to be finite it is necessary and sufficient that 8,< 27”.
Proof. The sufficiency of the condition is obvious, {and the proof of its necessity
follows from the observation that in VII.5, Exercise 4, we have proved without the
aid of the axiom of choice that if M is an infinite set, then the set UU(M) contains
a denumerable subset.
Now we are not able to prove without the aid of the axiom of choice that if m is
a cardinal number which is not finite, then §,< 2™.
5. Prove without the aid of the axiom of choice that if, for a cardinal number m,
we have m? >>, then m > No.
Proof. Let M denote a set of power m. In view of m? > &,, the set Mx M (which
is of power m?) contains a denumerable subset. Therefore there exists an infinite sequence
(p,> %)(Kk= 1, 2,...) of different elements of the set MxM. Thus we have p,« M and
q, « M for k = 1, 2,... If each of the infinite sequences p,, po, -.. and q, Qs, ... contained
only a finite number of different terms, then, all terms of the sequence (p,, q:), (Po, Ge), ---
could not be different from one another. Therefore at least one of the sequences ,, po, .-.
and q, q, -.. contains infinitely many different terms, the set of which forms a denumer-
able subset of the set M. Thus we have m= M>k8,. If m=), we should have m? —
=? =p, contrary to our assumption that m? > 8). Therefore m >No, q.e. d.
Now we cannot prove without the aid of the axiom of choice that, for cardinal
numbers, the inequality m? > n® implies the inequality m > n, or that the inequality
m >n implies the inequality m? > n’.
6. Prove without the aid of the axiom of choice that there is no cardinal number m
such that §)< 2 < 2“ (Tarski [5], Theorem 66, and Sierpinski [43]).
Proof. Let us assume that m is a cardinal number such that 8) <2™ < 2%. In
virtue of Exercise 3 we should have m < XN, 7. e. the number m, and therefore also the
number 2™, would be finite, contrary to the assumption that %)< 2™.
150 VIII. Inequalities
7. Prove without the aid of the axiom of choice that there is no cardinal num-
ber m such that 22" — 2%,
e 3 ot y
Proof. Let us assume that m is a cardinal number such that 22> = 2°. Thus the
: 5 ct m mt ¥ 9°
number m is not finite, and we have m < 2” < 22) = 2%°, therefore m < 2°°, whence,
° . * y nt A ° . . . x Tm rit
in virtue of Exercise 2, 2°°< 2", which is impossible since 2°° = 22° > 2”.
8. Prove without the aid of the axicm of choice that in order that a cardinal num-
ber m be not finite it is necessary and sufficient that we have 2°° < 2" (Tarski [5],
Theorem 69).
Proof. The sufficiency of the condition is obvious. In order to prove its necessity
let us assume that m is a cardinal number which is not finite and let M denote a set
of power m. For natural n let us denote by TZ’, the set of all n-element subsets of the
set M and let us associate with every set Z of natural numbers a set
Dien ler,
nNEZ
we shall have of course f(Z)« UU(M), and to different sets Z of natural numbers
would obviously correspond different elements of the set UU(M). Hence we obtain
the formula 2°° < 22", and since according to Exercise 7 the relation of equality can-
Mot-hold, 2°95 < 92". q. end.
9. Prove without the aid of the axiom of choice that if m is a cardinal number
such that 2" >, then 2™ > 2° (i. e. prove that a set which contains at least a denumer-
able aggregate of subsets has at least a continuum of subsets). (Tarski [5], Theorem 68),
Proof. Let m denote a cardinal number such that 2" >, and let MW be a set of
power m. In view of U(M) = 2™ >, there exists an infinite sequence of different non-
empty subsets of the set M: M,, M,,... Let S= M,+M,+...: we shall have M=
= (M—S)4+ M,+ M,+...; all the components of this infinite series of sets will be dif-
ferent and, except perhaps the first, non-empty. Thus the set M is the sum of an infinite
series of different non-empty sets, and therefore, as we have proved without the aid
of the axiom of choice in VI.6, Exercise 3, the set M is also the sum of an infinite series
of disjomt non-empty sets, whence, as in the proof of Exercise 2, we conclude that
os Oe eqne.cd:
10. Prove without the aid of the axiom of choice that if m, and m, are cardinal
numbers >1, then m,-+m, < m,m,.
Proof. If m, > 1 and m, > 1, then, in virtue of Theorem 1 of VIII.1 there exist
cardinal numbers n, and n,, both >0, such that m, = n,+ 1, m, = n,+ 1, whence m,m, —
= (n,+ 1)(n,4+ 1) =n, n,+n,+n,+ 1, and since, in view of n, > 0 and n, > 0, we have
nin, > 1, therefore m,m, > 1+n,+n,+1=m,+m,; thus m,+m,<m,m,, q.e. d.
Note. If one of the numbers m, and m, are =—1, then the theorem proved above
may be false, since for instance 1+ 3 > 1-3. But 1+8,=1-.-
11. Prove without the aid of the axiom of choice that if k is a natural number
and m,,m,,...,m, are cardinal numbers >1, then m,+m,+...+m,<m,m,... m,.
Proof. According to Exercise 9, the theorem which we want to prove is true for
k = 2 and for k = 1 it is obvious. Now let & denote a natural number >2 and let us
suppose that the theorem is true for k—1 cardinal numbers; let m,m,,...,m, be k
given cardinal numbers >1. Thus we shall have m = m,+m,-+...- m,_, < mm,...m,_,
and (since the theorem is true for two cardinal numbers and since m >m,_, > 1)
m-+m,<mm, and m,+m,+...+m,_,+m,<m,m,...m,_;m*%, which proves the
§ 2. Addition of inequalities 151
theorem for k cardinal numbers. Hence, by induction, follows its truth for an arbit-
rary finite number of cardinal numbers.
Note. For finite cardinal numbers m, and m,, both >1, even the inequality
m,+m, < m,m, can easily be proved to hold. This inequality, however, is not neces-
sarily true for cardinal numbers m, and m, which are both > 2 but not both finite, since
for instance 3+ 8, = 3-,. It can even be proved with the aid of the axiom of choice
that if m, and m, are cardinal numbers <0 of which at least one is not finite, then
m,+-m, = m,m,, but the proof is difficult. It can also be proved that this theorem is
equivalent to the axiom of choice (see XVI.2, Exercise 1).
12. Prove without the aid of the axiom of choice that Theorem T,, stating that
for every cardinal number n > &, there exists only one cardinal number p such that
n=,+P, is equivalent to Theorem T,, stating that every cardinal number is either
SNL Ole N oe
Proof. Suppose Theorem T, to be true and let m denote a cardinal number for
which the formula m < 8, does not hold. From Theorem 2 it follows that in any case
M-+ S$) >, and m+, >m; thus if m+, =), we should have S) >, contrary to
our assumption. Therefore n = m+, > 8, and, according to Theorem T,, there exists
only one cardinal number p such that n = 8,+p, and since n= m+, =N,+m, we
must have p =m. But we also have &)+ (S>+m) = (S)>+8))-+m = +m =n, which,
in view of Theorem T,, gives $,.+m=m, whence m>8,. Thus we have T,—T,.
Now suppose that Theorem T, is true, and let n denote a cardinal number such
that N > 8. In virtue of Theorem 1 there exists a cardinal number p such that n =
=N,)+p. Ifp < Ny, we should have n = &)+P <Ny+N, = Ny, Whence n < Np, contrary
to our assumption. Therefore it follows from Theorem T, that p > Np.
Hence it follows that a set of power p contains a denumerable subset, and there-
fore, by Theorem 4 of III.7, it will not change its power if we join with it a denumer-
able set. Thus ps, = p, therefore p = n, which proves that number p is unique. Thus
we have T,—T,.
The equivalence of Theorems T, and T, is thus established.
13. Prove that if the cardinal numbers m,n, p are such that m+n =n andn<p,
then m+p=p.,
Proof. In view of t <p and in virtue of Theorem 3, there exists a cardinal num-
ber g such that.p =n-+q. Hence, in view of m+n=n
Proof. Let m,n, m,, nm, be cardinal numbers satisfying (2.1), and
M,N, M,, N, — sets whose vowers are respectively equal to those
cardinal numbers. In view of (2.1) we can assume that MC NW and M,C N,.
According to the definition of exponentiation we shall have m™ = uM",
n™ = NV, But from the definition of power of sets (1.18) it immediately
follows, in view of MC N, that M@1C N™%, which gives m™ <n™. In case
of m, =n, we have formula (3.1). Now let us assume that m, ~1n,: in
view of (2.1) we have n, > m,, and, by Theorem 1 of VIII.1, there exists
a cardinal number p, > 0 such that n,= m,+),, whence, in virtue of
the properties of the exponentiation of cardinal numbers (VII.4), mn =
=n™n't, whence, by the Corollary to Theorem 3 of VIII.2, n>n™.
The inequalities m™ <1" and n= > n™ give again (in view of Theorem 1
of VIII.2) inequality (3.1).
Thus Theorem 1 is proved.
It will be observed that for the cardinal numbers m, n, p the ine-
quality m<n does not imply the inequality m <n” because, for
example, 2<c but 2°°—c™ (see VII.5). Similarly, for the cardinal
numbers m, p,q, the inequality p< q does not imply the inequality
m?® < m® since, for instance, 1 < 2 but xp = 8}.
Yet with the aid of the axiom of choice we can prove that, for car-
dinal numbers, the inequality m® < n” implies the SNE IEN m<n, and
the inequality m?’ < m* implies the inequality p < q.
§$ 3. Exponentiation of inequalities 153
It should be noted that, as has been proved by A. Tarski [3] (p. 10),
the inequalities m<n and m, <n, for cardinal numbers do not imply
the inequality m™ < n"..,
Here is an elementary proof by A. Tarski (different from the one
given in his work quoted above).
Let N, denote an arbitrary infinite set. We define the sets NV, (n=
= 1,2,:.) Dy induction putting NV,;= U(N,-1)-for n= 1,2,... (where.
U(A) denotes the set of all subsets of the set <A).
Let
(3.2) M = Nyt Net e+. .
In view of NV,= U(N,_1), we have, for natural n, N,1¢« N, and,
by, (5:2), Vey ¢ 1 for = 1.2, . Since, in view of NV,= U (Nn 1) and
by Theorem 1 of V.5, we have ¥, Swi 1, this gives N, = N, oN, =.
whence it follows that the sets Ny, N,, N2,... are all different from ote
another, and since they are elements of the set M, the set M contains
a denumerable subset. For m= M we thus have
(3.3) M>N.
Now let Q denote any subset of the set M; by (3.2) we shall have
(3.5) 2 me.
Now let P denote the set of all sets {{a}, {a, bs}, where ae M and
be M. As we know from 1.17, Exercise 9, we have P~M x M, whence
P =m. On the other hand, if ae M and be M, then by (3.2) we have
with certain natural k and l, ae Nx_1, be Nj_1, whence, in view of the
definition of the sets N,: {a}e Ny, {a,b} « Neyi+, and {{a},
{a, b}he
e Nga, M. Thus we have PC M, whence P<m, and since we
154 VIII. Inequalities
have previously found that P = m’, this gives m? <im, and since, by the
Corollary to Theorem 3 of VIII.2, m? > m, we have
(3.6) NV
(3.8) om mo,
Further, in view of (3.7) and (3.3), we have
2 Ny it = 2
(3.9) Sh alte.
Ne peM
Any
therefore N will be the sum of m non-empty disjoint sets. We have
proved without the aid of the axiom of choice that the formula m <n
implies the formula m <*xn. With the help of the axiom of choice it is
easy to prove that the converse is also true. This immediately follows
from Theorem 1 of VI.7.
Now we can prove without the aid of the axiom of choice that if
m<xn, then 2" < 2". Indeed, from the definition of the relation <x it
follows that if m <n and if NV is a set of power n and M a set of power my
then. there exists a decomposition
v=
>’ A(p)
peM
where A(p)(p « M) are non-empty: disjoint sets. Now for every subset 7
of the set WM let
ee
(C93 ae peT
EXERCISES. 1. Prove without the aid of the axiom of choice that if m is a car-
dinal number such that m<-*,, then M< Wp.
2. Prove without the aid of the axiom of choice that if m,n, p are cardinal num-
bers such that m<x*«n and p 40, then p™ <p" (Tarski [5], Theorem 4).
Proof. Suppose that m<«*n; we can assume of course that m ~ 0. Therefore
if N denotes a set of power n, and M a set of power m, then there exists a decomposition
N=» A(p)
peM
where A(p), for p « M, are non-empty disjoint sets. Let P denote a set of power p and
let f denote an element of the set P™, i. e. a function f(x), defined for « « M, having
values which are elements of the set P. Now denote by (x) a function defined for
a « N as follows. If x « N, then, in view of
N=
)' Alp)
peM
and the disjointness of the sets 4(p) for p « M, there exists one and only one element
pe M such that x « A(p). Let y,(x) = f(p). Clearly we shall have 9; « P%, and to dif-
ferent functions f « P” will correspond different functions g,, 1. e. to different elements
of the set P™ will correspond different elements of the set P™ . Hence it follows that the
power of the set P is < the power of the set PY, i. e. that p™ <p", q.e. d.
3. Prove without the aid of the axiom of choice a theorem of Tarski [5] (p. 303,
Theorem 58) stating that if pq<m-+n, then either p< m or g<*«n.
Proof. In view of pq <m-+n there obviously exist sets P,Q, M,N such that
P=p,Q=q,M=m, N=n, MN =0 and PxQc M+WN. Let us suppose that p< m
is not the case, and let q denote any given element of the set Q. In view of PxQc M+WN,
we have, for p< P, (p,q) « M+N. We denote by A (q) the set of all ordered pairs (p, q)
such that (p,q) « NV.
If we had A(q) = 0, we should have (p,q) « M for p« P and jthe set M would
contain a subset of power p composed of all the elements (p,q) where p« P and we
should have p<m, contrary to our assumption. Thus ‘we have A(q) #0 for qeQ.
But of course the sets A(q) where q«Q are disjoint, and since A(q) c N for q«Q, the
set NV contains the sum
YAW
aeQ
4, Prove without the aid of the axiom of choice a theorem of A. Tarski [5] (Theo-
rem 60) stating that if pq<* (m+n), then p<xm org<xn.
Proof. In view of pq <«(m-+n), there exist sets M, N, P and Q such that M =
=m, N=n, MN=0, P=p, Q=q, and, for peP, geQ, non-empty disjoint sets
A(p,q) such that :
M+N= »' Alp, q).
peP 7€Q
$4. Relation m= *«n 157
Let us suppose that p<*m is not the case, and let q denote a given element of
the set Q. Let
Big) =D) N*A(p.0).
peP
MSTA
A€F
where F is a family of 2" non-empty disjoint sets. The family F has 2?" subsets (sub-
families). For Fc F, let
La) Als
A€Fy
we shall then have 7'(f,)c M for Ff,c F. Since the sets A, forming the family F, are
disjoint and non-empty, we shall obviously have 7(F,) 4 T(f,) for F, cH, F,cF,
i, # F,. Thus the sets 7(F,) where F,cF form a family of 2" different subsets of
the set M. The set M of power m would thus have > 2?" > 2™ different subsets, which
is impossible. ;
It will be observed that in above-mentioned work A. Tarski stated (without
proof) several further theorems concerning the relation <*, pointing out that they
could be proved without the aid of the axiom of choice (e. g. Tarski [5], Theorems 24-28).
Of the theorems which are difficult to prove without the aid of the axiom of choice we
shall mention, as an example, the following one (Tarski [5], Theorem 27):
If m+p
<x 2m, thenp <*m.
CHAPTER IX
Proof. Let A and B be two given sets, and 4~B. Thus there exists
a (1-1) mapping ¢ of the set A on the set B. Therefore for each element xr
of the set A, m(x) is a defined element of the set B.
If, moreover, g(x)«A, then y(@) = (p(x) is a defined element
of the set B. Generally speaking, if, for a given natural number n, ~(«)
and by D, the set of those elements y (of the set B) for which
We shall show that the sets C,, C,, D,, D, satisfy conditions (1.1).
For this purpose it is of course sufficient to show that
and by Q, the set of all elements y (of the set B) such that
(ety news = A or aie for hae 1, Doss Mok. ANd o="(4y) 6 AaB.
The sets*P,,P,, .. and Q;, Q,--. are obviously disjoint.
From formulae (1.4) and the definition of the sets C,, D,, P, and Q,
it immediately follows that
From the definition of the set P, it follows that for every natural
number 7 the function (x7) is determined for x « P,. We shall show that
Indeed, let « denote an element of the set P,; according to the defini-
tion of the set P, we have formulae (1.6)..Let y = g(x); in view of (1.6)
we have yeB—A and g-"(y)=9—"(p(x)) = xe A—B. But gy) =
= p-(p(a)) = vw) e B for k=1,2,...,n—1, since if for a certain
natural p there exists an element g(x), then this element belongs to the
160 IX. Difference of cardinal numbers
set B. Thus, in virtue of the definition of the set Y, we have y « Q,. There-
fore, if we P,, then p(x) «Q,; the function p(x) is defined for # « P, and
we have g"(P,)C Qn-
On the other hand, let y denote an element of the set Q,; according
to the definition of the set Q, we have formulae (1.7). Let 7= -"(y);
in view of (1.7) we shall have w« A—B and 9"(x) = g"(p-"(y)) = ye B—A.
But g(a) = o'(pa"(y)) = oy) <A, for k= 1;2,:..,n—1, since it
for a certain natural q there exists an element y—4(x) which in virtue of
(1.7) occurs for g=1,2,...,n—1, then y-4%(a”)e« A. According to the
definition of the set P,, we thus have w« P,. Therefore, for each element y
of the set Q, there exists an element x of the set P, such that 7 = g-"(y),
whence v(x) = y; in view of we P, we thus have ye g"(P,) for y «Qn,
whence Q, C g"(P,). But, above, we have established the inverse inclusion;
therefore we have formula (1.9), q. e. d.
The function g"(x#) is of course invertible in every set in which "(z)
is a defined element (since the function g(x#) is invertible in the set A);
therefore it is invertible in the set P,.
Let us now define a function f(#) in the set C,, putting
(1.10) TC) GAO) POT) «re PRN i ke ene
Since the sets P,, P;,... are disjoint, the function f(x) will be defined
by formulae (1.10) in the whole set O,, and, in view of (1.9), we shal] have
(1.11) P20, ors Moat ee,
and since, in view of (1.10), the function f(a) is invertible in each of the
sets P, (n=1,2,...), and the sets Q,,Q.,... are disjoint, we conclude,
in view of (1.11) and (1.8) that the function f(x) is invertible in the set C,.
In view of (1.8) and (1.11) we also obtain f(C,) = D,. Therefore the func-
tion f maps in a (1-1) manner the set C, on the set D,, and we have the
formula C,~D,.
According to the definition of the set C, we have C,C A—B, there-
fore C,-AB = 0. If xe C,, then, for natural n, v(x) « A, and since also
y"(x) « B (y"(x) being a defined element), we have g”(#) « AB. Thus, for
natural n,g"(C,) is a defined set CAB. Let
(1.12) AB—
)' 9"(Q,) =R.
n=1
(1.13) AB = R+—(Cz)
+ (C5) + 9(C2) +...
whence
(1.14) C,+AB=
R+C,+ (Cs) +9°(C2) +...
§ 1. Theorem of A. Tarski and F. Bernstein 161
Now, for # « C,+ AB, let g(v)=«@ if we R, and g(x) = ¢(a) if x¢ KR.
Since, in view of (1.12), RC AB, and C,C A—B, we have RC, = 0
and, in view of (1.12), Re(C,) = 0 for n= 1, 2,... Thus, in virtue of the
definition of the function g and in view of (1.14), we have g(x) = y(2)
for r= 0,+ 9(C,)+¢(C,)+..., in view of (1.13) and observing that g(R)=
= RK while g(C,)=9(,) and g(¢(C,)) = ¢*(C,) for n= 1,2,..., for-
mula (1.14) gives
(1.15) g(C,+AB) = B+ (Cz) +9°(C2) + P( C2) +... = AB.
Since the function y is invertible in the set A and in the set C,+
+ y(C,)+¢7(C,)+...CA, we conclude, in view of g(k)=Rk, RC,=0
and Ry"(C,) = 0 for n= 1, 2,,.. and in virtue of (1.14), that the func-
tion g(x) is invertible in the set C,+AB. Thus formula (1.15) proves that
C,+AB~AB.
The proof of the formula D,+AB~AB is analogous.
Formulae (1.5) have thus been proved. Therefore the sets Cj, C,,
D,, D, satisfy conditions (1.1), which proves Theorem 1.
A. Tarski points out 1) that Theorem 1, expressed in the language
of the theory of cardinal numbers, takes the following form:
THEOREM 2. We are able to prove without the aid of the axiom of
choice that 1f m,p, q are cardinal numbers such that m+p = m-+ q, then
there exist cardinal numbers n, p,, 9, such that
p= n+7,, g= n+, m+Pp= M= M+q,.
Proof. Let us suppose that the set A is finite in the sense of Dedekind, and let B
denote a set such that A~B. In virtue of Theorem 1 there exist sets C,, O,, D,, Dz
satisfying conditions (1.1). We shall show that C, = 0. Suppose that C, 4 0. In view
of (1.1) we have 0,+ AB ~AB, and since O,:AB = 0, we obtain AB = A,+ A, where
A,~AB, A,~O,. Hence A = (A—B)+AB= (A—B)+A,+A,, where the sets 4—B,
A,, and, A, are disjoint. But, in view of A,~AB, we have A; = (A—B)+4,~(A—B)+
+AB= A. Therefore A = A,+ A, and A~A,. If we had A,=0, the set A would
be equivalent to its proper subset A;, in contradiction to the assumption that it is finite
in the sense of Dedekind. Therefore A, = 0 and, in view of A,~C,, also OC, = 0, q.e.d.
It can be proved in a similar way that D, = 0. Thus, in view of (1.1), dA—B= (0,,
B—A =D,, therefore A—B~B—A. Thus we have proved that the condition in our
exercise is necessary.
Now let us suppose that the set A is not finite in the sense of Dedekind, 7. e. that
it is infinite in the sense of Dedekind. It follows then (without the aid of the axiom
of choice) from III.8, Exercise 1, that the set A is equivalent to the set B= A- {p},
obtained by joining to the set A an element p not belonging to A. We shall have of course
A-—B=0, B—A = {p}, therefore 4—B non ~B-—A. Therefore if we have A—B ~
~B—A for every set B such that A~B, then the set A must be finite in the sense of
Dedekind. Thus we have proved that the condition of our exercise is sufficient.
2. Prove without the aid of the axiom of choice a theorem of A. Tarski stating
that in order that a cardinal number n satisfy the condition n 4 n+ 1 it is necessary
and sufficient that for any cardinal numbers p and q the formula n+p =—n-+q be
equivalent to the formula p = q (Tarski [12] p. 82).
Proof. Let us suppose that n is a cardinal number such that n 4 n+ 1, and that
for certain cardinal numbers p and q we have n+p = n+q. Let A denote a set of pow-
er n. If the set A were not finite in the sense of Dedekind, then it would be infinite
in the sense of Dedekind, and it follows from Exercise 1 in III.8 that the set A would
be equivalent to the set A, = A+ {p}, obtained by joining to the set A one element p
not belonging to A. But, in view of A =n, we have A, = n+ 1, and, in view of AW~A,,
we should have n =n+1, contrary to our assumption. Thus the set A must be finite
in the sense of Dedekind.
Now let C denote a set of power p, disjoint with respect to the set A. Thus we shall
have A+C =n+p=n-q, therefore A+C—=B+D where Pane’ D = gq, BD 0.
The set A is finite in the sense of Dedekind, therefore, according to Exercise 1, and in
view of A~B, we have 4—B ~ B—A. But it can easily be verified that, in view of
AC=BD=0, we have C=COD+(B—A), D=OD+(A—B), the sets OD, A—B,
B—A being disjoint. Hence, in view of A—B~ B—A, we obtain O~D, therefore
p =q. Thus we have proved that the formula n+ p = n+q implies the formula p = q.
Clearly the converse is also true. Therefore the formulae n+p =—n-+q and p =q are
equivalent.
On the other hand, if for any cardinal numbers p and q the formulae n+ p = n+q
and p = q are equivalent, then we cannot have n = n-+ 1, since it would follow hence
that n4+1—n+ 2, thus 1= 2, which is impossible.
Thus we have proved A. 'Tarski’s Theorem (without the aid of the axiom of choice).
P,—P = f(C,M)+h(g(C,M))~0,M+0,M
= P—P,,
(3.1) S, = (,+0,+P+f(PC,)+9(NN)) .
(3.2) 8, =P+(O,—P)+(0,—P)+f(PC,)
+g(NN,) -
' We have NN,C PO,+NN,C 0,4+NN,, and 0,+NN,~WN,, there-
fore PC,+NN,~NN,; since g(NN,)~NN,, we have g(NN,)~PC,+
+NWN,. On the other hand, we have f(PC,)~PC,, and since both the
left-hand and the right-hand sides of the last two formulae are disjoint,
being also disjoint with respect to the set (C,—P)+(C,—P), we have
(3.4) NV,(C,+0,P
+ f(P0,)) =0.
In view of NN,~g(NN,)C
NN, and (3.1) we thus have
By — NN
Cy C3 PCE Cyr (NN Cae Cai ee at (aCe a
whence S,~S,. The sets S, and S, are both of power n+p and, in view
of S,C M, S.C M and the existence of the difference m—(n+p), we have
M —S,~M—S,. Let h denote a (1-1) mapping of the set M—S, on
M—S8S,. We have C,-NN,=0 and, since the function g is invertible
in the set NN,+0,, we find that g(NN,)g(C.)=0 and naturally
g(NN,)g(PC,) = 0. On: the other hand, in view of g(PC,)C NN, and
(3.4), we have g(PC,)[C,+C,+P+f(PC,)] = 0. In view of (3.2) we have
g(PC,):S,= 0, whence g(PC,)C M—8, which proves the existence of
a set, h(g(PC,)) C M—S,. Let
P= M—-N=[(M—(N+N)]+(%—Y),
Q= M—N, =(M—(N+N,)]+(¥—™,).
Therefore, to prove that P~Q it is sufficient to show that V,—N~
~N—N,. Now we have Vj—-N=W,—-NN,, N—-N, => N—NN,. The
168 IX. Difference of cardinal numbers
sets N and N, have n elements each; let k denote the number of elements
of the set NN,: it will be a natural number or 0. Clearly the sets V,—NN,
and NV,—NN, will have n—k elements each, 7. e. they will be equivalent,
whence VN,—N~N—N,, q.e. d.
Thus we have proved Theorem 1.
CoroLLARY. If, for the cardinal numbers m and n, we have, with a cer-
tain natural n, M+n=n-+n, then M=nN.
Proof. Let p= m+n; we shall have also p= n-+vn. If m is a finite
cardinal number >0, then, p= m+n > n, which is of course true also in
the case of m not being a finite number (since » is a natural number).
Thus we always have p > n, and, in virtue of Theorem 1, the difference
p—n exists. But in view of p=m+n=n-+n we have p—n=m and
p—n=n, whence m—=—n, q.e. d.
Thus we have proved our Corollary (without the aid of the axiom
of choice).
It will be observed that in our Corollary the number n cannot be
replaced by s, since, for instance, 1+) = 2-++,), but we are able to
prove without the aid of the axiom of choice that if m and n are cardinal
numbers >s, and if m+%,—=n+s,, then m=n (since as we have
proved in Exercise 5, VIII.1, if m >x,, then m+, =m). On the other
hand, we are not able to prove without the aid of the axiom of choice
that if m and n are cardinal numbers such that m >, and n is not a finite
number, and if m+s,=n-+s,, then m=n. For it would easily follow
from such a theorem, without the aid of the axiom of choice, that every
cardinal number n which is not finite is >s,. (Indeed, taking n+», = m,
we should have m+s,=m->s, and n+s,=tm-+s8,, whence if our
theorem were true, then since n is not a finite number and m>x,, we
should have n= m= M+, > >).
EXERCISE. Prove without the aid of the axiom of choice that if m and n are
cardinal numbers such that n An-+1 and m>nNn, then the difference m—n exists.
Proof. In view of m > n there exists a cardinal number p ¥ 0, such that m = n+p.
Now let q denote any cardinal number such that m=n-+q. According to Exercise 2
(section 1) it follows hence without the aid of the axiom of choice that p = q. Thus
there exists one and only one cardinal number p such that m= n-+p. This proves the
existence of the difference m—n = p.
5. Proof of the formula 2™—m = 2™ for m >, without the aid of the
axiom of choice. We shall prove
THEOREM 1. We are able to prove without the aid of the axiom of choice
that if m is a cardinal number >, then
2" —m = 2™
(Tarski [5], p. 307, Theorem 56, without proof).
§ 5. Proof of the formula 2% —m = 2™ 169
(5.1) n(X)
= X+ Ei[we My, x ¢ f(g(X))]
(F, [W (a)] denotes the set of all elements x satisfying the condition W(« Me
If, for a certain set X C 8, we had h(X) « M,, then, in view 7 Me
= f(M), there exists an element ¢ of the set M such that h(X) = f(t).
But, in view of te M and M = g(M,), there exists an eee ce M,
such that t = g(x). Therefore we should have h(X) = f(g (@)) where we M,.
In view of «e M,, Xe S and M,S=0, we have «¢X and if we h(X)
formula (5.1) gives « ¢ f(g(#)) = h(X), which gives a contradiction; now,
if x¢ h(X), then in view of we M, formula (5.1) gives x « f(g(x)) = h(X),
which again gives a contradiction. Thus the assumption that for a certain
X C S we have h(X) « VM, leads in any case to a contradiction. This proves
that h(X)¢ M for X C 8S, and since, in view of (5.1), we have h(X)« T=
= M,+P, h(X)« P for XC 8. Now since in view of M,8 = 0, formula
(5.1) implies immediately that h(X,) ~ h(X,) for X,C 8, X,C 8, X, ~ X,,
the function h(X) is invertible in the set U(S) (of all subsets of the set
and its values are elements of the set P. Hence it follows that U(8) < wee
1. e. that p > 2°, q. e. d. Thus we have proved our lemma without the
aid of the axiom of choice.
Proof of Theorem 1. Let m denote a cardinal number >ys,. As
we know, 2™ > m; thus there exists a cardinal number p such that 2" =
= m-+p. In view of m>ys, we have, as we know, m+1—m and thus
2 he ee em, and since 2™-m2> 2", we obtam: 2" =—
=2™1m. Thus we have m+p = m-+2™. In virtue of Theorem 2 of IX.1,
there exist cardinal numbers n, p,, q, such that
S’= Myj+Mo+...
According to Theorem 8 of VI.7, we shall thus have S’~S which,
by (1.4), gives S’ =s.
Thus we have proved that to every infinite sequence (1.1) of cardinal
numbers corresponds a well-defined cardinal number $ such that if (1.2)
is an arbitrary infinite sequence of disjoint sets satisfying conditions (1.3),
then $ is the power of the sum of all sets of the sequence (1.2). Such a car-
dinal number ¢ is called the sum of an infinite series of cardinal numbers
m,t+m,+... and written
From the fact that the sum of an infinite series of sets is independent
of the order of the components and of the manner of joining them im-
mediately follows the commutativity and associativity of the sum of an
infinite series of cardinal numbers. Thus we can say that
Hvery infinite series of cardinal numbers has a well-defined sum, which
is a cardinal number, and this sum does not depend on the order of the com-
ponents or on the manner of joining them.
From the definition of the sum of an infinite series of cardinal num-
bers it immediately follows that if M,, M,.,... is an infinite sequence of
disjointed sets, then writing
S= M,4+WM,+4+...,
\
we shall have
6S
> 307 -.
EXERCISE. Prove that if 8 is a set such that g= m,+m,+..., then there exists
an infinite sequence M,, M,,... of disjoint sets Such that u, SS tt WOM We Nh P45 cons
and S = M,+ M,+ ...
Proof. Suppose that S is a set such that Ts m,+m,-+... For an infinite sequence
of cardinal numbers m,, m,, ... there exists, as we know, an infinite sequence of disjoint
sets M’? (n=1, 2,...) such that Mw Sith, AONE Wy) Se Pa, coe) Aner SS UE US
we shall have §’= m,+m,-+-..., and therefore S’ = S, which proves that
Thus 8’~S.
there exists a (1-1) correspondence gm between the elements of the sets S’ and 8. Writ-
ing 9(M/) = M, for n= 1, 2,..., we shall obviously obtain sets satisfying the condi-
tions of our exercise.
Tg fit ret
de oe
For let NV, (n= 1, 2, ...) denote an infinite sequence of disjoint sets
suchethat: N,)=sst, tore 4542... .Sinee myo; dorms by 2.8) there
exists for every natural number » a set VM, such that M, C NV, and M, —
=m,. Let S= M,+WM,+..., S’= N,+N.+... Of course we shall have
SCS) whence S =< 8/,-and since 8 =m, m,+...) 8’ =n, fn, +..., we
have m,+m,.+... <<m,+1,+..., q.e. d.
We can also easily prove that for any cardinal numbers the ine-
quality
(2 L) ie ite, Migs ae
their easy proof is left to the reader. Thus infinite series of cardinal num-
bers behave in the same wayias absolutely convergent series of real numbers.
M+Ng+...,
Syed = WN Na et et + on as
therefore
(3.1) ; So=%44+%7)4+.
as well as
in particular for m= sp, in view of the formula x8) = %>, formula (3.3)
gives
No a No +No + No +... 9
and for m= c, in view of the formula s,c =c, formula (3.3) gives
C= Usa
Cop coo ¢
Cait, “Santa,
then there exists at least one natural number » such that m,=c. In
particular, for m, = m, n= 1, 2,..., we obtain in view of formula (3.2),
the conclusion that if m is a cardinal number such that c=s,m, then
m= c. We have obtained this conclusion with the aid of the axiom of
choice, since the definition of the sum of an infinite series of cardinal
numbers makes use of that axiom and, besides, we have used theorem 15
of VI.7, which was proved with the aid of the axiom of choice. Without
the aid of that axiom we cannot prove that if c=s,m then m=c. On
the other hand, we can prove without the aid of the axiom of choice
(although it is difficult) that if » is a natural number and m a cardinal
number such that c= nm, then m= cc (Sierpinski [35], p. 32 and Sier-
pinski [31], p. 154).
From Theorem 2 of V.5 it immediately follows that if m,, mb), ...
is an infinite sequence of increasing cardinal numbers, 7. e. if m, < mM, <...,
then. 1,-- Wt, = 1, for m= 1,2, .
EXERCISES. 1. Prove that if, for cardinal numbers, we have p+gq =m,+m,+
+m,-+..., then there exist infinite sequences of cardinal numbers Pj, p,, ... and q,, Jo,-- »
such that p=p,+P.4+..., G=G+4+.., and m,=—p,+q, for n= 1, 2,...1).
Proof. Since p+q =m,+m,-+..., it follows from the axiom of choice that there
exists an infinite sequence of disjoint sets M,, M,, ... such that M, =m, forn = 1, 2,...
and that there exist disjoint sets P and Q such that P=p, Q =q and P+Q= M,+
+ M+...
Let P,=PM,, Q, = QM, for n=1,2,... Obviously we shall have P= P,+
Pot, O' = O10... =, EO, tor, n= 1,2, .3, the sete Pp kse endl
Qz,-.. being disjoint. Let p, = P,, q, = 2, for n = 1, 2,... It can easily be proved that
the sequences p,,P,,... and q,,q4,,... will satisfy the required conditions.
2. Prove that if m,,m,,... and ,,n,,... are two infinite sequences of cardinal
numbers such that m,=n,+m,,, for n=1,2,..., then my >n+n,+mj+...%).
Proof. Let m,,m,,... and n,,1,,... be two infinite sequences of cardinal numbers
such that m, = n,+- M44 for n= 1, 2,... As we know (see IX.1), in that case there
exist infinite sequences of sets M,, M,,...,.N,, .N,,... and M’, M!,... such that NM, =
iy Mita NM 0, Me NM fort = 1,2, .,. Since My a=
=™..= M! for n= 1, 2,..., we have M,1y~M, for n= 1, 2,..., and it follows from
the axiom of choice that there ex-sts an infinite sequence ¢, (n = 1, 2,...) where g,, is
a mapping of the set M,,, on the set ¢,(M,,,) = M,. Since M, = N,+ M,, we have
M,, = N+ On( M41) for m= 1, 2,..., and thus Mf, ——N7-49¢,(M,), M, = No--@, (M5), -::
Let us write py, = 9 and Y,11 = YnPn4, for n= 1, 2,... Thus we shall have
But y, 1s obviously a mapping of the set W,,, (for n= 1, 2,...) and it is easy to
prove by induction that the sets V1, y,(N2), Po(N3), «+> Ya_y(Na)> y, (M41) are disjoint.
het Ny wy, CN, ) for nm —=.2,3,.-.; we shall have Ni == 11, 1OLN) leu 2,...5) Les SOLS
forming the infinite sequence N,, Vi, N/,... being disjoint and contained in M,. Hence
Moa NENT
EN + and m, = Mf, > 1,4 np t..., q.e. d.
3. Prove that in order that a cardinal number s be the sum of an infinite series
_ of cardinal numbers m,+m,+-... it is necessary and sufficient that the following two
conditions be satisfied:
1) there exists an infinite sequence of cardinal numbers n,, n,, ..., such that n, = s
and nn, =™,+T, 1) TONE Pr aes Ul Bis, goons :
2) if py, P., ... is an infinite sequence of cardinal numbers such that p, = m,-+p, 41
for 715 2)..., then s<p, (Tarski [13]; p. 11, Theorem 1-36).
Proof. Suppose that s=m,+m,+... and let n,=m,+m,,,+M,,.+.. It is
easy to verify that the sequence nj, n,, ... satisfies Condition 1 (since m,+ Ta Mot
+..=m,+(m,.,+ M,, a+ ---) for n= 1, 2,...). It also satisfies Condition 2. Indeed,
if p,,P.,... IS an infinite sequence of cardinal numbers such that p,=m,+p,_, for
m=1,2,..., then, according to Exercise 2, we have p, >m,+m,+...=s, whence
$< p,. Conditions 1 and 2 are thus necessary in order that s = m,+m,+...
On the other hand, suppose that Conditions 1 and 2 are satisfied. By 1 and ac-
cording to Exercise 2, we have s =n, >m,+m,+... Now let p, = ™,,-- 11,4) + +.» for
m=1,2,...; it is easy to verify that we shall have p, = m,+ Pat LOTR eee
therefore, by 2, we shall have s < p,, 7. e. 8 << m,+m,-+... Thus s = m,+m,-+ ... Condi-
tions 1 and 2 are therefore necessary in order that s = m,+m,+ ...
From Exercise 3 it follows by Theorem 3 of VIII.2 that the concept of the sum
of an infinite series of cardinal numbers can be reduced (with the help of the concept
of the infinite sequence) to the concept of the sum of two cardinal numbers.
s= > M,;
é€E
where M;= wm, for ée H and M,M,=0 for é« BH, ne BH, EF.
Now let m and n be two arbitrary cardinal numbers, MW a set of
power m, # a set of power n, and suppose that
s= >’ Mm,
é€E
where M;=m for 颫 H#, and M:M,=0 for eH, ne H, EAN. Now,
for é« H, let us denote by M; the set of all ordered pairs (p, €) where
pe M. We shall have Mj=m= M,, whence M:;~M, for é« E, and
of course M:M; = 0 for feH, nie H, & = 7.
Let
i= ) u:;
é€E
Pe OV x
and
p=.
Further, let My, Mg, ... denote any infinite sequence of sets satisfy-
ing conditions (1.5). Thus we shall have M,~M, for n=1,2,... Let
iP eae SG Ma Cae
because, for instance, every partial product of the infinite product (6.2)
is finite, 7. e. <<), while the value of the infinite product (6.2) is >np.
The value of an infinite product of cardinal numbers need not necessarily
be the least cardinal number that is > each partial product of that
product.
On the other hand, however, it can easily be proved that for m, 4 0
(n=1,2,...) inequality (6.4) always implies inequality (6.3).
Assuming in formula (6.1) m=), or m= c, we obtain, in view of
the formulas s9°= c° = c, the formulae
then
My MgiMg... < My MeN...
For it follows from the axiom of choice, by (6.6), that there exist
infinite sequences of sets M,, M.,... and N,, N.,... such that i ie
N,=nt, and M,C N, forn—1,2,..., whence it follows that M,< Mox
x Msx...C NX N.XN,x... and thus the power m,m,m, of the set
M,x M,x... is < the power nnzgn3, ... of the set N,x N.x ...
§ 6. Properties of infinite products of cardinal numbers 181
De 203a= *:
As regards other properties of infinite products of cardinal numbers,
here are the formulae
Cie
Aer ne) LIL Stay eng
and
WP tP2+ Pst.» — yPimr2em?s... ;
m,M,...M,_,M,_,m,
> m,_,m,_,m,
> 2(k—1)m, > km, > m+mM,+...+M, ,
that P~P’. For that purpose let us define for f « P a function y(f) in the
following way.
Let f denote a given element of the set P, and & a given element of
the set H. Thus we shall have f(&)« M;z, and therefore (f (€)) « M:. Let
g(&) = 8(f(€)). The function g(é) will thus be defined (by the function f)
for €« H and
we shall have g(&) « Mz for « H, whence ge P’. Let g=
= y(f). It is
easy to verify that the function y maps in a (1-1) manner
the set P onthe set P’. Thus we have P~P’, g.e. d.
We have
proved that the cardinal number p defined above depends
only on the set # and on the function h(é), associating with every element
«EH a certain cardinal number A(é) = m;. This number p we call the
product of the cardinal numbers m¢ extended to all elements & of the set EH
and write
Dp = | |We¢ 4
f€E
(8.1) Sn ote:
EXERCISE. Prove that if m is a cardinal number such that m- 2°09 = 22°, then
m = 22%,
Proof. Suppose that m- 2° — 2, Thus we have m- 2°°>m, whence m< 2”,
Therefore it suffices to show that m < 2° is impossible. Suppose that this inequality
holds. Let H denote a set of power 2° and let m,= m,n, = 2% for € « H. Therefore
m, <n, for € « # which, in virtue of Theorem 1, implies formula (8.1). But obviously
from 2 = 2° we get
Dm, =m-2% and =ff n, = (2%) = 2%
écE §<E
(because (2°°)? = 2°°). Hence, in view of (8.1), we should have m- 2% < 2%°, contrary
to our assumption. Thus the assumption that m < 2? leads to a contradiction, q. e. d.
CHAPTER XI
ORDERED SETS
The set of all rational numbers will be ordered if, for the rational
numbers a and b, formula a <b means that a < b. However, it is pos-
sible to order that set differently, e. g. if a <b means that a > b. Another
ordering of the set of all rational numbers will be obtained in the fol-
lowing way. In III.4 we have defined a method of arranging all rational
numbers in an infinite sequence. If a<b means that in the sequence
number a precedes number b, we shall obtain a new ordering of the set
of all rational numbers.
The set of all natural pumbers, besides being ordered according to
increasing magnitudes or according to decreasing magnitudes, can be
ordered by assuming that a<b if and only if number a has smaller
natural divisors than number 0b, and in the case of an equal number of
divisors — if a<b. The reader will easily verify that the relation <
established in this way really orders our set (7. e. that it is connected,
transitive and asymmetrical).
We shall have here, for instance,
eeee Oo 08
The set of all complex numbers may be ordered by assuming that
a+bi<c+di if and only if a<e, or if a=ec and b < d. Thus we shall
have, for instance,
If we have a family F of sets such that one of every two sets belonging
to # is always a subset of the other, then we can order family / putting
first that one of two different sets belonging to it which is a proper
subset of the other.
It is remarkable that every ordered set A easily permits the construc-
tion of such a family F of sets: it suffices to take the family F' of all
sets A(a), for ae A, where A(a) denotes the set composed of the ele-
ment @ and of all elements x of the set A such that 7 <a.
We are not able to order every set (7. e to define in every set a con-
nected, transitive and asymmetrical relation); e. g. we are not able to order
the set of all sets of real numbers, or the set of all real functions of a real
variable. Also, it is difficult to give an effective example of an ordered
set of greater power than the continuum (Corollary to Theorem 3 of XVI.1).
2. Partially ordered sets. If, for the elements of the set A, a relation R
is defined, transitive and such that a non R a for ae« A (or, which is the
same, if the relation R is transitive and asymmetrical), then we say that
the relation BR orders the set A partially, or that the set A is partially ordered
by the relation R.
The set of all real functions of a real variable will be partially ordered
by the relation /Rg denoting that for every real number x we have
f(«) <g(#).
The set of all natural numbers will be partially ordered by the rela-
tion aRb denoting that number a is a divisor of number b, smaller
than b.
188 XI. Ordered sets
EXERCISES. 1. Prove (without the aid of the axiom of choice) that if an effectively
denumerable set A is partially ordered by a relation e, then we are able to define an order
arrangement @, of the set A such that for ae A, b « A formula agb always implies ag,b.
Proof. Let A = {a,, a, ...} denote a given effectively denumerable set, partially
ordered by a relation g. It is sufficient to prove that we are able to define a function f
which maps the set A on such a subset of the set of all rational numbers that for a « A,
be A the formula agb always implies f(a) < f(b).
Let 7,, 72, ... denote the infinite sequence composed, of all rational numbers (which
we are able to construct). Let f(a,) = 7,. Now let » denote a given natural number >1
and suppose that we have already defined numbers f(a,) for i < and that fork <n
and J < » the formula a, ga, always implies f(a,) < f(a,). We shall distinguish three cases:
1) In the sequence a,, a,,...,d,_, there are no such terms a, that a,ea,. In that
case we denote by f(a,) the first term of the sequence 7,,7,,... that is smaller than
any of the numbers f(a), f(d2),..-,f/(@,_,). (From the properties of the set of all ra-
tional numbers it follows that such a term exists).
2) Case 1) does not hold, and in the sequence ay, a2, ...,@,_, there are no such
terms a, that a, oa,. In that case we denote by f(a,) the first term of the sequence 7,
%,..., that is greater than any of the numbers f(a,), f(a), .--, f(Q,_)-
3) Cases 1) and 2) do not hold. We denote by p, the greatest of the numbers f(a,)
corresponding to elements a, such that k < n and a,ea,, and by q, the least of the num-
bers f(a,), corresponding to elements a, such that | < n and a,ea,; by f(a) we denote
the first term of the sequence 7,,7,,... that is different from f(a,), f(a), ---, f(@,_,),
greater than p, and smaller than q,.
In this way the infinite sequence of rational numbers f(a,) (n = 1, 2, ...) is defined
by induction. It remains to prove that if, for natural k and 1, we have a,ea,, then
f(a) < f(a).
Let us suppose, that it is not so. Thus there would exist an arrangement of
the natural numbers & and J such that a,ea, and f(a,) >f(a,). In view of the trre-
flexivity of the relation @ we should have k 41, and from the definition of the infi-
nite sequence f(a,) (n = 1, 2,...) it follows that all its terms are different from one
another. Thus we should have f(a,) # f(a,), and therefore f(a,) > f(a).
Among all the different arrangements of natural numbers k and | where a, oq,
and f(a,) > f(a,) there would exist an arrangement in which the number max(k, 1)
would be least. Let (p,q) denote such an arrangement of natural numbers. Thus we
have a, ea,,f(a,) > f(a,) and, if max(k,l) <= max(p,q), te. if k<n andl<n,
and if a,oa,, then f(a,) < f(a,).
If p > q, then n = p and, since a, ega,, we have a, ga,- From the definition of num-
ber f(a,) it follows that f(a,) < f(a,), which, in view of n=, contradicts the in-
equality f(a,) > f(a,).
If p <q, then qg=™ and, in view of a,ea,, we have a,ea,. From the definition
of number f(a,) it follows that f(a,) > f(a,), which, in view of n= q, again contra-
dicts the inequality f(a,) > f(a,). ’
Thus we have proved that for natural k and / the formula a, ea, implies the formula
f(a) < f(a), q.e. a.
2. Prove the following theorem T: If A is a partially ordered infinite set such that
each of its infinite subsets contains an ordered parr of elements, then there exists an ordered
infinite subset of the set A +).
1) The question whether theorem T is true has been put forward by a young Hun-
garian mathematician, A. Hajnal.
190 XI. Ordered sets
H, = xrEeA
F (weal+ eAF [aex)+ {a}
is infinite. Suppose that for a «A each set H, is finite. We shall define by induction
an infinite sequence a,, da, ... of elements of the set A as follows. Let us suppose that we
have already defined the elements a,, dz, ..., a, of the set A (where n is a given natural
number). The set Ho,-+Ha,-+ ...+Ha, is finite and therefore the set A— (Ha,+ ...+Ha,)
is non-empty; let a, 4a denote any of its elements. The elements a,, a,.,..., defined in
this way by induction (with the aid of the axiom of choice, since we have not indicated
the method of choosing them), are of course all different from one another, since from
the definition of the sets H, it follows that a, « H, for k = 1, 2,... In virtue of the as-
sumption concerning the set A there exists among those elements an ordered pair,
2. e. there exist natural numbers k and 1 > k such that either a, ea, or a,oa,. Since k < I,
we have
£ a i teh
Gihs del. = EF. (wea,|+- F (a, or)-+ {a,},
Ke eA
whence it follows that we can have neither a,ea, nor a,ea,, which gives a contradiction.
Thus we have proved that there exists an element b, of the set A such that the set
H,, = z€A
EF (»eb.]+ x€eA
F tb, er]+ 1}
is infinite. It follows that at least one of the sets
F [webs]
xeA
and ff [b, ea]
xéeA
is infinite. Let us denote by A, the first of these sets that is infinite. As we have proved,
then there exists an element b, « A, such that the set
is infinite. Let us denote by A, the first of these two components that is infinite and
let b, denote an element of the set A such that the set
Since b, « A,_, CcA, Cc A,_,, we have 6,0), or b,0b,, which, the natural numbers k and
1 > k being arbitrary, proves that the infinite set {b,, b,, ...} forms an infinite ordered
subset of the set A. Theorem T is thus proved.
Remark. We are not able to settle the question whether every non-denumerable
set satisfying the conditions of theorem T contains a non-denumerable ordered subset.
Prof. B. Knaster has put forward the question whether if A is a non-denumerable
§ 2. Partially ordered sets oH
partially ordered set such that each of its non-denumerable subsets contains an orde-
red infinite subset, there exists an ordered non-denumerable subset of the set A.
If the set A is finite, then every relation @ between its elements may be defined
by means of a table having as many rows and as many columns as there are elements
in the set A; in the table both the rows and the columns are supplied with headings
in which the elements of the set A are written out, while at the intersection of the row
with the heading a and the column with the heading b we write number | if agb and
number 0 if aeb does not hold.
Thus for instance the table
anes)
S|
Sh so
oro|!lH|
OD
|
Si
ma
—
defines a certain order arrangement of the set {a, b, c}, namely that in which a <b ~e.
a) |) eld
Gis WO AO Th ye al
Sea Ma OPEDaL
er rono 0/0
d}olojijo
defines an order arrangement of the set {a, b,c, d} such that bSaxdwJe.
aloe |-d
Cm COM set
a a i
e |0lo0lo0!o
d|0|o}o]o
Proof. It follows from the table of the relation 9 that we have agb and bee, but -
not ac; thus the relation g is not transitive, and therefore does not partially order
the set {a, b, c}. ‘
5. Calculate the number of partial order arrangements of a two-element set and
write out their tables.
Answer. There are three such partial order arrangements, and their tables are:
a ) ieuinonos Teale oO
set N of all natural numbers; the number of those is >2*°, since by associating with
every real number #, such that 0 < «<1, whose expansion into a proper infinite
dyadie fraction is
co
Te ele Gan
n=1
we shall obviously obtain 2“° different permutations of the set V. Therefore the num-
ber of all the different order arrangements of a denumerable set and, naturally, that of
all its partial order arrangements are >2”. The inequalities obtained, Z < 2% and Z > 2%
give Z = 2", q.e. d.
8. Calculate the number of different order arrangements (or of different partial
order arrangements) of a set equivalent to the continuum.
Solution. Their number is 27°. Indeed, on one hand, in view of the formula
(22%0)2 — 22%, we conclude that their number is <2”°. On the other hand, let X denote
the set of all real numbers, and 7’ — any set of positive real numbers; the set F of all
such sets is known to be of power 22°. For every set 7’ «I, let us denote by U(T) the
set X ordered in such a way that each number of the set T is regarded as preceding each
number of the set X—/F, and the numbers of each of the sets T and X—F are ordered
according to their magnitude. It is easy to verify that in this way to different sets T «Ff
would correspond, different order arrangements of the set X. The set of all the different
order arrangements and, naturally, the set of all the different partial order arrange-
ments of the set X are thus of power >2?° and since we proved before that it is of power
<2°, therefore it is of power 22%, q.e. d.
Remark. With the aid of the axiom of choice it can be proved that the set of all
the different order arrangements of an infinite set of power m is of power 2™.
9. Prove without the aid of the axiom of choice that the set of all the different
No, 2 28
order arrangements of a set of power 2° is of power 2?°”.
ss yz, while x(yz) is an element such that w(yz) sa and «(yz) 3 yz;
therefore, since yz sy and yz 32, we have a(yz) 3 yand «(yz) 3, and
if te and t sy, then t 3a/(yz). Thus if usa, uy, ue, then U32,
whence also Us x (yz). On the other hand, ay igan element such that
vy sx and xy 3y and if ssa and s sy, then s say, while (vy)e is an
element such that (« y)23 vy and (ay)z sz, and if ¢ is an element such
that tS ay and t Sz, then iS ey) 2. Since (ay)z xy and ay 3a and
xy 3y,we have (#y)ze3a, (ay)e3y, (ay)e32, and if wa, w3y
and w sz, then ws xy, whence also w = (wy)z. Thus we have proved
that a(yz) 3a, x a(yz) 3 y, (yz) 2, and that if ua, u sy and u 3%,
then vu 3 a(yz), and algo that (ay)z-Su, (wy)ze SY, (ay)e3z and if w = Le
w3sy, wsz2, then w 3S (ay)z.
Therefore we coun pecan that wu= (w#y)z and w = «#(yz), which gives
(xy)2 3 xv(yz) and x(yz)S (wy)z, whence (ay)z= «(yz), which proves the
truth of the first formula in 3).
The truth of the second formula in 3) might be proved in an ana-
logous way, or it could be observed that between the two formulas there
is a kind of duality. For if a set A partially ordered by a relation R is
a lattice, then the same set ordered by a relation R’ such that for a« A,
be A we have aR’d if and only if b Ra, is obviously also a lattice, and it
(for certain elements a, b, ¢ of the set A) we have c= a-+b in the first
lattice, then in the second we shall have c= a-b, and vice versa, and if
we have ¢ = ab in the first, then we shall have ¢ = a+b and vice versa.
In order to prove the first formula in 4) let us observe that if the
elements « and y belong to a set A constituting a lattice, then, according
to the definition of the product, we have x(#+y) 3 a, and since, in view
of the properties of the sum, we have «3 7+y and1 of course sw, there-
fore, in virtue of the definition of the product, we have a = “(e+y).
Thus «3 a(%+y) 3a, whence w(v7+y)= a, g.e.d. We can prove the
second formula in 4) in an analogous way, or by resorting to the principle
of duality mentioned above.
Thus we have proved that if a partially ordered set A is a lattice Ai
according to the definition given above — let us call it a lattice of the
1-st kind — then by defining in it in the above way the addition and
the multiplication of two elements weobtain a set in which both operations
are uniquely performable and the properties 1) to 4) hold. Let us denote
the set obtained in this way by K, and let K, = f(K,).
Now we call a lattice of the 2nd kind every set A (not necessarily par-
tially ordered) in which the addition and the multiplication of elements
of the set A are defined, uniquely performable in that set and satisfying
conditions 1) to 4). From what we have proved it follows that if K, is
a lattice of the 1-st kind, then K, = f(K,) is a lattice of the 2nd kind.
13*
196 XI. Ordered sets
different from A, and such that f(A7) = K,. The lattice A; would thus also
be a set A, but, being different from the lattice K,, it would be partially
ordered in a different way than A,. Thus there would exist two different
elements a and b of the set A such that a<b in the lattice A, and non
(a<b) in the lattice Aj, or non(a-—b) in the lattice K, and ab in
the lattice Ky;. Suppose that a<b in the lattice K, and non (a <b) in
the lattice Ay. In view of f(K,) = f(Ki) the same element of the set A
would be the product of the elements a and 6 both in the lattice K, and
in the lattice K;. But in the lattice K, the product of the elements a
and b is the element a, since, in view of a <b) in K,, we haveasa,ax3b
and, if sa and #3), then «3a. Now, in the lattice AK; the elementa
is not the product of the elements a and b, since, in view of ab and
non (a<b) in Ky, we cannot have asb in K;. We prove in a similar
way that in case of non (a<b) in K, and a<b in Kj the element a is
not the product of the elements a and b in K,, but it is their product in Kj.
Thus we have proved that the correspondence f between lattices
of the 1-st kind and lattices of the 2-nd kind is (1-1). The examination
of lattices of the 1-st kind is thus reduced to the examination of lattices
of the 2-nd kind. Since lattices of the 2-nd kind are defined in a purely al-
gebraic manner, the examination of partially ordered sets, which we
have called lattices (and then lattices of the 1-st kind), may be algebrized.
As regards conditions 1)-4), we shall observe that they are not
independent of one another. Namely, as already Dedekind (Birkhoff [1],
p. 18) observed, from 4) follows 1). For if, in the second formula of 4),
we replace y by «+ y, then, in virtue of the first formula in 4), we shall
obtain «= a#+ae(a+y)=a+a, and if, in the first formula 4), we re-
place y by xy, then, in virtue of the 2-nd formula in 4), we shall obtain
= Ue cy) = vw.
EXERCISE. Prove that conditions 1), 2), 3) and 4) are equivalent to conditions
1), 2), 3), 5) where 5) denotes the following condition:
5) For we A, ye A the formula cy = a is equivalent to the formula 7+ y = y.
Proof. Suppose that conditions 1), 2), 3) and 4) are satisfied and that wy = «;
by 2) and 4), we then have x+y = vy+y = y+cy = y. And if x+ y = y, then, by 4),
we have sy = «(a#+y) =a. Formulae cy = « and «+y=y are thus equivalent, 7. e.
condition 5) is satisfied.
On the other hand, if condition 5), is satisfied, then the formula w(x%+y) = aw is
equivalent to the formula #+ (#+ y) = «+ y, which is true by 3) and 1), and the for-
mula «+ xy = is equivalent, by 2), to the formula xy+ a= 4, which, by 5), is equi-
valent to the formula (xy)« = xy, which is true by 1) and 2). Thus conditions 1), 2), 3)
and 5) imply condition 4).
we shall say that the set A(o) is similar to the set B(oe’) and write A(e)~
~ B(o’) if there exists a function f, (1-1) mapping the set A on the set B
and such that if a, and a, are any two elements of the set A for which
4,043, then f(a,)0’f(d.). We say that the function f establishes similarity
between the sets A(o) and B(o’).
The relation of similarity between ordered sets is symmetrical, 7. e. if
A(o)~ B(o’), then B(oe’)~ A(e@). Indeed, suppose that A(e) ~ B(o’)
and let f denote a function establishing similarity between the sets A (@)
and B(o’), and f-! — the function inverse with respect to the function f.
Let b, and b, be any two elements of the set B such that b,0’b, and let
a, = f(b), a. = f(b2); we shall have a, « A and a, « A. If we had a,= a,,
we should have b, = f(a) =f(a.)=6,, which is impossible since }, 0’),
and the relation 0’ orders the set B. Therefore a, and a, are two different
elements of the set A, and since the relation @ orders the set A, we must
have either a, oa, or a,0a,. But if we had a,ea,, then, since the function /
establishes similarity between the set A(e@) and the set B(o’), we should
have f(a.) o’'f(a,), 2. €. b,0'b,, Which is impossible in view of b,0’b, and
of the asymmetry of the relation 0’ as the relation ordering the set B.
Therefore we must have a, oa,, 7. e. f(b) o'f(b). Thus we have proved
that if f-! is a (1-1) mapping of the set B on the set A such that if b, and b,
are any two elements of the set B for which b,0'b., then f—*(b,) of—(b.).
Thus the function f— establishes similarity between the set B(o’) and the
set A(e) and therefore we have B(o’)~ A(@), q.e. d.
It would also be easy to prove that the relation of similarity between
ordered sets is transitive, 2. e. that the formulae A(o)~+ B(e’) and B(o’) ~
~ C(0"’) always imply the formula A(e)~ C(e’’). Namely, it could easily
be proved that if the function f establishes similarity between the sets
A(o) and B(o’) while the function g establishes similarity between the
sets B(e’) and C(o’’), then the function g(f) establishes similarity between
the sets A(o) and CO(0’’).
For every ordered set A we have of course A(o0) ~ A(o); as the func-
tion f establishing similarity we can take here the identical mapping of
the set A on itself, 7. e. we can take f(a) = a for ae A. Thus we can say
that the relation of similarity between sets is reflexive.
If @ and o’ are two different relations ordering the same set A, then
A(e)~A(o’) may either hold or not. For example, if A = {1, 2,3,...}
and if, for the elements of the set A, zey means that 2 < y and the rela-
tion we’y means that the sum of the natural divisors of number @ is
smaller than the analogous sum for number y or that, if the sums of the
divisors of numbers # and y are equal, we have w < y, then the set A is ordered
by the relation o in a different way than by the relation 0’, but A(e) ~ A(0’)
will hold. We shall have 102030405o..., and 10’20'30'50'40'70'60'90’...
§ 4. Similarity of sets 199
If, however, xo’’y means that either number x has fewer natural divi-
sors than number y or,in caseof numbers # and y having the same
number of divisors, that » < y, then the sets A(e) and A(o’’) are not
similar.
The similarity of the sets A(o) and A(o’) here considered cannot
be established by means of an identical mapping.
However, it may happen that a non-identical mapping f of a given
ordered set A(e) upon itself has the property of making the formula xoy
equivalent to the formula f(x)oef(y); for example, in the set A of all inte-
gers, voy denotes that « < y and f(#) = «+5. But, if we wrote g(x) = —2,
then the formula vey would not imply the formula g(x) eg(y), since, for
instance, 1 <2 but g(1) = —1 > —2= g(2).
EXERCISES. 1. Prove that the set of all real numbers w such that —1 <a <1
ordered according to their magnitude is similar to the set of all real numbers ordered
according to their magnitude, and that the function
establishes the similarity of those sets. Give another function establishing the similarity
of those sets.
2. Prove that if a and b are real numbers such that a < b, then the set of all real
numbers a such that a < « < b ordered according to their magnitude is similar to
the set of all real numbers ordered according to their magnitude.
3. Prove that every function f(x) mapping the set of all real numbers ordered
according to their magnitude on itself in a similar way must be a continuous function
of a real variable.
Proof. Suppose that the function f(z) maps the set x of all real numbers ordered
according to their magnitude on itself in a similar way. Therefore the inverse function,
f(a), has the same property. Let 2, denote a given real number, and e — an arbitrary
positive number; f(x,)+« and f(x,)—e will be certain real numbers, and f(x#)—e <
< f(a) < f(%)+¢, whence a= f(f(x9)—e) < w% < f“f(x.)+¢) =, since the func-
tion f1 establishes the similarity of the set of all real numbers, ordered by means of
the relation <, to the same set, also ordered by the relation <. Now if « is a real num-
ber such that f"(f(a)—«) < # < f-(f()+ ¢), then, in view of the fact that the func-
tion f has the same property as the function f, to which we referred just now, and
in view of the fact that f(f(y)) = y for real y, we obtain
F < 2%°, and on the other hand, every function f(z) = w+ a, where a is a real number,
obviously belongs to F', whence F > 2°. Therefore F = 2%°, q. e. d.
5. Calculate the power F of all functions establishing the similarity of the set
of all rational numbers ordered according to their Magnitude to itself.
Solution. That power is the continuum. For, on one hand, the number of all
functions defined in the set of rational numbers and assuming rational values is equal
to the continuum (since such is the number of infinite sequences of rational numbers),
whence < 2%. On the other hand, let x denote a real number such that 0<a< 1
and let
be its expansion into a dyadic fraction whichis really infinite. Let us define a function
f,(v) of a rational variable r as follows: f,(r) = r for r < 0. If k is an integer >0, then,
in case of a, = 0, let £,(r) =r for k<r<k-+1, and in case of a, = 1 let £,(r) = k+
+4(r—k) for kx r<k+# and f(r) =k+3(r—k)—3} for k+4<r<k+1.
Itis easy to verify that fe F forO<a<1 andf,~f, forO<#<1,0<y<l,
x #y, whence F > 2’, Thus F = 2%, q..e.d.
5. First and last element of an ordered set. Cuts. Jumps. Density and
continuity of a set. If, in an ordered set A(o), there exists an element a
such that for no element « of the set A does wea hold, then a is called the
first element of the set A(o). If, in the set A, there exists an element Dd
such that for no element x of the set A does box hold, then b is called
the last element of the set A.
In the set of all natural numbers ordered according to their magni-
tude there exists a first element, and that is number 1, but no last
element. In the set of all negative integers ordered according to their
magnitude, there is no first element but there is a last element —
number — 1. In the set of all integers ordered according to their magni-
_ tude there are no first and no last element. In the set of all real numbers wv
such that 0 <a# <1, ordered according to their magnitude, there exist
both a first element, 0, and a last element, 1.
Obviously, if the set A(@) is similar to the set B(0’) and if the set A(o)
has a first element, then the set B(o') also has a first element. If a is the first
element of the set A(e) and f is a function establishing the similarity
of the sets A(e) and B(o’), then b = f(a) is the first element of the set
B(o'). For if there existed an element ¢ « B such that ¢o’b, then we should
have a’= f“(c) ef(b) = a, whence a’« A and a’oa, which is impos-
sible since a is the first element of the set A(o).
We likewise prove that 7f the set A(o) is similar to the set B(o') and
has a last element, then the set B(o’) also has a last element.
As an easy conclusion, we shall prove that two ordered sets, A(o) and
B(o'), which are similar to subsets of one another are not necessarily similar.
Indeed, let A(g) denote the set of all rational numbers 7 such that
§ 5. First and last element of an ordered set 201
0<r <1 ordered according to the magnitude of those numbers and let
B(e) = A(e)— {1}. A similar mapping of the set A(o) on a subset of
the set B(e) is established by the function f(#) = #/2, and a similar map-~
ping of the set B(oe) on a subset of the set A(e) — by the function g(x) =
Still, the sets A(o) and B(o) are not similar since the set A(o) has a last
element while the set B(o) has no last element.
Now, from a theorem of Banach [2] (p. 239, Theorem 2) it follows
that if A(e) and B(o) are ordered sets which are similar to subsets of
one another, then there exist sets A,C A and B,C B such that A,(0)~
~ Bye’) and A(e)—A,(e)
+ B(e’) —B,(e’).
A decomposition of an ordered set A(o) into two non-empty com-
ponents, A= A,+A,, such that for any two elements a, and a, such
that a, « A,,a,<« A, we have a,oea,, is called a cut of that set. A cut is
denoted by the symbol [|A,, A].
If in a cut [A,, A,] the set A, has a last element and the set A, has
a first element, then we say that the cut gives a jump. If, however, the
set A, has no last element and the set A, has no first element, we say
that the cut [A,, A,] gives a gap.
An ordered set which has no jumps is called dense, if it has nenber
jump nor gaps it is called continuous.
EXERCISES. 1. Prove that if [4,, A,] is a cut of an ordered set A(o) and if a « A,,
x«A,, and xoa, then xe A,.
Proof. Let [4,, A,] be a cut of an ordered set A(g@) and let ae Ay, we A and xea.
If we had we A,, then, in view of a« A, and the properties of a cut, we should have
aex, which is impossible in view of wea and of the asymmetry of the relation 9. There-
fore x¢ A,, and thus, since A = A,+ A, and we A, we obtain we A,, q.e. d.
2. Prove that if [4,, A,] and [B,, B,] are cuts of the same ordered set, then
either A,c B, (and A,> B,) or B,c A, (and B,5 A,).
Proof. Suppose that a = [A,, A,] and 6 = [B,, B,] are two cuts of an ordered
set A(o). Thus we have A = A,+4,=— B+ B,, A,4,— BB, = 0. Hence A, = A—A,,
B, = A—B,. Therefore if we had A, c B,, we should have A—A,5 A—B,, 7. e. A, 5 B-
Let us suppose that A,cB, does not hold. Therefore A,—B, + 0 and there exists
an element a« A,—B,. Thus we have ae A, and, naturally, ae A (since A,c A) and
a¢ B,; therefore a « A—B,, i. e. ae B,. Thus the element a belongs in the cut f to the
class B,. Therefore if x « B,, then wea. But if ae A, and wea, then, according to Exer-
cise 1, we have we A,. We have proved that if # « B,, then x « A,, which proves that
B,c A,, whence it follows also that A—B,5 A—Aj,, 7. e. that B,> A,. Thus we have
proved our theorem.
If A(g) is an ordered set, and a and b are two elements of that set such that ab,
then the set composed of the elements a and b and of all elements # of the set A such
that aexeb is called a closed interval (a, b).
We say that an ordered set A(e) satisfies the axiom of Ascoli if, for every infinite
sequence of closed intervals each of which contains the next, there exists at least one
element common to all intervals of the sequence in question.
202 XI. Ordered sets
3. Prove that every ordered set that has no gaps satisfies the axiom of Ascoli.
Proof. Let (a,,,) (n= 1, 2,...) be an infinite sequence of closed intervals of
an ordered set A(o) such that each interval contains the next.
It follows that for every natural number n we have a,ea,,, OF a, = G4,- If
there exists such an index k that a,=a, for n>k, then a is a common element
of all our intervals. If there is no such index k, then obviously for each term a,, of the
sequence @,, a2, ... there exists in that sequence a term a, such that m < n and a, oa,.
In that case let us form a cut [A,, A,] of the set A(o) assigning to the class A, each
element a of the set A such that a,oa for k = 1, 2,..., and writing A, = A—A,. Thus
if ac A,, then there exists such an index k that the formula a, oa does not hold; there-
fore, in view of the connexity of the relation @ in the set A, we have either aga, or a = a,,
and then, as we know, for a certain natural n we have a, ea, and therefore aga,. Thus
in class A, there is no last element, and, in view of the assumption that the set A has
no gaps, there must exist a first element 6 in the set A,. From the fact that the interval
(Qn41> %n41) is contained in the interval (a,,, b,) we easily conclude that a,b, for all na-
tural k and n. This proves that b, « A, for n = 1, 2,..., and since b is the first element
of the set A,, we have either b, = b or beb, for every natural number n, whence also
a, ebeb, since a, <«A,, b«A,. Thus in any case b is an element of the closed interval
(a,,6,) for n= 1, 2,..., 7. e. a common element of all intervals of our sequence. The
set A(g) therefore satisfies the axiom of Ascoli, q. e. d.
It will be observed that an ordered set may satisfy the axiom of
Ascoli and have gaps’).
THEOREM 1. In order that an ordered set A(o) be dense it is necessary
and sufficient that for each two elements, a and b, of it such that aob there
exist at least one element xe A such that aox and xob.
Proof. The condition of Theorem 1 is necessary. For let us suppose
that it is not satisfied, 7. e. that there exist elements a and b of the set A(o)
such that aeb but there exists no element x of the set A such that aowx
and xob. Let us denote by A, the set of all elements x of the set A such
that xeb and let A, = A—A,. In view of agb we have ae A,, whence
A, #0, and since beb does not hold (in view of the irreflexivity of the
relation @), we have b¢ A, whence b « A, and A, #0. The sets A, and A,
are thus non-empty. Now if # denotes such an element of the set A, that
aex, then, according to our assumption, veb does not hold; therefore,
according to the definition of the set A,, we have x¢ A,, which gives
a contradiction. Thus in the set A, there is no such element x that aoa,
which proves that a is the last element of the set A,. Now, b is the first
element of the set A, since, according to the definition of the set A,,
if the element «x of the set] A satisfied the formula web, then we
should have #e« A,; consequently, since A, = A—A,, we could not have
vce A,. Finally, if 2, ¢A,, %,¢« A,, then we have 2,0b but not 2,ob. In
view of the connexity of the relation o we thus have either x7, = b or boas.
tion 9, we have a, ea for 1 <k <1 <n, whence, in view of the irreflexivity
of the relation 0, we have a, ~ 2). The set of all elements of the set A(o)
lying between a and b cannot be finite. Our corollary is thus proved.
Remark. The above proof of the Corollary does not use the axiom
of choice since we have made only a finite number of choices (of the
elements %,, %,..-,%). We should not be able, however, to prove
without the aid of the axiom of choice that, in a dense set A(o), for each
two elements of it, a and b, such that aob, there exists an infinite sequence
of different elements of that set lying between a and Db.
It is easy to prove that a set which is similar to a dense set is dense
itself.
Tf a and b are elements of an ordered set A(e) such that aeb and if
there is no element x of that set for which aex and web, then the element b
is called the consequent of the element a and the element a — the ante-
cedent of the element b. Obviously each element of an ordered set has at
most one consequent and at most one antecedent.
EXERCISE. Prove that in order that an ordered set be dense it is necessary and
sufficient that no element of it have a consequent.
to their magnitude is of type w. Also the set of the terms of every infinite
sequence of different terms (a,, a, ...) ordered according to the magnitude
of the indices of those terms is of type wm. But the set of all positive num-
bers ordered according to their magnitude is not of type o.
Every ordered set of type wm is denumerable but not vice versa.
Every ordered set which is similar to a set of type w is obviously
(in view of the transitivity of the relation of similarity) itself of type o.
Each subset A, of an ordered set A(o) such that if ae A,, re A
and xoa (or aox), then we A,, is called a segment (or a remainder) of the
set A(o). A segment (or remainder) A, of a set A(o) is called a proper
segment (or remainder), if A, ~0 and A, 4 A. Obviously a segment of
a segment of a set A(o) is a segment of that set.
If a is a given element of an ordered set A(o), then the set of all
elements x of the set A(e) such that xea obviously forms a segment of
the set A(o) (which can be the empty set); we call it a segment corresponding
to the element a and denote it by A,(0).
Obviously, in a mapping of a set A(e) upon a set B(o’) which esta-
blishes their similarity, every segment of the set A(e) is transformed
into a segment of the set B(o’). The same applies to proper segments
and to remainders, as well as to proper remainders.
element of the set A(o); let f(a) = 1. If A,(e) #90, then (according to
the definition of a segment corresponding to an element a) we also
have aéA,(e), whence A,(e) # A(o) and the segment A,(o) of the
set A(o) is a proper segment. According to the conditions of our theorem
it is thus a finite non-empty set; therefore its power, 4,(o), is a natural
number. Let f(a) = A,(o)+1; f(a) will be a natural number +1. In this
way a certain natural number f(a) has been associated with each element a
of the set A(g). Now suppose that a and b are two elements of the set
A(e) such that aeb. Thus the element a belongs to the segment A;/(0),
whence it easily follows that the segment A,(o) (since it does not contain
element a) is a proper subset of-the segment A,(0). Since the sets A,(0)
and A;(e) are finite, we have A,(0) < 4;(0), and thus f(a) < f(b). Thus
the function f establishes the similarity of the infinite set A(o) to a certain
subset (also infinite of course) of the set of all natural numbers ordered
according to the magnitude of the numbers which belong to it. But ob-
viously every infinite set of natural numbers ordered according to their
magnitude is of type w since we can arrange the numbers of that set
in an infinite sequence according to their magnitude. It follows that
the set A(oe) is of type w. Thus the conditions of our theorem are suffi-
cient and Theorem 1 is proved.
We also have the following
THEOREM la. In order that an ordered set be of type w it is necessary
and sufficient that it have no last element and that each proper segment of
that set be finite.
The proof of Theorem la is an easy modification of the proof of
Theorem 1.
EXERCISES. 1. Prove that one of two segments of the same ordered set is al-
ways a subset of the other.
Proof. Suppose that A, and A, are segments of an ordered set A(g) and suppose
that neither A,c A, nor A,c A,. Therefore we have A,—A, ~ 0 and A,—A,40 and
there exist elements a and b such thata « A,—A,, b « A,—A,. Since A,c A and A,c A,
we have ae A and be A, where a # b since a« A, and b € A,. Thus in view of the con-
nexity of the relation @ we have either agb or bea. If aeb, then, since 6 is an element
of the segment A,, we have, in view of the properties of a segment, a « A,, which is impos-
sible. If b = a, then, since ais an element of the segment A,, we should have 6 « A,, which
is impossible. Thus the assumption that neither A, c A, nor A, c A, leads to a contradic-
tion. Our theorem is thus proved.
2. Prove that if a non-empty set 4, 4 A is a segment of an ordered set A(g) and
if we put A, = A—A,, then [4,, A,] will be a cut of the set A(o).
Proof. Suppose that 4,,0 #4 A, # A, is a segment of the set A(oe); A, = A—A,,
a,<«A,, a,¢ A,. Thus a, «A, a,¢ A, and a, a, since a, ¢ A,, a, ¢ A,. In view of the
connexity of the relation @ we thus have either a, ga, or a,ea,. But if we had a,ea,,
then, since @ belongs to A,, we should have a, « A,, which is impossible. Thus a, ea,.
208 XI. Ordered sets
We have proved that if a,<«A,, a, ¢-A,, then a,ga,, and since A = A,+A,, it follows
that [A,, A.] is a cut of the set A(Q), q.e. d.
From Exercise 1] in § 5 and from the definition of a segment it immediately follows
that if [4,, A,] is a cut of the set A(@), then A, is a segment of the set A non-empty
and, different from A. Thus we see that in order that the decomposition A = A,+ Ag,
of an ordered set A(g) into two disjoint components determine the cut [A,, A,]
of the set A (g) it is necessary and sufficient that the set A, be a segment of the set A (g)
SUCH unatn Oka eA — eA
3. Prove the following theorem:
In order that an ordered set be of type w it is necessary and sufficient that it have the
following properties: 1) it should contain a first element, 2) it should contain no last element,
and 3) each cut of it should give a jump.
Proof. It is easy to prove that the set of all natural numbers ordered, according
to their magnitude has the, properties 1), 2) and 3), whence it follows that the condi-
tions of our theorem are necessary.
Now suppose that an ordered set A(o) satisfies conditions 1), 2) and-3). According
to 1) the set A(g) has a first element: let us denote it by a,. Now let us define by induc-
tion a certain infinite sequence of elements of the set A(@), a,, d2,..., in the following
way. Suppose that, for a given natural number n, we have defined the terms a, dg, ..., A,
(which is true for n = 1). Let A, = Aa,(e)+ {a,} and A, = A—A,. In view of a, « A,,
we shall have A, ~ 0, and also A, # A since the set A, has a last element, a,, and,
according to 2), the set A has no last element. Thus A, = A—A, ~ 0. It is easy to
verify that, in view of the definition of the sets A, and A,, for x « A,, ye A,, we have
xoy. Thus the sets A, and A, determine a certain cut [4,, A,] of the set A(g). In virtue
of property 3) that section must give a jump; according to the definition of a jump (§ 5),
the set A, must have a first element; let us denote it by a,,,. We shall have of course
a, 0A, 41 Since a, « A,, a,,, € A,, and between a, and a, ,, there is no element of the setA
since such an element « would satisfy the formulae a, ga and xea,,, and it would belong
neither to the set A, nor to the set A,(a, 4, being the first element of that set).
Thus we have defined by induction an infinite sequence a,, a,,... of elements of
the set A(g); we have a,ea,ea3ea,..., and between a, and a, ,, there is no element of
the set A(g). It easily follows by induction that between a, and a,,, lie only the ele-
ments dz, dg, .--, M1, Of the set A(e) (for n= 1, 2,...). Further, it follows that if
there existed an element a of the set A(@) different from all the terms of the sequence
@&, G,,..., then we should have either aga, for n=1,2,..., or a,oa for n=11, 2,...
The first case cannot hold since a, is the first element of the set A(g). And if a, ea for
nv —1,2,..., then let B,— {a,, a,,...), B= A—B,. Since we have proved that for
y « A—B, we must have a, oy for n = 1, 2,..., therefore for x « B,, y « B, we have xoy.
Of course B, 4 0: if we also had B, 4 0, the sets B, and B, would determine a cut
[B,, B,] of the set A(e), and, in virtue of property 3), the set B, would have a last ele-
ment, which is impossible in view of B, = {a,, a@,,...} and a,ea,o0a;... Thus we have
B, = 0, whence A(o) = B, = {a,, dz, ...}, Which, since a,ea,Qa3..., proves that the set
A(@) is of type w. Thus the conditions of our theorem are sufficient.
4, Prove the following theorem:
In order that an ordered set A(g) be of type w it is necessary and sufficient that it
have the following three properties:
1° it should have a first element, a,,
2° each element of the set A(@) should have its consequent,
3° if a,« XC A(e) and the set X contains the consequent of each of its elements,
then X = A (see Kuratowski and Mostowski [1], p. 151, Theorem 1).
§ 7, Sets of type w 209
It will be observed that an ordered set A(g) which satisfies only conditions 1°
and 2° is not necessarily of type w. It is even easy to give an example of a set of rational
numbers, not being of type w, ordered according to the magnitudes, having a first ele-
ment and such that every element has the consequent, and every element, except the
first one, has the antecedent.
As can easily be verified, such is the set composed of the numbers 1—1/n, 1+ 1/n
and 3—1/(n+ 1) where n= 1, 2,... We have here
Oy he a Le a Se 8 Sh
There exist also non-denumerable ordered sets A(g), having the properties 1°
and 2° and such that each of its elements that is different from the first element has
an antecedent.
5. Prove the following theorem:
In order that an ordered set be similar to the set of all integers ordered according to
their magnitude it is necessary and sufficient that it have neither a first nor a last element
and that each cut of it give a jump.
6. Prove that if an ordered set A(@) is of type w, then by removing an arbitrary
/
element of that set we shall obtain a set B(@) which is also of type ow.
the former composed of the terms of sequence (8.1) and the latter of the
terms of sequence (8.2), and a function f of the terms of sequence (8.3)
as follows.
Let. p,= 4,, ¢,= 01, f(p1) = g@. Now let » denote a given natural
number >1 and suppose that we have already defined all terms p,, q@,
Diss Qa, +2) Potts In=1) ond (thatthe: function: 7(p;)—4¢;, Lor 4— 1,2, ee,
nm—1, establishes the similarity of the sets {p,, po, -.-, Pn-rt(e) and
NG eGoaicoes Une (On) aWLC D piss ULUe 4Or a7) Ly,
The ordered set {p,, Ps, .--5 Pn—1}(e), being finite, it is, in virtue of
Theorem 1 of X1.6, similar to the set {1, 2, ..., ~—1}(<), and if y(t) = pz,
(¢=1,2,...,n—1) is a function establishing similarity between the
set {1,2,..., 7—1}(<) and the set {p;, Do, :-»5 Pri} (0), then
(8.5) Pky 0 Pky @ Pkg +++ Pkn—» Pkn—1
Pk,
CP koQ +++ OPKk,CPnOPk,
410 +++ OPkn_y
and
Gk O'UhgO +++ O'Gk, O'InO'Vkpy
yO +++ OFOkn_ 9
§ 8. Sets of type 7 Zi
be infinite sequences formed, respectively, of all! different elements of the sets 1’,
Mie basanda Bia.
Now let us define by induction four infinite sequences
whose terms belong, respectively, to the sets A’, A’, B’, B’, and a function f of the
terms of sequences (8.11) and (8.12) as follows.
(Let p= 4, G =O, pi = ay Gi! = bY, F(pi) = a Flpy') = a"
Now let » be a natural number >1 and suppose that we have already defined the
terms Pi»4;, p;’ and gi’ for i < n, and a function f establishing similarity between the set
<Pie Pi 1 Pas Paces Deis Pag (@eana. whe set 4%, Q4. 9a dy es Gy dy ys (@) and such
that f(p;) = q and f (pi) = gy’ for 1< n. We shall distinguish two cases:
1) nm is an even ey As p} we shall take the first term of sequence (8.7) dif-
ferent from p;,p3,..-,p,_, and as p}’ the first term of sequence (8.8) different from
Dr's Po’, «+» pi_,- In view of the properties of the sets B’ and B”, we shall find (as in the
proof of Theorem 1) the first term of sequence (8.9), q/ such that if we take f(p/) = qi,
then the function f will establish similarity between the sets {p/, pi’, ..-. Ph_1> Pi_1> p}(@)
and {Q1, 91's-++> Gr_1> Mr_1> 9,}(0’); next we shall find the first term of sequence (8..0),
gq, for which if we take f(p/’) = q/’, then the function f will establish similarity between
the sets {p1, Pi’, -+> Daa» Pn_1> Pn» Pn3(Q@) amd {G55 G45 +> Una Ina» Ine In 3 (0’)-
2) n is an odd number. Here we repeat the reasoning of case 1), replacing every-
where the letter » by the letter q and vice versa, B’ and B” by A’ and A” and vice
versa, @ by @’ and vice versa, sequences (8.7), (8.8), (8.9) and (8.10), by (8.9), (8.10),
(8.7) and (8.8) respectively, and leaving the formulae f(p/,) = q/ and f(pi’) = qf’ un-
changed.
As in Theorem 1, we prove next that sequences (8.11), (8.12), (8.13) and (8.14)
differ from sequences (8.7), (8.8), (8.9) and (8.10) respectively at most in the order of
their terms, and that the function f establishes similarity between the sets A(g) and
B(o’), and we have f(A’) = B’ and f(A”) = B”, q.e. d.
2. Give a decomposition of the set & of all rational numbers ordered according
to their magnitude into an infinite series of disjoint sets, dense in LR.
Answer. Let p, denote the n-th successive prime number?!) and, for » > 1,
let A, denote the set of all numbers of the form 1/py where k is a natural nuntber and /
an integer, prime with respect to p n >
let A, = R—-(A,4+A,+...). R= A,+4,4+... will
be the required decomposition.
3. Give the decomposition of the set & of all rational numbers ordered according
to their magnitude into 2*° almost-disjoint sets *), dense in R.
Answer. Let p, denote the n-th successive prime number and @(n) — the set of all
numbers of the form l/p, where J is an integer prime with respect to p, and such
1) Since we do not know any simple formula expressing the n-th successive prime
number as function of n, the infinite sequence p, (n= 1,2,...) may be defined here
in a different way, e.g. by taking p, = 2°41 for n=1, 2,...
2) As regards the definition of almost-disjomt sets (see [V.14).
214 XI. Ordered sets
= Ol Q"(2Hna+ 1)),
n=1
and
=R-
> Ala),
z2>0
where > denotes the sum extended to all positive real numbers.
z>0
R=
y Ate)
r>0
co co
Ap==y)
= gl@) aad
: oe
ez (72)
where A 4 — B©B® — 0 for k <1 and where, for n = 1, 2,..., the sets A and B™
are dense in the sets A and B respectively, then there exists a function f establishing
similarity between the sets A(o) and B(g), such that f(A™) = B® for n= 1, 2,...
5. Prove that every dense ordered set contains a subset of type 7.
6. Prove that if A is a set of type 7, B its segment, and C the remainder of the
set A corresponding to that segment, then at least one of the sets B and C is of type 7.
Proof. Obviously if the set B contains no last element and is not empty, then B
is of type 7; otherwise C is of type 7.
7. Let & denote the set of all rational numbers ordered according to their magni-
tude. Give an example of such a set Bc R that neither of the sets B and k—B is of
type 7.
Answer. An example of such a set B is the set formed of all rational numbers
lying inside the interval (0,1) and of the numbers 3 and 4.
8. Prove that if A is a set of type 7 and B — any subset of the set A, then at least
one of the sets B and A—B contains a subset of type 7.
Proof. If the set B is dense in the set A, then obviously it is of type 7. Otherwise
there exist in the set A two elements a and b, a 4 b, between which there is no element
of the set B. The set of all elements of the seb A lying between a and b will be a subset
of type 7 of the set A—B.
there exists an element b, « B, such that b,0’s, whence ),C s and b, 48.
But, in view of b,¢ B,, we have bo’b, for each element be B,, whence
bCb,; thus it follows from the definition of the sets s that sC b,, which
is impossible since b,C s and b, #s8.
Thus the assumption that there exists a cut [B,, B,| of the set B(o)
determining a gap leads to a contradiction. We have proved Theorem 1.
Remark. If the dense ordered set A(ge) had no first element, then,
in the proof of Theorem 1, when defining the set B we should not have
to include the empty set among its elements; the set B(o’) would also
have no first element. It is likewise obvious that if the set A(o) has no
last element, then the set B(o’) also has no last element.
Theorem 1 might also be expressed as follows:
Every dense ordered set is a dense subset of a continuous ordered set.
This should be understood in the following way: if A(o) is a dense
ordered set then there exists a continuous ordered set B(o’) containing
the set A, the relation 0’ being equivalent in the set A to the relation 0,
and the set A(o’) is dense in the set B(o’).
This theorem is usually proved by joining to the set A(o), as new
elements, all its cuts [A,, A,] giving gaps, and defining the relation o
as follows: for ae A, ae[A,, A] holds if and only if ae A,, and[A,, A,]oa
holds if and only if ae A,; finally if [A,, A,] and [Aj, Az] are two diffe-
rent cuts of the set, A giving gaps, then we write [A,, A,]o[Ai, Az] if
and only if A,C Aj. Then we prove that the new set obtained in this
way will be continuous and that the set A(o) will be dense in it.
In particular, if we applied a proof of this kind to the set of all rational
numbers ordered according to their magnitude, we should obtain Dede-
kind’s theory of irrational numbers, the irrational numbers being the
gap-giving cuts of the set of all rational numbers (see Dedekind [1]).
THEOREM 2. Two continuous ordered sets having no first and no last
elements and containing, respectively, similar dense ordered subsets are
similar.
Proof. Let A(e) and B(e’) be two continuous ordered sets having
no first and no last elements, and let A’(o) be a dense subset of the set
A(o) and B’(o’) a dense subset of the set B(o’), A’(e) + B'(o’). Thus there
exists a function f establishing the similarity of the sets A’(o) and B’(0’).
Now let «« A—A’. Let us denote by O the set of all elements a « A’ such
that aoxv. Since the set A(o) has no first element and the set A’(o) is dense
in A(e), we easily conclude that the set C is not empty and has no last
element. Similarly, since the set B(o@) has no last element, we conclude
that the set @,= A’—C, is non-empty and has no first element. Thus
[C,, C.] is a cut of the set A’(o) determining a gap.
§ 9. Dense ordered sets as subsets of continuous sets 217
10. Sets of type 4. An ordered set which is similar to the set of all
real numbers ordered according to their magnitude is said to be of type 2.
Thus, for instance, the set of all real numbers ordered according to their
magnitude is of type 4. The set of all real numbers lying inside an arbi-
trary interval (a,b) and ordered according to their magnitude is also
of type A. (See Exercise 2 in XI.4).
THEOREM 1. Jn order that an ordered set be of type A tt is necessary
and sufficient that it have the following three properties: 1) it should have
neither a first nor a last element, 2) it should have a denumerable subset
dense in tt, 3) it should be continuous.
Proof. As can easily be seen, the set of all real numbers ordered
according to their magnitude, which is of type A, has the properties 1),
2) and 3). (A denumerable subset dense in it is formed, for instance, by
the set of all rational numbers, while the continuity of the set of all
real numbers is proved in Analysis,’ in Dedekind’s theory of irrational
numbers (cf. § 9). Since properties 1), 2) and 3) are obviously valid in
transformations establishing the similarity of sets, it follows that the
conditions of Theorem 1 are necessary. In order to prove that they are
also sufficient it is enough to show that if each of two ordered sets,
A(o) and B(o’), satisfies conditions 1), 2) and 3), then those sets are
similar.
218 XI. Ordered sets
Let A(o) and B(o’) be two ordered sets having properties 1), 2)
and 3). According to property 2) the set A(o) has a denumerable subset
A,(o) dense in it and the set B(o’) has a denumerable subset B,(o') dense
in it. The set A,(0), aS a dense subset of the set A(o) which has property 1),
must obviously itself possess that property. Similarly, the set B,(e’) has
property 1). Further, the set 4,(o) being dense in a set that is continuous,
i. e. dense, is dense itself. Similarly the set B,(o0’) is dense. Thus the sets
A,(o) and B,(0’) satisfy the conditions of Theorem 1 of XIJ.8 and are
both of type 7, 7. e. similar. The continuous sets A(e) and B(oe’) thus
have neither a first nor a last element and contain, respectively, the
subsets 4,(e) and B,(o’) dense in them, those subsets being similar. Accord-
ing to Theorem 2 of XI.9 the sets A(e) and B(o’) are thus similar. We
have proved Theorem 1.
Tt will be observed that there exist ordered sets which satisfy condi-
tions 1) and 3) but are not of type A. Indeed, let Y denote a square com-
posed of points (~, y) of a plane, where 0 <a%<1, 0<y <1, and let
us order its points by means of a relation o where (a, b)o(c, d) means
that either a <¢ or a=ec and b <d. We leave it to the reader to prove
that, after removing from a set QY ordered in this way the points (0, 0)
and (1,1), we shall obtain an ordered set U satisfying conditions 1)
and 3). However, the set U does not satisfy condition 2), since, in view
of (7, 0)o(x, 1) for every real number x such that 0<a#< 1 each dense
subset of the set U contains at least one element (x, y,) where 0 < y, <1
for 0 <a <1, 2. e. itis obviously of the power of the continuum. It follows
that the set U is not similar to any subset of a set of type 4. Thus it is
not every ordered set of the power of the continuum that is similar to
a certain set of real numbers ordered according to their magnitude.
It can easily be seen that every set of type / is such that if we remove
from it an arbitrary element, the ordered set obtained in this way will
no longer be of type A (since it will have a gap).
If A(e) is an ordered set and a and b are two of its elements such
that aoeb, then the set composed of the elements a and b and all those
elements w of the set A(o) for which aov and «ob is called a (closed)
interval (a,b) (X1.5, Exercise 2). Two intervals, (a,b) and (c, d), where
aoc, are said to be non-overlapping if bec or b = c. It is easy to prove that
every ordered set A(g@) containing a dense denumerable subset is such that
4) every set of non-overlapping intervals in it is finite or denumerable.
Indeed, suppose that P= {p,, p.,...} is a denumerable subset of
the set A(o) dense in it, and let Z denote the set of non-overlapping
intervals of the set A(o). Let us associate with each interval belonging
to Z the first term of the sequence p,, p., ... that belongs to that interval
and is different from its end-points. (Such an element exists since the
§ 10. Sets of type 4 219
EXERCISES. 1. Prove that the set composed of number 0 and of all real num-
bers # such that |v%| > 1 ordered according to the magnitude of the numbers that
compose it is of type A.
2. Prove that if we remove from the set X of real numbers ordered according
to their magnitude the sum of an arbitrary set of non-overlapping intervals each of
which contains only one of its end-points, then the remaining set will be of type A.
3. Find how many different subsets of type /4 are contained in the set X of all real
numbers ordered according to their magnitude.
Solution. The number of such subsets is equal to the continuum. Suppose that
Ac X is a subset of the set A that is of type 4. Let x denote an element of the set X —A,
A, the set of all those numbers of the set A that are less than «, A, the set of all those
numbers of the set A that are greater than x. Let us denote by 6, the upper bound of
the numbers of the set A,, taking b, = — co if A, = 0, and by 6, the lower bound of
the numbers of the set A,, taking b, = + oco if A, = 0; let us associate with number «
the interval 6(x) = (b,, b,). We have here of course b, < «4 < b,, b, = db, being impos-
sible since then the cut [A,, A,] of the set A would give a gap, which is impossible in
view of the set A being of type 4, i. e. continuous. From the continuity of the ordered
set A it easily follows that one and only one of the numbers b, and b, belongs to the
set A. It also follows that if « and w’ are two different elements of the set X —A, then
either 6(”%) = 6(a’) or the intervals d(x”) and d(x’) do not overlap. Hence we draw the
conclusion that the set A will be obtained by removing from the set X the sum, finite
or infinite, of non-overlapping intervals, each of them containing only one of its end-
points; let us denote that sum by (A); we shall have A = X—S(A). But, in virtue
of Theorem 1 of III.3, a set of non-overlapping intervals is finite or effectively denumer-
able. Thus in order to determine the sum (A) it suffices to give a finite or infinite se-
quence of left-hand and right-hand end-points of intervals whose sum is (A), and another
sequence of those end-points which belong to the intervals in question. The set p(A)
can thus be determined by two infinite sequences of real numbers and since, as we know,
there are 2“° pairs of such sequences, there are at most 2*° sums S(A), whence it fol-
lows that the set X has at most 2*° subsets of type 4.
On the other hand, every interval without end-points is a type / subset of the
set X, whence we conclude that there are > 2*° such subsets.
Thus we have proved that the set X has 2°° different subsets of type A, q. e. d.
Remark. It is possible to construct a family, of the power of the continuum, of
disjoint subsets of the set X each of which is of type 4. Indeed, let A denote the set
1) Several works have been devoted to this problem, e.g. Kurepa [1], p. 2-4,
Maraham [1], Kurepa [3], p. 187, Kurepa [2], p. 1051, Kurepa [4], p. 327 and p. 1084,
Marezewski [1], p. 303, Inagaki [1], p. 25-46, Inagaki [2], p. 145-162 and 191-201,
Miller [1], p. 673-678, Sierpinski [46].
220 XI. Ordered sets
of all real numbers lying inside the interval (0,1) and being of the form
va
co
my —2n
n=]
where a, (n= 1, 2,...) is an infinite sequence, formed of the numbers 0 and 1 and
containing infinitely many terms <0. It is easy to prove that the set A, ordered accord-
ing to the magnitude of the numbers which belong to it, is similar to the set X of all
real numbers lying inside the interval (0,1), ordered according to their magnitude,
i.e. it is of type 2. The function establishing the similarity of the sets 4 and X is
foe} co
l= on 7 —n
A> a, 2 ae
n=1 n=1
Now let us denote by 4,, for ¢¢ A, the set of all numbers «+ ¢/2 where x « A. The
sets A, where t « A are of course similar to the set A, i. e. they are of type /, and ob-
viously A,A, = 0 forte A, we A, t #u. The sets A,, where t « A, form a family, of the
power of the continuum, of disjoint subsets of the, set X each of which is of type 2.
4. Prove that if Q denotes the set of all irrational numbers ordered according
to their magnitude, and if we remove from it an arbitrary finite or denumerable set,
then we shall obtain a subset of the set Y which is similar to Q.
Proof. Let X denote the setwof all real numbers, & the set of all rational numbers,
D an arbitrary finite or denumerable set of irrational numbers, all of them ordered, ac-
cording to the magnitude of the numbers which belong to them. From theorem 4 it
follows that the sets R and K+D are similar: thus there exists an increasing function
f(z), defined in the set & and mapping that set upon the set R+D. As in the proof of
Theorem 2 of XI.9, we prove that there exists a (unique) increasing function of a real
variable, y(#), such that p(x) = f(a) for « « R. The function g(a) obviously establishes
similarity between the sets 9 = X—R and Q—D = (X—R)—D.
5. Prove that every ordered set containing a denumerable subset, dense in it
is similar to a certain set of real numbers ordered according to their magnitude.
ci] Proof. The proof should be based on theorems 6 and 8.
6. Prove that every continuous ordered, set contains a subset of type 4.
7. Construct a set of disjoint intervals contained in the interval (0, 1) which,
‘when ordered according to the magnitude of the abscissae of the left-hand end-points
of the intervals belonging to it, is of type 7.
Solution. Let us divide the interval (0,1) into three equal intervals 6), 0,, d2
and let us assign to a set Z the middle interval, i. e. 6, = (4, 3). Each of the remaining
two intervals 6, and 6, let us again divide into three equal intervals 699, do, Oo, and
Seo, Oo1, O22, and let us assign to the set Z the middle interval of each triplet, 7. e. the
intervals 69, = (4,4) and 6,, = (4, 8). Let us repeat the same procedure with regard
to each of the remaining four intervals 699, do2, 5205 022, dividing it into three equal
intervals, assigning the middle one to the set Z, etc. In this way we shall obtain an in-
finite sequence of intervals
(10.1) Or; Oo1> Oer> Ooo» Ooe1> Oo01> Ooa1 > Oooo1> Ooo2 ? O2001> eens,
composed of the interval 6 and of intervals of the form Once opt? where k is a natural
number and a,, a,..., a, are numbers 0 or 2. Those intervals, ordered according to the
magnitude of their left-hand end-points, form a set of type 7, since they obviously
satisfy the conditions of Theorem 1 of XI.8.
§ 10. Sets of type A 221
ay Ay , Oy, “a)
DP [isa ere a at pee
SS Dia Roa
% a, eT 1 Bpa Bure
u +—4.,..-+
3 | 92 - gh gk | 3h ta y gk +2 Foss
. €
o=2(S4 2+.)
where ¢, = 0 or 1 fori = 1, 2,... and where c, # 0 for infinitely many natural numbers 7.
It can easily be seen that if we associate with number x of the set A the number
we shall obtain a similar mapping of the set A upon the set of all real numbers lying
inside the interval (0, 1). The set A ordered according to the magnitude of the numbers
that belong to it, is thus of type 4, i. e. of the power of the continuum. The set
O=A-+D, which is called the nowhere dense perfect set of Cantor is therefore also
of the power of the continuum.
It is easy to prove that if we have two different numbers of the set C, then either
they are the end-points of a certain interval of the set Z or infinitely many different in-
tervals of the set Z lie between them.
CHAPTER XII
2. Sum of two order types. Let a and f be two given order types.
Thus there exist ordered sets A(o,) and B(o,) such that 4 =a, B= fp.
We can assume here that the sets A and B are disjoint. Otherwise it would
suffice to replace the set A by the set A, of all ordered pairs (a, 1) where
ae A, ordering the set A, by means of the relation oj where (a,, 1) o1(a2, 1)
holds in the set A, if and only if we have a,o,a, in the set A. Similarly
we should replace the set B by the set B, of all pairs (b, 2) where be B,
ordering the set 6, by means of the relation 03 where (b,, 2) 03(b,, 2) holds
in the set B, if and only if we have b, 0,6, in the set B. We shall have of
course A,(e1)~A(a), Bye) ~ B(@), whence A,=o, B,= 6 and
A,B, =.0.
Therefore let us suppose that A= A(o,), B= B(o,), d=a, B= B,
AB = 0. Let S = A+B and let us order the set S by the relation 0 defined
by the conditions: if s, « S, s,« 8, then the formula s, es, holds if and only
lime hesAr as, erAront S109, )00us; € Bs 28,icB “and <3, 08,4 oresre Ay 1s.) © B:
It is easy to prove that the relation @ defined in this way will order the
set S. In other words, we order the sum S of the sets A and B in such
a manner that for each two elements of that sum both belonging to A or
224 XII. Order types and operations on them
—-t< -4<-4<..<57<4<1.
Types m*-+-@ and w+ * are different from one another: the former has neither
a first nor a last element, while the latter has both. The former has no gaps, while the
latter has one.
This example proves that a swum of] order types may depend on the order of the
components. Thus the components of a sum of two types play a different part each:
therefore they have been given different names: the first is called angendus and the
second addendus.
Let us take another example: the sums 1-+m and w+ 1. According to the defini-
tion of the sum of types, in order to obtain a set of type 1+ w we must take a set con-
taining one element only, e. g. a), and a set of type @, e. g. the infinite sequence a,, do, ...,
ordered according to the successive indices; having formed the sum of the two sets,
regard the element a) as preceding each term of the sequence a, (k = 1, 2, ...), in which
§ 2. Sum of order types 22 Ol
we retain the former ordering. In this way we shall obtain the ordered set
git,
<A ses 5
which is obviously also of type wo.
Now, for the type »+1 we obtain the ordered set
pA ean ne
whose type is different from m, since it has a last element while type wm has not. Thus
we have 1+o0 =o, o+1+ wo and therefore 1+o4~o+1.
Similarly we easily obtain 1+ 0* #4 o*+1= o%.
In certain cases, however, sum of two order types may be independent of the
order of the components, e. g. when both types are finite or when the components are
equal. Here is another example. Let £ = w+ *: obviously 1+&=é&+1 (here both
sums are equal to &).
(o+l1l)+o=—o+(1l+o)=o+0.
yt+1l+n=y7 .
At+14+A=i.
If we formed a cut of the set R,[Ri, Ry], where Rj denotes the set
of all rational numbers < 2 and R3j—=— R—R{, we should have Rj=
= Ri = ny, whence, in view of R= hkj+ Rj, we should obtain the formula
U fs bred Bega Be
But :
2 -— A SH i, 9
2. Prove that if € denotes the order type of the set of all irrational numbers
ordered according to magnitude, then
Ai = (wef(@)1, A, =é.
We shall further distinguish two cases:
1) a Af(a) for ve A. Let A, = A—A,. If w¢ A,, then, according to the defini-
tion of the set A,, xef(x) does not hold, and since, by 1), x ~€ f(a), in view of the con-
nexity of the relation @ in the set A, we must have f(x) ox. Hence we easily conclude
that
A,= Ef (@) ee].
Now if a, <« A,, a,¢ A,, then we have a, ~ az, a ef (a1), f(a) 0a,. If we had a, ea,,
we should have hence f (@2) ea@2 0a, of (a,), and thus f(d,) ef (a), which, as we know, gives
4, 0d; this, in view of a,ea,, is impossible. Thus if a, « A,, a, ¢ Az, then a, ~ a, and
4, 0a, does not hold; therefore, in view of the connexity of the relation o@ in the set A,
we must have a,ea,. Thus, from the definition of the sum of order types it follows that
A = A,+A,. Now we shall prove that the function f maps the set A, on the set A,.
Let t¢ A,; we thus have tof(t) =u, whence f(u) of (t), i. e. f(f(t)) ef(t), which proves
that f(t) « A,. On the other hand, if we A,, then f(w)ew and, writing ¢ = f-1(v), we ob-
tain f(t)= wu, whence f(u)of(t), which gives tow, t.¢. tof(t), and thus te Aj, i. e.
f-(u) « A,. The function f maps the set A, on the set A,, and, since it inverts the order
of elements, it follows that A, = A* = &*, Thus a= A= 4,44, = €+&.
2) There exists an element a « A such that a = f(a). To begin with, we shall prove
that there may be only one such element. Indeed, if we had also b = f(b) for b 4a,
then we should have either agb or bea; suppose that aeb, whence f(b) oef(a), 7%. e. boa.
which gives a contradiction. Similarly we find that A = A,+ {a}+A, where
Now if a, <¢A,, then a, ef(a,) and, in view of a= f(a), we must have a, 4a. If
we had aea,, we should have f(a,)ef(a) = a, whence, in view of a, ef(a,), a, ea, which
is impossible. In view of a, 4 a and the connexity of the relation @ in the set A, we
thus have a, ea. Therefore, if a, « A,, then a,ea. We prove similarly that if a, « A,, then
aea,. In view of A = A,+ {a}4+-A, and the definition of the sum of order types, we
have A = A,+1+4,. But, as in case 1), we obtain f(A,) = A,, whence A, = A*, and
if we take A, = &, we shall have a = + 1+ &*.
Thus we have proved that if a = a*, then there exists an order type € such that
a= &+ &* or such that a = €+ 1+ &* (it is possible for those cases to hold simultane-
‘ously, ¢. g. 7 = 7* =y+7* =7+1+7*). On the other hand, it is easy to verify (by
formula (2.1) and a** =a) that, for every order type &, we have (€+ é*)* = ¢+ &*
and (€+ 14+ é*)*= €+ 1+ &*. The required proof is thus completed.
5. Prove that, for an order type a, we have a+ 1=a if and only if a= &+o%,
where € is an arbitrary order type (which may be equal to 0).
Proof. Suppose that a is an order type such that a+ 1= a. According to Exer-
cise 1, we have 1+ a* = (a+ 1)* = a*, and, according to Exercise 4, there exists an
order type ¢, such that a* m+, whence a = (a*)* = (w+C)* =€*+ o*. Taking
* — & we shall have a = + w*: thus the condition is necessary.
On the other hand, in view of w*+ 1 = w* and the associativity of the sum of
types, we have for every order type &: (€+ m*)+1= &+(w*+1) = + o*; thus the
condition is sufficient.
6. Prove that, for an order type a, which is not finite, we have 1+ a = a-+ l, if
and only if there exists an order type € (which may be equal to 0) such that a = w+ &+
+ m* (and then we have also 1+a=a).
7. Determine the order type of the set of all infinite sequences composed of num-
bers 0 and 1 and having only a finite >0 number of terms <0, a) ordered according
to the principle of first differences‘), and b) ordered according to the principle of last
differences.
Answer. In case a) the required type is 1+ 7, in case b) it is w. In case a) it suffices
to prove that the set has a first element, has no last element and is denumerable and
dense (and to refer to Theorem 1 of XI.8). In case b) a mapping of the set in question
on the set of all natural numbers is obtained by associating with each sequence 4@,, a2,
a,,... @ natural number 1+a,+ 2a,-+ 2?a,+ ...
8. In Exercise 7 replace the words ,numbers 0 and 1“ by ,,integers‘. What is
the answer?
Answer. In case a) the type is 7, in case b) it is 7+ w*+o+ 7.
9. Prove that there exist only four different dense denumerable order types, na-
mely the types 7, 1+7,7+1 and 1+7+1. 3
Proof. Let a denote a dense denumerable order type, A(g) a set of type a. Thus
the set A satisfies conditions 2) and 3) of. Theorem 1 of XI.8. If, moreover, the set A
satisfies condition 1) of that theorem (i. e. has neither a first nor a last element), then,
in virtue of that theorem, it is of type 7. If the set A does not satisfy condition 1), then
of course only the following three cases are possible:
1’) The set A has a first element — a, but has no last element.
1’) The set A has a last element — 6b, but has no first element.
1’’) The set A has a first element — a, and a last element — b.
From properties 2) and 3) of the set A it easily follows that in case 1’) the set
A, = A— {a}, in case 1’) the set A, = A — {b}, and in case 1’) the set A; = A—{a, b}
will satisfy conditions 1), 2) and 3), 7. e. will be of type 7. In these three cases we shall
have, respectively, A = {a}+A,, A = A,+ {b}, A = {a}+A;+4 {6}, and since a is the
first element of the set A in cases 1’) and 1’), and b is the last element of the set A in
cases 1’’) and 1’’’), we shall find that the types of the set A in cases 1’), 1’’) and 1’”)
are 1+7,7+1,1+7+1 respectively. The proof that these types are denumerable,
dense, different from one another and different from 7 presents no difficulties.
10. Prove that if a, 6, a, and £, are order types such that a+ 6=a,+/,, then
there exists an order type y (which can be equal to 0), such that either a, =a+y, B=
=yt+f,, or a=a+y, Bp=y+8.
1) 4. e. so that (a,, dg, ...) = (b,, bg, ...) if and only if, denoting by k the least na-
tural number for which a, 4 b,, we have a, < 6,.
§ 2. Sum of order types 229
Proof. Let # denote an ordered set of type a+ 8. From the definition of the sum
of order types it follows that, in view of a+ $f=a,+ ,, there exist ordered sets A,
B, A, and B, such that A=a, B=, A, =a, B,=f,, F = A+ B= A,+B,, [A, B]
and [A,, B,] being cuts of the set F. From Exercise 2 in XI.5 it follows that either A c A,
and B,c B, or A,c A and Bc B,. In the first case the set A will obviously be a segment
of the set A, and the set B, — the remainder of the set B, and we shall have A, = A+
-+(A,—A), B= (A,—A)+B,, in the second case the set A, will be a segment of the
set A and the set B — the remainder of the set B,, and we shall have 4d= A,+(A—A,),
B, = (A—A,)+8. Taking y = A,—A in the first case and y = A—A,, in the second
case, we shall have either a, =a+ty, B=y+,, ora=a+t+y, fj} =y+f, q.e.d.
11. Give an example of order types a, B,y, aq, f,,y, such that af~Aa,, P-Af,,
ea+p=aq-p, a4=aty, pP=y+tBp, ¢=aty.- pi = i+B-
AMEWOR, C= 7, (i = IS Q, = i = We Uy hi = 7 DA = He
Remark. It can be proved that there are no such order types a, a, and y that
AAG, h=a+y, a=aty.
3. Product of two order types. Let a and / be two given order types;
there exist ordered sets A(o,) and B(o,) of types a and f respectively. Let
P= AXB and let us order the set P according to the principle of last
differences, 7. ¢., for (a, b)e« P, (c,d)eP, let (a, b)oe(e, d) if and only if
bod, or b= d and aoe. It is easy to verify that the relation o orders the
set P.
If we take, instead of the set A(o,), an ordered set A’(o;) also of
type a, and, instead of the set B(o,) — the set B’(o2) of type f, and if
we write P’ = A’xB’, ordering the set P’ by the relation 0’ according
to the principle of last differences, we have, as can easily be proved,
P(o)~ P(o’). It follows that the order type of the set P(e) does
not depend on the choice of the ordered sets A(o,) and B(@,), provided
the former is of type a and the latter of type 6. Thus the order type y
of the set P(e) depends only on the order types a and f (and on their
order). We call it the product of the order types a and #6 and write y =
ap.
From the definition of the product of order types it follows im-
mediately that a-l1=1-a=a for every order type a.
EXAMPLES. 1. Let us calculate the product 2-m. As the set 4(0,) we take the set
{1, 2}, and as the set B(o,) — the set of all natural numbers ordered according to their
magnitude. The set P will be the set of all ordered pairs (a, b) where a = 1 or 2, and b
is any natural number; ordering the set P by the relation @ according to the principle
of last differences, we shall obviously obtain
2. Let us now calculate the product wm-2. As the set A we can take the set of all
natural numbers, as the set B — the set {1, 2}, both ordered according to the magnitude
of the numbers composing them. Ordering the set P = A xB by the relation @ accord-
ing to the principle of last differences, we obviously obtain:
whence it follows that the set P(g) is of type a+. Thus we have
o-2=o10,
and therefore w: 2 4 2-w since the set P(g) is not of type w, its proper segment formed
by the element (1, 2) not being finite, in contradiction to Theorem 1 of XI.7 on the
type w. Therefore
The product of order types does not possess the property of commutativity.
The concept of the product of order types is generalized by induc-
tion to the product of an arbitrary finite number of types, and, as can
easily be seen, that product always has the property of associativity. E. gq.
(m-2)-oa=@o-(2-0)=o-o.
As regards the law of distributivity of the multiplication of order types
with respect to their addition, it holds only in one of its forms, namely
when the second factor (multiplier) is a sum. It can easily be proved .
that, for all order types a,f and y, we have
(8, + Bot...
+ Bn) = af, + a8.+...+
a8, ,
(3.3) Fe ee
For a natural n, the product of » order types each of which is of
type a is denoted by a”. From formula (3.3) it immediately follows by
IMOUCHION Lhab qe —= 7) LOT MWe 1,
EXERCISES. 1. Prove that for any order types a and fS we have the formula
(aB)* = a*B*.
2. Prove that if an order type a is dense and has no first or last element, then
for every order type 6 the type af is dense, but the type fa is not necessarily dense.
3. Prove that if a set of type a-f has a first element, then both a set of type a
and a set of type f have a first element. Prove that the word ,,first“ can be replaced
here (twice) by the word ,,last‘.
4. Prove that aw* ~< w*o.
Proof. We can prove the above by showing that the type w*m has segments of
type w*, while the type ww* has no such segments.
5. Prove that if a is any denumerable type, then ya = 7.
Proof. Let a denote a given denumerable order type. From |Exercise 3 and from
the observation that the type 7 has no first element it follows that type na has no first
element. We similarly prove that type ya has no last element. From Exercise 2 and
from the density of type 7 it follows that type ya is dense. Finally, from the denumer-
ability of types 7 and a it follows that type ja is denumerable. Thus type ya satisfies
the conditions of Theorem 1 of XI.8 and therefore is equal to 7, q.e. d.
6. Prove that if a is any denumerable type having no last element, then
(nt+lha=y.
Proof. Since the order type 7+ 1 is dense it follows from Exercise 2 that type
(n+ 1)a is dense. Since type 7+ 1 has no first element, and type a, according to the
assumption, has no last element, it follows from Exercise 3 that type (7+ 1)a has neither
@ first nor a last element. Finally, from the denumerability of types 7+ 1 and a follows
the denumerability of type (n+ 1)a. Thus type (7+ 1)a satisfies the conditions of
Theorem 1 of XI.8, and therefore it is equal to 7, q. e. d.
7. Prove that if ais any denumerable type having no first element, then (1-+ 7)a = 7.
232 XII. Order types and operations in them
10. Give an example of three different order types whose squares are equal.
Answer. Such are, for instance, the following types: (w*+@)7, (w*+ @)(y+1)
and (w*+ w) (1+).
Remark. Anne C. Davis has given an example of infinitely many different order
types whose squares are equal. They are the types & = (@*+ )(w+on+k) where
k= 0,1, 2,... (see XII.4, Exercise 4). She has also given an example of a denumer-
able order type a for which the number of different order types & satisfying the equa-
tion £2 =a, and, more generally, the equation é” = a, where n = 2, 3,..., is equal to
the continuum. She has likewise proved that for each cardinal number m = 0, 1, 2,
suey No, 2° there exist 2“° denumerable types a such that for every natural number n > 2
the equation &" = a has exactly m different solutions (in order types &). In particular,
for given natural m and n, there exist exactly m-+ 2 order types satisfying the equa-
tion &” = (w*+ @)myn: they are &, = (w*+@)myn and Ga ok = (w*+ ow) (k+ mn)+ (m—k)
LODE One, Serceaite
1l. Give an example of three order types a, 6 and y such that for every natural »
we have a Ap’ #y" 4a’ but
gy ae p" = y" ; .
12. Prove that (1+ 7) (n+ 1) = (n+1)(1+n) = 7, and that for natural n we have
(1-+.n)" = 1+ and (y+1)"=n+1.
Proof. In view of the distributivity law we have (1+ )(y+1) = (l+7)n+1+7.
But, in virtue of Exercise 7, (1+ 7)7 = 7, and since, as we know, 7+ 1+ = , we have
(1+ 7)(n+1)= 7. We similarly obtain (7+ 1)(1+7) = (7+ 1)+ (n+ ))n=9+14+7=
= yn. Further, we have (1+ 7)(1+ 7) = (l+7)+ (l+7)yn = 1+7+7=1+7, whence
by induction (1+ 7)"=1-+y for natural n. Finally (y+ 1)(y4+1) = (n4+1)n+n4+1,
and since, in virtue of Exercise 6, (7+ 1)7 =%, it follows that (y+ 1)? =7+y7+1=
= +1, whence by induction (y+1)° =y+1 for n=1, 2,...
13. Give an example of infinitely many different order types satisfying the equa-
tion €-2=—&.
Answer. Types 1+ 7+ (n—1), where » is a natural number, are obviously all
different from one another and we have [1+ 7+ (n—1)]-2=1+77+ (n—-1)+1+
+ n+ (n—1) = 14 nj + n+ m+ (n—1) = 14+ n(n+ 14+ )+ (n—-1) = 14+ 27+ (n—]).
14. Give an example of an order type a such that a-n=a for n= 1, 2,..., but
aw ~ a.
Answer. a=7-+1 since, if for a natural n we have (n+1)n=7+1, then
(n+ 1) (m+ 1) = (n+ 1) nt (9+: 1) = (n+ 14+ (9+ 1) = (H4+-14+9)+1= 741, whence, by
induction, (y+ 1)n = 7+ 1 for n= 1, 2,..., but, in virtue of Exercise 6, (7+1l)a=7
and n+1 7.
15. Prove that the types A-n(n= 1, 2,...) are all different from one another.
Proof. The proof follows from the observation that the type A-n has exactly » gaps.
16. Prove that A? ~ 2.
Proof. The proof may proceed as follows. We have 4+ 1+.A4= A, whence #2? =
=A(A+1+4+A) = P+A+ 72? = + (A+ 27). Hence it follows that type #2 is the sum of
two order types /? and A+ 22 of which the first has no last element and the second has
no first element. Type 2? thus has a gap, whence /? 4 /. -
17. Prove that types nA (n= 1, 2,...) are all different from one another.
Proof. The proof proceeds as follows. If n and k are natural numbers such that
n > k, then a set of type nd contains elements a and b #0, between which there lie
nm—2>k—1 different elements of that set, and a set of type kA contains no such
elements.
18. Give an example of an order type m for which there exist order types a and fp
such that mp = 2a+ 1 = 26 (i. e. of type g which is both odd and even) and of an order
type y for which there exists neither a type a such that y= 2a+ 1 nor a type f such
that y = 26 (i. e. of type y which is neither odd nor even).
Answer. We have w* = 2w* = 2w*+ 1, and type 7 is neither of the form 2é
nor of the form 2+ 1 since both forms give types which are not dense.
19. An infinite sequence of order types a, (n= 1, 2,...) for which there exists
an order type g such that a,,,=a,+ 0 forn = 1, 2,..., is called, an infinite arithmetical
progression. Give an example. of an infinite arthmetical progression composed of dif-
ferent terms each of which is the square of an order type.
Answer. For example a, = (w-n)? for n= 1, 2,..., since we have here O41 =
= [a(n+ 1) P= w(n+4+ 1) = w'n4+ w? = (w-n)?+ ow = a,+ w? for n = 1, 2,...
Remark. According to a certain theorem of Fermat (of 1640) there are no four
different finite order types forming an arithmetical progression and being each
a square. We can have three such types, e.g. 1, 5°, 7°. ,
234 XII. Order types and operations on them
20. Give an example of three order types a, f and y such that each of the six per-
mutations of those types gives a different sum.
Answer. Por example a—1, B=o, y= 7.
21. For every natural number k = 1, 2,3,4,5 give an example of three order
types a,, B,. 7, Such that in any permutations of those types we obtain exactly hk dif-
ferent sums.
ADS WOR: C=O, pr 0-2, Vy == fe) Opel ps On 7, ny Ae ge
=o ily Wy — Os Be Gn = Thy fin Oy SHS OP Ch = @, a= Wel vee Oe We Wane
here, for instance, a,+/;+y,; = @+(@+1)+o? = o+0+4+ (1+?) = 040+? =
=o(1t+1+o)=o0-o=o, B+a+y; = (@+1)to+0? = 0+0+0?= ow, Bs+yst
+a,=(o+1)+0?+o= 079+ 0, a4,+7,+f; = 0+ w+ (o+ 1)=0?+0+1, 75+ 6s+ a=
=w?+(o+1)+o=o?+o-2, ystast+f; = o+ot+(o+1)—e@?+q@-2+1, and the
types w?, w+ 0, w+ o+1, w+ 0-2, o?+o-2+1 are all different from one another
(see Sierpinski [48], p. 252).
22. For every natural number n give an example of m order types a,, dy, ..., a,
such that each of n! permutations of those types gives a different sum.
PACING WiCT Cee /Go17) a OTGe=— sl ge ae ae
23. Find all order types a 40 such that a4+y=y+a.
Answer. They are all order types of the form 7+ &+ 7 where € is an arbitrary
order type. Indeed, if a is an order type 40 such that a+ 7 = 7+ a, then ais a segment,
different from 0, of type 7+ a, and since every non-empty segment of type 7-+a is
of the form 7+ é, we have a=7+é, whence, since 7+ =, we obtain a= 7+ €&=
=y+yn+E=yn+a=a+n=7+E+7. Thus a=7+&+7. On the other hand, if, for
an arbitrary type €, we take a = 7+é+7, then obviously a+7=y+a=a.
24. Find all order types a #0 such that a+/A=A-+a.
Auswer. These are all order types of the form A-n, where n is a natural number,
and all the types of the form wj+ + w*/, where & is an arbitrary order type (which
can. be equal to 0).
25. Prove the theorem of B. Dushnik and E. W. Miller stating that every denu-
merable ordered set is similar to a proper subset of itself.
Proof. Let A(g) denote a denumerable ordered set. Let us decompose the set
A(e) into components, assigning two elements of the set A(g) to the same component
if and only if only a finite number (+0) of the elements of the set A(@) lies between
them. Obviously the set A(@) decomposes into disjomt components, each component
being either a finite set or a set of one of the following types: m, w* or w*+. If A,
and A, are two different components of the set 4 (g), then every element of one component
precedes every element of the other; thus the components of the set A(g) form an or-
dered -set.
If the set A(g) contains an infinite component Y, then obviously the ordered set
Q(oe) (as a set of type w, m* or m*+) is similar to a certain proper subset of itself;
denoting by P the set of all elements of the set A(e) which precede all elements of the
set Y (or, respectively, by & the set of all elements of the set A(o) which follow all
elements of the set Q), we shall have A = P+Q+8. Mapping the sets P and R identic-
ally each on itself, and the set Q similarly on its proper subset, we shall obviously ob-
tain a similar mapping of the set A(g@) upon its proper subset.
If the set A(g) contains no infinite component, then, obviously, the set A (@),
ordered by the relation @, where, for two components A, and A,, formula A,0,A, means
§ 3. Product of two order types 235
that a,ea, for a, « A,, a, « A,, will be a dense ordered set, since two finite components
cannot lie side by side (which immediately follows from the definition of components).
Taking from each (finite) component its first element, we shall obtain of course
a dense subset D of the set A(e@), denumerable (since the set A(g) is denumerable).
Thus, according to Exercise 9 in XII.2, the set A(g) is of type 7, 1+ 7, n+1 or 1+
+n-+ 1, and therefore contains a subset of type 7. Hence it follows that the set A (@)
contains a subset O of type 7. In virtue of Theorem 2 of XI.8, and in view of the equa-
lity 7 = n+ 7, the set A(g) is similar to a certain proper subset of the set CO, 7. e. also
to a certain proper subset of itself.
Thus the theorem of B. Dushnik and E. W. Miller is proved.
26. Give an example of a non-denumerable ordered set having a first element
and such that each element of it has a consequent and each element except the first
has an antecedent.
Answer. A set of type w+ (w*+ o)d.
27. Prove that an infinite ordered set has a first element, and is such that each
of its elements has a consequent and each element except the first has an antecedent
if and only if that set is of type w+ (w*+ w)& where & is an arbitrary order type (which
can be equal to 0).
28. Give an example of infinitely many different denumerable order types & for
which & = é.
Answer. Types €= ny where n= 1, 2, ...
Remark. It would be more difficult to give an example of 2°° different denumer-
able order types & for which &? = &, as well as an example of a non-denumerable type &
such that = €.
29. Prove that if y and # are arbitrary order types, and y = yw+0-+yo*, then
p+y = y+ (Aronszajn [1)).
30. For every type y, at most denumerable, give an example of a type y, of the
power of the continuum, such that p+y=y-+¢.
Solution. As follows from Exercise 29, such a type will be yw = yw+ A+ ga*.
31. Prove that [@(1+ 7)!’ = (1+ y) and [(@+1)(n+ 1] = (®+1)(n+1) for
Tf, ss MW Dan oe
32. Prove that (o+oy7+n) = o+on+n for n= 1, 2,...
33. Prove that the order types w+1, w+2 and w+3 give a different product
in each of their six permutations.
Proof. For natural n,, n., n3 we easily find: (w+ ,)(@+ N2)(@+ Ns) = w+ w'ng3+
+ on,+ n,, and w?+ wks-+ wk,+ ky= w3+ w?n3+ won,+ nif and only if k, = n, fori =1,2,3.
Hence it follows that with each change of the order of the factors w+1, w+ 2, w+ 3
we obtain a different product. ;
34. Find how many different values are given by the product of the order types w,
o+1, w+ 2 with all possible changes of the order of the factors.
Answer. There are five different values since w(w+1)(w+ 2) = w+ w?-2+ 0,
w(o+ 2)(@+ 1) = w?+ w?4+ w-2, (w+ 1)@(@+ 2) = w3+ w?- 2, (w+ 2)@(@+ 1) = w+ w?,
(+ 1)(o+2)o
= (0+ 2)(o+ 1)o
= a. |
35. For every natural number n give an example of n order types whose product
has a different value in each of their n! permutations.
Answer. Types a+ 1,@+2,...,@+N.
2356 XII. Order types and operations on them
36. The set of all infinite sequences whose terms are 0 or 1, the number of those
terms which are different from 0 being finite (> 0), is ordered according to the principle
of last differences. Prove that the order type of that set is m and determine a function f
establishing similarity between that set and the set of all natural numbers.
Answer. f(a, d, ...) = 1+ a,+ 2, a .+ 2?a3+...4 oo ae ... (Cf. XII.2. Exercise 7).
4. Sum of an infinite series of order types. Let a,, a, ... denote a given
infinite sequence of order types. From the axiom of Hilbert (VI.3) it
follows that there exists an infinite sequence of disjoint ordered sets
An(On) (n=1,2,...) such that A,(o,) is a set of type a,1). Let S=
= A,+A,+... and let us order the set S by the relation o defined as
follows. If the elements a and b of the set S belong to the same com-
ponent A,, then we shall write aob if and only if ao,b. And if ae Ax,
be A, where k ~#n, then we shall write aob if and only if k < n. It is easy
to verify that the set S will be ordered by the relation o. Let S = co. We
shall prove that the order type o depends only upon the infinite sequence
of order types a, a,..., and not upon the choice of ordered sets An( en)
disjoint and such that A,(0,) =a, for n=1,2,.
Let A,(o,) (1 = 1, 2,...) denote another infinite sequence of disjoint
ordered sets such that A,(o,) = a, for n= 1, 2,... Thus we have A,(0,)~
~ A,(o,) and it follows from the axiom of choice that there exists an in-
finite sequence of functions /7,,f,,..., such that, for n=1,2,..., the
function f, establishes a similar mapping of the set A,(o,) on the set
A,(o,). Let us define the function f in the set S as follows: if ae A, then
let f(a) = f,(a). Obviously, the function f establishes similarity between
the set S(o’) and the set S’ = Aj+A3+..., ordered by the relation 0’
defined by the conditions that a’ob’ if and only if we have either a’ « A,,
b’<« A, and a’o,b’ for a certain natural n or a’ e«Aj, b’e A, and k <n.
Thus 8’ = S =o.
We have proved that to every infinite sequence of order types a,
d,,... corresponds an order type o, well defined by that sequence, such
that if A,, A,,... is an infinite sequence of disjoint ordered sets where
A, = a forn=1,2,... and if we order the set S = A,+A,+... retaining
1) Here also the existence of such an infinite sequence of sets A,(0,) (vw = 1, 2, ...)
could be deduced from the axiom of choice (instead of the axiom of Hilbert), for we
shall prove in XVI.1 that from the axiom of choice follows the existence of a function f,
associating with each cardinal number m a set f(m) of power m. For n = 1, 2,..., let
A, =f(a,) and denote by ®, the family of all the order arrangements of the set A,
according to type a,. Since /(m) =m for cardinal m, we have A, = a,, and therefore
there exists at least one ordering of the set A, belonging to ®,. Thus it follows from the
axiom of choice that there exists an infinite sequence of order arrangements Q,, Q2, ..-;
such that 9, ¢«®, for n= 1, 2,..., 4. ¢. that A,(o,) =a, for n = 1, 2, ... The substitu-
tion of disjoint sets for the sets A,, A,,... presents no difficulty.
§ 4. Sum of an infinite series of order types 237
for the elements of the same component that order arrangement which
they had in that component, and — if the elements belong to different
components of the sum S — regarding that element as coming first which
belongs to a component that comes first, then we shall have S = o. The
order type o is called the swm of an infinite series of order types a,+a,+..
and we write
oO = Oy += Ag Og...
o=1414+1-3.,
but also
w= 242424..=142434...=
2427+ 23+...
ht+yntyt+..=y-ltyn-lty-14+...=n(14+1+...) = no
= 7;
whence n»+y+7+...=Y7
We also find in general that
am =a(14+1+...)=a-l4ta-1+...=atea+...;
w=ototot...,
But, like the sum of two order types, the sum of an infinite series
of order types has no commutativity law. H.g. it can be proved that
the sum of the infinite series of order types
n+2n+3y-+...+nyn+...
changes its value with every change of the order of its components (7. €.
assumes 2” different values).
238 XII. Order types and operations on them
(5.2) pt1i+a+gy+1+a+p+1i+a,+...
§ 5. Power of the set of all denumerable order types 239
6. Power of the set of all order types of the power of the continuum. We
shall prove
THEOREM 1. We are able to define a set of power 2” ‘of sets of real
numbers, of the power of the continuum, of which no two different sets ordered
according to the magnitude of their elements are similar (see Sierpinski '[53],
p. 305-307).
Proof. In XI.10, Exercise 10, we have defined the perfect nowhere-
dense set C of Cantor, which is a complement to the closed interval (0, 1)
1) It will be observed that, analogously, we could prove in general (with the aid
of the axiom of choice) that the set of all order types of power m is of power <2™,
§ 6. Power of the set of all order types 241
2. Let X denote the set of all real numbers ordered according to their magnitude.
Prove that for every non-empty set AcX there exist exactly 2° different sets cX
similar to the set A.
Proof. If AcX, Bc X andtif the sets A and B are jsimilar, then there exists a real
function f establishing their similarity, 7. e¢. a function defined for xe A and increasing
in the set A. But, as follows from Exercise{2 of VI.4, we can prove with the aid of the axiom
of choice that the set of all such functions is of power 2°. Hence the set of all differ-
ent sets cX similar to the set A is of power <2". Thus it remains to show that the
above set is of power > 2%. If the set A is bounded (even from one side only), then it
is easy to prove that all translations of the set A along a straight line are different from
one another and their number is 2*°. On the other hand every set of real numbers is
similar to the set of all real numbers lying inside the interval (0, 1) (since both sets
are of type A). Thus for every non-empty set Ac X there exist at least 2“ different
sets cX similar to the set A, q.e. d.
Now let m denote the power of the set of all different order types of the subsets
of the set X of all real numbers ordered according to their magnitude. From the theorem
proved in exercise 2 it immediately follows that all subsets of the set X can be divided
into m disjoint classes, each of which contains 2“° different sets. Hence the equality
m.-2%0 — 28°, which, as follows from Exercise of X.8,p.181,impliesm = 22°°. Thus we have
proved that the set of all different order types of the subsets of the set of all real num-
bers ordered according to their magnitude is of power 2?°, which immediately follows
from Theorem 1. :
A= »'A,
ped
and let us order the set A in such a way that for two elements of the set A
belonging to the same component we shall retain the same order rela-
tion which existed between them in the set A,, and of two elements of
the set A belonging to two different components of the set A we shall
regard as coming first that element which belongs to the component
that comes first (2. e. the component whose index ¢ in the set ® comes first).
Obviously in this way the set A will be ordered and we shall easily prove
with the aid of the axiom od choice (as we did in XII.5 for the sum of
an infinite series of types) that the type o of the set A will depend: only
upon the ordered set ® of types gy, and not upon the choice of sets A,,
disjoint and such that A, = y for ye @®. The type o obtained in this way
1) We could resort here to the axiom of choice and not to Hilbert’s axiom Cf.
an analogical remark concerning the sum of an infinite series of types, § 4.
IGS
244 XII. Order types and operations on them
nays
Thus every ordered set of types gives a well-defined sum; that sum
depends in general upon the order arrangement of the components, but
it has the property of associativity. Particular cases of such a sum of
types are obviously finite and infinite series of types.
In particular, if the set ® is of type y and if each of its elements
constitutes the same type y, then it is easy to prove that
c= Me=ory.
gpeD
++ Ag, Ag, Ay y
there always exists the least natural number ék such that a, ~ b,. Those
sequences form the infinite Cartesian product of type a*
ie eat Ae A Xe AG
then we shall obviously establish similarity between our set of sequences and the set A
of all real numbers « such that 0 < x < 1, ordered according to the decreasing magni-
tudes of its elements. If we reversed the orderof the numbers of the latter set, we
should obtain, as we know, a set of type 4+ 1. It follows that the set A is of type
(A+ 1)* = 1*+4* =1+4. Therefore w® = 1-+4.
2. Let us calculate the infinite product of type w*
i. e. the power 2®'. It will obviously be the type of the set of all infinite sequences ay,
a, ..., composed. of numbers 0 and 1, ordered according to the principle of first differences.
1) In general, given a set F of type » of sets ordered by the relation @, we can define
the Cartesian product of type g of those sets as the set, of all sets {a,}4-, where a, «A
for A «Ff, ordered in such a way that for A, «Ff, A, «Ff, A, 4 A, we have Ag, 344, if
and only if 4,0A, in the set F.
246 XII. Order types and operations on them
. OF ww* ow*|o,
i. e. the power (w*w)”. It will obviously be the type of the set of all infinite sequences
Ny, —Ny,N3,—Ny,..., Where n,,%,,... are natural numbers ordered according to the
principle of first differences, 7. e. so that if m,, —m,, m3, ... and 21, —Ne, Nz, ... are two
different sequences of this kind and if k denotes the least natural number for which
M, F= Ny» then the first of those sequences is regarded as preceding or following the
second according to whether (—1)*t*m, < (—1)'t'n, or (—1)*t?*m, > (—1)'t',. Now if
we associate with each sequence n,, —N2, M3, —N4, ... an irrational number whose deve-
lopment into an infinite continued fraction is
4} cory
NM, |Ns
then we shall obviously establish similarity between the set of our sequences n,, —Nez»
Ng, +... ordered according to the principle of first differences, and the set of all irrational
numbers of the interval (0, 1) ordered according to their magnitude. Hence the conclu-
sion that (w*w)® is the type of the set of all irrational numbers ordered according to
their magnitude (Hausdorff [2], p. 151).
The infinite product of type w* of order types obviously has the property of as-
sociativity of its factors but not of their commutativity. It will also be observed that
two infinite products of type w* of order types may have all their partial products
respectively equal and still be different from one another. Thus for instance in each of
the infinite products 5
Spee oihogy. 6 ehiysl | sae SRA Oso,
all the partial products are equal to wm, and the products themselves are different from
each other since the first is obviously equal to wm and the second can easily be shown
to be of the type of the set C— {1} where C is the perfect nowhere-dense set of Cantor _
ordered according to the magnitude of the numbers belonging to it.
EXERCISES. 1. Prove that for every order type a0 there ‘exists an order
type €#~0 such that éa=&
Proof. It suffices to assume that =... aaa =a”.
_ 2. Prove that for every order type a 4 0 there exists an [order [type & ~ 0 such
that a& = é,
Proof. Since a # 0 we can of course take a= 8+1+y where # and y are order
types which can be equal to 0. Let us denote by é the sum of the infinite series of type
o*+q of order types
E=..+ aB+ oP B+ aB+ at ay+ ay+ ay+...
§ 8. Infinite products of order types 247
2. Give an example of infinitely many different order types any two of which
are remainders of each other.
Solution: Types n+ w*w where n= 0,1, 2,...
3. Prove that types /?, 2?-+- A, #?+A-+ 1 are different and each of them is a segment
of each of the other two.
Proof. The proof should be based on the equality 4= A+ 1+ A, whence
BHA=(REAL +A, BPSAAt1+A) = (PLATE = (PEAT D)+AT®).
THEOREM 1. Of two segments (remainders) of the same order type
at least one is a segment (remainder) of the other.
Proof. Let a=o+o@=o,+0,. Let A denote an ordered set of
type a. From the definition of the sum of order types it follows that
there exist decompositions of the set A, A= S+R=8,+R,, such that
[S,R] and [8,,R,] are cuts of the set A. But, as we have proved in
exercise 2, § 5, Chapter XI, we have either SCS, and ROR, or SOS,
and RC R,, whence it follows that either the set S is a segment of the
set S, and the set R, a remainder of the set F or the set S, is a segment
of the set S and the set R a remainder of the set #,. In the first case type o
will be a segment of type o,, and type 0, a remainder of type o, while in
the second case type o, will be a segment of type o and type o a remainder
of type 0.
THEOREM 2 (of A. Lindenbaum). If an order type a is a segment of an
order type B, and the order type B a remainder of the order type a, then a= f.
In other words, for order types the equalities
(9.3) B= 8 +7(8i:)+P(8)
+... +78) + 7"4(B)
+ ("(B) +... + f(R) +h «
§ 9. Segments and remainders of order types 249
It follows from formula (9.2) that f"(S,) C f*(B) and /”+*(8,) C fet+*(B);
in view of S,f(B) = 8,B,= 0 and of the invertibility of the function /
we obtain /"(S,)/*+1(B) = 0, and since f(B)C B, whence /{*+*(B) C fe+1(B)
for natural k, we have /*+*(8,)C f-+1(B) and thus /*(8,)/"+*(S,)= 0 for
n= 0,1, 2,... and for natural k. Therefore from formula (9.3) it follows
that BD 8,+/(S,)+f(S,)+...; the components of this sum are disjoint
and of two of its elements belonging to different components always
that one comes first which belongs to the component that comes first, every
element of this sum preceding every element of the set B that does not
belong to this sum. Hence follows the existence of a set 7 such that
Solution. Let & denote the set of all rational numbers ordered according to
magnitude. In virtue of Theorem 1 of III.2, we can arrange the numbers of the set &
in an infinite sequence R = {7,,7,...}. For n=1, 2,... let us denote by ZN the set
of m systems (7,, 1), (7,5 2). ---, (7,, 7). Let
and let us order the set A by a relation 0 defined as follows: (7,,,k)o(r,,4) if 1, <p
or if 7, = 7, and k < l. We say that the set A(g) has 2° different segments and as many
different remainders.
Indeed, let » and y be two real numbers, x < y, and let us denote by A(x) the
segment of the set A composed of those of its elements (r,, &) for which 7, < x. Since
a2 <y, then there exists a natural number p such that « <r, < y. Hence it obviously
follows that the segment A (y) of the set A contains a component (in the sense of Exer-
cise 25 of XIT.3) 4, , consisting of p elements, but the segment A(x) of the set A con-
tains no such component. Therefore A(x) 4 A(y) for « ~y. Thus the order type A
has 2° different segments. We prove in a similar way that the type A has 2‘ different
remainders.
7. Prove that if a and f are order types such that a-2 = 8-2 then a= f.
Proof. Since a-2= 6-2, a+a= 6+. Types a and f are segments of the same
type. Thus it follows from the proof of theorem 1 that either a is a segment of type 6,
and $ a remainder of type a, or f is a segment of type a, and a remainder of type f. In
both cases, in virtue of theorem 2, we shall have a= f#.
8. Prove that if y is a finite order type, then for any order types a and f the for-
mula a+ y = 6+ y implies the formula a = f, and the formula y+ a= y+ 6 also implies
the formula a = 8, while if y is an infinite order type, then the following cases may
hold: 1) for any order types a and 6, at+y=f+y implies a= f, and y+a=y+ 6
implies a = f, 2) for any order types a and 6, a+ y= f+y implies a= 6 but there
exist order types a, and f, such that y+ a,= y+ f, but a, ~ f,, 3) for any order types a
and B, y+a=y+ implies a = 6 but there exist order types a, and 8, ~ a, such that
a+y=f,+y, 4) there exist both order types a, and 6, € a, such that qa+y=f,+y
and order types a, and B, ~ a such that y+a,= y+ py.
Proof. The proof of the first part of the exercise presents no difficulty. For the
second part we have the following easily provable examples: 1) y=-w*+o, 2) y=
Or, (ne Ih hie Dl Wes Oy Ch sil) fsa, 2b) Mew Caea th Sse lh, CRE
Bo=1+7.
9. Find two order types o and 6 £a such that a+7y=f+y and ynta=n7+8
Answer. a=1,f=1+4+ 7+1.
of order types which are left-hand divisors of one another: we shall give
one in Chapter XIV.12. We do not know whether there exist two different
denumerable order types which are left-hand divisors of each other.
Neither do we know whether there exist two different order types which
are both left-hand and right-hand divisors of each other.
It is possible to give an example of a denumerable order type hav-
ing 2” divisors which are both left-hand and right-hand divisors, as
well as an example of a set of power 2 of denumerable order types such
that any type belonging to that set is a right-hand divisor of any other
type belonging to that set (Sierpinski [51]). It can also be proved that
there exist 2°° denumerable order types for which type 7 is a right-hand
divisor.
If a= gy, then the right-hand divisor y of type a is called comple-
mentary to the left-hand divisor » and conversely the divisor is called
complementary to the divisor y. Thus each (left-hand or right-hand)
divisor of a given order type has at least one complementary divisor.
For natural numbers, as we know, to every divisor corresponds a well-
defined complementary divisor, but a divisor of an order type may have
more than one complementary divisor. H. g. to the right-hand divisor
of the type w correspond infinitely many different complementary divisors,
since w = n-o for n= 1, 2,... (but to the left-hand divisor w of the type
® = ow-1 corresponds only one complementary divisor, 1). To the left-
hand divisor 7 of the type » correspond 2” different complementary
divisors as follows from Exercise 5 of XII.3. To the right-hand divisor
of the type w* correspond infinitely many different complementary divisors
since w? = (w-n)-w = (w+n)-o for n=1,2,... To the left-hand divisor
y+1 of the type 7 correspond infinitely many different complementary
divisors since for instance 7 = (y+1)-(@-+n) for n=1,2,..
EXERCISES. 1. Determine all the left-hand and all the right-hand divisors of
each of the types w, w*,7,7+1,1+7.
Solution. The left-hand divisors of type w are obviously the types 1, 2,3,...
and w (since w= nw for n=1,2,...) and those types only, the right-hand divisors
are only the types 1 and w. The left-hand divisors of the type w* are the types 1, 2, 8, ...
(since w*=nw* for. n=1,2,...), and the right-hand divisors are ,the types 1,
and w*. The left-hand divisors of the type 7 are the types 1,7, 1+7,7+1 and 1+7+1
(since: ny = (1+ )n = (1+ 7) = (1+7+1)y) and those types{only since leach {left-hand
divisor of a dense type is either equal to 1 or dense; thus type7 has’ exactly
five different left-hand divisors. The right-hand divisor of type 7 is every finite order
type 40 or denumerable type (cf. Exercise 5, XII.3); thus type 7 has 2°° different
right-hand! divisors. The left-hand divisors of type 7+1 are only the types 1, 7+ 1
and 1+7+1 (since 7+1=(1+7+1)(n+1) and eachi left-hand divisor of type
n+ 1 must be dense), and the right-hand divisors of the type 7+ 1 are all finite types
+0 and all denumerable types having a last element, and only those types. The left-
hand divisors of type 1+ 7 are [only the types 1, 1+ 7 and 1+ 7+1 (since 1+7 =
252 XII. Order types and operations on them
= (1+7+1)(1+7) and the right-hand divisors of the type 1+ 7 are all finite types 40
and all denumerable types having a first element, and only those types.
2. Prove that if 6 is a left-hand divisor of.an order type a ¢ 0, then there exist
order types 6 and y (which may be equal to 0) such that a = 6+6+y.
Proof. If 6 is a left-hand divisor of an order type a ~ 0, then there exists an order
type € £ 0 such that a = 6&. Since & # 0, there exist order types w and vy (which may
be equal to 0) such that €=yw+1+». Hence a= 6&= d(u+1+4+7) = du+6+6y and
taking du = B, dv=y, we have a= f+ 6+), q.e. d.
The order type 6 is called a portion of the type B+ 6+ y. The theorem proved
above can therefore be expressed as follows: Wach left-hand divisor of an order type
different from 0 is a portion of that type.
3. On the basis of Exercise 2 prove that the only left-hand divisors of type / are
the types 1, 4,4+ 1 and 1+4.
4. Prove that the types @, w*, w*+ @w*+o-2, w*+ w?, (w?)*+q@ are right-
hand divisors of type 4, and the type w+ w* is not a right-hand divisor of type 4.
Proof. The fact that the types w, w*, w*+ 0, o*+@-2, w*+o*, (w?)*+o are
right-hand, divisors of type 4 follows from the formulae 2= (A+ 1)w = (1+ A)wo* =
= (14-4) (@*+ @) = (1+ 4) (@*+ @- 2) = (1+ 4) (@*+ @?) = (A+ 1) ((@*)*+ 0), which can
easily be obtained from formulae (1+ A)m = 1+/, (a+ B)* = B*¥+a*, (aB)* = a*B* and
from the distributivity law.
It would be more difficult to determine all right-hand divisors of type. 4, because
they form a non-denumerable set.
In order to show that type w-+ w* is not a right-hand divisor of type 4 we should
use Exercise 3, which implies that if type w+ w* is a right-hand divisor of type A, then
at least one of the formulae 2=1-(w+o*), A=A(w+o*), A= (A+ 1)(@+ 0*), A=
= (1+ A)(@+ w*) holds; that, however, is not the case since 2 4 w+ w* and A(@+ w*) =
=A+AAZA, (A+-1)(@4+ w*) =A+A414/~, (14 A) (+o*) =14A4A Fd.
5. Prove that none of the types w, n+1, 1+7 and 1+7+1 is a product of
two order types different from it, but each of the types 7 and A is a product of two
order types different from it.
Proof. For type w the above immediately follows from the observation that this
type has only two right-hand divisors: 1 and w. In order to prove that type 7+ 1 has
the required property we shall observe that, as follows from Exercise 1, the only left-
hand divisors of type 7+ 1 are the types 1, 7+ 1 and 14+7-+1; thus if type 7+ 1 were
a product of two types different from it, then we should have 7+ 1= (1+7+1)f.
If the type 6 were not dense, then there would exist types y and 6 (which could be equal
to 0) such that 6 = y+ 2+ 6, whence 7+ 1 = (1+y+1)y+14+7+2+7y+4+14 (14+ 7+1)6
and the type 7+ 1 would not be dense, which is impossible. Thus type f is dense
and also it must have a last’ element since otherwise the type (1-++7+1)6 = 7+ 1 would
not have a last element, which is impossible. Moreover, the type f cannot have a first
element since otherwise the type n+ 1= (1+ 7+ 1) would have a first element. The
type B is at most denumerable as a divisor of a denumerable type. Thus it is a denumer-
able dense type having a last element and no first element. Therefore we must have
Bp =7-+1. Thus we have proved that type y+1 is not a product of two types dif-
ferent from it.
If we had 1+7= af where a41+7 and B#~1+y, we should have 7+1=
= (1+ 7)* = a*f*, where a* ¢ (1+7)* =n+1 and 6* #~7+1, which is known to
be impossible. Thus type 1+7 has the required property.
§ 10. Divisors of order types 253
Answer. Such are for example the types w and 7 since nw = yn, but for every
order type € 4 0 type wé is not dense while type 7& is dense. Such are also the types w
and w* since 7a = nw*, and it can easily be proved that for any order types 4 0
and , 40 we have wi 4 w*é,.
It would be more difficult to give an example of two order types which are com-
mon left-hand divisors of a certain order type but are not common right-hand divisors
of any order type, or an example of two order types which are neither common left-
hand divisors nor common right-hand divisors of any order type (see Chapter XVI.12).
9. Give an example of two order types such that among their common lIcft-hand
divisors there is no divisor for which each common left-hand divisor of those types
would be a left-hand divisor.
Answer. The types w and w*, since their only common left-hand divisors are
natural numbers.
10. Prove that if a and f are two order types, then: a) a common left-hand divisor
of the types a and f is always a left-hand divisor of type a+ f, b) a common left-hand
divisor of the types a and a+ £ is not necessarily a left-hand divisor of type f, ¢) a com-
254 XII. Order types and operations on them
mon left-hand divisor of the types 6 and a+f is not necessarily a left-hand divisor
of type a.
Proof. a) If a= da, B = 6f,, then a+ Bf= d(a,+ f,).
b) We have 7+ (1+) =7 and type 1+ 7, having a first element, cannot have
the left-hand divisor 7.
c) We have 1+w:1=q@-1 and for type 1 type o is not a left-hand divisor.
11. Prove that if a and B are two order types, then a) a common right-hand divisor
of the types a and f is not necessarily a right-hand divisor of type a+ f, b) a common
right-hand divisor of the types a and a+ is not necessarily a right-hand divisor of
type 8, c) a common right-hand divisor of the types 8 and a+ f is not necessarily a right-
hand divisor of type a.
Proof. a) Type 2 is a common right-hand divisor of the types n = 7-2 and 2,
but is not a right-hand divisor of type 7-++ 2 since if we had 7+ 2 = y- 2, we should have
n+2=y+y, whence y ~ 1 and thus type 2 would be a remainder of type y; there-
fore y=6+2, whence »=6+2+6, which is impossible {since 6+2+6 is not
a dense type: ;
b) We have w-2+1= (w+ 1)-2 and type 2 is not a right-hand divisor of type 1.
c) We have 1+1-m=1-q@ and type w is not a right-hand divisor of type 1.
12. Prove that type 1+7-+1 is a left-hand divisor of type 7+ 2+ 7.
Proof. The proof follows from the equality 7+ 2+7 = (1+n7+1)(n+2+7).
The proof of some theorems concerning divisors of order types is difficult, e.g.
the proof that every order type whose right-hand, divisors are the types 2 and 3 has
the right-hand divisor 6. A. Lindenbaum has given without proof a more general theo-
rem stating that if & and J are relatively prime numbers, and a and # are order types
such that ak = fl, then there exists a type € such that a = &l, 6B= ék (Lindenbaum
et Tarski [1], p. 321, Th. 14).
It can be proved that if n is a natural number and a and £ are order types such
that na = nf, then a = B. We could also replace the number n by the types w, w*, w*+
+q@,x7-+2, but we could not replace it either by n or by +1 since 7-2 = 7-1, while
(n+ 1)-2 = (y+1)-1. It can also be proved that if n is a natural number and a and 8
are order types such that an = fn, then a= f. But aw = fw does not imply a= 8
‘because 1-w = 2-q@; still, it can be proved that a(w+1)=£8(w+1) always implies
a =f and also a(1+o*) = B(1+o*) implies a = 6B (Sierpinski [45], p. 1-2).
11. Comparison of order types. Let a and / be two given order types,
A and B — ordered sets such that 4 = a, B=. One and only one of
the following cases holds:
1) The set A is similar to a certain subset of the set B but the set B
is not similar to any subset of the set A.
2) The set B is similar to a certain subset of the set A but the set A
is not similar to any subset of the set B.
3) The set A is similar to a certain subset of the set B and the set B
is similar to a certain subset of the set <A.
4) The set A is not similar to any subset of the set B, and the set B
is not similar to any subset of the set A.
It is easy to prove that if one of the above cases holds for the ordered
sets A and B and if A, and B, are ordered sets such that A,~A and
§ 11. Comparison of order types 255
B,~B then the same case holds for the sets A, and B,. Hence it follows
that the question which of the cases k (where k=1,2,3,4) holds de-
pends only on the order types a and 6 and not on the choice of ordered
sets A and B such that A=—a, B= f.
If it is case 1) that holds for the order types a and #6 we shall say,
after R. Fraissé [1] (p. 1330) (cf. also N. Cuesta [2], p. 132), that type
a is smaller than type f and type f greater than type a, and we shall
write a < 6 and 6 > a. Thus if it is case 2) that holds for the order types a
and f, then 6 <a and a>f.
If it is case 3) that holds for the types a and f, we shall say that
the types a and f are equivalent and we shall write a~f. Obviously in
that case we also have B~a.
If it is case 4) that holds for the types a and f, we shall say that
the types a and f are incomparable and write all.
It is easy to prove that if we have, for the order types a, 6 and y,
a—p and 6<y, then a4<y. If a<, then neither ~ < a. nor_a~f").
Tfia<f and f~y, thena<y;if a< 6 and a~y, then a < f. Thus order
types are partially ordered by the relation <. The relation of equivalence
of order types is obviously transitive.
EXAMPLES. It can easily be seen that if n is a natural number and a denotes
an infinite order type, then n < a. From Theorem 2 of XI1.8 it follows that for every
denumerable order type a we have a < yn ora 7. If a, B and y are order types such
that a = B+ y, then obviously 6 < aor Braand y <a orywxa. Since n = (n+ 1)+7,
we obtain 7+1# 7; thus two different types can be equivalent.
Each subset of an ordered set of type w is known to be either finite or of type w.
Each subset of an ordered set of type w* is either finite or of type w*. Since w 4 w*,
we conclude that the types w and w* are incomparable: || w*.
EXERCISES. 1. Prove that for every infinite type a one of the following four
formulae holds: a>w,axo,a>o*,avra*.
6. Give an example of an infinite series of order types a,+ a,+... with the sum o
and of a type f# such that
Answer. a, = (w*)?, a,—=o* for n=2,..., B= (w*)?+ o.. For we have here
a+ a,+...+ a, = (w*)?+ w*(n—1) = w*(w*+4+ n—1) = (w*)? for n= 1, 2,..., while o=
= 4+ ag+... = (w*)?+ H*w, and (w*)? < (w*)?+ a < (w*)?+ w*o.
7. Give an example of an infinite series of order types a,-+-a,+... with the sum o
and of types 6 and y such that
Che p Gh onapn i eo BO NN Mo oa
a +a,+... <A (Sierpinski [53], p. 260, Th. 7, and p. 263, Th. 8). Now,
with the aid of the axiom of choice it can be proved that an ordered set
of type A is the sum of two subsets of itself, each of which is of type < 4.
Moreover, with the aid of the axiom of choice we can prove the fol-
lowing theorems.
There exist two order types a and f, of the power of the continuum,
such that a</<A and such that there is no order type & such that
o<€< 6 (Sierpinski [53];p-261, Th. 6).
There exist two order types g <A and y <A, of the power of the
‘continuum, for which there is no order type é, of the power of the con-
tinuum, such that <q and <y (Sierpinski [53], p. 261. Th. 10).
There exists a set, of the power of the continuum, of order types
<A each two of which are incomparable (Sierpinski [53], p. 258, Th. 4).
Furthermore: there exists a set, of the power of the continuum, of subsets
of a set of type 4 each two of which differ only in two elements and are
incomparable (Sierpinski [53], p. 258, Th. 5).
It can be proved that for every infinite order type there exists an
order type which is incomparable with it.
WELL-ORDERED SETS
2° a is not the first element of the set A. Thus there exist elements
of the set A which precede a; let a’ denote any of them. The element a’,
since it precedes a, cannot belong to OC, a being the first element of the
set O. Thus we have a’ none C= A—B, whence, in view of a’ « A, it
follows that a’ « B. Therefore each element of the set A that precedes a
belongs to B, whence, according to property 2), we have ae Bb, which is
impossible since ae C= A—B.
Thus the assumption that A CB does not hold leads to a contradic-
tion. We have proved that AC B.
Let P denote the property of a well-ordered set A according to
which A has a first element and every set B satisfying conditions 1° and 2°
contains the set A. We have proved that every well-ordered set A has
the property P. Let us now prove that every ordered set having the pro--
perty P is well-ordered. Let A denote an ordered set with the property P
and let A, denote any non-empty subset of the set A and let us suppose
that the set A, has no first element. It follows that the first element
of the set A (which exists in virtue of the assumption that the set A has
the property P) precedes each element of the subset A,. Let us denote
by B the set of all those elements of the set A which precede each element
of its subset A,; thus the set B will satisfy condition 1°. We shall prove
that it will also satisfy condition 2°. Indeed, let a denote such an element
of the set A that each element of the set A preceding the element a be-
longs to B. If we had aé¢ B, then the element a would not precede each
element of the set A,, and since, according to our assumption, the set A,
has no first element, there would exist in the set A, an element a, preced-
ing a; in view of the definition of the element a, we should have a, « B,
which is impossible since A,B = 0. Therefore ae B and from the defini-
tion of the element a it follows that the set B satisfies condition 2°. Thus
the set B satisfies conditions 1° and 2° and from the assumption that
the set A has the property P it follows that AC B, which is impossible
since Ay G Ay AT 2 0 and i406 0:
Thus we have proved that each non-empty subset A, of the set A
has a first element. Therefore the set A is well-ordered.
We have proved that if an ordered set A has the property P, then
it is well-ordered, and since we proved before that also the converse
holds, the property P is a characteristic of well-ordered sets.
Now let A denote a well-ordered set having a first element, and T —
any theorem satisfying the following two conditions:
1° Theorem T is true for the first element of the set A,
2° If a is such an element of the set A that theorem T is true for
each element of the set A that precedes a, then theorem T is true for the
element a.
§ 2. Principle of transfinite induction 261
and, by 1°, we shall have S £0. If S # A then, by 2°, there exists a se-
ement 7 of the set A such that SC 7C Band T 4S. But, in view of
TC B and the definition of the set S, 7’ is one of the components of the
sum S; therefore we have TCS, which contradicts SC TAS. Thus
we must have S= A and, since SC B and BCA, we obtain A= B.
Theorem 1 is proved.
EXERCISE. Prove that among the ordered sets A(@) sets having no gaps, and
only those, have the property P according to which any theorem T (on an element of
the set A) is true for each element of the set A if that theorem satisfies the following
two conditions:
1. there exists an element a« A such that theorem T is true for each element «x of the
set A such that xea,
2. if be A and if theorem T is true for each element x « A such that xeb, then there
exists an element ¢ « A such that bec and that theorem 'T is true for every element x of the
set A such that xee.
Proof. The fact that ordered sets having no gaps have the property P follows
from the proof of Theorem 2, in which we use only the property of the set of all real
numbers (ordered according to their magnitude) consisting in their having no gaps.
Suppose now that an ordered set A (go) has a gap; let [A,, A,] be a cut of the set A (@)
giving a gap. The sets A, and A, are thus non-empty, the set A, having no last element
and the set A, having no first element. Now let T denote a theorem on an element x
of the set A stating that x « A,. Theorem T satisfies condition 1) since the set A, is
non-empty and is a segment of the set A; thus if a denotes any element of the set A,,
then theorem T is true for each element x of the set A such that xoa.
Suppose now that 6b « A and that theorem T is true for every element x « A such
that web. We cannot have b « A,, since in that case, the set A, having no first element,
there would exist an element b, « A, such that b, eb; in view of b,eb and of our assump-
tion concerning the element b, theorem T would be true for the element b,, which is impos-
sible because in view of b, € A, we have b, € A,. Therefore b « A,, and since the set A,
has no last element, there exists an element ¢ « A, such that bec. In view of ¢ e A, and
of the fact that the set A, is a segment of the set A we conclude that x « A, for xoc,
i.e. that theorem T is true for each element x of the set A such that xec. Therefore
theorem T satisfies condition 2).
Thus theorem T satisfies conditions 1) and 2) and yet it is not true for each element
of the set 4 since it is not true for any element of the non-empty subset A, of the set A.
Therefore the set A does not have the property P. We have proved that if an ordered
set A(ge) has a gap, then it does not possess the property P.
‘This completes the required proof.
*
264 XIII. Well-ordered sets
each non-empty set of segments of a given well-ordered set there exists a smal-
lest segment.
If two well-ordered sets are similar, then to each segment of one
set corresponds a similar segment (only one) of the other set. Now we
shall prove the inverse theorem, namely
THEOREM 1. Jf, for each segment of a well-ordered set A(e), different
from A, there exists a similar segment of a well-ordered set A’(o’), different
from A’, and vice versa, then the sets A and A’ are similar.
Proof. Let a denote an arbitrary element of the set A; according
to our assumption, for the segment A, of the set A there exists a similar
segment Sj of the set A’, and, as we know, only one such segment. The
element a’ of the set A’ is thus well-defined by the element a of the set 4;
let f(a) = a’. The function f establishes a similar mapping of the set A
on the set A’. Indeed, let a’ denote an arbitrary element of the set A’;
to the segment Aj, of the set A’ corresponds, in virtue of our assumption;
a similar segment A, of the set A and it follows from the definition of
function f/ that f(a)= a’. Thus each element of the set A’ is an image
of a certain element of the set A. Finally if a, and a, are elements of the
set A such that a,oa,, then, as we know, A,, < A,, and, for aj = f(a),
az = f(s), Ag < Ag (since Ay,~A, and Ag,~A,,), therefore aj gaz,
whence it follows that each element of the set A’ is an image of only
one element of the set A and the function f establishes the similarity of
the sets A and A’. Theorem 1 is proved:
It will be observed that Theorem 1 is generally not true for ordered
sets; e. g. it is false for a set A of type 7 and a set A’ of type 7+1.
Now let A(oe) and A’(o0’) be two given well-ordered sets and sup-
pose that in the set A(o) there exists a segment, different from A, that
is not similar to any segment of the set A’(o) different from A’. Let S,
denote the smallest of such segments of the set A. We shall prove that
each segment of the set A’ that is different from A’ is similar to a segment
of the set A, different from Aj.
Indeed, if there exist segments of the set A’, different from A’, not
similar to any segment of the set A, different from A,, then let Sj} de-
note the smallest of them. Thus each segment Aj; of the set Aj, different
from Aj, being a segment of the set Aj that is smaller than Aj, is similar
to a segment A, of the set A, different from A,, and we have A, < A,.
Therefore each segment of the set Aj different from Aj/ is similar to
a segment of the set A, different from A,. But also each segment A,,
of the set A, different from A is, according to the definition of the se-
ement A,, similar to a segment Ag of the set A’ different from A’, and
it follows from the definition of the segment Aj that Ay < Aj. Thus
§ 5. Properties of segments of well-ordered sets 267
set A, are similar to a certain segment of the set A’ different from A’,
and that the set Ai, and therefore also the similar set A’ are similar to
a certain segment of the set A different from A. Hence it follows that
the set A is similar to a certain segment of a segment of itself, different
from A, contrary to Corollary 4 of XII.4. Thus we have proved Corol-
lary 2.
Corollary 2 is not always true for ordered sets, e.g. it is not true
for sets of which one is of type 7 and the other of type 7+1.
Theorem 2 and Corollary 2 immediately imply
COROLLARY 3. If A and A’ are two well-ordered sets, then either they
are similar or the set A is similar to a certain segment of the set A’ different
from A’ and the set A’ is not similar to any subset of the set A, or finally
the set A’ is similar to a certain segment of the set A different from A, and
the set A is not similar to any subset of the set A’.
Now, if we recall the definition of inequality for order types, given
in XII.11, from Theorem 1 of XIII.3 we immediately deduce
COROLLARY 4. If A and A’ are two well-ordered sets, then one (and
only one) of the formulae
AT We A aA Vee Wea
holds.
In particular, the inequality A < A’ holds for well-ordered sets A
and A’ if and only if the set A is similar to a certain segment of the set A’
different from A’. From Theorem 2 also immediately follows
COROLLARY 5. If A and A’ are two well-ordered -sets, then always one
(and only one) of the formulae
Ae ba Lowry
holds.
Thus for the powers of two well-ordered sets we have trichotomy.
CHAPTER XIV
ORDINAL NUMBERS
EXERCISE. Prove that if a,, aj, a3, ... is an infinite sequence of ordinal numbers,
then there exists an increasing infinite sequence of natural numbers k, < ky, < ks < ....
such that a, Ken <a, Kn sy for n= 1, 2,... (in other words that from an arbitrary: infinite
sequence of ordinal numbers we can pick out a non-decreasing infinite sequence).
Proof. Suppose that there exists an infinite sequence picked out from the sequence
Ay, Az, ++. &. g. the sequence a,, a, ... where I, <I, <..., in which there is no term
that is not smaller than each term of the given sequence. Then for the term a, there
exist terms a, such that a, <a,. Let a, denote the first of them; we shall have a, <
s s 81 a
<4, for 1 < s,, whence, in view of aq, <a , we shall have a, >a, fori<s,. If
: sy 8 G
we had a >a, for i> s,, then the term a would not be smaller than each term
sy v &
Increasing indices s, < s, < s;,..., Such that a. <a, <..., therefore there exists an
increasing infinite sequence picked out from the sequence a,, a. ...
. . . . . . ‘1 52
Thus if it is impossible to pick out from the sequence a,, a, ... an increasing in-
finite sequence, then every increasing infinite sequence picked out from the sequence
@,, M,... contains a term that is not smaller than any term of the extracted sequence.
In particular, in the sequence a,, a, ... itself there exists a term a,,, > a, fori = 1, 2,...
Removing from that sequence the terms q,, a, ..., An» We shall obtain an infinite se-
quence G45 Oyo ss picked out from the sequence a,, a, ... in which there exists a term
Gn, = 4; for t > m,. Similarly in the set a, 1,4, 19+ there exists a term a, > a; for
«> m, ete. In this manner we finally obtain an infinite sequence a, > 4, > On, = +
and m, < m, < ... In this infinite sequence the sign > cannot appear infinitely many
times, since then we should obtain a decreasing infinite sequence of ordinal numbers,
which is impossible. Hence it follows that from some place in the sequence Ons = oe Shh
the sign = must appear. Thus there exists a non-decreasing infinite sequence (of equal
terms) picked out from the sequence a,,4:,... °
The required proof is thus complete.
1) For example, we can assume B, to be the set of all ordinal numbers <a, and
take A, = B, x {§}. ;
272 XIV. Ordinal numbers
to the same component A, of the sum S that order relation which existed
between them in the set Az is retained, and of two elements belonging
to different components of the sum S we shall regard that one which
belongs to the component with the smaller index as preceding the other.
From the definition of the sum of types, given in XII.7, it follows that
the type of the set S ordered in this way will be o. Thus in order to prove
that o is an ordinal number, it remains to show that the set S is well-
ordered.
Let 7 denote any non-empty subset of the set S. Since SD T # 0 and
S= »' Ap,
0<é<
there exist indices € <q such that A;T ~0, and among them a smallest
index a (since the set of all ordinal numbers & such that 0 <é<@ is
well-ordered according to their magnitude). Thus we have A,T <0.
The set A,7, as a non-empty subset of the well-ordered set A,, contains
a first element, a. Obviously a is also the first element of the subset 7’
of the set S. Therefore each non-empty subset of the set S has a first
element, which proves that the set S is well-ordered, q.e.d. Thus we
have proved
THEOREM 1. The sum of every well-ordered sequence of ordinal numbers
is an ordinal number.
From Corollary 1 of XIII.5 immediately follows
COROLLARY 1. The sum of a well-ordered sequence of ordinal numbers
is not smaller than any of its components.
Thus if we add number 1 to such a sum, then in view of o+1>6
(since o is the order type of a segment of a set of type o+1 different
from that set), we obtain
COROLLARY 2. For every set of ordinal numbers there exists an ordinal
number greater than any number of that set.
The sum of two or more ordinal numbers depends in general on the
order of the components. The function /(n), expressing, for natural n,
the greatest number of different values that can be assumed by the
sum of » ordinal numbers in all the n! permutations of its components
has been investigated (Wakulicz [1], p. 254, Wakulicz [3], Erdos and
Rado [1], p. 127). It has been calculated that f(1) = 1, f(2) = 2, f(3)=5,
F(4)= 13, f(5) = 33, f(6) = 81, f(7) = 193, f(8) = 449, f(9) = 337, f(10) =
= 33°81, f(Cil) = 81%) (12) = SES103.- 4 (3) = 19528 =) (14) aoe sae
J(LD) = 33-18 17,5f (16) S09 C7 re Od 9S.) ULE) ae Mle ee
= 193%, #(20) = 33-813 and |
jin) =S8li(m—5), for m2 21.
§ 3. Sum of ordinal numbers 273
Thus we know the function f(7) for all natural numbers n. We have
f(n) <n! for n>3 and
limf(n)/nt = 0.
For each natural number k <5 there exist three ordinal numbers
whose sum, in all their permutations, gives k different values (Sierpin-
ski [48], p. 252). Similarly, for each natural number k < f(4) = 13 there
exist four ordinal numbers whose sum, in all their permutations, gives
k different values (Wakulicz [2], p. 23). But, as has been proved by Wa-
kulicz [1] (p. 261), there are no five ordinal numbers whose sum assumes 30
different values, although there exist five ordinal numbers such that
their sum, in all their permutations, assumes 31 different values. (Such
are for instance the numbers w?,@+1, w-2+2, w-3+3, w-4+4).
3. Find how many different values are assumed by the sum of numbers 1, 2, 3,
4,q@ in all their permutations.
Answer. 11.
otota:-3+o-5+(0-2+1)=e?+o-ll+1.
8. Prove that the sum of n ordinal numbers w-+ 1+ 2+ 22+...+ 2”~*, in all the
possible permutations of the components, assumes 2”~* different values (namely it may
assume each of the values w+ k where k = 0, 1, 2,3, ..., 2"-*—1 and only those values).
Cardinal and ordinal numbers 18
274. XIV. Ordinal numbers
where y #,, ¢. g. y > yi. There exists, aS we proved just now, at least
one ordinal number 6 > 0 such that y = y,+6, whence, in view of (4.1)
and the associativity of the addition of order types:
a=B+y=6+(y146)=(6+y)+é6=a+6,
which is impossible since, by theorem 1, we have a+6> a.
Thus Theorem 2 is proved. As an immediate conclusion we obtain
COROLLARY 1. If, for ordinal numbers B, y and y,, the formula B+y =
= 6+, holds, then we must have y = ¥,.
It would also be easy to prove that if 6 is an ordinal number, and y
and y, are order types such that 8+ y= 6+ y,, then y = y,. Now, w*+1=
== SM
EXERCISES. 1. Prove that if a, 8, y and 6 are ordinal numbers such that a> B
and y > 6, then a+y>+6, and if a>f and y > 6, then a+y > B+.
2. Give an example of ordinal numbers a, f, y and 6 such that a > 8, y > 6 holds
but the inequality a+ y > B+6 does not hold.
Answer. a= 2, B= 1, y= 6=o. We have here 2+ 0= 1+ o.
3. Prove that if a and f# are ordinal numbers such that a > B, then we have a+
+n>B+n for n=1,2,...
Proof. As we know, between the numbers f and 8+ 1 there is no intermediate
number; therefore if a > B then we cannot have a < 6+1, t.¢. a>f+1, whence
atn>f6+1+n. But, since 1+n > n, we have 6+1+n > B+n. Thus a+n> B+n,
Qa e.2d.
4, Prove that for an ordinal number a the equality 1+.a = a holds if and only
ifalSoa.
Proof. On one hand, if a < w, then number a is finite and we have l+a> a.
On the other hand, if a > mw, then, by theorem 2, there exists an ordinal number y > 0
such that a=w-+y whence l+a=1+(0+y)=(l+o)+y=o+y=a.
5. Give an example of ordinal numbers a and B > a 1) such that a+ 8 < B-+4a,
2) such that a+ $f > B+a.
Answer. 1) a> 1 and 1+a#<o+1, 2) o+2>o+1 and (#+1)4(@+2)>
> (@+ 2)+ (@+ 1).
18*
276 XIV. Ordinal numbers
(2-lhoa=o#~2-o—-1-wo=0,
(2—1)(@+
1) =@4+14 2(@4+ 1)—(@4+1)=1.
13. Prove that if y is an order type whichis not an ordinal number, then there
exists one and only one decomposition y= a+y where a is an ordinal number >0
and y is an order type having no first element.
Proof. Let F denote an ordered set of type y. Let us assign to class A each element
ae such that the set of all elements of the set F that precede a is well-ordered.
Let B= f—A; if b<« B, then the set of all elements of the set F that precede 6 is
not well-ordered (since in that case we should have b« A, which is impossible) and
therefore we obviously cannot have b < a or b= a for any a« A. Thus a 3 b for ae A,
b « B. We have here a cut [A, B] of the set F (class A may be empty).
The set A is well-ordered, for if it contained a subset a,> a,\a,;© ... of type w*,
then the set of all elements of the set # preceding a,, as containing the subset a,,
Gz, +.., of type w*, would not be well-ordered, contrary to the fact that a, « A. The set B
cannot be empty since in that case we should have Ff = A, in contradiction to the as-
sumption that the type @ of the set # is not an ordinal number. The set B has no first
element since for such an element 0b the set of all elements of the set F that precede b
would be identical with the set A, 2. e. it would be well-ordered and we should have
b ¢ A, which is impossible. Let A = a, B = y; thus p = a+y where a is an ordinal num-
ber y — an order type having no first element. Thus the required decomposition exists.
Suppose now that we have two different decompositions: p=a+ty=at+y
where a and, a, are ordinal numbers and py and y, are order types having no first ele-
§ 4. Properties of the sum 277
ment. From XII.2, Exercise 10, it follows that there exists an order type y such that
either a, =a+y and p=y+y, or a=at+y,w=yty.
If we had y = 0, then we should have, in either case, a, = a and y = y,, which
is impossible because the decompositions a+ y and a,+y, are different from each other.
Therefore y ~¢ 0. In the first case the formula y = y+y, implies that the type y has
no first element, which is impossible since a, = a+ y and a, is an ordinal number. In
a similar way we obtain a contradiction in the second case. Thus there is only one
decomposition of type g having the required properties. The desired proof is complete.
14. Prove the following theorem of Z. Ohajoth: In every infinite ordered set we
can change the place of one element so that the order type of the set will be changed (Cha-
joth [1], p. 133).
Proof. If the set in question is well-ordered of type a> wo, then 1+a—=a and
a—l=a, therefore (a—1)+1—a+4+1>a; thus moving the first element of the set
to the last place we shall change the type of the set. And if the ordered set in question
is not well-ordered, then, in virtue of Exercise 13, it is of type a+y where a is an
ordinal number and y an order type having no first element. Thus there exist order
types y, and yw, such that y= y,+1+y,., type y, having no first element, whence it
follows that a+1+y,ty,4a+y,+1+y4,. By changing the place of one element
of our ordered set we can obtain an ordered set of a different type.
15. Prove that every ordered set is a subset of an ordered set of a different type,
having only one element more than the given set.
(5.1) a=étQ;
the number & satisfying equation (5.1) is called a segment of number a
corresponding to the remainder o. For given a and o there may exist
more than one ordinal number é satisfying equation (5.1); e.g. for a=
= w-2, 0 =o formula (5.1) holds for =o and for é=w-+n where
n=1,2,...; thus a segment is not uniquely determined by a number
and its remainder, but among the segments of a given number cor-
responding to the given remainder of it there always exists a smallest
segment. H.g. the smallest segment corresponding to the remainder w
of number w-2 is number ow.
In virtue of Corollary 1 of XIV.3, formula (5.1) gives o <a; thus the
remainders of an ordinal number are not greater than the number itself.
THEOREM 1. Every ordinal number has a finite number of different
remainders.
Proof. Suppose that o and 0, are two different remainders of num-
ber a and that, for instance, o > o,. Thus there exist ordinal numbers &
and é, such that a=é+o and a= é,+0,, whence §+9=—£¢,+0,. We
shall show that & < é,.
278 XIV. Ordinal numbers
(5.2) a=€+e=¢,+6
and > é, since (see the proof of theorem 1) to the greater remainder
corresponds the smaller segment. Thus, by theorem 5, there exists an
ordinal number 6 > 0 such that € = &,+ 6 and formula (5.2) gives 6,+6+
+o= &+0,, whence, according to Corollary 1 of XIV.4, we have 6+ 0=
= 0,, which proves that o is a remainder of number @,, q. e. d.
§ 5. Remainders 279
Suppose now that 0 is a positive ordinal number such that for every
ordinal number & < @ formula (6.1) holds. If number o were not a prime
component, there would exist ordinal numbers mw and y such that @=
= pty, uw <o,»<o. By formula (6.1) for € < @ we should have »+ o= @
and w+o= oe, therefore
OO We?) Oe (0 8) = OO,
which is impossible since, in view of 9 > 0 and according to theorem 4,
we have 90+0 > o. Thus number 0 is a prime component. We have proved
that the condition of Theorem 1 is sufficient.
Theorem 1 is thus proved. It implies that a prime component absorbs
every component that goes immediately before it and is smaller than 1.
EXERCISE. Prove the following theorem:
If @ is a prime component and a any ordinal number, then
If we had 0’ < 0’, then, in view of the fact that o’ as the smallest
positive remainder of number a is, by Theorem 2 of XIV.5, a Pane
component and it follows from Theorem 1 that we should have 0+ 0’ = 0’,
therefore, by (6.6),
(6.7) (6, == a’ +o
§ 6. Prime components 281
ber a, contradicting the definition of a’. Thus we have proved that 0’ > 0’
must hold.
If we had a’ > 0, then, denoting by 0’ the smallest positive re- Q
whence, in view of (6.10) and of the observation that 0, > 0. >... > 6,
we should obtain 9 > a, contrary to the assumption that o@ <a. Thus
we have proved the following
282 XIV. Ordinal numbers
0 > @,. Thus @ > wu and @ >», and since 0, as a number of the set P, is
a prime component, we have, according to Theorem 1, a+o0= (uw+y)+
+o0=p+(y+e)= u+e=e, whence a < og, contrary to the definition
of number a. Thus Theorem 3 is true.
THEOREM 4. If a= 0,+0.+...4+ 0, is a decomposition of an ordinal
number a> into a finite series of non-increasing prime components,
then all the different positive remainders of number a, ordered according
to their decreasing magnitudes, are the numbers og+ Og41 +... +0, where
| TP etn
LEMMA. If a= e+06 where o@ is a@ prime component and if t is a re-
mainder of number a smaller than a, then t <6.
Proof of the lemma. If we had 1+ > 6, then, since (as has been
shown. in the proof of theorem 1 of XIV.5) to a greater remainder of
the same ordinal number always corresponds a smaller segment, the
segment 0, corresponding to the remainder 6 of number a, would be greater
than the segment € of number a, corresponding to its remainder t; thus
we should have 9 = +y where y > 0, whence (in view of the fact that
a prime component has no positive remainder different from itself), we
should have y= o. Therefore 0 = +0, which, in view of the formula
a=é4+17, gives €+0+6=¢+t1, whence, in virtue of Corollary 1 of
XIII.4: a= o+6=1T, in contradiction to the assumption that 1 < a.
Thus our lemma is proved.
Proof of Theorem 4. From the lemma it immediately follows
that if a= o+6 where o is a prime component and 6> 0, then there
is no intermediate remainder of number a between 6 and a; if, moreover,
6< a, then 6 is the greatest positive remainder of number a that is
smaller than a. In particular, let us consider decomposition (6.10) of
the number a into non-increasing prime components. Suppose that n> 1
and let
(6.12) 02 + O03 ++ + On = O43
we shall have, by (6.10):
(6.13) = 01+ 4;
which, as we know, gives a > a. If we had a= a,, formula (6.13) would
give a= o,+a, whence ’
1 2 n
gf
— i { ie lead .
O04 Oy oes Oy OS
10) TO ae e+ 1 01 5
therefore, in view of 0, > 0, >... > @, and (6.10), a < a, which is impos-
sible. Thus a, <a and therefore, according to the conclusion from the
lemma, formula (6.13) proves that a,, 7. e. number (6.12), is the greatest
positive remainder of number a that is smaller than a.
234 XIV. Ordinal numbers
Av lines,
E<A
im 7,==.0..
since 24 > a; for € < gy, numbers g; are all > 0, and since they are remainders
of number 4 (by (7.4)), we have oe; > @ for §& < gy. Number 4, as we know,
is of the 2-nd kind, therefore 0 is a transfinite number. All numbers o¢,
where <q, are thus not smaller than a certain transfinite number;
this fact alone proves that the limits of transfinite sequences behave in
a different way from the limits of infinite sequences of real numbers.
Since 0 is a remainder of number 4, there exists an ordinal number
ff > O90 such that
5
(7.5) A=p+e.
In view of (7.5) and 0 > 0, we have A > yw; thus, by (7.3), there exists
a number y for which formula (7.2) holds. By (7.2) and in wirtue of Theo-
rem 2 of XIII.4 there exists, for y <& <q, a number t; > 0 such that
lim(n+n)=o<o+o=lmn+lmn.
na<@w n<ow n<w
But we have
THEOREM 2. The formula
lim ag = A
é<p
(7.7) lim
E<q
(y+ag)= y +A.
Proof. Since
A —— hm ag
E<@p
we have A> a, for <q, whence y+A>y-+a, for é <q. On the other
hand, suppose that uw is an ordinal number such that w > y+ a; for <9.
Therefore we have w > y and, according to Theorem 2 of XIII.4, there
exists an ordinal number 6 > 0 such that uw = y+ 6, whence y+6>y+a¢
for <q, which gives 6 > az for <q; since 4 is the smallest ordinal
number greater than any of the numbers a for <q we must have
6>A, whence w= y+d>y+4. Hence y+A is the smallest ordinal
number that is > y+ ag for < y, which proves the truth of formula (7.7).
Thus Theorem 2 is proved.
But the formula
limag = A
E<—p
does not imply the formula
lim(ag-+y) = 4+y
E<o
for every ordinal number y, since for instance, in view of o+1> a0,
we have
lim(n +1) < limn +1.
1<@ n<o
if ag > Bz then
lim de < lim Be ‘
E<p é<
8. Infinite series of ordinal numbers and their sums. Let {az}:<, denote
any transfinite sequence of type qm of ordinal numbers (not necessarily
increasing), where m is any ordinal number (not necessarily a number of
the 2-nd kind), and let o denote the sum of all numbers a: where & < 9g,
ordered according to the magnitude of their indices €. In virtue of Theo-
rem 1 of XIV.3, o will be an ordinal number. We shall write
(8.1) oe
Let
(8.2) = dog for v<g.
The numbers o, are partial sums of the series (8.1). Thus for instance
we shall have o, = a, +a,+...+a,-1 for natural n, og = a) +a,+a,+...,
Cota—— Og O75 Cas O~.9 = Ag Oy Gy 4- +--+} Ao + Anti Cmte + «.-
We shall prove that if g is an ordinal number of the 2-nd kind and
ag > 0 for <q, then
(8.3) oe = limo; ,
v<@p
S= )' Ag,
E<p
where A; is a set of type az, for é <q, each element of the set A; preced-
ing each element of the set A; for €<¢<y. Since t<a, t is the type
of a certain segment 7’ of the set S different from S. Let a denote the first
element of the (non-empty) set S—T; T will be the set of all the elements
of the set S that precede a. Since
there exists an index y <q such that ae A,, whence it follows that t is
the type of a certain segment of the set
pre
é<y
which is of type o,,1 and, in view of y <q and of number @ being of the
2-nd kind, we have y+1<p@. Thus t <o,41, which proves that t > 0,
for » <q is impossible. We have proved that formula (8.3) holds.
It will be observed that the sum of an infinite series of positive
ordinal numbers (unlike the sum of an infinite series of positive real
numbers) can change its value with the change of the order of its com-
ponents. For example, since w+? = w(1+o) = w?, we have
o+o?+14141+..=0?+a,
but
w+towti+i4+14+..=e?+o-2>ae%+o.
EXERCISES. 1. Prove that the infinite series (of type w) w+ 1+2+3-+... does
not change its sum with any permutation of the components.
2. Prove that the sum of the transfinite series of type w+ 1:
assumes infinitely many different values with the change of the order of its components.
Proof. Let m-denote any given natural number. Interchanging the components n
and w in.the series (8.4) we shall obviously obtain the series (of type w+ 1) 14+ 2+...
t(m—1)+@+ (n+ 1)4+ (n+ 2)+...+”=@-2+n, and for n = 1, 2,... all these sums are
different from one another.
3. Prove that the sum of the transfinite series of type w-2
o-l+o@-2+o@-34+..+142434.
assumes infinitely many different values with the change of the order of the components.
Proof. Interchanging the components w-n and 1, we shall obviously obtain from
our series a series (of type w-2) whose sum is w?+ (n+ 1), and these sums are all
different from one another for n = 1, 2,...
4. For every natural number n give an example of an nfinite series (of type w)
of positive ordinal numbers whose sum, in all the permutations of the components,
assumes exactly n different values.
Answer. The infinite series a)+a,+a,+... where a =o*,a,=@ for i=l,
2,...,n—1, and a, = 1 fori > n. Indeed, if after a given permutation of the components
of this series. the number of all the components equal to wm and preceding the component w?
is k (where 0<k<n-—1), then the sum of the series is obviously w?+ w(n—k) and
these numbers are al) different from one another for k = 0,1, 2,...,n—1
Dee
E<a
where a is a given ordinal number are called triangular. Determine all the successive
transfinite triangular numbers < ?.
Answer. 0, :2, o-3+1, w:-44+2,..., w-n+(n—2),..., w?, w?+@-2, w?+
+o:-4+1, w?+o-6+2,..., w+q@-2n+ (n—1),..., w?-2, w?-21+0-3, w?-2+@-6+
+1...,, w?-2+@-3n+ (n—1),..., w?-3,..., w?-k, wo? -k+ow(k+ 1), w?-kt+o-2(k+1)+
+1,..., wo -kto-n(k+1)+ (n—1), ..., w%.
Remark. The number
on Ds E<w2
is the smallest transfinite triangular ordinal number which is the cube of an ordinal
number. In virtue of a theorem of Fermat, there exist no finite triangular numbers >1
that are cubes.
au —
» ES "=",
E<w”
4. What values can be assumed by the product of two positive ordinal numbers a
and # for which a+fB=o-+1.
Answer. w+k where k= 0,1, 2,...
whence it follows that the number (w+ 1)”-2” lies between two successive n-th powers
n
13. Give an example of two ordinal numbers a and # such that for every natural
number > 1 the number af” is, and the number f”a” is not, the n-th power of an
ordinal number.
Solution. a= +1, B = o. For we have here (w+ 1)°w” = (?)", but @"(@+ 1)” =
= oot nore cody ha!
(o@ + o) = Or RG ee eo (a+ 1)" < OG as
+o+1= (o?+o-+ 1)", whence it follows that the number w"(w+ 1)” lies between the
n-th powers of two successive ordinal numbers and therefore it cannot itself be the
n-th power of an ordinal number.
14. Give an example of two ordinal numbers a and 6 such that a+6=f+a
but a?+ 62+ 64 a.
Answer. a=o+l1, B=a@-2+1. For we have here a+ f=f6+a=—o:-3+1,
a+B? = w7?-3+ 0-24 1'> @-3+4+ 0+ 1 = f+ a.
15. Prove that there exist infinitely many pairs of ordinal numbers a and f such
that a? = 6? and that we do not have a = 73, 6 = 7, where t is any ordinal number.
Proof. a=o”"+o"", p=o"+o, where n=1,2,.., or a=o'+o”-24
bo 29 p=—o
+o -24-2 for n= 1,2.
EXERCISES. 1. Prove that if, for the ordinal numbers a and 6 we have a-2=
== /HoD, WN -G == [Xe
Proof. The proof immediately follows from Corollary 3.
2. Prove that for ordinal numbers a and f the formulae af = fa and a?f? = fa?
are equivalent.
Proof. If af = Ba, then af? = a(aB)B = a(Ba)B = (af) (aB) = (Ba) (Ba) = B(aB)a =
= B(Ba)a = Ba’.
On the other hand, if aB > fa, then a?6? = a(aB)6 > a(Ba)6 = (aB) (af) > (Ba) (Pa)=
= fp(ap)a> B(fa)a= fa", therefore a?B? > Ba?.
Similarly, if Ba > af, then fa? > a26?. Thus a?6? = fa? implies af = fa.
3. Prove that for a natural number » and ordinal numbers a and f the formulae
ap = Ba and a’p” = "a" are equivalent 2).
1) More generally, m and n being natural numbers, and a and f ordinal numbers,
the formulae af = Ba and a™p” = f"a™ are equivalent.
§ 11. Theorem on the division 295
(11.2) at +a, = a.
Solution. The natural left-hand divisors of the number of the 1-st kind B=
= w&+k are divisors of number k and only these. Indeed,} if I|k, k =1k,, then B=
= wE+k = I(wE+k,) and 1 is a left-hand divisor of number f#. On the other hand, if
the natural number J is a left-hand divisor of number $ then 6 = w&+ k = I(wé,+ k,) =
= wé,+ lk, and it follows from Theorem 2 (for a = w) that = & and k = lk,, which
proves that J is a divisor of number k.
3. Prove that if a is an ordinal number >0, then for ordinal numbers f of the
1-st kind we have (a+ 1)6 > af and for ordinal numbers f of the 2-nd kind we have
(a+ 1)B = af.
Proof. If 6 is a number of the 1-st kind, 6 = y+ 1, then (a+ 1)6 = (a+ 1)(y+1)=
=(a+|l)ytat+le>ay+a+1=al(y+1)+1=a8+1> af,
§ 11. Theorem on the division 29%
If 6 is a number of the 2-nd kind, then we have 6B = wé. But since 2w = w and
a > 0 we have aw< (a+ 1)0< (a+ a)o = (a: 2)m = a: (2m) = aw, whence (a+ 1)o=
=ao and (a+ 1)6 = (a+ 1) wé = awE = af.
4. Prove that the product of two ordinal numbers >0 is a number of the 1-st
kind if and only if both factors are of the 1-st kind.
Proof. On one hand we have (a+ 1)(6+ 1) = (a+1)6+ a+ 1; on the other hand,
if aisa number of the 2-nd kind, then a = wé, whence, for B > 0, a8 = (w&)B = w(&f) is
a number of the 2-nd kind, and if 6 is a number of the 2-nd kind, then it follows from
the definition of the product of order types that (for a > 0 and f > 0) the type af
has no last element.
5. Prove that if a > 0 is a number of the 2-nd kind and 6 a number of the 1-st
kind, then (a+ 1)6 =af+1.
Proof. It 6 is a number of the 1-st kind, then we have 8 = wé+n where n is
a natural number, and according to exercise 3 we have (a+ 1)w& =awé. If a > 0 is
a number of the 2-nd kind, then we have a > and according to exercise 4 in § 4 we
have 1+a=a, whence (a+ 1)n = (a+1)4+ (a+ 1)+...+ (a+ 1)+ (a+ 1) = atoat..+
t+ta+(a+1)=a-n+1. Thus we have (a+ 1)6 = (a+ 1) (m+n) = (a+ 1) w€4+ (a4 1l)n=.
=awét+a-n+1=—a(woé+n)+1=af+1.
It will also be observed that if 6 is a number of the 2-nd kind, then it follows from
Exercise 4 that the equality (a+ 1)8 = aB+1 cannot hold for any ordinal number a,
while if a and £8 are numbers of the 1-st kind, then the above equality may be either
true or false, because, for instance, it is true for a= 6 = 1, as well as for B=ow+1
with an arbitrary a (since, in virtue of exercise 3, (a+ 1)@ = aw, whence (a+ 1)(@+ 1) =
= (a+ 1l)o+a+1=ao+a+1=a(w+1)+1), but it is false fora=—1, B = 2.
6. Prove that if 4 is an ordinal number of the 2-nd kind and & and n are natural
numbers, then (An)* =n.
Proof. If 4 is an ordinal number of the 2-nd kind, it follows from Corollary 1
that A= wé&, where é is an ordinal number >0. Thus, if for natural &k and n we
have (An)* = 2'n (which is true for k = 1), then (An)**? = (An)*An = A*ndn = A*nwén =
= win = i'*'n (since nw = w). Thus the proof follows by induction.
Brook, at
A= lim Qe ,
é<p
then we have 41> oa; for <q, whence for y>0; yA> ya; for <q.
On the other hand, suppose that wu > yo; for E<g. By Theorem 2 we
have u = y6-+e where 0 < y, whence y(6¢+1)= y6ty>yte=u> ya.
298 XIV. Ordinal numbers
for <q; therefore y(€+1) > yag for E<g, which gives ¢+1 > a; for
— <q. In view of
lima=i,
E<o
= yx0x~ < y16,, Which is impossible. Therefore 6, > 6,, and from Exercise 4
of XIV.4 it follows that numbers 6, and 6, must be of the 1-st kind (the
number a being of the first kind). Therefore 6,=—¢,+1, 6.=¢4+1.
Hence ypdg = yelCxt+1) = vebetye and y6;= wility. Since a= yd, =
= 10, we thus have y,xl,+y,= yidity. If we had ype, <j), then,
since yz< yw, we should have y,7;+y,% < yi; +71, which is impossible.
Therefore. we have y,¢, > y¢;; if we had ¢,<¢,, then, since »,< ,
we should have y,¢, <+,¢;, which is impossible. Therefore we have
én C7, whence op C. +1 > ¢,+1 = 0), v. €. 0, >0;. Therefore 6, > 0,
for k<l, whence 6, >06,>..., which is impossible. Thus an ordinal
number a of the first kind cannot have infinitely many different left-
hand divisors. But from Exercise 1 of XIV.10 it follows that every ordinal
number of the 2-nd kind has infinitely many different left-hand divisors.
Theorem 2 is thus proved.
Here is another proof of the theorem stating that every ordinal
number of the first kind has a finite number of left-hand divisors; this
proof is based on Theorem 1.
Let a denote a number of the first kind. As we know, to each left-
-hand divisor of the number a corresponds a well-defined complementary
right-hand divisor. If the same complementary right-hand divisor f
corresponded to two different left-hand divisors, y and y, > y of the num-
ber a, then we should have a= yd = y,6, and, according to Exercise 4
of XIV.10, 6 would be a number of the first kind; in view of y < y,, and
by Corollary 2 of Theorem 2 of XIV.10, we should have yd < y,6, which
is impossible. Thus to different left-hand divisors of the number a cor-
respond different right-hand divisors of that number, and since the
number of the latter, by Theorem 1, is finite, the number of left-hand
divisors of the number a is also finite, q. e. d.
EXERCISES. 1. Prove that every number of the first kind gives a finite number
of different decompositions into a product of two factors, and also into a product of
three factors.
Proof. The fact that every number of the first kind gives a finite number of dif-
ferent decompositions into a product of two factors follows from the observation that,
in every such decomposition, a = yd, the number y is a left-hand divisor and the num-
ber 6 a right-hand divisor of the number a, and by Theorems | and 2 a number of the
first kind has a finite number both of left-hand and of right-hand divisors. In every
decomposition a = fy6d of the number a into a product of three factors, the number £
is a left-hand divisor of the number a, the number 6 — a right-hand divisor of the num-
ber a, and the number y — a left-hand divisor of the right-hand divisor yd of the num-
ber a, which, according to Exercise 4 of XIV.10, is also a number of the first kind.
Hence, and by Theorems 1 and 2, it follows that the number of all decompositions of
@ number of the first kind into a product of three factors is finite.
2. Prove that, for every natural number m, every ordinal number of the first
kind gives a finite number of different decompositions into a product of m factors.
300 : XIV. Ordinal numbers
3. Give an example showing that a right-hand divisor of each of two given ordinal
numbers need not be a right-hand divisor of their sum.
Solution. The number w-w+o-o = @*-2 is not divisible on the right by o,
since if w?-2 = éw, we should have < w-w (because w?-2 < w?-w) and, by Theo-
rem 1 of XIV.11 we should have € = wk-+l, where k < wm and 1 < w, whence w =
= (wok+1)o = w < w?-2, which is impossible. But a left-hand divisor of each of two
given ordinal numbers is a left-hand divisor of their sum, since 6a+ 66 = 6(a+ ).
13. Prime factors of ordinal numbers. Every ordinal number >1 that
is not the product of two ordinal numbers smaller than that number
is called a prime factor or a number indecomposable with respect to multi-
plication.
If a> 1, then there exist right-hand divisors of the number a that
are greater than 1 (e.g. the number a itself), and among them there
exists the smallest, z,. It is easy to prove that z, is a prime factor. Other-
wise, in view of the definition of prime factors and of x, > 1, the number z,
would be the product of two ordinal numbers smaller than 2,: 2, = py
where uw <a, and »<2,, and since u<a,, we should have y>1 and
the number » would be a right-hand divisor of the number a, greater
than 1 and smaller than z,, which is contrary to the definition of num-
ber. z,.
Since z, is a right-hand divisor of number a, there exists an ordinal
number a, such that a = a,. If a4,=1, then we have a= 2, and the
number a is a prime factor. And if a, > 1, then, in view of z, > 1 and by
Theorem 1 of XIV.10, we obtain a > a,. Number a, can be dealt with in the
304 XIV. Ordinal numbers
1) A. Schoenflies in his book [1], p. 161, gives a theorem on the unicity of de-
composition, which is probably an ovérlooked mistake. Cf. F. Sieezka [1], p. 172.
Sis Prime factors 305
then, by Theorem 1, of XIV.10, we should haveau < a& and ay < aé;
therefore, in view of 90= ag, this would give au<o and ay <o, and
since, by (14.1), oe= a&=a(u+yv)=au+ay, the prime component o
would be the sum of two ordinal numbers smaller than 0, which is impos-
sible. Thus number & is not the sum of two ordinal numbers smaller than &,
z. €. it is @ prime component.
Thus Theorem 2 is proved.
(0,
0, 0) 3 (1; 0, 0) S (0, 1, 0) S (1, 1, 0) = (0,0, 1) = (1,0, 1) $(0, 1, 1)
S(1, 1, 1).
For 4 = 3, a= 2 we obtain nine two-term sequences:
(0, 0) S (1, 0) S (2, 0) S (0, 1) S (1, 1) S (2, 1) S (0, 2) S (1, 2) S (2, 2).
It can easily be seen that if the numbers uw and a are finite, then
the ordered set Z(u,a) will be of type wm.
For any positive ordinal numbers uw and a, let us agree to denote
the type of the set Z(u, a) (ordered by the relation <) by the symbol p..
Thus, for any positive ordinal numbers wu and a, u* will be a well-defined.
(by ~ and a) order type.
It can easily be seen that w'= mw for every ordinal number yu > 0.
Now we shall prove that for any positive ordinal numbers yu, a and f we
20*
308 XIV. Ordinal numbers
(15.1) FD ei lay 1
Let c= {Cg}sca+g denote any sequence belonging to the set Z(u, a+ f).
The term c, of the sequence ¢ splits this sequence into two sequences:
the segment a= {dag}ecq and the remainder b = {Cg}acgca+g- Clearly the
sequence a belongs to the set Z(u,a), and the sequence b (regardless
of the symbols used as indices) — to the set Z(u, B). Conversely, if we
take an arbitrary sequence b, belonging to Z(u, f), followed by an arbi-
trary sequence a, belonging to Z(u, a), then we should obtain the se-
quence ¢=a+b, belonging to Z(u, a+).
Furthermore, it is obvious that if e=a+b and c’=a’'+bD’ are
decompositions of the sequences ¢ and c’ belonging to Z(u,a+), then
we shall have c<ce’ in the set Z(w, a+) if and only if b <b’ in the set
Z(u, B), or if we have simultaneously b) = b’ and a<a’ in the set Z(u, a).
Hence, and in view of the fact that the sets Z(u, a), Z(u, 6) and Z(u, a+6)
are of types wt, uw’ and yt+? respectively, it follows that (according to the
definition of the product of two order types) formula (15.1) holds, q. e. d.
In particular, for 6 = 1, formula (15.1) gives
(15.2) | path = uu
for any positiwe ordinal numbers yw and a.
Now we shall prove that for any positive ordinal numbers u and a
the type wt is an ordinal number.
Suppose that for a certain positive ordinal number w there exist
positive ordinal numbers a such that uw is not an ordinal number. Thus
among such numbers there exists the smallest, which will be denoted by ».
If number » were of the. 1-st kind, »y=a+1, then by (15.2), we
should have w’= utu. From the definition of number »v it follows, in
view of a<y, that the type y* is an ordinal number; thus the type wp’
would be the product of two ordinal numbers, 7. e. an ordinal number,
contrary to the definition of number v. Thus number » is of the 2-nd kind.
Since yw’ is not an ordinal number, the set Z(u, v) (which is of type py’)
is not well-ordered. There exists in it a non-empty subset Z,, having
no first element. Let a= {ag}cc, denote a sequence belonging to Z,.
Since only a finite (>0) number of terms a; of this sequence is different
from 0, there exists the greatest ordinal number 4 < y such that a, £0.
(If we always had ag= 0 for 0 <&<», then the sequence a would of
course be the first element of the set Z(u,v), 2. e. also of its subset Z,,
in contradiction to the assumption that Z, has no first element; thus
number / exists). From the manner of ordering the set Z(u, v) it follows
§ 15. Exponentiation 309
ee WE:
E<a
310 XIV. Ordinal numbers
Now, for a given ordinal number uw > 1 and for every ordinal num-
ber a > 0, let f(a) = mw. Thus f(a) will be a function of an ordinal number,
satisfying the following three conditions: .
1) f)=4,
2) f(a+1) = f(a)-w for every ordinal number a,
3) If a is a number of the 2-nd kind, then
f(a) = limf(é) .
E<a
Now we shall prove that for every ordinal number y« > 1 there exists
only one function f(a) of an ordinal variable a satisfying conditions 1),
2) and 3).
Let us suppose that for a given ordinal number w > 1 there exist
two different functions, f,(a) and f.(a) of an ordinal number a satisfying
conditions 1), 2), 3). Since the functions /, and f, are different from each
other, there exist ordinal numbers é such that f,(é) #/.(é) and among
them there exists the smallest, a. Thus we have f,(a) ¢ f,(a), but
such that for a given ordinal number p we have a <q) a certain element
h({ag\e<a). Then there exists one and only one function f(a), defined for
every ordinal number a (or for every ordinal number a <q), such that
and since, as has been proved above, there is at most one such function fe,
by (16.2) and in view of 0<q, we obtain
h({ag}eca) = limag ,
é<a
(171) [] 4:
<p
as follows.
314 XIV. Ordinal numbers
H. Le TIS
E<p
value of the product does not depend on the order of the factors (Sier-
pinski [56], p. 21 and 24).
Now, a transfinite product of type +1 of increasing ordinal num-
bers may give infinitely many different values if we change the order
of its factors. H.g. it is easy to find that the transfinite product
(of type w+1) 1-2-3...@, gives the value wn if we interchange the
factors n and. w, these numbers being different for n= 1, 2,...
For ordinal numbers a let us write
but
limn? = wm < @?.
n<o
Also
o® ~ limn?
na<o@
316 XIV. Ordinal numbers
== noer,
k<o
whence
lim k f
Wye 8 ee Ie =
k<o
But in view of € < y formula (18.1) holds for 6 = ¢; therefore (u2)* = py.
Thus we have
(U2)? ite
é<y
But, according to Theorem 3 of XIV.11, in view of
IDR as See
é<y
we have
JONES Ye rea pie
é<y
§ 18. Properties of the powers of ordinal numbers 317
Lama t* = uv.
S<y
liimn = w
n<w
while
lim g(n) = lim(n-2) = @ < g(w) = @-2
nN<w Row
and
liimh(n) =hlmn? = wo < h(w)=@?.
n<w N<w
oO (22)
2. Prove that n® =o for natural n> 1.
Proof. In view of 1+. a =o we have wm? =w'*® =w-q@”, and since, as we know,
@
for natural n > 1 we have n® = @, therefore, by (18.1), n°” = 27° = (n®)°” = w®
3. Give an example of a function f(a, f) associating with every ordered pair a, B
of positive ordinal numbers a positive ordinal number f(a, #) and increasing with respect
to a for every given f and with respect to f for every given a.
Solution. Such is the function (a+ 1/8 -_ a, since for a < a, we have (a+ 1)P ax
“<< (a,+ te a <(a,+1)'+a,. and for B < Bi: (a+ 1)? + a> (a+ 1) > (atift=
= (a+ 1)%(a+ 1) = (a+ 1)'a+ (at 1% S (at 18+ (a+ 1) > (at Uf +a.
4, Prove that for every ordinal number a we have
224+ 3% = B24 27,
Proof. This is obvious for finite a. If a= om. then a = wé+ k where €>1 andk
“is a finite number >0. Hence 2%+4.3% = 20F+*1 grbth _ (9@)F oF1 (35. gF 2 wy. oF
+ hs F = w®- (2° 3°) = w° (3+ 2") = 37+ 2°
5.; Prove that (@-+ 1)? = o®.
Proof. For natural n. we have: wo < (w+1)" < (w+) = (w-2)" = w'-2<
<0'oO= or whence, on account of the fact that
lim w” = lim w” tT
1
— @®,
nN<w n<o
‘we easily conclude that
lim (@-+ 1)” = w®
N<w
Ave I coe
N<w!
we shall have {(A) =A.
Proof. As the function f(&) is increasing, we easily deduce by transfinite induc-
tion that f(€) => é for every ordinal number &. Since f(a) #~a, we have f(a) >a,
and thus a,= f(a) = f(a) > a=a,, whence a, > a. Now if we have a, >
> a,_, for a certain natural n, then, since the function f is increasing, we obtain hence
F (Qn) > f(Gn_i)> 4. @- G4, > a,. Hence it follows by induction that this inequality holds
for »= 0,1, 2,... Thus there exists a limit
A = lima, .
n<w
(18.3) Ue ei
Proof. From formula (18.2) it follows that (for given w>1 and
& > 0) there exist ordinal numbers y such that w > &, since for example
§ 18. Properties of the powers of ordinal numbers 319
9
pitt > ub > &; let » denote the smallest of them. If the number » were
of the 2-nd kind, then, as we know XIV.15) the number ~” would be
the smallest ordinal number >! for €< ». But also, if » were a number
of the 2-nd kind, then, for ¢<¥v we should have ¢+1<¥, which, in
view of the definition of the number » gives the inequality ptt! < ,
therefore uw? < & Thus the number é < w” would be greater than any of
the numbers uw for ¢ <», in contradiction to the properties of the num-
ber uw’. Therefore number »y is of the 1-st kind and we can write y= a+1,
whence, in view of the definition of number », we obtain formula (18.3).
We have proved the existence of an ordinal number a for which
inequalities (18.3) hold. It can easily be seen that there is only one such
number, because if, besides the inequalities (18.3), also the inequalities
wP<E< pot) held, we should have pt < pot? and pf < ptt, whence,
by (15.3), a< 6+1 and 6 < a+1, therefore a < 6 and 6 <a, whence f=a.
We have proved theorem 2.
(19.1) Oe Pe OCs
According to the corollary to Theorem 1 of XIV.14 the smallest
prime component >o* is wt w= w%t!; the inequalities (19.1) prove
that 0 = w*. We have proved
THEOREM 1. Prime components are powers of number w (whose
exponents are ordinal numbers) and conversely.
From Theorem 1 and Theorem 2 of XIV.6 it immediately follows
that every ordinal number a> 0 may be represented uniquely in the form
a= +. we? ar ee on,
where m is a natural number and p,, Bo, .--, bm are ordinal numbers such
that By > pp >... > Bm > 0. Joining equal components in this decomposi-
tion of number a we immediately obtain
THEOREM 2. Hvery ordinal number a > 0 may be represented uniquely
in the form
(19.2) a= w74,+ w%a,+...+ wa,
where * and 4a,, a,,..., a are natural numbers while a,, a2, ..., ax 18 a de-
creasing sequence of ordinal numbers.
Formula (19.2) is called, after Cantor, the normal form of the ordinal
number a and the number a, is called its degree. By (19.2) and (18.2)
we have a>o7>a,; thus the degree of a number (in its normal form)
is not greater than the number itself.
EXERCISES. 1. Determine the normal expansion of the number
(m+ 1)2(m4+ 1)3(m+ 1)4.
Answer. 3-44 @?- ogi m:2+ 1.
2. Determine the normal expansion of the number 2% where a is a given ordinal
number.
Solution. As we know from XIV.11, every ordinal number a may be represented
(uniquely) in the form a = wé+k where & is an ordinal number >0 and k is an inte-
ger >0. Since 2° = w, we have 2% = 2°°+* — (2°)§.9* — w*.2*. Thus the normal ex-
pansion of number 2% is 2% = w*- 2". ;
3. Prove that prime components >1 are powers of number 2 whose exponents
are numbers of the 2-nd kind and vice versa.
Proof. If 9 is a prime component >1, then, by Theorem 1, there exists an ordinal
number & > 0 such that 9 = o*, and since w = 2°, we have 9 = 2°, which shows that @
is a power of number 2 with an‘exponent which is an ordinal number of the 2-nd kind.
On the other hand, if a is a number of the 2-nd kind, a=&, then 2% = 2° = (2%)§ = w®
and it follows from Theorem 1 that 2° is a prime component (>1 of course).
4. Knowing the normal expansion of positive ordinal numbers a and f determine
the normal expansion of the numbers a+ and af.
§ 19. The power wm" 321
Answer. Let a = w%a,+ o™”a,+ ...4 oa, and B= wtb, + wb, + ...4 ot, be the
normal expansions of numbers a and f. If a, < f,, then a+f=. Otherwise there
exist indices 1<k such that a, > f,; let 7 denote the greatest of them. If a, > fy,
then the normal expansion of the number a+ is w%a,+ oa,.+ ...+ o%a,4 wtb, 4
a7 el ee wlth, , and if a,=,, then the normal expansion is wla,;-+ w"a,+ ...4+-
+ w%—la, »+ cot (a, + b,)+ w2b,+ wF8b,+ ...+ wht,.
If number
f is finite (i.e. B =56,), then af = w%a,b,+ w%a,+ w8as+ ...F w%a,.
If number f is transfinite (i. e. B, > 0) and of the 1-st kind (i. e. 6, = 0), then af =
= mAthip+ matfapt 4+ oa +Piap, s+ ola,
b, + w%a,+ ...+ w%a,. Finally, if number f
is of the 2-nd kind (i.e. B, > 0), then af = w% 11d, + wt ap, + ... + wt th,.
5. Knowing the normal expansion of an ordinal number a > 0 determine the nor-
mal expansion of the number a”.
Solution. If a=1, then a® = 1. If 1 < a < a, then, as we know, the normal
expansion of the number a® is w. If a => and the normal expansion of number a is
(19.1), then we have a, > 1 and a+ watt — mt! therefore w% <a < w4*' whence
wt < g® < mt", But, according to exercise 3 of $10, we have (in view of a, > 1)
(a, +1) = aw. Thus a? = o%®. The normal expansion of the number a® (where
a>) is therefore a? = w%®.
6. Prove that if « is an ordinal number >1, k — a natural number, a,, a, ..., a, —
ordinal numbers such that a, > a, >... > a, > 0, and »,, »,...,% — ordinal numbers
greater than 0 and smaller than mw, then w%y,-+ u™r_+ ...4+ uy, < watt.
Proof. In view of », << pw and w > 1 we have wy, < ww = ons Thus the inequa-
lity which we are to prove is true for k = 1. Now let us suppose that it is true for a na-
tural number k& and let a, > a, > a, >... > a, be k+1 ordinal numbers not smaller
than 0 and 7), 7, ¥2,..-,%—k+ 1 ordinal numbers greater than 0 and smaller than wy.
. ; +)
We shall have uv,+ w%y,+ ...+ wy, < w%yyt wT,
But, since a) > a,, we have a,+1<a,. Therefore p“ovp+ ut? < wy, + jis
= p(y +1) < wn = wt, because, in view of ») < uw, we have »+1< yp. Thus
Oy + wat? < n%**, whence it follows that our inequality is true for k+ 1 components.
It has been proved by induction for an arbitrary finite number of components, q. e. d.
(19.5) a= p 'y,+a’,
where 0 <a’ < w™ <a and, in view of (19.4) 0<»< yp. If a’ > 0, then
we can further write
(19.6) C= 7,4 ae
we reach the conclusion that numbers a,, dz, ... a, aNd 11, 2, ..-) MR are
well defined by number a, which proves that there is only one decom-
position (19.2) (where numbers aj, ag,...,a, and ,%,...,% Satisfy
known. conditions).
Theorem 3 of XIV.19 is thus proved.
In particular, if ~ = 2, we obtain for every ordinal number a > 0
the expansion
Q 27 ee
where a,, a), ..., 4, 1S a decreasing finite sequence of ordinal numbers > 0,
there being only one such expansion for every ordinal number a > 0.
Thus every positive ordinal number is the sum of a finite number of
powers of number two (just as is the case with natural numbers).
It will be observed that it is not every ordinal number that can be
represented by numbers smaller than itself with the aid of expansion
(19.1), or even, more generally, with the aid of expansion (19.2). There
exist ordinal numbers a > 1 such that for any uw < a the first exponent a,
in formula (19.2) is equal to a; we shall get acquainted with such numbers
in the next paragraph.
It is almost self-evident that with certain mw for certain a the ex-
ponent a, in formula (19.2) is equal to a; e. g. we have w = 2° and, more
generally, .0 == n° 10n n= 1,72...
(20.2) Ores
Cantor calls every ordinal number « satisfying equation (20.2) an
epsilon-number. Thus they are the critical numbers of the function f(é) =
= w*. We shall prove that number (20.1) is the smallest of such numbers.
Indeed, suppose that w*= a. We have hence a> 1, therefore a= w* >
>o=gy, whence a>g,. On the other hand, if for a certain natural
number » we have a>g,, then a= > w™= @ai1, Whence a > gait.
We infer by induction that a>, for n=1,2,..., and thus
a= limg, =e,
n<o
tones Ook Cs
For every ordinal number y there exists an epsilon-number > y.
Indeed, let »,=y+-1 and y=" for, 7=—2,3,... From Formula
(18.2) it follows that w’+t > y+1, and since the left-hand side is a num-
ber of the 2-nd kind and the right-hand side of the 1-st kind, we
have w?tt>y-+1, and thus y.>y,. Hence, by induction, we easily
conclude that y, < y, < y;... and then we prove, in the same way as we
have done above for number «, that the number
H(y) = limy,
n<o
Indeed, let
a= lim €e.
E<A
From the properties of the power of ordinal numbers it follows that
wo = limw®.
E<A
a=lme,
E<a
a is the smallest number that is greater than any of the numbers «; for
—& <A. It follows hence that a= «,. Thus
6, = lime: , ).g-e.7d.
E<A
In particular, the next epsilon-number after the numbers ¢é9, &, é, ..-
will be*
So es
n<o
= nee
<a
ot = limo’ <a,
E<a
(20.7) of = pa;
such are for instance the numbers a= w and 6 = ew. Indeed, by (20.2)
we have ew = ww = w*t1, and since, by XIV.11, Exercise 3, we have
(2 Dw co, this) cives: b2==.(em)P = (w*t!)? = wo6tDo— o@.— of, Num-
ber « may of course be replaced by any epsilon-number. Thus equation
(20.7) has infinitely many solutions in different ordinal numbers a and f.
Among finite ordinal numbers a and />a satisfying equation (20.7)
there is, as we know, only one pair: a= 2, B= 4.
Equation (20.7) for ordinal numbers will be dealt with in greater
detail in XIV.27, where all its solutions will be determined.
ys > 0, By < ax; therefore 6 < m”, which is contrary to the assumption.
Thus y,;= 0 and number y is of the 1-st kind, whence (cf. Exercise 4
of XIV.19) we obtain a= py = w™*%¢,+ ote, +... + ot t"—I¢,_ 4 +
+ w'tb,¢, + wb, +... + we, whence, by comparison with the expansion
(Oe) ery ODUALL! Mie eG RN Opt teh Depa syeinpee Pp Sey U1Ce=
= Ok—pii, Jo = Op—ria; +. 0p = a. The natural number r may assume k
different values, and for a given r the numbers #,, f.,..., 8, and b.,
bs, ---, b, are well defined (by number a) while, in view of b,|a,_,41 num-
ber b, may assume only a finite number of values. Thus there exists only
a finite number (> 0) of left-hand divisors 6 of number a greater than w”.
Theorem 1 is thus proved.
THEOREM 2. For any ordinal numbers a, B and y the following ine-
quality holds (Shermann [1], p. 111):
EXERCISES. 1. Prove that an ordinal number that is greater than 0 has a right-
hand divisor of the 1-st kind greater than 1 if and only if it is not a prime component.
Proof. Suppose that an ordinal number @ > 1 is a prime component. By The-
orem 1 of XIV.19 it is a number of the 2-nd kind. Suppose that number ¢ has a right-
hand divisor of the 1-st kind y+1> 1; therefore »y> 0 and there exists an ordinal
number f such that @ = B(y+1), and 6 > 1 since number o is not of the 1-st kind.
Hence @ = fy+ 6 > By > B (since y > 1), and thus 9 > fy and e > f, and the prime
component @ is the sum of two ordinal numbers smaller than itself, which is impossible.
Thus our condition is necessary.
On the other hand, if an ordinal number a > 0 is not a prime component, then,
by Theorem 1 of XIV.19, it is not of the form w> where € > 0, and then in the normal
expansion (19.2) we have either k = 1 and a, > 1, and number a = wa, has a finite
right-hand divisor a, > 1, or k > 1 and number a has a right-hand divisor of the 1-st
kind 17 %a,+ w%2”%*a,4+ ...4 @%-1~%q, _ ,+a,. Thus our condition is sufficient. The
required proof is complete.
2. Prove that if each of two given ordinal numbers is divisible on the right by
a natural number n, then their sum is also divisible on the right by n.
Proof. The proof follows easily from the theorem proved at the beginning of
XIV.21 and from the formula for the addition of normal forms of two ordinal numbers.
3. Prove that if an ordinal number y is >, then number y+y is not divisible
on the right by y.
Proof. If y is an ordinal number > q@ and if wc, is the first component of its nor-
mal expansion, then the first component of the normal expansion of number y+y is
@’12c,. If number y+ y were divisible on the right by y, we should have y+ y= &y
where € is an ordinal number > 0. But it is easy to find that the first component
of the normal expansion of number €y has the form w*c, where B > y,. In view of y+ y=
=&y and the uniqueness of the normal expansion of an ordinal number, we should thus
have 2c, = c,, which is impossible since ¢, is a natural number.
4, Give an example of two order types each of which is divisible on the right
by 2, but the sum of which is not divisible on the right by 2.
Answer. Such are the types 7 and 2.
5. Prove that if a and f are arbitrary ordinal numbers y >a and y=, then
a+B+y=B+at+y.
Proof. The proof is easily carried out with the aid of the normal expansions of
numbers a, 6 and y. ;
6. Prove that in order that an ordinal number & satisfy the equation wé = € it
is necessary and sufficient that number w® be its left-hand divisor.
7. Find all systems &, 7 of ordinal numbers such that &-.2 = 7.
Solution. Suppose that € and 7 are positive ordinal numbers such that &.2 = 7?,
and that & is of the first kind. Then 7 is also,of the first kind, and the numbers & and 7
cannot be finite (because V2 is irrational). Thus the normal expansions of &-2 and 7?
have at least three components and, since &:-2 = 7?, the number of their components
is equal. Comparing in these expansions the first, the middle and the last components,
we easily obtain a contradiction. Thus é is a number of the 2-nd kind and we have
&-2 = (&-2)2, 1. e. (E> 2)? = 7?, which gives &-2=7. Thus all positive ordinal num-
bers & and 7 such that &*- 2 = 7? are: 1 = €-2, where é is an arbitrary ordinal number
of the 2-nd kind.
332 XIV. Ordinal numbers —
8. Prove that there exist no transfinite ordinal numbers § and y such that =
=7>-+ 1.
and, in view of uw < w+, we cannot have uw > w for € < 4. Thus there exists
a number ¢, < A such that w < 1, and similarly we conclude that there
exists a number ¢, < 4 such that » < w. But, as we know from Theo-
rem 1 of XIV.19, number A = wé is a prime component: therefore if ¢, < A
and ¢, <A, then we must have ¢,+¢, <A; thus py < wm? = white < @),
in contradiction to the equality #4 = uy. Therefore number ow” is a prime
factor, q. e. d. We have proved
THEOREM .2. In order that an ordinal number of the 2-nd kind be
a prime factor it is necessary and sufficient that it be of the form wo” where E
is an ordinal number > 0.
Combining Theorems 1 and 2 we immediately obtain
THEOREM 3. All numbers of the form w’+1 where y is an ordinal
number +0 and all numbers of the form a” where € is an ordinal number > 0
are, besides finite prime numbers, the only ordinal numbers that are prime
factors.
This theorem immediately implies that among ordinal numbers
there exist infinitely many pairs of successive ordinal numbers which are
prime factors: they are the numbers w® and w® +1 where é is an arbitrary
ordinal number > 0.
304 XIV. Ordinal numbers
But if z is not a finite prime factor, 7+2 is not a prime factor, be-
cause if m= w’+1, we have 7+2= 3m and 3<a2+2, a<2+2, and
if «= w”, we have 7+2= 2(x+1) and 2<2”+2, a4+1<a2-+2.
EXERCISES. 1. Find the smallest ordinal number >1 that is not the sum of
a finite number of prime factors.
Answer. o?.
2. Find the smallest transfinite ordinal number a such that between a and a-2
there is no prime factor.
Answer a=q@-+1 since the next prime factor after w+ 1 1s w?+ 1 > (@+1)-2=
=@-2+1. This proves that the so-called postulate of Bertrand is not true for ordinal
numbers.
3. Prove that in order that an ordinal number be a prime factor it is necessary
and sufficient that it have two and only two different right-hand divisors.
Proof. Cf. the last section of XIV.13.
4. Prove that in order th t an ordinal number be a prime factor of the 1-st kind
it is necessary and sufficient that it have two and only two different left-hand divisors.
Proof. Suppose that an ordinal number z is a prime factor of the 1-st kind. Thus
it has at least two different left-hand divisors: 1 and z. If it also had a left-hand divisor
w~Al1 and w~2x, we should have 7 =~. If y= 2, we should have 1-7 = u-a, and
on account of the fact that z is a number of the 1-st kind and according to Corollary 3
of XIV.10 we should have 1 = uw, which is impossible. Therefore » 4 a and number a
is the product of two numbers u and vy smaller than 2, which is contrary to the assump-
tion that z is a prime number. Thus our condition is necessary.
On the other hand, if an ordinal number a has two and only two left-hand divisors,
then they are 1 and a (where a > 1). Then there is no decomposition a = wy where
“u <aand vy < a because in that case we should have «= 1, whence v = a. Number a
is thus a prime factor and it must be of the 1-st kind since otherwise it would have
infinitely many different left-hand divisors: 1, 2, 3,... Thus our condition is sufficient.
4. Find all ordinal numbers having three and only three different left-hand divisors.
Solution. As regards finite numbers, such are, as we know, only the squares of
prime numbers. Suppose that number a is transfinite and has only three different left-
hand divisors. Number a must be of the 1-st kind (since a number of the 2-nd kind
has infinitely many left-hand divisors, e. g. all natural numbers). Thus besides numbers 1
and a the number a has only one more left-hand, divisor, w. Therefore we have a = py
where 1 < uw < a. If number p were not a prime factor, we should have u = u,7, where
fy < wand », < mw; therefore | < uw, < mw and m, would be a left-hand divisor of num-
ber a such that 1 < uw, < u < a, which is impossible because a has only three different
left-hand divisors: 1, w and a. Thus number yw is a prime factor. If number y were not
a prime factor, then, in view of »y > 1 (since a = uv > pw), we should have v = u.¥, where
1 < py < v and, in view of a = wyyv,, number py would be a left-hand divisor of num-
ber a such that 1 < uw < wy, < a, which is impossible. We have proved that if a trans-
finite ordinal number a has only three different left-hand divisors, then it is of the 1-st
kind and is the product of two prime factors (of the 1-st kind of course).
Suppose now that a transfinite ordinal number a is the product of two prime
factors of the 1-st kind, a = 2,2,, at least one of the numbers z, and z, being transfinite
since such is number a. Number a has at least three different left-hand divisors: 1, z,,
§ 22. Determination of prime factors 335
and a. Suppose that number a has one more left-hand divisor mw, different from the other
three. Thus a = pr.
If x, is a finite number, i. e. a prime number p, then we have wy = pa, % = o?+1
where & > 0, therefore wy = ore p. Number w+ p, as we know from XIYV.21, does
not possess a natural right-hand divisor >1, and since vy > 1 in view of u +a, » can-
not be a natural number. 2-1 p being a number of the 1-st kind, vymust be a transfinite
number of the 1-st kind. If w were also a transfinite number (of the 1-st kind of course),
then the normal expansion of the product wy would have at least three components
which is impossible since this expansion is w°?-++ p. Therefore « must be a natural num-
ber. But number a+ p has (see Exercise 2 of XIV.11) only two natural left-hand
divisors, 1 and p, whence follows a contradiction since ~~ 1 and w~2,=p.
If a, is a finite prime number, z, = p, then wy= 1p, 1 = w+ 1 where €, > 0,
whence wv = wolp+ 1. Number wtp+ 1, as we know, does not possess a natural left-
-hand divisor >1, and since w 4 1, ~ must be a transfinite number (of the 1-st kind),
We conclude as above that number » cannot be transfinite because the normal ex-
pansion of the product uv = w*!p+1 has only two components. Thus » is a natural
number, and since number wp+ 1 has only two natural right-hand divisors, 1 and p,
therefore y= 1 of y= p. But if y= 1, a = wy would give «=a, which is impossible,
and if »y= p, we should have wp = 2,p, whence uw = xz, which is also impossible.
Finally let us suppose that numbers z, and a, are both transfinite, and since they
are prime factors of the 1-st kind, therefore 2, = wo +1, ii == w2+ 1, where &,>0
and & > 0; thus wy = w+ o t+ 1. This number has no left-hand or right-hand
natural divisors different from 1, and since « ~£ 1 and vy # I (in view of u ~ a), we con-
clude that numbers yu and v are both transfinite. Let u = w“!m,+ o?m,.+ ...4+ wo*—Im,_1 ++
+m, and vy = own,-+...+ 0"-In,_,+n, be their normal expansions. The normal ex-
pansion of number wy will thus be oT n,+ wMtT”n,-+ ...-+ Ml Tlg, + om, Ne+
+om,+-...+m,. By comparing both normal expansions of the product uv, we ob-
tain k+1—-1=3, i.e. k+l=4, and since k > 2 and 1>2 (numbers yw and » being
transfinite of the 1-st kind), therefore k= |= 2; further, the comparison of the normal
expansions gives w,+ 7, = 6+ &, w= &, m = Mn, =m, = 1. whence m, = m, = 4 =
= —= sls UNCLOLOT ONi — ot 1] = m,, Which is impossible.
We have thus proved that in each of the three cases investigated number a has
only three left-hand divisors. We have proved that in order that an ordinal number a
have three and only three different left-hand divisors it is necessary and sufficient that
it be either the square of a finite prime number or a transfinite number of the 1-st kind
which is the product of two prime factors (different or not), one of which may be finite.
Thus for instance the following numbers are ordinal numbers having three left-
hand divisors: o+ p, op+1, w?+p, w*p+1 where p is a finite prime number, as well
as (w+ 1)(@?+ 1) = w?+ 0+ 1 and (w?+1)(@+ 1) = w+ w+ 1.
5. For every natural number n give an example of an ordinal number a which
is the product ot two prime factors and has more than n different right-hand divisors.
Solution. Such is, for natural n, the number a = w”. Indeed, w” is the product
of two prime factors, w”~?+ 1 and », and, in view of w" = w"—*w" for k= 0,1, ..., 7,
numbers 1, w, w?,..., @", Which are all different from one another, are its right-hand
divisors.
6. Find all the ordinal numbers having three and only three different right-hand
_ divisors.
Solution. The necessary and sufficient condition for an ordinal number a to have
three and only three different right-hand divisors is rather complicated. We shall prove
336 XIV. Ordinal numbers
that in order that an ordinal number a have three and only three different right-hand
divisors it is necessary and sufficient that it be either the square of a finite prime number
or a transfinite number which is the product of two prime factors satisfying one of the
following two conditions: 1) the second factor is a number of the 1-st kind (finite or
transfinite), 2) both factors are of the 2-nd kind, the first being not smaller than the
second.
For the proof it is of course sufficient to investigate the case of number a being
transfinite. Therefore let us suppose that number a is transfinite and has only three
different right-hand divisor, i. e. besides the right-hand divisors 1 and a one more
divisor, v, different from the other two. Thus there exists a smallest ordinal number yp
such that a = wv. We have uw > 1 because vy £ a. If w were not a prime factor, we should
have w= .47,, where pw, < uw and », < mw, and a= ,,». If »»=», we should have
a = wy, which, in view of uw, < mw, is contrary to a definition of number uw. Thus »yv ¥ »,
therefore v,y > v, and on the other hand »y < a, since a= wy > vy, and the equa-
lity a = »» is impossible in view of », < w and the definition of number yw. Thus num-
ber a would have at least 4 different right-hand divisors 1, y, v,y, and a, in contradic-
tion to the assumption that it has only three right-hand divisors. Thus number yp is
a prime factor.
It is also easy to prove that number » must be a prime factor since otherwise it
would have a right-hand divisor 6 such that 1 < 6 < », and that divisor would also
be a right-hand divisor of number a such that 1 < 6 < » < a, which is impossible since a
has only three right-hand divisors: 1,» and a.
We have proved that if a transfinite ordinal number a has only three different
right-hand divisors, then it is the product of two prime factors. It is impossible simultane-
ously for the first of them to be a finite number and for the second to be a number of
the 2-nd kind since in that case number a would be equal to the second factor, i. e.
it would be itself a prime factor and therefore, according to Exercise 3, it would have
only two different right-hand divisors. Thus if the first factor is a finite number, then
the second is a transfinite number of the 1-st kind. Suppose now that the first factor
is a transfinite number of the 1-st kind, and the second a number of the 2-nd kind
not smaller than the first factor. By Theorem 1 and 2 we thus have a = 2,2, where
m%=ol+l, m= Boe &>0 &>0 and 2,<a2,; therefore w#!+1< on whence
wol< eae Wa Covey << w2, which, on account of the fact that number w*? — as a prime
factor — absorbs the preceding component &, smaller than itself, gives &+ wo?
= w®,
>) €2 é2 : = ‘ A
whence a = ™2, = (w+ 1) @°" = wt? — w?” = n,, which is impossible, since the
prime number z, has only two right-hand divisors. Thus if the first factor in the de-
composition a = 7,7, is a transfinite number of the 1-st kind, and the second a number
of the 2-nd kind, then we must have a, > a,. We shall also prove that 2, < 22. Indeed,
*s ay &2. : : €2.
suppose that 2,>73. Thus wot+t1> wo”, whence w! > wo” (since if w! < ow,
we should have w!-+-1< we?) which gives &, > w*?-2; therefore £,— w*?-2+-5 where
56> 0. Thus we should have a = pfitotee —- ofte+d+o% 514 number p= om tetomt
would be a right-hand divisor of number a such that 2, < » < a; number a would thus
possess at least four different right-hand divisors, 1, z,,», and a, which is impossible.
é é2. ¥
Therefore 2, < 2, < 22 and thus o°" <@!+1<o 2 whence Roe <@ml< oot 2
and ow? < &, < w-2. Therefore &,= w°!+7 where 0<1t < w. If €& = 0, we have
0<+t< 1, whence t=0 and t+ ow? = w? and if €& > 0, we also have t+ we? = wm?
in view of t < wo and by Theorems 1 of XIX.14 and 1 of XIV.6. Therefore &,+ o =
§ 22. Determination of prime factors Goospa
és a 7 5
= wo?+ 7+ wo = w?-2, and thus a= aa = who = mito? — we? — zz. Condi-
tion 2) is satisfied.
Finally let us suppose that both factors, a, and 2,, are of the 2-nd kind and that
é1 é2 Ey §2 :
Ty < %,. We have 2, =, 1 =o” , OX €, < &, and wo” < w””, whence wo! < w®
E ea ase E14 gySe €2 Fe Pe |
and w*!+ w= w*; therefore a = 12,= 0° w= ot?” = w®” = ay, whichis impos-
sible. Thus if both factors, a, and z,, are of the 2-nd kind, then we must have 2, > a,
2. e. condition 2) must be satisfied.
We have proved in this way that if a is an ordinal number having only three
different right-hand divisors, then it is the product of two prime factors satisfying one
of the conditions 1) and 2). Thus in order to prove our theorem it remains to show that
if a is a transfinite ordinal number which is the product of two prime factors satisfying
any of the conditions 1) and 2), then number a has three and only three different right-
hand divisors. Therefore let us suppose that a = 2,72, where 2, and a, are prime factors
satisfying condition 1). The second factor, a, is thus a number of the 1-st kind. If z,
is a finite number, then 2, = p where p is a prime number and, since a = 2,7, is a trans-
finite number, z, must be a transfinite number. If z,, is a number of the 1-st kind, then
we have xz, = w*!+ 1 where &, >-0; therefore a = a,p = op + 1. This number, as we —
know, has no natural right-hand divisors other than 1 and p. If number a had, besides
1, p and a, another right-hand divisor y, then » would have to be a transfinite number,
of course of the first kind (since such is number a). We should have a = wy. Let w=
=o m, + ..+o"m, and vy = w'n,+...+@"m, be the normal expansions of numbers yz
and »v. By comparing the normal expansions of numbers wv and a we obtain k+1J—1= 2.
But 11> 2 since y is a transfinite number of the 1-st kind. Hence k = 1, therefore uw =
= olm,. We cannot have yw, > 0 since a = wy, i. e. wis also a number of the 1-st kind.
Thus ~,=0, whence w= m, and p is a natural number. But number a= op+ 1
has no natural left-hand divisor >1. Therefore « = m, = 1, and thus, in view of a=
= wv,v =a, which is contrary to the assumption concerning number v. Now if x, is
a number of the 2-nd kind, then we have z, = ow? where & > 0, therefore a= p=
--1—1 = 2. Since number a has neither a left-hand nor a right-hand natural divisor > 1,
therefore, if y A 1 and yA a i.e. wu£1, then numbers mw and y are transfinite. If we
had 1 — 1, number » would be a power of number @ with a positive exponent and the
number «wv would also be a power of number w, which in view of uw» = a is impossible.
Therefore | > 2 and from the equality k+1/—1= 2 it follows that k= 1 and l= 2.
Hence pr = wo!m,(@"'n, + ny) = 0! "In, + co! m,n, and by comparison with the normal
expansion of number a we obtain m,+ », = wi+té,, Mi, w'', therefore v, = &; further
m, = 1 and m,n. = 1, whence n, = 1, and thus »y = w*?-- 1 = z,. Thus in any case num-
ber a has only three right-hand divisors.
We have proved that if a is an ordinal number which is the product of two prime
factors satisfying condition 1), then a has only three different right-hand divisors.
Suppose now that a= 2,2, where a, and z, are prime factors satisfying condi-
: Ey &2 4 oe Me
tion 2). Therefore m= 0° ,7%,=@0° , m =a, and thus & > & > 0. If number a —
fe
mel +
E
=o “had one more right-hand divisor, besides the right-hand divisors 1. a and a,
E14 $2 :
then we should have m® °°” = wy, whence we easily conelude that » must be a power
of number w, vy = w1 and we obtain w®'*t ieee wt, whence w!-+ wo? — 4, +»,. Num-
ber », is thus the remainder of the number w*'-- w*? where &, > &,. But the latter, as
follows from Theorems 4, p. 283 and 1, p. 320, has only two different positive remainders:
wo? and w*!-- w?. Thus either », = o*? whence »y = w’! = 2,, which is impossible, or
vy, = w8'-+ w, whence » = w = a, which is also impossible. Thus we have proved that
if a is an ordinal number which is the product of two prime factors satisfying condition 2,
then a has only three different rnght-hand divisors.
Our theorem is thus proved.
7. Find all the ordinal numbers giving three and only three different decomposi-
tions into the product of two ordinal numbers.
Answer. Obviously they are all the ordinal numbers having three and only three
left-hand divisors (7. e. the numbers determined in Exercise 4).
8. Prove that if an ordinal number has three and only three left-hand divisors,
then it also has three and only three right-hand divisors but not vice versa.
Proof. The proof immediately follows from Exercises 4 and 6. 5
9. Give an example of a transfinite ordinal number having the same three and
only three left-hand and right-hand divisors.
Answer. Such is, for instance the number (m+ 1)?. In general, they are numbers
(w+ 1)? where & > 0 and only such numbers.
§ 22. Determination of prime factors 309
10. Find for what transfinite ordinal numbers a the numbers 2*+ 1 are prime.
Solution. For ordinal numbers a of the 2-nd kind and only for such transfinite
numbers a. Indeed, if a transfinite ordinal number a were of the 1-st kind, we should
have a = w&+k where & is an ordinal number greater than 0 and & a natural number.
Hence 2° = 2°+* — (2)°.9* — m>.2*, therefore 2°+1=o*- 9°41 1 = (w+ 1)-2",. and
thus the ordinal number 2%+ 1 is the product of two ordinal numbers smaller than
that number, i. ¢.it is not a prime number. But if a is a number of the 2-nd kind, then
ce £ ' F ; 5
a = wé where — > 0 and we have 2°+ 1 = 27% +1 = w* +1, which is a prime number-
Note. It is much more difficult to find for what finite ordinal numbers a the num-
bers 2*-- 1 are prime numbers. We know only five such numbers a, namely 0, 1, 2,
4,8 and 16, and we do not know whether there are more of them.
11, Ordinal numbers of the form F = 2%*+ 1 are called Fermat numbers. Prove
that every Fermat number with a transfinite index is a prime number.
Proof. Ifa is a transfinite number, then a = w+ & where = O and therefore the
number 2% = 2°°5 — 2°.2° — w- 2° is of the 2-nd kind. According to the preceding exer-
cise, number fF, = 2%*+ 1 is a prime number, q. e. d.
Note. As regards numbers F, for finite a we know that they are prime for a =
0,1,2,3,4 and composite for 4< a< 12. We do not know whether numbers F,,;
and F,, are prime or composite. Numbers /’,, and F’,, are composite.
Proof. Let a denote an ordinal number > 1, 6 = wy. where y is an ordinal number
>0O and suppose that there exists an ordinal number € such that &P =a. In view of
a> 1, we must have > 1. If € is a natural number then, in view of € => 2, E°= o,
therefore g = &? = &°” = @’.. If € is not a natural number, then > w. Let w lx, denote
the first component of the normal expansion of number €; we shall have &, > 0. Hence
«8 <M) & 4 !& .
we obviously have &°e — w*!, therefore a = &eB gO” _. (@51)? — @?, Thus our condi-
tion is necessary.
On the other hand, if a = m’, then for every natural number n > 2 we have n° = o,
therefore n? = n °Y — ~” — a and every natural number n > 2 is a B-th root of number a.
If a=", where rt > 0, then for every natural number n we have (@ ny oS gp TnMOY see
== (1) as oY
— ~? — g, the numbers w™” (n = 1, 2, ...) being all different from one another,
and again there exist infinitely many $-th roots of number a. Thus our condition is
sufficient and the required proof is complete.
2. Determine all m-th roots of number ow”.
Answer. They are all ordinal numbers € such that wo < & < w”.
3. Find the smallest ordinal number >1 of which there exists an (w+ 1)-th root.
. =i
Answer. It-is the number 2°° = - 2.
4. Find the smallest transfinite ordinal number of which there exist no roots of
degree > 1.
Answer. w+. Indeed, on one hand, 2° = wm. On the other hand, if for a certain
ordinal number f > 1 we had e? — w+ 1 and it — were a finite number, then we should
have > 2 and f > @ (since n® = w < w+1 for n=2,3,...), whence & > 9¢+? —
=o-:2 >o+1, which gives a contradiction. Therefore we should have €>o, and
gh 10)" > w+ 1, whence we again obtain a contradiction. Thus there are no roots of
degree P > 1 of number m+ 1, q. e. d. 5
§ 24. Roots 343
Proof. We shall say that two ordinal numbers a and / satisfy condi-
tion C if there exist an ordinal number & and non-negative integers k
and J for which formulae (25.2) hold.
If ordinal numbers a and f satisfy condition C, then, in view of the
distributivity of multiplication of ordinal numbers with respect to addi-
tion we obtain, by (25.2):
that it will be the number $= & where & is the smallest ordinal number
such that for a certain natural k’ we have a = ék’. The number é will thus
be the smallest left-hand divisor of the number a for which the comple-
mentary right-hand divisor of the number a is a natural number. And
since, as we know from XIV.21, the greatest natural right-hand divisor
of the number (19.1) is a,, this number é will be determined from the
equation a= éa,, whence it is easy to find that &= o1+o7%a,+...+
ao Oe. <
Thus: Lf the normal expansion of an ordinal number a> 0 is (19.1),
then the smallest ordinal number a, > 0 additively commutative with num-
ber a 18 a) = @'+07%0,-.. + otae
Let us now determine all ordinal numbers 6 > 0 additively com-
mutative with a positive number (19.1). By Theorem 1 they will all be
numbers f= él where | is a natural number and é a left-hand divisor
of number a for which the complementary right-hand divisor is a na-
tural number, ¢. e. a divisor of number a,. Thus in order to determine
number é we must solve the equation a= &k’ where k’|\a. If a, = k’ky,
then we easily obtain §= w%ki+o"a,1+...+o%a,, therefore 6 = l=
=on+wa,t+...t@%*a, where n= kil is a natural number. On the
other hand it is easy to verify that for any natural » number $ = w%n +
+ wa,+...+ 0a, is additively commutative with number a. It follows
that im order that an ordinal number 6 >0 be additively commutative
with number a > 0 having the normal expansion (19.1) it is necessary and
sufficient that B = w n+ w®a,+...+ o%a, where n is a natural number.
Thus we have 6 = ayn and, conversely, for any natural n the number
fb = aon is additively commutative with the number a. Thus we have
THEOREM 2. If a is an ordinal number >0 and ay denotes the smallest
ordinal number > 0 additively commutative with number a, then every ordinal
number p additively commutative with a is of the form 6 = aps where s is
an integer >0, and vice versa.
From Theorem 2 immediately follows
COROLLARY 1. For every ordinal number a> 0 the set of all ordinal
numbers additively commutative with number a is denumerable.
From Theorem 2 we can also easily deduce
COROLLARY 2. Jf a, B and y are ordinal numbers such that a 40,
a+f=B+a and a+y=y-+a, then B+y=y+6.
Proof. If a, 6 and y are ordinal numbers such that a> 0, a+ p=
= f+a and a+y=—y-+a, then, by Theorem 2, there exist integers >0,
s and r, such that 6 = ays, y= ayr, whence B+ y = as+apr = a(s+r)=
‘= ao? +8) = apr + as = y+, whence B+y= y+, q.e. d.
From Corollaries 1 and 2 immediately follows
§ 25. On ordinal numbers commutative with respect to addition 345
that the sum of numbers a,, a, and a, is independent of their order. Thus the condi-
tion is sufficient.
6. Let » denote a natural number >1. Prove that in order that the sum n of
positive ordinal numbers a,, @,..., a, be independent of the order of those numbers
it is necessary and sufficient that there exist natural numbers k,, k,.,...,k, such that
Thy == C1) = oo == Cla bic
7. Prove that Corollary 2 to theorem 2 is not true for order types.
Proof. If a=1,.8 = o+o*, »y=o+1+o*, then a+ $= f+a,a0-y—y+7a@
but B+yAy+ fp.
8. Prove that an ordinal number a > 0 is not additively commutative with any
ordinal number >0 smaller than a if and only if number a has no natural right-hand
divisor greater than 1.
9. Prove that if a is an ordinal number > 0, then all ordinal numbers > 0 additively
commutative with number a form an increasing transfinite sequence of type @ whose
limit is number aw.
Proof. It is easy to prove that if (19.1) is the normal expansion of number a > 0,
dg
then the sequence in question is the sequence of numbers w n+ w@d,+ w7dg-+ ...
l . . - = Es |
+ oa, where n = 1, 2,..., whose limit is the number m2" = aw.
10. For an ordinal number a > 0 find the smallest ordinal number f such that
number a is not additively commutative with any ordinal number => f.
Answer. It easily follows from Exercise 9 that this number is 6 = ao.
11. Prove J. Stupecki’s theorem stating that if m is an infinite order type, then
there exist order types w and » such that p= pwtvAv+u.
Proof. If m is a transfinite ordinal number, then, as we know, y = 1+ , and,
according to Exercise 1, we have 1+ 9 4 y+ 1; thus we can assume that uw = 1,vy= 9.
If y is not an ordinal number, then ~ is the order type of an ordered set /,, which is
not well-ordered, 7. e. it contains a transfinite sequence of type m*. Denoting by A the
set of all elements of the set / that precede every term of that sequence and writing
B = F—A, we shall obviously obtain a cut fF= A+B of the set F in which the set B
has no first element (and the set A may be empty). Let 4 =a, B = f. If the type @
has a first element, then of course gy= a+/ + B+ a, since then a has a first element
while £ has not. And if y has no first element, then let a denote any element of the set F,
let us denote by WV the set of all elements of the set / that precede a and let VN= F—M.
The set J/ will thus have no first element and the set N will have such an element,
viz. a. Writing M = yw, N =», we shall thus have g=putyvA~Ar+ypm. We have proved
J. Shupecki’s theorem.
12. Give an example of two ordinal numbers a and f such that a®+ p° = p®+ | a®
Bee ie eRe n Ce pee ¢
butcesepe- p +a for n=1,2,...
:
ADswer. a=, Bp = w*;2. for
¢ we have here a® = f° = w® anda +f =o 2 ° <o an,
+ i]
ae aK z= p Bt ee
13. Prove that if a and f# are ordinal numbers >1 such that a+ 6 = P+ a, then
a® = 6° but not vice versa.
Proof. If a and f are natural numbers >1, then we have a® = f° = o. If a and B
are transfinite ordinal numbers with the normal expansions a= w“a,+... and Bp=
8. os :
= w'b,+... and if a+ f= fB+a, then we must have a, = f,, therefore a? = 0% =
= wi — B°, Now, w® = (w)® but oto? < w+o.
§ 25. On ordinal numbers commutative with respect to addition oad.
14. Prove that if a and f are transfinite ordinal numbers of the 1-st kind and if
for a certain natural m > 1 we have a”+ fp” = p”+ a”, then a= f.
Broof, Wet a= w4a,4-...-- wk la, +a, and p= @ Lb ack i | ene b, be
the normal expansions of numbers a and $. Thus we have a, > 0, B;> 0 (since a and P
are transfinite). It is easy to calculate that for a natural m > 1 the sum of the first k
terms of the normal expansion of number a” will be
ay(m—1)-+ ay ru” =D) et the, ay("—1)
Oa eo a+... jo A,Uy,
and the sum of the first | terms of the normal expansion of number ~”ym will be
m B —1)+-£8 5. —1)+ 7. 34(m—1
wht b,+ oi—Y 2 ae lee ai) rPr_ip, Sain phi” dh, Ds
the expansion of number a” having mk—m-+ 1 components and the expansion of num-
ber 6”, ml—~m—1 components. If a” + p” = p”"+ a”, then the normal expansions of num-
rs a” andand p”
bers p° maymay differ
differ atat most
most in ththe coefficient
ff t of the
of the highest
highest power
er of of number
numbe w;
thus mk—m+1= ml—m-+1, whence k =I; further a,m = f,m. whence a, = f,,
and a,(m—1)+ a, = B(m—1)+, whence, in view of a, = a we obtain a, = f,, etc.,
IU Ch 5 [Gh a Gly = Oy, Che Wynccon Cheoy =b, ,, &a, = b,b,. But in the expansion
of a” the term free of @ is a, and in the expansion of f” it is b, = b,; thus we must have
a, = b,, whence, in view of a,a, = b,b,, we obtain a, = b,. The normal expansions of
numbers a,a and f, are thus identical, i.e. a= p, ¢
15. Prove that for positive ordinal numbers a and f the formula a+f = p+a
is equivalent to the formula aw* = po*.
Proof. If a+fP=f+a and numbers a and f are positive, then by Theorem 1
of XIV.25 there exist an ordinal number € and natural numbers m and ” such that
=a a= CN CCN OO” —= GIO) =O) |== CNG) —=
On the other hand, if a and f are positive ordinal numbers such that aw* = pw*,
then there exist a natural number m such that a is a remainder of number fim and
a natural number » such that 6 is the remainder of number an. Thus there exist ordinal
numbers y and 6 such that Pm =y+a, an=6+ f. If a=o™a,+...4 0%a, and B=
= wb, +...+ ob, are the normal expansions of numbers a and f, then we have
ob, m+ w2b,+ ...4 o'lb, =yto%a,+..+o0%a, and wa,n+ w%d.+ ...+ wa, = 0+
a 10 =... eke Sinie we easily find that fp, > a, > f,, therefore a, = f,; further
Cig == [855 Gaon =P olin = Oy coon Cs = Ors EHNG! oe € = w+ w%a,+ ...+ o%a,, We
obtain a Sia. = a5 == <b, theretore a 3 —= ba.
16. Give an example of ordinal numbers a and f for which aw = pw but a+ pF
# B+ a.
ANSwer a=, f)—@-— I.
Give an example of order types a and £ for which aw* = pw* but a+ Pp~ B+ a.
AISWiers @— 0, p— Teo”
18. Give an example of order types a and f for which a+ 6 = f+ a but am* ~ pa*.
Answer. a=1, B =w+o*
(26.3) : an = pm,
The question arises whether for each two transfinite ordinal num-
bers, a and f, multiplicatively commutative there exist an ordinal num-
ber € and natural numbers m and » for which formulas (26.2) hold. We
find that this is the case only for transfinite ordinal numbers of the 1-st
kind (the proof of this theorem, which will be given later, is difficult);
for numbers of the 2-nd kind it need not be so. Indeed, let
therefore af = fa.
Suppose now that for a certain ordinal number & and natural num-
bers m and » we have formula (26.2). Number & would thus have to be
of the 2-nd kind. If we had m = 1, we should have é= a= w?+o > a’,
and, since é = @?+ w <@*+ w? = B, n > 2, whence p = &" > & > 4, which
is impossible as 8 = w*+ w? < w*. Thus m > 2, therefore w?+@= é" > ®,
whence @ < w?-2 = (w-2)*, which gives €< m-2. Since number ¢ is of
the 2-nd kind, we have = w, whence w*?+ wm? = &" = w”, which is impos-
sible.
Thus formulae (26.2) cannot hold for numbers (26.4) with any ordinal
numbers € and natural numbers m and n.
We find, however, as will be proved later (see Theorem 1), that in
order that transfinite ordinal numbers a and f be multiplicatively com-
mutative it is necessary and sufficient that there exist natural num-
bers m and n for which formula (26.3) holds.
EXERCISES. 1. For numbers (26.4) find natural numbers m and mn for which
formulae (26.2) hold.
Answer. m= 2, n= 3, because a? = f? = w*+ @?.
2. Find all transfinite ordinal numbers that are multiplicatively commutative
with number a= o®+ o.
350 XIV. Ordinal numbers
Weulnters in vilewrOteGa >:= San ulnciul Sym — Nome) Oy == Symite fl ==)9m =) eee rent oe
= p,, 1+ 6, = 6,+ 1, whence we find that 6, , = 1, ae Say cae,” Ba =I—1. By com-
paring the coefficients in the normal expansions of numbers af and fa we obtain b, = 1,
(Dy =o, Oy = yh soon We = Dg 1 lays, WN, CAE, Oh Se, Oy SD = cen = Dy se he MR
ei ern nae wo k+ ok + + wk+-k, which, as can easily be verified, is equal to
number (@+ k)'~". Thus ne
=a ~' where |—1 (in view of 1 > 1) is a natural number,
Ms Gs Ok
Prove that if a, f and y are ordinal numbers such that af = pa and ay = ya,
then also a(fy) = (by)a
Proof. In view of our assumptions we have a(fy) = (ap)y = (Ba)y = B(ay) =
= B(ya) = (By)a, therefore a(By)= (By)a, q.e. d.
6. Prove Jacobsthal’s theorem stating that if a, f,y +40 are ordinal numbers
such that ay = ya and a(yf) = (yf)a, then af = fa.
Proof. In view of our assumptions we have y(af) = (ya)B = (ay)B = a(yB) =
= (yB)a = y(Ba), therefore y(af) = (Ba), which, in view of y#0, gives af = fa,
Gio Cn Bh
§ 26. On ordinal numbers commutative with respect to multiplication 351
(26.20) ~ 6’ = wo a, +o + ax.
In view of (26.16) and (26.20) we obtain
(26.22) aay,
Cardinal and ordinal numbers to[o)
354 XIV. Ordinal numbers
where
(26.23) pee
and ap, = pya.
Proof of Lemma 21). Suppose that a, 6 and 6 are transfinite
ordinal numbers of the 1-st kind and a>f> 6, af= fa, ad= da.
According to Lemma 1 there exists an ordinal number (’ such that a=
= Bf’ = B’B and af’ = f’a; the number f’, being greater than 1 (since
a>fP and a= fp’) and multiplicatively commutative with the trans-
finite number of the 1-st kind, a, must also be transfinite of the 1-st
kind. Let £, = 6’6: it will be a number of the 1-st kind greater than o.
In view of 6 < f we shall have f, = p’6 < f’B = a, therefore B, <a and
apy = af'6 = p’ad = f'da = f,a, and thus af) = f,a. By Lemma 1 (for a
and f,) there exists a number f, such that a= 6,f,= 6,f) and af,=
= fi.a “Wurther,. swe {have a ==!6,0; = poppy, aud. “smce..a%—7 85.
B = 6f,. If B = £,, we should have hence 6 = 6f, and since # is a number
of the 1-st kind, 6=w+1, we should have w+1= 6(u¥+1)= 6u¢+
+6>u+0, therefore w+1 > 4+6, which gives 6 <1, which is impos-
sible, 6 being a transfinite number. Thus 6 ~ 6,, therefore, in view of
6 = 68,,8 > B,. Lemma 2 is thus proved.
Proof of Theorem 2. Let a denote a transfinite ordinal number
of the 1-st kind. There exist transfinite numbers >1 of the 1-st kind
multiplicatively commutative with number a, e. g. number a itself. Let a,
denote the smallest number of the 1-st kind, greater than 1, multiplica-
tively commutative with number a; as we know, it will be a transfinite
number. We shall show that a is a power of number a, with a natural
exponent. This would be obvious in the case of a = a,. Therefore let us
assume that a ~ a); from the definition of number a, it follows that
a >a). By Lemma 1 there exists a number ag such that a = aoa = ajay
and adg = aga.
We shall have a> ag since a) > 1 and a= agay. Number a, being
wultiplicatively commutative with the transfinite number of the .1-st
kind, a, is also transfinite of the 1-st kind In view of aaj= aja and of
the definition of number a, we thus have a > ay. If aj = a), we should
have a= a0 = aa.
Therefore let us suppose further that a, = aj >a). By Lemma 1
there exists a number a, such that a, = apa, = aya) and aya. = Q20,; Since,
in view of a,=— ao, we have a,a= aa,, and since aa,= aja, we have
Ap Ms = O09A, = Ayaa,, whence a4 = aa,.
') The idea of the proof is the same as Jacobsthal’s [1] (p. 483), but his proof is
somewhat longer than ours.
§ 26. On ordinal numbers commutative with respect to multiplication 355
and ay = ya, the numbers 6 and y are also finite and py = yf.
If a is a transfinite number of the first kind, then, in view of af = fa
and of ay = ya, the numbers f and y will also be transfinite of the first
kind, and in that case, by Theorem 2, there exist natural numbers p
and gq such that 6 = af, y = 0, whence fy = yf.
Finally, if a is a number of the 2-nd kind, then, in view of af = pa
and ay = ya, the numbers 6 and y will also be of the 2-nd kind, and in
that case, by Theorem 1, which has already been proved for numbers
of the 2-nd kind, and in view of af = fa there exist natural numbers m
and » such that a" = p”, and in view of ay = ya there exist natural num-
bers p and q such that a? = y7. Hence we have fp = ar" = y%, and apply-
ing Theorem 1 to numbers of the 2-nd kind, 6 and y, we again infer
that By = yp.
Theorem 3 is thus proved. It will be observed that it is not necessarily
true for order types, for it can be shown for instance that for a= 2,
B= on, y= on we have af = pa and ay = ya but By A y6.
We shall now prove Theorem 1 for transfinite ordinal numbers of
the first kind. Suppose that a and f are transfinite ordinal numbers of
the first kind. By Theorem 2 there exist natural numbers m and n» such
that == 0550 = Bos whence a =p"
Conversely, suppose that a and / are transfinite ordinal numbers
of the first kind such that for certain natural numbers m and n» we
have a”?= Bp". We have of course 6f"= BB, therefore fa* = a”B, and
since also aa” = a"a, we have, by Theorem 3 (for the numbers a”, 6 and a),
ONO /6x0s
Theorem 1 is thus proved for transfinite ordinal numbers of the
1-st kind, and since it has already been proved for numbers of the 2-nd
kind, it is now proved in full.
From Theorem 1 follows
COROLLARY 1. For every ordinal number a > 1 the set of all ordinal
numbers multiplicatively commutative with number a is denumerable.
§ 26. On ordinal numbers commutative with respect to multiplication ODT
Solution. They are all numbers of the form w?o(?"-+ ) where k = 0,1, 2, ...
The smallest of them is a. If they were all represented by the formula &” (m = 1, 2, ...)
where é is an ordinal number (greater than 1 of course), then we should have =a
and there would exist no ordinal number f multiplicatively commutative with nuim-
ber a and such that a? < 6 < a’, while in fact B = w?*(w®*-+ @) is such a number be-
cause
a = m°7*(m°2 +) and a= w°*(w?* +o).
5. Prove that each two of the order types 7, 1+7, 4+1 and 14+ 4+ 1 are multi-
plicatively commutative.
6. Determine all order types, at most denumerable, that are multiplicatively
commutative with the type 7.
Solution. They are types 1, 7, 14-7. 4+1 and 1+ 7+ 1. Their multiplicative
commutativity with type 7 follows from Exercise 5. On the other hand, if a is an order
type at most denumerable and such that an = ya, then from Exercise 1 of XII.10 it
follows that ya = 7; therefore an = 7 and a is a left-hand divisor of type 7, 7. e., accord-
ing to Exercise 5 of XII.3, a must be one of the types 1, 7, 1+ 7, 7+1 or 1+ +1.
7. Give an example of a non-denumerable order type, multiplicatively commutative
with type 7.
Answer. Type 7/7.
8. Give an example of a non-denumerable order type € such that 2-€= €-2.
Answer. € = wAn.
9. Give an example of a non-denumerable order type &€ such that wé = éw.
Answer. €= An.
10. Give an example of denumerable order types f, y and 6 such that 2-6 = p- 2,
2-y=y-2, 2-6= 6-2, but Py ~ yB, BO ~ OB, yd H$ Oy.
Answer. B=on, y= ow*n, 6 = (w*+ ow).
11. Give an example of two different infinite order types both additively and
multiplicatively commutative.
Answer. Types w+@* and (w+ o*)?.
12. Prove that there exist no two different transfinite ordinal numbers both ad-
ditively and multiplicatively commutative.
Proof. Suppose that there exist two transfinite ordinal numbers a and f such
that a #4f, a+ f = B+ a, af = Ba. In view of af = Ba, numbers a and f are known
to be either both of the 1-st kind or both of the 2-nd kind.
Let us suppose that the transfinite ordinal numbers a and 8B ~£ a are both of the
1-st kind and assume, for instance, that a > 6. By Theorem 2 (in view of af = fa)
there exists an ordinal number a, (transfinite of course) and natural numbers m and n
such that a= /, f =a, and, in view of a > B, we have m > n, therefore m = n+ p
where p is a natural number. In view of a+ f = B+a we have a, +a, = a,+a" there-
fore aj(aj+ 1) = aj(a+ aj), which gives a}+1=1+ a> which, as we know, is impos-
sible since number | is not additively commutative with any transfinite number.
Suppose therefore that number a and 6 ~a are of the 2-nd kind. In view of
a+ Pp=f6-+a and by Theorem 1 of XIV.25 there exists an ordinal number é, of the
2-nd kind of course, such that = wé,, and natural numbers k and 1 such that a =
=ék, 6 = él. Thus we have af = ékw&,l= €0&,l, Ba = Elwé,k = Ewé,k, therefore, in
view of af = Ba, (fm&,)l = (wé,)k, which gives | = k, therefore a = B, which is contrary
to the assumption.
® § 26. On ordinal numbers commutative with respect to multiplication 359
where 1, %.,... 18 an arbitrary infinite sequence of natural numbers satisfies the condi-
tion 2-€= &-2 and that &(m,, me, ...) = &(m, me, ...) only if m =n, for k= 1, 2,...
16. Give an example of 2° denumerable order types no two of which are multi-
plicatively commutative.
Answer. Types &(n,, 2, ...) from Exercise 15.
17. Find what ordinal numbers a may be represented in the form a = yy where
Ly F ve.
Answer. All transfinite ordinal numbers that are not of the form (wo + 1)”, where&
is an ordinal number, n =1, 2,..., and only such numbers.
wy g
= (wa?!b = wo” awe
19s b 21
a" = w ap’ b
B wE+ by by; iP Pt
a =a SAAT as
being a number of the 2-nd, kind, (1+ 4,)a= (t+ 4,)wé = tw§ = ta.
Thus £* = w™. Therefore we have af = fe, 7. e. formula (27.1).
Theorem 1 is thus proved.
From Theorem 1 it immediately follows that the solution of equa-
tion (27.1) in the smallest transfinite ordinal numbers a and f >a will
be obtained by setting a= m and 6 = ew where « is the smallest epsilon-
number, 7. e. the number (20.1). Another solution will be obtained by
setting a= w-2 and P= «w- 2.
From Theorem 1 we also immediately obtain the following
COROLLARY. All solutions in ordinal numbers & 4 w of the equation
a = & are obtained by setting — = tw where t denotes any epsilon-number.
This corollary can also easily be proved directly (Sierpinski [52],
p. 49). ;
In XIV.20 we have proved that for every ordinal number y there
exists an epsilon-number >y. Thus from Theorem 1 it immediately fol-
lows that for every ordinal number a of the 2-nd kind and for every
ordinal number y there exists an ordinal number f > y satisfying (27.1).
However, there are ordinal numbers a of the 2-nd kind for which there
exists no ordinal number 6 <a satisfying equation (27.1). H.g. there
is no such number f/ for any ordinal number a such that wo <a< «qa,
e. g. for a= e. It can be proved however that for every transfinite ordinal
number a (of the 2-nd kind) there exists at most one ordinal number B < a,
satisfying equation (27.1).
Indeed, let a be a transfinite ordinal number of the 2-nd kind,
a= ow a,+...+ oa, — its normal expansion, and suppose that 6 <a
is an ordinal number satisfying equation (27.1). In that case number f,
as we know, must also be of the 2-nd kind: let 6 = w%b,+...+ 0b; be
its normal expansion. By Theorem 1 in view of a>, we must have
a= tf where t is an epsilon-number >f. Thus a= 18 = 0B = w'"b,+
+o",
+ ...+.0°°%b,, whence, by comparison with the normal ex-
pansion of number a, we obtain /= k,b; = a; for i= 1, 2,...,1,7+fi= a,
LOrt.== 1, 2.2. 51. Whee: .B; —30)— 7 fOr a= 2s. 1. Number deiedt
exists) is thus well defined by number a.
aioe Le Cm AAMC aiam= Tee LNs rete ty ead t= Pb > Ga Anew. OL etsai—
=T3T,a we have 1t,at,= 7,7,aT,. But, since epsilon-numbers have the property of
absorbing positive factors smaller than they are and immediately preceding them (see
Theorem 1 of XIV.20, formula (20.4)), we have at, = t, and 73;7,aT, = 7T,; thus equality
Tet, —= TstyAt, LIVES 72 = %,, which is impossible since 7? = 7,7, > 7,-
2. Find all systems of positive ordinal numbers a, f and y such that a8” = (a*)’.
3. Find out for what systems of positive ordinal numbers a, 6, y we have a8” < (a’)”.
Solution. The above inequality may hold only for a > 1, y > 1 and then it is
equivalent to the inequality p’~* < y. The latter inequality holds of course for
B=1 y>1. Hy>o, then we have y-l=y and, in view of y>1 and of
formula (18.2), we obtain p’—* = B” > y which proves that for 6B > 1, y > @ our inequa-
lity does not hold. Therefore let us suppose that 6 > 1, 2<y< @. By means of an
easy induction we prove that 2” > m+ 1 for natural n. Thus B’~* > 2’ > y for B>1
and 2<y< wo and in this case our inequality does not hold.
Thus finally we reach the conclusion that inequality a6” < (a°)” holds for positive
ordinal numbers a, 6 and y if and only ifa>1, B=1, y > 1.
28. Natural sum apd natural product of ordinal numbers. Let € and 7
be two given ordinal numbers. As we know (cf. the proof of A. Tarski’s
theorem in § 21) they can be represented in the form ¢ = wm, + wm, +
+... om, , (pire om, + oN, + jee oN, where ¢,,¢5,...,¢, are decreas-
ing ordinal numbers, and} m,, m2, ...,m, and 1, %,...,”, are integers >
> 0. Let
EXERCISES. 1. Prove that for any ordinal numbers a and f we have a+ P<
<a(+)f.
2. Determine the smallest ordinal number a such that a+a < a(+)f.
Answer. a=—a-+1l. We have here a+a=q@-1+1 and a(+)a=o-2-+ 2.
3. Determine the smallest ordinal number a such that a(-)a < a-a.
Answer. a=o-+1. We have here a-a=o?+o+1, a(-)a=o@*+@-2+1.
A closer investigation of types 2”, 7°. 2°° and (w*w)°” is left to the
reader. The last type can be proved to be the type of the set of all irra-
tional number, ordered according to their magnitude.
We shall also mention here another definition of the power of order
types, constituing a generalization of the definition of the power of
ordinal numbers given in XIV.14. Let a and w be two given types, A and JW
two ordered sets such that 4 = a, M = yw, and assume that the set M
has a first element, m,. Let us consider the set Z of all functions f(x)
defined for the elements « of the set A, whose values are elements of the
set M, the inequality f(r) #4 my holding only for a finite number (> 0)
of the elements # of the set A. Thus if 7, and f/, are two different functions
belonging to the set 7, then among the elements « of the set A, for which
f(x) A f.(x), there exists (in the’set A) a last,-a. Let us assume that
hfe if f(a) <fxa) (in the ordered set M), and /.<j, if f.(a)<f,(a)-
It is easy to verify that in this way the set Z will be ordered, and its order
type will depend only on types a and w. This could be accepted as a defini-
tion of the power uv*. According to this definition we should have, for
instance, 2°Qo* = 14-7 (while according to the definition given at the be-
5 Q@ *
ginning of this section type 2” is of the power of the continuum).
CHAPTER XV
Py + Oy + Ay +...
A=@+%+a.4+...,
whence
Dero Hag Ossetia
and since gy, = No, and a, <x, for n= 1, 2,..., we have A= Sis, Ue (On Avis
a number of the 2-nd class. Theorem 1 is thus proved. The proof is based
on the axiom of choice, since it makes use of the theorem (which we are
unable to prove without the aid of the axiom of choice) stating that the
sum of an infinite series of cardinal numbers a, <s, is a cardinal num-
ber =.
It will be observed that we are unable to prove Theorem 1 without
the aid of the axiom of choice.
As regards the limit of an increasing infinite sequence of type w
of numbers of the 1-st class, it is of course always number wo.
. From the theorem stating that the sum of a denumerable series of
numbers of the 2-nd class is always a number of the 2-nd class it im-
mediately follows that for every denumerable set of numbers of the 2-nd
class there exists a number of the 2-nd class greater than any number of
that set.
As an immediate conclusion we obtain
THEOREM 2. The set of all numbers of the 2-nd class is non-denumerable.
The proof of Theorem 2 given above has been based on the axiom
of choice. We are able, however, to prove Theorem 2 without the aid
ot the axiom of choice, namely in the following way.
368 XV. Number classes and alephs
Suppose that the set Z, of all numbers of the 2-nd class is denumer-
able. Therefore the set Z = Z,+Z, of all numbers of the 1-st and the 2-nd
classes would also be denumerable. The set 7, as a set of ordinal numbers,
is well ordered (according to the magnitude-of those numbers). Let us
denote its type by 2; it will be an ordinal number, and from our assump-
tion that Z =», it follows that Q=x,, 7. e. that Q is a number of the
2-nd class, which means that it is an element of the set Z. But every
ordinal number is known to be the type of the set of all ordinal numbers
smaller than that number; thus each number of the set Z is the type of
all numbers of the set Z which are smaller than that number, 7. e. the
type of the segment which it forms in the set Z Thus number 2, which
is the type of the well-ordered set Z, would likewise be the type of a cer-
tain proper segment of that set. Thus the well-ordered set Z would be
similar to a certain proper segment of itself, which is known to be impos-
sible (see XIV.5, Corollary 5). Therefore the set Z, cannot be denumer-
able. And being infinite, as it contains a denumerable subset composed
of numbers m+n where n= 0,1,2,..., it is (effectively) non-denumer-|
able, q.e: d.
From Theorem 2 it also immediately follows that for every denumer-
able set of ordinal numbers of the 2-nd class there exists a number of
the 2-nd class different from any number of that set. This conclusion
we are also able to prove without the aid of the axiom of choice. But we
cannot prove without the aid of that axiom that for every denumer-
able set of numbers of the 2-nd class there exists a number of the 2-nd
class greater than any number of that set.
For ae Z, let us denote by A(a) the set of all ordinal numbers <a;
it will be a set of type a, 7. e. a denumerable set. Let F denote the family
of all sets A(a) for ae Z,. For ae Z,, Be Z, a< 6 we shall have A(a)C
C A(f). The sum of all the sets of family F will of course be the set of
all ordinal numbers of the 1-st class and of the 2-nd class, 7. e. it is a non-
denumerable set. Thus there exists a family F/ of denumerable sets such
that for each two different sets of family F one is contained in the other
and the sum of all the sets of family F is a non-denumerable set.
It is also easy to give an example of a family with the same properties
formed of 2” different denumerable sets. Such is the family of all proper
segments of an ordered set of type 72 (where 2 denotes the order type
of the set of all ordinal numbers of classes one and two, ordered according
to their magnitude). From the formula 7a = 7 for every ordinal number > 0
of the first and of the second class (following from Exercise 5 of XI.3)
we easily conclude that each element of a set of type 72 determines
a segment of type 7 of that set. The type 7 is known to have similar pro-
perties. Thus from the fact that each element of a given ordered set,
§ 1. Numbers of the l-st and of the 2-nd class 369
72= »'n,
a<Q
formed of all (different) rational numbers. Let us divide the set X of all
real numbers into s, disjoint subsets as follows.
For a given real number «x let us denote by
Wap nD i) aa)
taken out from sequence (2.3). The set of all terms of sequence (2.3)
ordered according to their magnitude is a certain denumerable order
type g(x). If p(x) is not an ordinal number, then we shall assign number »
to the set X, and if g(x) is an ordinal number a (of the 2-nd class of course,
since the set of numbers (2.4) is denumerable), then we shall assign num-
ber x to the set X,. We shall have of course
where the sum is extended to all ordinal numbers a of class two. Thus
there are aS many components in sum (2.5) as there are ordinal numbers
§ 2. Cardinal number ys, 371
of the 2-nd class, 7. e. s,, the components of sum (2.5) obviously being
disjoint. Finally, in order to prove that they are non-empty we use
theorem 2 of VIII.8 according to which the subset of the set of all rational
numbers, ordered according to the magnitude of the numbers, can re-
present any denumerable order type. Thus for any ordinal number a
of the 2-nd class there exists an increasing infinite sequence of natural
numbers 71, .,... such that the set {7q,,1,,-..}, ordered according to
the magnitude of the numbers that belong to it, is of type a. Writing
Ee ED ES le
we shall obviously obtain a real number belonging to the set X,. Thus
we have X, £0 for ae Z,. Thus we have proved (without resorting to
the axiom of choice) the following
THEOREM 3. We ave able to give an effective decomposition of the
set of all real numbers into s, non-empty disjoint sets *).
Using the relation <-« introduced after A. Tarski in VIII.4, we im-
mediately obtain the following
COROLLARY. We are able to prove without the aid of the axiom of choice
that s; <2".
The decomposition (2.5) given by H. Lebesgue [1] (p. 213) has been
investigated in detail by N. Lusin [1] (p. 1-95) in his extensive work on
analytic sets. As to the nature of the components X, of the sum (2.5),
Lusin has proved that for a> oq they are B-measurable (7. e. they can
be obtained from intervals by means of a denumerable aggregate of addi-
tions and multiplications), while the set X, is not B-measurable but it
is an analytic set (7. e. a projection of a B-measurable plane set, or,
which is the same, a continuous image of the set of all irrational numbers).
It could also be proved that, with a suitable choice of the index a,
X, may be a set whose class (in the classification of B-measurable sets)
is higher than an arbitrary ordinal number of the 2-nd class.
In virtue of the axiom of choice there exists a set N containing one
and only one element from each of the sets forming the components of
series (2.5); the set N is of course equivalent to the set of all components
of series (2.5), 7. e. it is of power s,. Thus we have proved with the aid of
the axiom of choice that there exists a set of real numbers of power s,,
whence follows Theorem 2.
I have given another effective decomposition of the set of all real
numbers into s, non-empty disjoint components in Fund. Math. 29 (1937),
p. 2-3.
1) Tt can easily be shown that each of those sets (in the decomposition (2.5)) is
of the power of the continuum (Church [1], p. 183).
24%
372 XV. Number classes and alephs
p(L)
= SX,
aeT
numbers a set of real numbers f(A) different from each of the sets contained
an the set A.
Indeed, let 4 denote a given at most denumerable set of sets of real
numbers. Since the components X, of sum (2.5) are disjoint, non-empty,
and their number is s,, while the set 4 is at most denumerable, there
exist among them components which do not belong to the set 4. Let
mw denote the smallest ordinal number (>) such that X,¢ A and let
f(A)= X,. Thus f(A) will be a set of real numbers not belonging to
the set A.
Hence it easily follows that we are able to associate with every na-
tural number n and with every at most denumerable set A of sets of real
numbers, contained in the interval J,=[n—l<a<n], a set of real
numbers f,(4;,), contained in the interval J, and different from each of
the sets belonging to the set A,.
Now let F denote the set of all sets of real numbers and let us sup-
pose that F is the sum of a denumerable aggregate of denumerable sets
of sets of real numbers. Thus there would exist an infinite sequence
Gi, Go,..., where G, (n=1,2,...) is @ denumerable set of sets of real
numbers, such that F=G,+G,+... For n=1,2,..., let us denote
by #H, the set of all sets-1,4 where 4 <G,. The sets A, (v= 1,27)
will of course be at most denumerable sets of sets of real numbers of the
interval J,. Now let
(2.8) Q = fi( Hy) + foe) +... :
we shall of course have Qe; therefore, in view of F = G,+G,-+...,
there exists a natural number » such that @ « G,, whence in view of the
definition of the set H,, we have J,Q « H?. But from the definition of
the function f, it follows that /,(H,)¢H, and f,(H,) «J, and, the inter-
vals I,, I,,... being disjoint, formula (2.8) gives J,Q = f,(H,). Thus we
should have J,Q ¢ H,, which is contrary to the formula J,Q « H, proved
above.
Thus we have proved without resorting to the axiom of choice A. Tar-
ski’s theorem stating that the set of all sets of real numbers is not the sum
of a denumerable aggregate of denumerable sets whose elements are sets of
real numbers.
In VI.7 sets constituting sums of a denumerable aggregate of de-
numerable sets have been termed D,-sets. Therefore the above theorem
can also be expressed as follows:
We are able to prove without the aid of the axiom of choice that the set
of ‘all sets of real numbers is not a D,-set.
Analogously, we could prove without the aid of the axiom of choice
that a set of power 2% is not a D,-set. For this purpose we should base
§ 2. Cardinal number 3, 375
(uh) Pat oa
Let. us denote this supposition by H. By Theorem 2 of XV.2 and
formula (2.1) it follows from H that we must have
(3.2) cee
since otherwise s, would be an intermediate cardinal number between Xo
and 2°
On the other hand, from formula (3.2) and Theorem 1 of XV.2 it
follows that there is no cardinal number 1m satisfying inequality (3.1),
which means the same as hypothesis H.
Hypothesis H is thus equivalent to equality (3.2). However, our
proof of this equivalence has been based on the axiom of choice since
we have used in it Theorem 2 of XV.2, proved with the aid of
the axiom of choice. Without the aid the axiom of choice we are unable
to prove that hypothesis H implies formula (3.2).
THEOREM 1. The hypothesis that 2°—vs, is equivalent to the theorem
stating that the set of all points in a plane is the swm of two sets of which
one is at most denumerable on every parallel to the axis of abscissae and
the other is at most denumerable on every parallel to the axis of ordinates
(Sierpinski [5] and [12], p. 179; Tietze [1], § 18, p. 307).
Proof. Let us assume that 2°°=—x,. The set X of all real numbers
is thus of power s, and therefore is equivalent to the set Z—= Z,+Z,
(of type 2) of all ordinal numbers of the 1-st class and the 2-nd class
(ordered according to their magnitude). Thus there exists a transfinite
sequence of type 2
points of the set A. On the other hand let a denote a given real number,
é. g. @=t, where ae Z. Points of the set B= P—A with the abscissa a
are obviously the points (f,, t2) where & < a; in view of ae Z we conclude
that the line « = a contains an at most denumerable aggregate of points
of the set B.
Thus from the assumption that 2°°—s, follows the existence of
a decomposition of the plane P= A+B satisfying the required condi-
tions. Now we shall prove that, conversely, the existence of such a de-
composition implies the formula 2°° = x,.
Let us assume that 2° ~s,. By Theorem 2 of XV.2 we should have
2" — sy, and there would exists a set Q formed of x, lines parallel to the
axis of abscissae. Let us denote by N the set of all points of the set A
lying on the lines forming the set Q. Since each of the lines belonging
to Y contains av at most denumerable aggregate of points of the set A
‘(according to the properties of the set A), and the set Q contains ys, lines,
we easily conclude with the aid of the axiom of choice that the set N is
of power <,Nq, 7. €. invirtue of the Exercise 1 in XV.2, of power <y,,
whence, by Corollary 3 of theorem 1, Chapter VI, we conelude that
also the projection of the set N on the axis of abscissae is of power <x.
Thus in view of 2° >, there exists a-point x, of the axis of abscissae
that is not a projection of any point of the set N on that axis. Hence
it follows (in view of the definition of the set N) that each point of
intersection of the line x= a, with the parallels to the axis of abscissae
belonging to Q belongs to the set P—A = B. The line x= x would thus
contain a non-denumerable aggregate of points of the set B, which is
in contradiction to the properties of that set. Therefore we must have
2° — 9,
‘Theorem 1 is thus proved (with the aid of the axiom of choice).
As has been observed by N. Lusin, this theorem can also be ex-
pressed in a different way. Every set of points in a plane that has one
and only one point in common. with every parallel to the axis of ordinates
is called a curve of the form y = f(x), and every set of points in a plane
that has one and only one common point with every parallel to the axis
of abscissae is called a curve of the form x = f(y). Theorem 1 may obviously
be expressed as follows:
Now let us call a curve every set of points in a plane that is con-
gruent (by translation or rotation) to the graph of a one-valued function
of a real variable (7. e. to a curve of the form y = f(#)). It is by no means
378 XV. Number classes and alephs
easy to prove that a plane is not the sum of a finite number of curves:
it was proved in 1933 by S. Mazurkiewicz [2]. In the same year I
proved with the aid of the continuum hypothesis (using Theorem 1)
that a plane is the sum of a denumerable aggregate of curves congruent
to one another, and also that there exists a function of a real variable (x)
whose graph y = y(#) may be divided into a denumerable aggregate of
fragments (corresponding to the intervals k <#<k+1) with which by
suitable changes of position (by translation and rotation) we can cover
the whole plane (Sierpinski, Fund. Math. 21 (1933), p. 39-42).
It has not been resolved whether from the assumption that a plane
is the sum of a denumerable aggregate of curves follows the continuum
hypothesis.
Without the aid of the continuum hypothesis we are able to prove
that a plane is not the swm of two sets, one of them finite on every parallel
to the axis of abscissae and the other at most denwmerable on every parallel
to the axis of ordinates (Tietze [1], p. 291, and Sierpinski [59], p. 6-7).
We can also prove without the aid of transfinite numbers that the
continuum hypothesis is equivalent to the proposition that every plane
set that is not of the power of the continuum is the sum of two sets of which
one is finite on every parallel to the axis of abscissae and the other finite
on every parallel to the axis of ordinates (Sierpinski [63], p. 297).
Proof. Let a denote a given number of the 2-nd class and of the
2-nd kind. The set of all ordinal numbers é < a is thus denumerable and
does not contain a greatest number. Let
(4.3) == iM ar
n<w
for which o, < a. We shall have N(a) 4 N(f) for different numbers a
and f of the 2-nd class since if (,, /,,... is a sequence, associated with
number f, of all ordinal numbers </, then, in the case of N(a) = N (fp),
the formula oa, < q, for any natural k and /, would be equivalent to the
formula $6, < 6;, which is impossible because the set {a,, a, ...}, ordered
according to the magnitude of the numbers that compose it, is of type a
while the set {6,, B., ...} is of type $8 4 a. Now let
ie)
= 2 Eo n n
oa n
)
where ”,, %.,... are numbers of the set N(a) ordered according to their
magnitude. We shall have «(a) £ «(f) for different numbers a and f/ of
the 2-nd class. The set of all real numbers «x(a) corresponding to the
ordinal numbers a of the 2-nd class is thus of power ys, 1).
Let it also be observed that A. Church has deduced the formula
s, < 2 without the aid of the axiom of choice from the assumption that
there exists a function associating with every ordinal number a of the
2-nd class an infinite sequence with the upper bound a (Church [1], p. 191,
Hardy [1], p. 87-94, Kurepa [6], p. 175-176).
Finally, it will be observed that it is possible to prove that if we know
a function f(a) establishing a (1-1) correspondence between the set of all or-
dinal numbers of the first and of the 2-nd class and the set of all real num-
bers, then we should be able (with the aid of the function /) to define a func-
tion g(a), associating with every ordinal number a of the 2-nd class
a certain infinite sequence a,, a,,..., formed of all ordinal numbers < a
(Sierpinski [59], p. 4).
From Theorem 1 it easily follows that every number of the 2-nd
class and of the 2-nd kind is the sum of an infinite series (of type w) of
positive ordinal numbers smaller than the number in question. Indeed,
if a is a number of the 2-nd class and of the 2-nd kind, then we have
TORMIIIA (4,5), “were: dg, <0, < .... Thus, for 7 ='2, 3, 2250p = Gna +p,
where 6, >0 for »—2,3,..., and a,+/.+ f.+...+6,—= a, for n=
= 2,3,..., whence we conclude that
a= lima, = a,+$,+ 63+...
n<w
If a set whose elements are numbers of the first or of the 2-nd class,
ordered according to the magnitude of the numbers that compose it,
gives an increasing transfinite sequence of type 2, f{as}ecqg such that
It can be proved (with the aid of the axiom of choice) that the set Z
of all ordinal numbers of the 1-st and of the 2-nd class is the sum of
a denumerable aggregate of sparse sets. We are unable, however, to
decompose the set Z effectively into the sum of an infinite series (of
type w) of sparse sets; it can be proved that if we were able to do it, we
should also be able to define a function associating with every ordinal
number a of the 2-nd class a certain infinite sequence formed of all
ordinal numbers <a (see Sierpinski [33], p. 289-296).
EXERCISES. 1. Prove the following theorem:
For every transfinite sequence of type 2, (dg}ecg whose terms are ordinal numbers < 2,
and for every ordinal number uw < 2 there exists an ordinal number 4 of the 2-nd hind
such that w<24< 2 and
(4.4) dp < A for Ns
Since f, > 1 and the sequence {f,},_9 is increasing, y, = By, > Py = V1, t. & Y2 > V-
Now let » denote a given natural number and suppose that y,., > y,. Since sequence
(4.5) is increasing, therefore, by (4.6), we have y,.. = ee Si Vane this way
we have proved by induction that y, << y.< ...
By (4.6) and since numbers (4.5) are ordinal numbers <2 we conclude that the
ordinal numbers y, (n= 1, 2,...) are also <Q. Thus, by Theorem 1, of XV.1 there
exists a number
(4.7) A = limy, <a Q
Now
and
Now let € denote an ordinal number <A. By (4.7) and (4.8) there exists a natural
number m such that € < y,, whence B, < Pap =Ymi1 <A and, by (4.5), we naturally
have a, < A. Formula (4.4) is thus proved.
In an analogous way we can prove the following theorem:
Tf f(&) is @ function associating with every ordinal number € an ordinal number f(é).
then for every ordinal number wu there exists an ordinal number 2 > yw such that
o
Hint. To prove this it is sufficient to write, for ordinal numbers 6,
and to define an ordinal number 4 as in the solution of exercise 1. We shall have Be <A
for € < A, whence, by (4.10), we obtain formula (4.9).
3. Prove the following theorem:
There exists no function f(f) defined for numbers of the 2-nd kind € < Q and such that
and
Proof. For an ordinal number a < @ let us denote by H, the set of all ordinal
numbers <a.
Suppose that there exists a function f(f), defined for numbers of the 2-nd kind
€ < Q2 and satisfying conditions (4.11) and (4.12). For the ordinal number a < Q let
us denote by g(a) the least number of the 2-nd kind greater than any number of the
set pa) where tf (.) denotes the set of all ordinal numbers of the 2-nd kind € < Q
for which {(¢) « H,. We shall have g(a) < Q since, in view of a < Q, the set H, is at
most denumerable, as well as the set i :) (which follows from (4.12)), and thus there
exists an ordinal number of the 2-nd kind <Q, greater than any number of the set
fei i.): We shall prove that
Indeed, if we had g(a) < a for a certain ordinal number a < Q, then (on account
of the fact that g(a) isa number of the 2-nd kind) we should have, by (4.11), f(g(a)) <
< g(a), therefore f(g(a)) <a, which gives f(y(a))«H,, whence g(a) «f '(H,), which
is contrary to the definition of the number g(a). Thus formula (4.13) holds. Thus the
infinite sequence (of ordinal numbers <Q)
(4.15) A =limg"(@)
N<w
(i. e. the smallest ordinal number greater than any of the terms of the sequence (4.14)
is a number (4.15) of the 2-nd kind <Q. By (4.11) we have f(A) < 4, therefore, by (4.15),
for a certain natural number m we have f(A) < g”(w), which gives f(A) « H, aw)? and since
A=f (f(A), we have 4 « f'(Ayney)> which, in view of the definition of the function g,
gives g”**(w) = g(g"(w)) > 4, in contradiction to (4.15). Thus the assumption that there
exists a function f having properties (4.11) and (4.12) results in a contradiction,
q. e. d
From the above theorem we easily deduce the following paradoxical theorem of
J. Novak:
“We take away one element s, from the given infinite countable set 4,, we add
a new infinite countable set A, to the remainder, from the set
is
» 41,- Boies
A<2
384 XV. Number classes and alephs
we take away one element s,, add the new infinite countable set A, and we continue
in this way so that from the set
eu
d, A,— ‘8Wika
1
A<a
(unless it is empty) we take away one element s, and then we add a new infinite coun-
table set A,. Then there exists an ordinal number # <2 such that the set of all
given and added elements is the same as the set of elements taken away, 7. €.
.) 2°
ye A, = Shes
: A<B
(Nioweikes lal ep emaed):
For let us suppose that we can choose the elements s, in such a way that
mar 7 =
» A,#~{s},-5 for @<Q.
A<B
Since 4,(A < 2) are arbitrary denumerable disjoint sets, we can put A, =
Be
= {0., Garr> Gara> } Or evienne:
even simply A, a= {wA,@A+1,
l wA+2,‘ ...}.) Let f(wa) 2505
= *a for
a <Q. In view of
Suq \!
f€ yi A, De Sire
a
8)54
(oar
A<a
we shall have f(ma) < wa and f(wma) ~ f(md) for 2 < a < Q, which is contrary to the
theorem of Exercise 3. Thus we have proved Novak’s theorem.
4. Prove that an ordered set of type Q will not change its order type if we remove
from it an arbitrary finite or denumerable aggregate of elements.
5. Prove that there is no order type 40 for which the types m and 2 are right-
-hand divisors.
Proof. Suppose that such is the type gy € 0, i. e. that gp = aw = BQ. In view of
go #0, we havea 40 and § #0. Let A,B,C and D be ordered sets of types a, 8, w
and © respectively, e.g. C= {1, 2,3, ...}, D= {S}eeg-
Let us order the Cartesian products 4 xC and Bx D according to the principle
of last differences. In view of aw = BQ, the ordered sets 4 xO and BxD obtained in
this way, will be similar and let f denote a similar mapping of the former upon the
latter. Let a be any element of the set A: we shall have (a, n).«AxCO for n= 1, 2,...
In the set A xO there is no element that would succeed each of the elements (a, »)
where n = 1, 2,..., since each element of the set A xC is of the form (a, k) where «« A
and k = 1, 2,..., and in the set A xC we have (a, k) < (a, k+ 1). In view of the simi-
larity of the sets A xU and Bx D, also the set BxD has no element succeeding each
of the elements f((a,)) where n = 1, 2,... But, in view of f((a,”))«BxD for n=
= 1,2,.., and D= {&),-,. for every natural number n there exist one (and only one)
element 6, of the set B and a (unique) ordinal number &, < Q, such that f((a,)) =
='(0,,'6,). In view of €, <= O'forn = 1, 2; ..; numberase: (2 = 1, 25%..) are or the: esp
or of the 2-nd class, and since we have proved in XV.1 (with the aid of the axiom of
choice) that for every denumerable set of numbers of the 2-nd class there exists a num-
ber of the 2-nd class greater than any number of that set, there exists a number uw < Q
such that w > &, for n = 1, 2, ... Thus in the set B x D we shall have f((a, n)) = (b,é,) <
< (b,) and (b, u) e Bx D, which contradicts the fact that in the set BD there is no
element succeeding each of the elements f((a, )) = (b,, &,) for n = 1, 2, ... The assump-
tion that there exists an order type 40 whose right-hand divisors are types w and Q
leads to a contradiction, q. e. d.
§ 4. Properties of ordinal numbers of the 2-nd class 38: wt
Ges Ihnen, «
N<@
Let f(@) = (0, 1, 2,...). Now let a be an ordinal number of the 2-nd class >
and suppose that we have already defined a function f(&) fora <& <a. Ifa=f+1,
then let us denote by f(a) an infinite sequence whose first term is 6, and the (n+ 1)-th
term is the n-th term of the sequence f(f) (for n = 1, 2,...). If a is a number of the
2-nd kind
a = lima nm?
Now
then let us arrange the double sequence f(qa,), f(q),... into an ordinary sequence by
the diagonal method and remove from the sequence thus obtained all terms equal
to any of the preceding terms. Let us denote the remaining sequence by f(a). The func-
tion f(a) is thus defined by transfinite induction for all numbers a of the 2-nd class
and obviously satisfies the required condition.
7. Prove the following theorem of A. Denjoy [2], p. 1394: If f(&) ts a@ function as-
sociating with every number & of the 2-nd class a certain positive ordinal number f(&) < &
(not necessarily of the 2-nd class), then there exists an ordinal number a < 2 for which
there are non-denumerably many ordinal numbers —< 2 such that f(§) =a.
Proof. Let f(€) denote a function, defined for # < € < 2 whose values are ordinal
numbers, such that f(€) < € for om< € < 2 and suppose that Denjoys’ theorem is not
true, 7. e. that for every ordinal number a < 2 the set
BE,= KUO=a)
é=Q
is at most denumerable (or empty). For every given ordinal number uw < 2 let
it will be an at most denumerable set of ordinal numbers <2 and thus there exists
the least ordinal number g(mu) < 2, greater than any number of the set H,. Moreover,
Cardinal and ordinal numbers 25
386 XV. Number classes and alephs
we have ae cH, for up<<» < Q, therefore p(u) < p(v) for uw< » < Q. In view of we Hy 5
we have u < p(u) < 2 for wu<2. The infinite sequence
A = lim”(1) ;
n<u
it will be an ordinal number of the 2-nd class. In view of the properties of function f
we shall have a = f(A) < A; thus there exists a natural number n such that gy” *(1) =
<a< q’(1). In view of f(A) = a we have Ae HE, and since H,c H,, A« H,, whence in
view of the definition of function y we obtain g(a) > 2. But in view of a < y"(1) and
v(t) < y(v) for uw < » < 2 we obtain p(a) <y"" (1) < 4, therefore g(a) < /, in contra-
diction to the inequality p(a) > A obtained above. Thus Denjoy’s theorem must be true.
Having proved this theorem (by a different method from ours), A. Denjoy [2],
(p. 1396) writes: “Thus we see how illusory, intangible, paradoxical is the nature of
the 2-nd class of ordinal numbers’.
8. Find all the ordinal numbers € for which &? = Q°.
Answer. These’are all the ordinal numbers é such that Q< € < Q®. For if 9 =
= 2°, then > (because for § < 2 we had &° < 2 < ®) and also & < 2® since
(G2E\P ss Ge, :
On the other hand, if Q< é < ®, then, in view of
Q® = lim",
n<mw
there exists a natural number » such that € < Q”, whence Q° < &® < Q” = Q” (since
nw =), therefore €° = Q®. Since there exist uncountably many numbers & for which
2<=& <Q” (such are, for instance, all the numbers 2+ & where 0 < & < Q), the equa-
tion €® = 2° has uncountably many solutions for the ordinal numbers &.
9. Prove that for each positive ordinal number a <2 we have ya=y7 and
(1+A)a = 1-+A (where 7 is the order type of all rational numbers and A the order
type of all real numbers ordered according to their magnitude).
10. Prove that the order type 72 is 47 but to each element a of any ordered set
type of type 72 corresponds a segment of the type 7.
11. Prove that the order type (1+4)2 is #41-+A, but to each element a of any
ordered set of type (1+4)Q2 corresponds a segment of the type 1+.
12. Prove that for € = (14+A)@ any ordered set A of the type + é* has a gap,
but the axiom of Ascoli is true in the set A(cf. XI.5).
5. Transfinite induction for numbers of the i-st class and of the 2-nd class.
Theorem 1 of XV.4 imples the following
THEOREM 1. Jf a given set A of ordinal numbers
1) contains number 0,
2) contains number a+1 if it contains number a
3) contains the limit of every increasing infinite sequence (of type wo)
whose terms are elements of the set A,
then the set A contains every ordinal number of the 1-st or of the 2-nd class.
fa
§ 5. Transfinite induction 387
In other words, the set of all ordinal numbers of the 1-st or of the
2-nd class is the least set of ordinal numbers that satisfies conditions 1),
2) and 3).
Proof. Suppose that a given set jA, satisfying conditions 1), 2)
and 3), does not contain a certain ordinal number of the 1-st or the 2-nd
class; let ~ denote the least number of the 1-st or of the 2-nd class that
does not belong to A; by 1) we must have w > 0. If w were a number
of the 1-st kind, then we could write = a-+1 where a < yu: in virtue
of the definition of number uw, the number a, being smaller than uw, belongs
to A, whence, according to property 2) of the set A, we conclude that
number w= a+1 also belongs to A, in contradiction to its definition.
Thus w must be a number of the 2-nd kind. But then, by Theorem 1
of XV.4, we have
fineealinneyem
n<w
where oa, <u for n=1,2,..., and from the definition of number « it
follows that a,« A for n=1,2,..., whence, according to property 3) of
the set A, we conclude that we A, which is contrary to the definition of
number pu.
Theorem 1 is thus proved. Just as in XIIT.2 we immediately deduce
from it the principle of transfinite induction for numbers of the L-st class
and of the 2-nd class, which runs as follows: in order to prove that a the-
orem T is true for every number of the 1-st and of the 2-nd class it suffices
to prove that 1) theorem VT is true for number 0, 2) if theorem T ts true for
number a, then it must also be true for number a+1 and 3) if theorem T is
true for numbers a,, dg,..., forming an increasing infinite sequence (of
type wm), then it is also true for the limit of that sequence.
For the proof of a theorem T for every number of the 1-st class
finite induction, aS we know, is sufficient; it consists in proving that
theorem T satisfies conditions 1) and 2).
a = lima:
£<a
t x
to
388 XV. Number classes and alephs :
if for every real number ¢ > 0 there exists an ordinal number w= ule) < a
such that |a;—a! < «for « <&<a. It can easily be proved that in order
that we have
lima: =a
E<a
where a < Q it is necessary and sufficient that for every infinite sequence
of ordinal numbers &, (n= 1, 2,...) such that
line eo
n=oo
we have
Hin Sn
G2. a
n=oco
lim f<(7)
E<a
in the sense defined in the first part of this section. That limit is then
the iteration f,(a).
It can be proved that if for a given function f(x) of a real variable
the iteration fo(x) exists, then we have f(fo(«)) = fo(x), therefore also f,(~) =
= fo(xv) for every ordinal number a > 2 (Sierpinski [68]). Thus there is
no need to consider iterations of orders higher than Q.
With the aid of the axiom of choice it can be proved that there exists
a function of a real variable f(x) for which all iterations f,(7) where a < Q
exist and are different from one another.
§ 7. Initial numbers, alephs and their notation 389
Proof. Suppose that there exist ordinal numbers £ for which the
cardinal number x; does not exist and let a denote the smallest of them.
390 XV. Number classes and alephs
Thus there exist all numbers x; for & < a, whence also all sets Ug = Z(x¢)
for <a. Let us denote by S the set composed of all numbers of the
1-st class and of the set
ues
E<a
a<fP~+1 and 6<a+1, whence f=a. Thus the class Z(s,) is the set
of all ordinal numbers E satisfying the inequalities wy <& < Wg41.
Let us now assume that a is a given ordinal number and m a trans-
finite cardinal number such that m<s,. Number m is thus the power
of a certain subset M of the set 7 of all ordinal numbers & < a, (which
is of type a, 7%. €. of power @ =,). But each subset of the set 7 (which
is well-ordered) is similar either to the set 7 itself or to a segment of
that set. Still the set M cannot be similar to the set Z because M =m
while 7’ is of power s, > m. Thus the subset / must be similar to a segment
of the set 7, e. g. to the one determined by number é of that set. Hence
iM = E, since the segment determined by number é in the set 7 is the
set of all ordinal numbers <é, therefore a set of type &. In view of é« 7
and the definition of the set 7 we have & < a; thus if we denote by Z(s;
the class to which number & belongs, then we shall have ¢€ < a. We have
proved that for a certain ¢< a we shall have m= M = b= Ne.
Thus we have proved that if for a given ordinal number a a cardinal
transfinite number mM is <s,, then there exists an ordinal number 6 < a
such that m = s;. Hence it immediately follows that there is no cardinal
number greater than each of the numbers s;, where ¢ < a, and at the same
time smaller than s,. This proves that for every ordinal number a number sq
is the next cardinal number after all numbers s- where 6 < a.
In particular if a is a number of the 1-st kind, a= £+1, then the
inequality €< a= 6+1 is equivalent to the inequality ¢ <<, and from
the theorem proved above it follows that the inequality m < s,+1 implies
the inequality m<s,. This proves that for every ordinal number f, the
cardinal number Sg41 18 neat after 83, 1. €. between Xz and Sp11 there is no
intermediate cardinal number. (We do not forejudge whether there exists,
besides 8,41, another cardinal number n, being not an aleph and such
that between s, and 1 there is no intermediate cardinal number. This
question will be resolved when we prove in Chapter XVI with the aid of
the axiom of choice that every transfinite cardinal numbers is an aleph).
In particular there is no intermediate cardinal number between s,
and s,, which has been directly proved (Theorem 1 of XV.2). The
cardinal number immediately following s, is s,, then comes s,, etc. The
cardinal number immediately following all numbers of the sequence s,
(7— es) IS Spy UNEN “COMES Soir, CtC.
In XV.2 (in the proof of Theorem 2) we have associated to every
real number w @ certain denumerable order type g(x). Now let us denote
by Q the set of all real numbers w such that y(#) is an ordinal number
satisfying the condition
(7.1) 2 = Rox «
392 XV. Number classes and alephs
se =a Te
(8.1)
1) See Sierpinski [17]. p. 191. The first example of such a set has been constructed
with the aid of the theory of analytic sets by N. Lusin [1] (p. 92) and also in his
book [3].
I have been asked why a non-empty set in which we are unable to point out any
element cannot be defined more simply as a set of numbers of the interval (0, 1) if the
continuum hypothesis is true. and as a set of numbers of the interval (2,3) if the conti-
nuum hypothesis is false. Now, in such a set M we are able to point out an element,
for instance the number 0 if the continuum hypothesis is true and the number 2 if it
is false.
§ 8. Formula 53 = &, 393
assume that wu < w, and v < w,. In view of u < a, we have uw < sq (since w,
is the least ordinal number of power s,), whence, as we know from § 7,
it follows that uw <s, or w= s~ where & < a. Similarly, in view of » < a,,
we shall have » < s, or » =x, where 7 < a. Let us denote ¢ = max(é, 7);
we shall have €< a ae bw <s: and » <s;. Hence, by (8.2), we obtain
uty <setee = 2np <8? =, which proves that for 4= w+» we have
A <s; where 6 <a, therefore 7<s, and A<a,, q.e.d. Thus we have
proved that formula (8.3) holds.
Now let us consider, for a given 2< a@,, the set P,, 4. e. the set of
all systems (u,v) where w+yv= A. For every given uw <A there exists,
as we know, one and only one ordinal number v such that w+yv= A.
The systems (u,v) forming the set P, can be arranged according to the
increasing magnitudes of numbers wu; the set obtained in this way will
of course be well ordered of type 4+ 1 (since as uw we can take only ordinal
numbers satisfying the inequalities 0 < wu <A).
Now let us arrange the set P, regarding that one of the two systems
(u,v) and (w’, »’) as preceding which belongs to a preceding component
P, (vt. e. gives a smaller sum uw+y); and if both systems belong to the
same component P, we shall retain for them the same order relation
as in the set P,, ordered as above. Clearly, the set P will be well ordered
in this way. We shall prove that its type will be P = o,.
Let us assume that P > w,. Thus there exists a proper segment of
the set P which is of type w,; suppose it is the segment A determined
by the element (u,, »,) of the set P. In view of (u,, »,) « P, we have uw, + 7, =
=A, <,. Further, it can easily be seen that for every element (p, v)
of the segment A, since it precedes (,,%,), we Shall have uw <A, and
y <4,. Since the set of all numbers uw <4, is of type 4’ = 1,+1, where
2’ <q in view of 2, < a, and thus of power 2’, therefore segment A is
of power A < Aji’.
But in view of 2’ <a,, we have 4’ < Sa, which, as we know from
XV.7, gives 2’ = x; where é < a; thus we have TE sz where é < a, there-
fore? by (8.2), a <<, and since xz < 8, for € < a, we have ee Sa, which
contradicts the assumption that A is a segment of type a,.
We have proved that P >, is impossible. On the other hand? we
have P > @, because the systems (uw, 0), where « <@,, form a subset
of type ,, of the set P. Thus we have P = w,, whence P = gq.
But from the definition of the set P and of the definition of the
product of two cardinal numbers it immediately follows that Beas
In view of the formula P = Na obtained just now we should have so Ra»
in contradiction to the definition of number a. Therefore formula (8.1)
must be true for every ordinal number a. Theorem 1 is thus proved.
394 XV. Number classes and alephs
and of one pair, (@, 0), belonging to the w-th line and to the 0-th column.
The set P,41 18 formed of the pairs
(0, o+1), (1, -o@+ 1), (2; m+. 1), ..., (, 1),(o -), 0),
the last of which belongs to the 0-th column, the last but one to the 1-st
column and all the remaining ones to the column of order m+ 1.
The set Py. is formed of the pairs
(0, we 2), (L.@>2)5, (2,000 2) sees Diy @) (ol GD), (0 2 i}) 205 gel = 2 Ne
@ (Oy ly (Oe Vl ae
Thus @7 <@, <a", whence @,—a@".ay Numbers o, and w™ thus
belong to the same class, whence, on account of the fact that wm, is an
initial number, we obtain w, < w%, and since wm! <@,, we have w,= w” ’
and thus @, is a power of number @, q. e. d.
§ 8. Formula s5:=Nz, 395
: :
since for a < 6 we have x, <sg, whence NaX8s <3; 2 = 8s and on the other
hand, of course SuNg > Nz-
Another corollary to Theorem 1 is given by the formula
iB) Sots—=,
(ee) for a<f
because for a < 6 we have sq <4, whence sa+ 8s < 8g-+8s = 28 < Ses =
= ,, and on the other hand we have of course s,+, > &,. It is equally
easy to prove that for every ordinal number a we have n+sx,=s, for
ell seen hs
From Theorem 1 we easily deduce by induction the formula
(8.6) SC ae Ne
for every ordinal number a and every natural number 7.
However, we have
Shorea, <
No = No,
since, as we know from VIII.5, 5° = 2°°. Still, there exist cardinal num-
bers m for which m”°— m; such is for instance the number m= 2”.
As regards number s,, we obtain, in view of the inequality 2 <s, < 2°°
(proved with the aid of the axiom of choice in XV.2, Theorem 2), 2°° <
<s7° < (2%)*° — 2 whence
(8.7) pie 0%
(we are unable to prove this formula without the aid of the axiom of
choice); it follows hence that the question whether x=, or si° >, is
equivalent to the question whether the continuum hypothesis 2 = s, is true
or false.
It will also be observed that we have a formula which is, in some
sense, reciprocal with respect to (8.7), namely
(8.8) gh en ON
396 XV. Number classes and alephs
ie Doce 2.
QK<aK<wg
into s, disjoint non-empty components, which can be proved without
resorting to the axiom of choice.
. Indeed, let a denote an ordinal number such that 2 <a < @,, thus
a —s,. In view of Q =x, we have a = Q, and since a is the order type
of the set #, of all ordinal numbers <a, and 2 the type of the set Ho of
all ordinal numbers <Q, there exists a (1-1) correspondence f between
the numbers of the sets Hg and H#,. Let R denote the relation defined in
the set Hg by the conditions that Ry for <2, 4 <2 if and only if,
T(€) < f(m). By the relation R the set He is obviously well-ordered accord-
ing to type a.
Now we shall define a Woouion f(x,y) of two real variables in the
following way. If # and y are given ae numbers, then, by (2.5), there
exist ordinal numbers § < 2 and 7 <2, well-defined by x and y, such
that we X,+<, ye Xo+,- If, moreover, we have Ry, then let f(x, y) = 1;
otherwise let f(#, y) = 0. From the definition of the set ®, it easily fol-
lows that fe @®,. Thus the sets ® are non-empty for Q <a<@y,, q.e. d.
It is easy to prove without the aid of the axiom of choice that the
set F is of power 22°. Thus we are able to prove without resorting to the
axiom of choice that a set of power 27° is the sum of s, disjoint non-
empty sets, 7. ¢. that we have the formula s, < *2?°. It easily follows
that 2°° < 22°, whence x, < 2”°. Let us also observe that it is pos-
sible to prove without the aid of the axiom of choice that s, < 2%! and,
after A. Tarski, that 2°¢+? < 27° and x,41 < *2"¢ for every ordinal number a.
We can also say that the power of an infinite well-ordered set does not
change if we subtract from it a set of less power.
As an application of formula (10.1) we shall calculate the power
of the set of all ordinal numbers forming the class Z(s,). In § 7 we have
proved that the class-Z(s,) is the set of all ordinal numbers & satisfying
the inequalities wm, <& < wyii1. Since that set will be obtained by sub-
tracting from the set of all ordinal numbers <a@,+1, which is of type @g41,
i.e. Of power @y+1—= Nai1, the set of all ordinal numbers <a, which is
of power @, = &., the power of the set Z(s,) 1s, by (10.1), So41—Sa = Not1>
Hence we obtain
THEOREM 1. The class Z(s.) has the power Sq+1-
The set S of all ordinal numbers € < m, is the sum of disjoint sets
where S, denotes the set of all numbers of the 1-st class (7. e. of the num-
bers €é < m). For if €< @,, then é<@,=8, and in that case either ¢ is
a finite number, and thus £« 8, or & is an aleph and thus = 3 <»,,
whence 6 <a and therefore ¢« Z(s,) where fp < a.
Since, by Theorem 1, the set Z(xz) is of power sei and S,= x),
S=s,., we obtain by (10.2),
(10.3)
eee
Rees pAb NE ai
B}
E<a
o
(10.6) Sot
8, +o t+... << NyNoNs-.-5
Pde since s-< ky form = 152, 1, S88... << Ne, 1ormula (10:6) gives,
by (10.4),
(10.7) peas.ce
It is easy to prove that
a
| | Ne = Ra
E<a
for numbers of the 2-nd kind, proved in 1925 by Tarski [3] (p. 11), who
also proved that for numbers a of the first kind we have
4 a
[]s:=s%.
§<a
Kor sums it would be easy to deduce from formula (10.3) the follow-
ing formulae:
and
. Se=, for a of the 2-nd kind.
ata Dye aa
(10.9)
( NP
Di) =SS 2x
ONales for
On =n
Pe Oe 5) Re:
since, by (11.1), a> a; for é<q. Number @, is thus greater than any
term of the sequence {a,,}s<g- On the other hand, it is easy to prove
that @, is the least ordinal number greater than any term of the sequence
under consideration. For if we have y <a, then p<, since a, is the
least ordinal number of power s,. In view of y<s,, however, we have
Y=: (see XV.7) for a certain € <a. By (11.1), for the ordinal number
¢<a there exists an ordinal number u«<g such that a, >¢, whence
Na, > Xo = wy; therefore a, > Ys which proves that number y cannot be
greater than each term of the sequence {wa,}:<,. In view of the definition
of the limit of a transfinite sequence of ordinal numbers we conclude
that equality (11.2) holds.
Therefore the function of an ordinal variable f(a) = w, is increasing and
continuous. .
Thus we have also proved that the limit of every increasing transfinite
sequence of initial numbers is an initial number.
In particular, we have for instance
Og = nos
n<o
o = lino”,
n<o
we have
Oo(2) = LUMO,» -
n<=o
For we obviously have wo, >a for every ordinal number a since
the- set of all ordinal numbers § < m, contains a subset {m}s<, of type a;
thus if a is a number of the 2-nd kind and ow, > a, then in view of
Og = ima:
E<a
We have hence a,= ao, > w= a,, therefore a, > a,. Suppose that
for a certain natural n >1 we have a, >a,_1. We should have hence
Day > g,_,) therefore a,11 > a,. Therefore we infer by induction that
the sequence a, (n= 1, 2,...) is increasing. Let
a= litota,3
now
which gives formula (11.3). The number @,, however, is not regular,
being the limit of an infinite sequence of type m < a,. It can easily be
proved that the number w, which we have defined above is the least
initial number satisfying equation (11.3). For if wm; = 6, then we have
Op > ®@ = a,, and from the assumption that wg > a, it follows that Wop >
> Day yt. €. Op > Ont1, Whence ws > a, for n= 1,2,;...; and thus w, >@,:
Neither can we solve the question whether every initial number,
with an index of the 2-nd kind, of power <2” is singular. The assump-
tion that if is so is of course weaker than the continuum hypothesis;
it can be proved that it follows already from the assumption that 2° < Sor,
(Sierpinski [23], p. 153). With its aid it is possible to prove some important
§ 11. Regular and singular initial numbers 405
theorems which previously could only be proved with the aid of the
continuum hypothesis (Sierpiiski [23, p. 153-161; 19, p. 214; 20, p. 1],
Ulam [1, p. 140; 2, p. 22)).
K. Gédel has proved that the assumption that there exist no regular
initial numbers with an index of the 2-nd kind is consistent with the
axioms of the theory of sets (if these are consistent); it is not known if
the same apples to the assumption that such numbers exist.
Regular alephs s, whose indices are numbers of the 2-nd kind have
been termed inaccessible by K. Kuratowski't). With the aid of Cantor’s
hypothesis regarding alephs (according to which for every ordinal num-
ber a we have 2°*= s,11) it can be proved that the above definition is
equivalent to the following definition of inaccessible transfinite cardinal
numbers:
A cardinal number m > s, is termed inaccessible if for every trans-
finite sequence {mMg}:<, of type uw, where 0 << m, formed of cardinal
numbers m; < m for é< mu) we have
[ [ me<m
E<u
|
\) Tanita
i’ Z
xeX
2. ifm < m, then there exists a cardinal number p such that n < p< m.
The same author calls a cardinal number m inaccessible in the nar-
rower sense if m~0 and if m satisfies condition 1 and the following
condition:
1) Without the aid of the continuum hypothesis we are not able to prove that
there are no inaccessible numbers among the cardinal numbers <2“. In their book [1]
(p. 233) Kuratowski and Mostowski write that the addition to the set theory axioms
of the assumption that all cardinal numbers are accessible does not result in a con-
tradiction, and that the proof thereof is outlined by K. Kuratowski [3] (p. 146) and by
Firestone and Rosser [1] (p. 79).
406 XV. Number classes and alephs
§ 1. Equivalences 409
Among the alephs that are not <m there exists of course a smallest:
and (in view of the corollary proved above) that aleph — which we shall
continue to denote by s(m) — will obviously satisfy conditions (1.1).
It is also clear that between the cardinal numbers m and s(nt) there is
no intermediate cardinal number (cf. Tarski [14], p. 26-32).
We have m<m-+s(m) since obviously m<m+s(m) and m=
= M+(M) give s(m) <m, which is impossible.
THEOREM 3. For every given ordered set M we are able to define an
ordered set N of greater power than the set M (Sierpinski [41], p. 137).
Proof. It suffices of course to prove Theorem 3 for infinite sets M.
Let M denote a given infinite ordered set and f — a function satisfying
Theorem 2. Thus we shall have f(J7)C UUU(M)=T. As we know,
there exists an ordered set M’ similar to the set WM and disjoint with
the set 7. Therefore M’-f(M)—0 and M’= M. Let N= M’+f(M)
and let us order the set N, retaining for the components M’ and f(J/)
their previous order arrangement (f(J/) is, by Theorem 2, a well-ordered
set), and regarding the elements of the component M’ as preceding the
elements of the component f(M/). Let M =m; thus f(M) = 8 (Nt), and,
by Theorem 2, s(t) <m does not hold. In view, of M’-f(M)= 0, we
shall have n= NVN=m+s(m). Thus n>m. If we had n=M, 7. e.
m-+s(m) =m, we should have s(m) <™m, which is impossible. There-
fore n ~m, and thus n> m. The ordered set N is thus of greater power
than the set M. We have proved Theorem 3 (without the aid of the axiom
of choice).
In particular, if W/ is the set of all real numbers ordered according
to their magnitude, then from Theorem 3 we immediately obtain the
following
CoroLLARY. We are able to define an ordered set N and to prove without
the aid of the axiom of choice that the set N ws of greater power than the
continuum.
Similarly, in virtue of Theorem 3, we are able to define an ordered
set NV, of greater power than the set N, then an ordered set JN, of
greater power than the set N,, etc. But we are not able to define an
410 XVI. Zermelo’s theorem
ordered set with regard to which we could prove without the aid of
Cantor’s hypothesis on alephs that it is of power > 27°. From Cantor’s
hypothesis it follows that the set N is of power 22°, and so is the well-
ordered set composed of all ordinal numpers & < ay.
from the assumption that every set can be ordered follows the axiom of
choice for an arbitrary aggregate of finite sets’). Indeed, let Z denote
a set of finite non-empty disjoint sets, S — the sum of all sets belonging
to Z. The order arrangement of the set S determines the order arrangement
of each of the sets belonging to Z. But every finite ordered set has a first
element. The set N of all the first elements of the sets forming the set Z
will be a set containing one and only one element from each of those sets.
It is also easy to prove without resorting to the axiom of choice
that the proposition T, that every set can be ordered is equivalent to the
proposition T, that every set in which every ordering is a well-ordering
is finite. The proof that T,->T, presents no difficulties. Suppose now
that proposition T, is false for a set A. Thus in the set A there exists no
order arrangement, 7. ¢.gbhat set satisfies the assumption of proposition T,
and therefore, in virtue of that proposition, it is finite, 7. e. it can be
ordered, which gives a contradiction. Therefore T,—>T,. (Tarski [2], p. 82).
W. Kinna has proved that if for a given set A we know the function /
associating with every subset B, composed of more than one element
of the set A, a certain proper subset f(B) of the set B, then we are able
to order the set A (see XVI.4, p. 4382, Exercise).
Proof. In view of y < m, there exists a cardinal number m, such that m= m,+ 9,
and, since we are able to prove without the aid of the axiom of choice that y+" = y,
we have m+s=(m,+8)+N=m,+ (N+) = m+ =m, whence m+N=—m, q. e. d.
2. Prove without the aid of the axiom of choice A. Tarski’s proposition that if
a cardinal number m is not finite and y(m) denotes the least aleph that is not <m, then
between the number m and m+y(m) there is no intermediate cardinal number.
Proof (after A. Tarski). Suppose that there exists a cardinal number n such that
(a) m<aon<mm+y(m).
(b) n= m’+ 9’
where
(since if Pc M+N, then P= M,+N,, where M,= MPcM, and N,= NPC N).
Further, we shall distinguish two cases:
1) s’=s(m). By (b) we have 8’ <n, whence, in virtue of Exercise 1, n+’ =n,
and since, by (a), n > m, we have n=n+y’ >m-+W’ and thus, by 1), n>m+x(m),
which contradicts (a).
2) ys’ £N(m). In view of (c) and the observation that for alephs trichotomy holds
(which is proved without the aid of the axiom of choice), we have x’ < s(m). Since
s(m) is the least aleph that is not <m and wy’ < ¥(m), we must have y’ < m, whence,
according to exercise 1, m-+¥’=m, which, by (c) and (b), gives m>m’+y’=n
contradicting (a).
Thus the assumption that there exists a cardinal number n satisfying inequalities (a)
leads in any case to a contradiction.
3. Prove without the aid of the axiom of choice that for cardinal numbers m
and n the inequality 2™ < 2" implies the inequality m < n.
Proof. Suppose that for cardinal numbers m and m the inequality m <n does
not hold. As we know, from the axiom of choice follows the theorem on trichotomy,
and, in view of (m < n) not holding, it follows that m > n, which, as we know, implies
the inequality 2 > 2". Thus if we have 2™ < 2" then m <n, q.e. d.
(2.1) Tee
Ot ea Or One
ai rh,
(2.2) Naat LOT meray ae eae
(2.3) itt it for” iti an;
(2 4) Wie as TORAt) de
iit =o dh
l
From formula (2.1) it immediately follows that no.cardinal number
that is not finite can be the sum of two cardinal numbers smaller than that
number. This theorem thus follows from the axiom of choice. On the other
hand, as has been observed by S. Lesniewski, this theorem implies the
theorem on trichotomy. Indeed, if the cardinal numbers m and n are
not finite, then in virtue of this theorem their sum m+n cannot be greater
than each of those numbers, and since of course m+n >m and m+
+n >n, therefore either m+n—n, whence n <i or m+n= n, whence
m <n. And since, in virtue of Theorem 6 of XVI.1, the theorem on tri-
chotomy is equivalent to the axiom of choice, we have
THEOREM 1. Yhe proposition that a non-finite cardinal number is
not the swm.of two cardinal numbers smaller than that number is equivalent
to the axiom of choice (cf. Kiyoshi Iseki [1], p. 109).
Now let a cardinal :uinber m be termed prime if it is not the product
of two cardinal numbers :maller than that number.
THEOREM 2. The axicin of choice is equivalent to the proposition that
every non-fnite card na’ number is a prime number (Sierpinski [41],
ps 11742).
§ 2. Theorems equivalent to the axiom of choice 415
where 7 igs am arbitrary (non-empty) set and m, (for te 7’) are positive cardinal
numbers.
416 XVI. Zermelo’s theorém
Proof. For T finite the proof is easy. For JT not finite we have T = 2-7 and
m? — [J m,
: ueT
therefore
(ey rn a (Sita eae ame pati,
teT ueT teT ueT eT
(7. e. that every set, either whole or with one element removed from it,
can be decomposed into disjoint pairs, which intuitively seems almost
self-evident).
From formula (2.1) it follows that if a cardinal number n is not finite,
then n= 2n= 2n+1. It immediately follows that a set which is not
the disjoint sum of two sets of the same power as the set itself is finite.
This theorem we are also unable to prove without the aid of the axiom
of choice‘ (Tarski [2], p. 93, Def. V).
THEOREM 3. The axiom of choice is equivalent to the proposition that
the difference mw—n ewxists for every cardinal number m and every cardinal
number 1 <— m+).
Proof. It follows from the axiom of choice that the difference m—n
exists for every cardinal number wt and every cardinal number n < m.
Indeed, if m is a finite number and n < m, then the existence of the dif-
ference m—11 is obvious. Therefore let 1m be a non-finite cardinal number.
In order to prove the existence of the difference m—n it is necessary
and sufficient to prove, in accordance with the definition of the difference
of two cardinal numbers (see [X.1), that there exists one and only one
cardinal number p such that m=n+p. That such a number p exists
1) This theorem was given without proof by A. Tarski [5], p. 312, th. 82, A. I gave
the proof in [37], p. 125, Theorem 5,
§ 2. Theorems equivalent to the axiom of choice 417
follows from the fact that, by formula (2.1) (which follows from the
axiom of choice) and on account of the fact that number m is not finite,
we have m= n+m for n <m. Thus it remains to show that if for a cer-
tain cardinal number p we have m=n+p and n<m, then p=m. In
view of m=n-+p we have m>p; if, we had m>p, then number m
would be the sum of two cardinal numbers smaller than itself, in contra-
diction to Theorem 1. Therefore p= 1m, q. e. d.
Suppose now that the proposition that the difference m—n exists
for every cardinal number m and every cardinal number n < ™ is true.
Let m denote any non-finite cardinal number. In XVI.1 we have proved
without the aid of the axiom of choice that there exists an aleph s(m)
such that s(m) <m does not hold (Theorem 1 of XVI.1). We have of
course m+s(m) >s(m). If we had m+s(m) >s(m), then from our
assumption of the existence of the difference of cardinal numbers would
follow the existence of the difference [m+s(m)]—s(m), and thus the
existence of only one cardinal number p such that m+s(m) = p+s(m).
But the latter formula is of course true for p=m, as well as for p =
= m-+wx(m) since, as we know (see XV, (8.5)), s(m)+ (mM) = s(m).
Thus we should have m = m-+s(m), whence s(m) <m, which contradicts
the property of number s(m). Thus the assumption that m+s(m) >
>s(m) leads to a contradiction. Therefore we must have m+s(m) =
=x(m), whence m<s(m), whence it follows that the cardinal num-
ber m (which is not finite) is an aleph.
Thus from the assumption of the -existence of the difference of
cardinal numbers m—n (for 1<™m) it follows without the aid the of
axiom of choice that every non-finite cardinal number is an aleph. And
this, as we know, implies Zermelo’s theorem and the axiom of choice
GxcV fell).
Theorem 3 is thus proved.
THEOREM 4. The axiom of choice is: equivalent to the theorem on the
addition of inequalities for cardinal numbers, 1. e. to the proposition that
for any cardinal numbers m,n, m, and n, the inequalities
Proof. Suppose that the axiom of choice is true and let us assume
that for cardinal numbers m, 1, m, and n, inequalities (2.5) hold. If the
cardinal numbers m and m, are both finite, then the number m+ 1, is
also finite, and inequalities (2.5) (both in the case where numbers 1 and 1,
1) A. Tarski, Fund. Math. 5(1924), p. 147-154.
Cardinal and ordinal numbers bo “1
418 XVI. Zermelo’s theorem
are finite and in the case where one or both of them are not finite) imply
inequality (2.6). Suppose therefore that at least one of the numbers m
and m, is not finite. The axiom of choice, in virtue of Theorem 6 of XVI.1,
implies trichotomy; thus we have either m<m, or m, <m. Suppose
that m << m,: number m, is thus not finite and, by formula (2.1) (resulting
from the axiom of choice), we have m+ ™m, = m,. Hence, by (2.5), we
obtain m+m, = m, <n, <n+n,, and thus formula (2.6). Therefore the
axiom of choice implies the theorem on the addition of inequalities for
cardinal numbers.
Suppose now that the theorem on the addition of inequalities for
cardinal numbers is true and let m denote any cardinal number that is
not finite. Let
(2.7) Teale 3
whence we immediately obtain
(2.8) 2 2, — t(2Ko) — Is, — It -
From (2.8) and from the formula for alephs 2s = s proved in XV.8,
we should have the formula
n+x(t)<n+s(n),
1 eS ey: mM, 9
ne
a eeuNeTte
Proof. Formula (2.10) for cardinal numbers m that are not finite
immediately follows from formula (2.1); thus formula (2.10) follows
from the axiom of choice.
Suppose now that formula (2.10) is true for every cardinal number m
that is not finite and let m denote any such number. Let m= s(m). In
virtue of our assumption, we shall have:
(2.11) SA eh) le NW, (m+ n)?=m-+n
and since (m+n)? = n?+ 2mn+1, by (2.11), we have
m+n=m+2mn4+n,
(2.19) n = me,
(2.20) p=n+rx(t),
(ON) q = n-s(n).
By (2.19) we obtain
(2.22) n2 = (m0)? — mo? — me =n,
whence, by (2.20)
In view of n>1 and x(n) >1 we have n+ x(n) < nKx(n), whence
Ms (11) < [11+ 8 (11)]+ 118 (11) < THs (11) + 18 (11) = 1128 (11) ] = Ts (11)
whence
(2.24) Ms (11) = [11 + 8s(1) ] + 1s (11)
Formulae (2.23) and (2.24) give the equality
(2.25) p? = its (11).
By (2.21), (2.22) and (2.25), we obtain
EXERCISES. 1. Prove A. Tarski’s [1] (p. 150) theorem stating that the axiom
of choice is equivalent to the proposition that for non-finite cardinal numbers m and n
we have mn = m-—n.
Proof. The axiom of choice, by Tbeorem 6 of XVI.1, implies trichotomy: thus
if m and n are non-finite cardinal numbers, then m<n or Mn, e.g. m<n. But,
as we know, the axiom of choice implies also formula (2.1): thus mn = m+n,
Suppose now the proposition that for non-finite cardinal numbers mt and n we
have mn = m-+-n to be true and let m denote an arbitrary non-finite cardinal number.
Let n = §$(m); we shall have formula (2.13), which, as we have shown in the proof of
Theorem 6, implies that the cardinal number m is an aleph. Further proof is obvious.
2. Prove A. Tarski’s ([1], p. 154) theorem stating that the axiom of choice is equi-
valent to the proposition that for any cardinal numbers m,n and p the inequality m-+-
+ <n-+p implies the inequality m <n.
Proof. As we know, the axiom of choice implies trichotomy; thus if m < n does
not hold, then m =n, whence m+ p >n-+p. Thus it follows from the axiom of choice
that if m+ p <n-+p, then we must have m <n.
Suppose now that for cardinal numbers m,n and p the inequality m+p <n-+p
always implies the inequality m <n. Let n denote an arbitrary non-finite cardinal
number. We have of course $(n) << n+wN(n). If S(n) < N+N(N) then, in view of $(m)-+
+8(n) = S(n) we should have §$(n)+ (nN) < n+8(n), whence, in view of the assumed
proposition (form = $(n), p = 8$(n)), S$(N) <n, which contradicts the property of num-
ber N(n). Therefore we must have S$(n) = n+8(n), whence n <N(n) and number n is
an aleph. Further proof is obvious.
3. Prove A. Tarski’s theorem stating that the axiom of choice is equivalent to
the proposition that for any cardinal numbers m, n and p the inequality mp < np implies
the inequality m <n.
Proof. The proof is analogical to the proof of Exercise2, the sums of cardinal
numbers being replaced by their products.
4. Prove A. Tarski’s') theorem stating that the axiom of choice is equivalent to
the proposition that for any cardinal numbers m, p and g the equality m+ p = im+q
implies that either p=g or p<m and g<m.
Proof. Suppose that for cardinal numbers m, p and q we have the equality m+ p =
= m-+q andp =q does not hold. In virtue of Theorem 6 of XVI.1, the axiom of choice
implies trichotomy: thus we have either p > q org > p, e. g.p > q. If we had alsop > m
and if the number p were finite, then the numbers m and q would also be finite and
it would follow from the equality m+p =m-+q that p=4q, which is contrary to the
assumption. Thus if p > m, then the number p is not finite and, by formula (2.1) result-
ing from the axiom of choice, we obtain m-+ p =p, therefore p =m-+q where p > m
and p > q, while, by Theorem 1, it follows from the axiom of choice that a non finite
number p may not be the sum of two cardinal numbers smaller than itself. Thus p > m
is impossible, therefore we have p< m, and, in view of p > q. q <m. Of course if
gq >p, we should similarly obtain q<m and p < m.
Thus we have proved that from the axiom of choice follows the proposition that
for any cardinal numbers m, p and q the equality m+ p= m-4q implies that either
De qorep— Nl andad site
Suppose now that the latter proposition is true and let n denote any unon-finite
cardinal number. As we know, n <n+y(n) and y(n)+n = y(n)+[n+(n)]; thus the
assumed proposition implies (for m = y(n), p =n, gq=n+y(n)) that n < y(n), whence
it follows that the cardinal number n is an aleph. Further proof is obvious.
1) This theorem was given without proof (see Lindenbaum et Tarski [1]). I gave
the proof in my paper [47].
§ 3. A. Lindenbaum’s theorem 427
cardinal number m is not finite it follows that the set Z is infinite since
of course n« Z for n= 1, 2,... It is easy to verify that the set Z is such
that if ae Z and €<a then fe Z. The set Z is well-ordered according to
the magnitude of the ordinal numbers that belong to it; let ¢ denote its
order type. If we had ¢ « Z, then the well-ordered set Z would be similar
to its proper segment formed by the element ¢ of the set Z, which is
impossible. Therefore ¢¢ Z and if we set s/(m) =, then we shall not
Se
Let A denote a given set, U = U(A) — the set of all subsets of the
set A. Let us denote by J’ the set of all functions f7;(X) defined for X « T
where 7C U and such that fr(X)e¢«X for XY « 7. In particular, for in-
stance, if 7, denotes the set of all subsets of the set A consisting of one
element only and if we set f;,({v}) = x for we A, then we shall have fr, € 1’.
Let fr,<9r, if frel, gr,e DT, TyC Ts, Ty ATs, and fr(X)= gr,(X)
fore West...
The set /’ contains subsets ordered according to the relation x,
e.g. the subset 4) = {f7,} (A one-element set CJ is regarded as ordered.
according to the relation <).
Let @ denote the family of all subsets of the set /’, ordered accord-
ing to the relation S. Thus J, « G. We shall prove that the family @ is
closed.
Let G, denote a chain contained in G and let
Sea —
ahi:
HeG,
s= >a,
HeG,
there exist sets H, « G, and H, « G, such that fr, « H,, Gr, « Hz, and since G,
is a chain we have either H,CH, or H,C H,, e.g. H,C H,. But then
fr,« H, and Gr, «H,. Since H,¢G, it follows from the definition of fa-
mily G that the functions forming the set H, are ordered according to
the relation <; in view of fr, 4 gr, we have either fr, Sgr, or gr, < fr,-
The set (of functions) S is thus ordered according to the relation < and,
in view of the definition of family G, it follows that S « G.
We have proved that family @ is closed. In virtue of theorem Z
there exists a set V eG, not contained in any other set of family @. In
view of V eG, V is the set of certain functions f;, ordered according to
the relation <.
We shall prove now that if XC A and X 40, then there exists
a function fre V such that XY « 7. Indeed, suppose that for a certain non-
empty set X,C A.and for every function freV we have X,¢ T. Let
x,e X, (such an element exists because VY, 4 0) and let us define a set O
and a function fe as follows. Let 0 = YT where the summation is extended
over all sets 7’C U for which there exists a function fr such that fre V.
Now if X «0, then, in view of the definition of the set 0, for a certain
§ 4. Theorems of Zorn and Teichmiiller 429
= Ly Aig’
E<a
it will be a set of family # (in view of its being closed) and therefore it
is contained in a certain set, different from it, of family 7. Thus there
exists a least ordinal number 4,<g such that 8,C A, and SA Ap
The ordinal numbers A, are thus defined by transfinite induction for
every ordinal number a and /, < 9.
1) Cf. Hausdorff [2], p. 140, where the author proves the existence in every partially
ordered set of a maximal ordered subset.
430 XVI. Zermelo’s theorem
Now let a and f be two ordinal numbers such that a < p. Hence,
in view of
S:= >) Ar,
é<p
we obtain. A; C 8s, and since Sp C Aas, we have Ajo An. If we had
Aq = 4g, then, in view of A,,C SpC Ag, we should have S; = Aig, which
is impossible. Thus A, 4 4g. On the other hand, in view of S8,C 4,C A a5
we obtain S,C A;,, and since 4, is the least ordinal number <q such
that S,C As, A, <dAg. Thus A, < dg. Therefore A, is an increasing func-
tion of the ordinal variable a, whence, as we know, it follows by trans-
finite induction that 2, > a for every ordinal number a and, in particular,
Ay > v, Which is impossible, because for every ordinal number a we have
Aa < gy. Therefore theorem Z must be true.
Theorem 1 is thus proved ?).
We shall say that a given family F of sets has property T if the condi-
tion Ae F is equivalent to the condition that each finite subset of the
set A belongs to F.
Property T of a family of sets obviously implies its being closed
(but not vice versa). Indeed, if F,C F, if family F, is ordered according
to the relation C, if
s= A
A€F,
and if. P denotes a finite subset of the set S, e.g. P = {pry Do, --»5 Pn}
then there exist sets A,, A,,..., A, of the family F, such |that p; «A;
for +=1,2,...,”. But the family F, is ordered according to the rela-
tion C; thus among the sets A,, A,,..., A, there exists such a set A
that A;C A, for += 1,2,...,. Thus the set P is a finite subset of the
set A, of family F; if family F has property T, then Pe F. Thus each
finite subset of the set S belongs to #, whence, again in view of the pro-
perty T of family F, we conclude that S« #. Family F is thus closed,
ire. .e:,
Feichmiller’s theorem runs as follows:
T. In every family of sets that has property T there exists at least one
set not contained in any other set of that family (Tarski [16]; Bernays [1],
j es) a)
THEOREM 2. Theorem T is equivalent to the axiom of choice.
1) Various authors deduce Zorn’s theorem directly from the axiom of choice
without using Zermelo’s theorem, e.g. Kneser [1], p. 110-134; Szele [1], p. 254-256;
With [1], p. 434-438.
*) G. Birkhoff [1], p. 42, theorem (AC 3), attributes this theorem to J. W. Tukey.
§ 4. Theorems of Zorn and Teichmiiller 431
S= AcF.
AEC
We shall prove that the set S is not contained in any other set of
family Ff. Suppose that SC A,eF and SA#A,. Let C, = C+{S+A,)}.
Since ACS for A « 0, we have ACS+A, for Ae C,; from the formula
for C, and the observation that OC is a chain it follows that the family C,
of sets is a chain. But CCC, and C4~C, because S+A,¢C (for if
S+A,« C, we should have S+A,C 8S, which is contrary to Aj #4SC A,).
Chain C would thus be contained in another chain contained in F, which
is contrary to the property of chain C.
We have proved that the set S (belonging to family F’) is not con-
tained in any other set of family F.
We have proved that T—Z and thus the proof of Theorem 2 is
complete.
It will be observed, moreover, that we can prove (without resorting
to the axiom of choice) that theorem Z is equivalent to a certain theo-
rem Z,, concerning partially ordered sets.
Let A denote a partially ordered set, A, — an ordered subset of
the set A. If there exist in the set A elements succeeding each element
of the set A, and if among those elements there exists one that precedes
all the others, then it is termed the wpper bound of the set A,. A partially
ordered set A whose each non-empty ordered subset has an wpper bound
is called inductive. Each element Of a partially ordered set A that does
not precede any other element of the set A is termed a maximal element
of the set A.
Theorem Z is equivalent to the following theorem Z,:
Z,. In every inductive set there exists at least one maximal element
(Bourbaki [1], p. 37; Wallace [1], p. 278).
432 XVI. Zermelo’s theorem
sets of family F',, there exist sets Y, and Y, of family F, such that
a,¢« Y, and a,« Y,, and since family F, is monotonic, we have either
Y,C Y,or Y,C Y,. Thus in any case one of the sets Y, and Y, of family 7, CF
contains two different elements of a certain set X, of family A, contrary
to the definition of family F. Hence the set Z contains at most one element
from each of the sets of family A.
Suppose now that for a certain set X, of family A we have X,Z = 0.
The set X, is not empty since it belongs to family A; accordingly there
exists an element ae X,. Thus we have aé Z and it follows from the
definition of the set Z that a is not an element of any set of family F,.
Denote by F, the family composed of the sets of family F, and of the set
Q= {a+ DY.
YeFy
{Xo, {vo}|} « F,, which is impossible iin view x, ~ #, and the fact that the
sets of family F', are disjoint.
Hence the set Z contains at most one element from each set of
family A.
Suppose that for a certain set X, of family A we have X,.4 = 0.
Since the sets of family A are non-empty, there exists an element % « Xy-
Let {X, {v}} denote any set of family F,. It follows from the definition
of the set Z that w« Z, and thus XZ ~0, which, in view of X,Z = 0,
proves that XY 4 X,, whence, the sets of family A being disjoint, it fol-
lows that YX, = 0. Accordingly the set {X,, {x }} belonging to family F
(since w),¢« X,¢ A), is disjoint with each set of family #,, contrary to
the assumption that F’, is the maximal subfamily of family / composed
of disjoint sets.
Thus the set Z has at least one element in common with each set
of family A.
The set A of non-empty disjoint sets being arbitrary, we have proved
that theorem T implies the axiom of choice, q. e. d.
It will be observed that the sets {X, {v}} in our proof could not
ie
be by the sets {X,x} since, for instance, in the case of
= {1}, {{1}} we should have F = (a, fe Hay, atl, and the
maximal subfamily of disjoint sets of Aes ow seen consist of one set
only, whence it would follow that the set Z has only one element, while
in fact it is the set fi, {1}} (only) that contains one element from each
set of family A.
But if p>s,, then 2° < 2°+p < 2°+2° — 2°t+1— 2°, whence 2?+
Se ae
In view of (5.3) we thus have
1 nt
(5.4) eet =O 2g Oe Oe OF ge
(5.5) Qe == 2", 22 OM be
it nt
Pe K . ; ogm .
———
‘ gm
>
2
a aw
o2 —
22 .
(5.7)
~ SCD 24 eee oe
m 2"
If'we had s(m)+2? = 2° , then, by (5.5), we should have s(m)=
mt . . . as *
= 2" +m>n and the cardinal number n (which is not finite) would
be an aleph.
; mt
Theorefore suppose that s(m)+
22" = 2 does not hold. By (5.7)
we thus have
; Z POA iC oe
2? wi RG)
Se 22 a2
: ; ott
whence, by theorem G applied to number 27, we have
: F m : Pee gS) , 5
Therefore suppose that s(m)+2™ 4~ 2?°, whence, by (5.8), we obtain
which, in view of theorem G, gives s(m)+m= m, and thus s(m) < 1m,
contradicting (5.6).
We have proved that the cardinal number n must be an aleph, and
since n is an arbitrary non-finiie cardinal number, this, as we know
from XVI.1, imphes the axiom of choice.
We have proved that theorem G implies the axiom of choice.
Now we shall prove that theorem G implies theorem C.
Let a be any ordinal number. We have 2° = 9. (see XIII) -and
Na <Nai1, and we know that that there is no cardinal number between x,
and 8,41. AS we know, theorem G implies the axiom of choice, and thus,
in virtue of Theorem 6 of XVI.1, it implies trichotomy. Therefore either
QO 81g OF 2° > key. Th we had 2% <oxecy,, then in view Of 02“2-+5,;
the cardinal number 2™¢ would be intermediate between s, and +1,
which we know to be impossible. Thus 2°*>x,41. If 2%>,11, then
we should have xg < %ai1 < 2, which contradicts theorem G (for m=
—N8,, = Na41). Therefore we must have 2°*=s,13, 7. e. formula (Ono
Thus we have proved that G+C.
Suppose now that theorem C and the axiom of choice are true and
let m be an arbitrary non-finite cardinal number. From the axiom of
choice it follows (as we know from XVI.1) that m is a certain aleph;
thus there exists an ordinal number a such that m—=s,. Now, theorem C
implies formula (5.2). Thus if there existed a cardinal number n for which
inequalities (5.1) would hold, we should have s¢ < 1 < S41, which we
know to be impossible. Thus there is no cardinal number 1 satisfying
inequalities (5.1). Therefore theorem G is true. We have proved that
the validity of theorem © and the axiom of choice implies the validity
of theorem G.
§ 5. Generalized continuum hypothesis 437
power x, forming a chain; let # denote the family of those sets. For each
element a of the set A there exists a set B(a) « F such that a« B(a). Let
= » Bla)
aéAy
From the definition of the set B it follows that for no finite sequence
b,, by, ..., 5, of different numbers belonging to B the formula r,b,+
+7.b,+...+7nbn = 0, where n is a natural number and 7,, 72, ..., 7, are
rational numbers ~0, is true. Indeed, if we assumed that this formula
holds and that the numbers b; (¢= 1, 2,...,) are written in the same
order as they appear in the sequence (1.1), we should have
Dn = 8,0, + 89bg+...+ Sp_10n-1 ,
(1.4) Gi = Oy ETS Dy ho Tm
where m (2=1,2,...,”) are natural numbers, pi? tea RA a
1=1,2,...,) are numbers of the set B and rp Cee Le roaeg HIS 0 a=
=1,2,...,”) are rational numbers 40. Introducing formulae (1.4) into
formula (1.2) we obtain for number #,, by reduction, an expression of
the form (1.3), which contradicts the assumption that x», cannot be re-
presented in that form.
Thus we have proved (with the aid of the axiom of choice) the
existence of such a set B of real numbers that every real number x
different from 0 can be represented in one manner only in the form (1.3)
where 0; («=1, 2,...,m) are different numbers of the set B and 7;
(¢=1,2,...,m) are rational numbers 40. A set B with such proper-
ties is called the basis of Hamel (of the set of all real numbers) (Hamel
[1], p. 459-462). :
So far we do not know any effective example of Hamel’s basis, and
neither can we prove its existence without the aid of the axiom of choice,
Now let b, denote any given number belonging to the basis B. Let
us define a function f(#) of a real variable, setting f(#)= 0 for r=0
and for every real number x for which in the expansion (1.3) number b,
does not appear (7. e. for which in formula (1.3) we have b; ~ b, for +=
=1,2,...,m). If in the expansion (1.3) number b, does appear with
the (rational) coefficient r, then we shall set f(x) =r. It is easy to verify
that the function f(x) of a real variable defined in this way satisfies the
functional equation
However, our function f(x) is not of the form Aw+B where A and B
are real numbers independent of x, since /(b,) = 1 and f(b) = 0 for every
number b ¢ b, belonging to the basis B, there being obviously infinitely
many such numbers b (since the set of all real numbers is non-denumer-
able while there is a denumerable aggregate of numbers of the form (1.3)
where 0,, b,,..-, bm belong to a finite sequence of numbers). Now, the
function Av+B, provided it is not constantly 0, has at most one root.
The existence of Hamel’s basis implies thus the existence of non-
linear solutions of the functional equation (1.5). However, we do not know
any effective example of a non-linear function satisfying the functional
equation (1.5), and neither can we prove the existence of such a function
without the aid of the axiom of choice.
It can be proved without the aid of the axiom of choice that every
non-linear function satisfying the functional equation (1.5) is disconti-
nuous (which was already proved by Cauchy), and even non-measurable
in the sense of Lebesgue *).
Numerous works have been devoted to functions satisfying equa-
tion (1.5) (Ostrowski [1]; Maceaferri [1], p. 92-101; Alexiewiez and Or-
licz [1], p. 314; Haeperin [1], p. 221-224).
However, we can easily give examples of discontinuous functions
f(x) of a real variable satisfying the inequality
g(ay) = g(@)gly)
for any real « and y; in order to obtain such a function it suffices to set
g(0) = 0 and, for real x 4 0, g(x) = ef(slr),
But it can easily be proved that, besides the function f(a) constantly
equal to 0 and the function f(a) = a, there is no real function of a real
variable satisfying the functional equation (1.5) and the functional
equation
*) It was proved with the aid of the axiom of choice by M. Fréchet [1] (p. 390-392).
I proved it without the aid of the axiom of choice in [6] (p. 116); see also Banach [1],
p. 123, and Kae [1], p. 170.
§ 1. Hamel’s basis 443
for any real x and y (see below, Exercise 1). However, it can be proved
with the aid of Zermelo’s theorem that there exist complex functions
f(z) of a complex variable, not constantly equal to 0 and different from
the functions f(z) = 2 and f(z) = 2’ (where 2’ denotes a complex number
conjugate with number z), which satisfy the functional equations
for any complex numbers 2, and 2, (Lebesgue [2], p. 532, and [7], p. 296-
300; Schoenflies [1], p. 181-184).
It has also been proved that there exist non-trivial functions satisfy-
ing (for any complex z, and 2,) the functional equations (1.7) and as-
suming every complex value once and only once (i. e. establishing the
automorphism of the set of all complex numbers with respect to addi-
tion and multiplication (Segre [1], p. 419, and Kestelman [1], p. 1).
EXERCISE. Prove without the aid of the axiom of choice that besides the func-
tion f(x) constantly equal to 0 and the function f(z) = » there is no real function of
a real variable satisfying the functional equations (1.5) and (1.6).
Proof. By (1.6), we have f(x?)= [f(x)]* for real «, whence f(t)> 0 for t> 0. By
(1.5), we have f(x) = f(~—y)+ f(y) for real « and y, therefore f(x) > f( +ffoe o> y.
If we had f(1) = 0, then, on ee of the fact that, by (1.6). f(x)= f(a)f(1), we should
have f(x) = 0 for real x. In the contrary case we have f(1) 4 0 and, by (1. zewe obtain
f(1) = 1 for x = y = 1, By (1.5) we easily obtain f(ma) = mf(x) for natural m, whence
for «= l1/m, in view of f(1) = 1,.we have f(1/m) = 1/m, and since jf(lx) = If(a) for
natural 1, we have
1 1 i
i(-.) Su) (5) mM
Thus, for positive rational r, we have f(r) = 7, and since, by (1.5), for « = y = 0,
we obtain {(0) = 0, whence for y = —~x, f(—«”) = —f(x), we conclude that the formula
f(r) =r is true for any rational r.
Now ne that there exists a real number w such that f(x . Thus either
iD) SS 8 Oe (Ca) <a Reex) x «x, then there exists a rational ea r a that f(x) >
SPS oO weave f(r) > f(a); therefore, in view of f(r)= 7, r > f(x”), which is impos-
sible. If f(x) < z, Ateae ci a rational number 7 such that f(x) < r < x, whence
f(r) <f (x); therefore r < f(x), which is impossible.
Thus we must have f(”%) =« for every real number x and the ae es, proof is
complete.
With the aid of Hamel’s basis it is easy to prove that there exists
a set of points P on a straight line such that the family of all the different
sets lying on that straight line and congruent to the set P is denumer-
able +).
1) Without the aid of the axiom of choice we are unable to prove the existence of
such a set.
444 XVII. Applications of Zermelo’s theorem
Indeed, let B denote Hamel’s basis and b any given element of it.
The set P, formed of number 0 and all real numbers x for which we have
b; £6 for i=1,2,...,m in the expansion (1.3), obviously has the re-
quired property. If we denote by P(a) the translation of the set P along
the straight line for a distance of a, then it is easy to prove that for ra-
tional 7; and 7, where 7, #7, the sets P(r,b) and P(r.b) are disjoint
and that for every real number a there exists a rational number 7 such
that P(a) = P(rb) (Sierpinski [44], p. 161).
It will be observed, however, that I. Kapuano asserts that there
exists no plane set Q such that the family of all different plane sets
congruent with QY (by translation or by rotation) is denumerable.
With the aid of Hamel’s basis it is also easy to prove E. Cech’s
proposition that there exists a non-empty set of real numbers A different
from the set of all real numbers and such that for every real number x
in the infinite sequence A, A(x), A(2ax), A(3a@),... there is only a finite
number of different sets (W. Sierpinski, Fund. Math. 35(1948), p. 160).
Indeed, let A denote a set formed of number 0 and of all real num-
bers x for which in the expansion (1.3) numbers 7; (2 = 1, 2,..., m) are
all integers. The set A is not the set of all real numbers because, for in-
stance, if b is an element of Hamel’s basis, then the number b/2 does not
belong to A. Now let x denote any real number, n — the common de-
nominator of the rational numbers 7,, 72, ...,7%m in the expansion (1.3) of
number v. Clearly nxe A for n=1,2,... and A(nw) = A. Hence it im-
mediately follows that each term of the infinite sequence of sets A, A(z),
A(2x), A(3a), ... is equal to one of the sets A, A(x), A(2z), ..., A((n—1)a).
Gech’s theorem is thus proved.
In connection with translations of linear sets let us also observe
that the following proposition is not trivial: for every non-empty set A
of real numbers that is not the set of all real numbers there exist infinitely
many different sets of real numbers congruent with the set A by trans-
lation (W. Sierpinski, Fund. Math. 35(1948), p. 159, Th. 1).
We say that a given set X of real numbers is formed of rationally
independent numbers if for every finite sequence «,, 2, ...,%m of dif-
ferent numbers of the set X and for every sequence 7,, 72, ...,%m Of ra-
tional numbers the equality 7,7, +7,0,+...+7m@%m—= 0 implies the equa-
it Nea Op fOnei ea ed a. 00.
We say that a real number z is rationally independent of the numbers
of the set X if for every finite sequence 7,, %,..., %m of different numbers
of the set X and for every sequence 7,, 72, ..., 7m of rational numbers 40
we have the formula + #7,0,+7,0,+...+1m@m-
It can easily be seen that every basis of Hamel is formed of rationally
independent numbers. It is also easy to prove that if A is a set of rationally
§ 1. Hamel’s basis 445
Q(r,n)
= DY)fay.
(r,a)
a<Q
s= » dB,n), hoard
446 XVII. Applications of Zermelo’s theorem
. 2f . .
where the summation > extends over all rational numbers r different
r
is obviously the set of all real numbers different from 0, whence we easily
conclude that also S is the set of all real numbers ~0. Thus the latter
set is the sum of a denumerable aggregate of Hamel’s bases, q. e. d.
It can be proved that also conversely: from the assumption that
the set of all real numbers 40 is the sum of a denumerable aggregate
of Hamel’s bases follows the continuum hypothesis. Therefore the conti-
nuum hypothesis is equivalent to the proposition that the set of all real num-
bers different from 0 is the sum of a denumerable aggregate of Hamel’s bases
(Erd6s and Kakutani [1], p. 459, Theorem 2).
2. Plane set having exactly two points in common with every straight
line. We shall prove
THEOREM 1 (of S. Mazurkiewicz). There exists a set of points in
a plane that has two and only two points in common with every straight
line lying in the plane.
Proof (Mazurkiewiez [1], p. 382-383). Since the set of all points
in a plane is of the same power as the continuum and the set of all straight
lines lying in a plane is of the same power as the continuum (in virtue
of Theorem 2 of IV.8 and Exercise 2 of II.6) and from Zermelo’s theorem
follows the existence of the least ordinal number @ of the power of the
continuum, there exists a transfinite sequence of type @, {pels<,, formed
of all different points in a plane and a transfinite sequence of type
@, (Sshe<p, formed of all different straight lines lying in a plane.
_ Now let us define by transfinite induction a transfinite sequence of
type @, {dehe<y Of points in a plane as follows.
Let us denote by q, the first point of the sequence {pe}s<, lying on
the straight line S,. Now let a denote an ordinal number such that 1 <
<a<g and suppose that we have already defined all points gq: where
E<a. Let us denote by Q, the set of all points gq: where & < a; in view
of a <q we have Y, < 2”, therefore also the set Q.XQ, is of power <2,
and so is the set 7, of all straight lines having at least two points in com-
mon with the set, Q,. Since the number of all straight lines lying in
a plane is 2", there exist lines that do not belong to the set 7',; thus there
§ 2, Plane set 447
exists the first term S, of the sequence {S¢}:<, which is such a line. Let &
denote the set of all intersection points of the line S,, with the lines of
the set 7,; since each of those lines intersects the line Sz at one point
only and the set 7, is of less power than the continuum, the set Rk will
also be of less power ‘than the continuum, whence it follows that the
linee Ss, contains points belonging neither to Q, nor to R (since Oe Rie
ZO. +R < 2°, for it follows from the axiom of choice that the sum of
iv sets of power less than the continuum is of power less than the conti-
nuum). Let us denote by q, the first term of the sequence {pets<,, Which
is exactly such a point.
In this way the points q (a <q) have been defined by transfinite
induction. Let us denote their set by Q. We shall prove that the set Q
has two and only two points in common with every straight line lying
in a plane. Suppose that a certain straight line S contains more than
two of the set Q; let qa,, 4a, and q,, be those three of them which have
the least indices; thus a, < a,< a3, therefore q,¢Q., and q.,¢€Q,, and
thus S« T,,, while from the definition of point q,, it follows that it does
not lie on any straight line belonging to 7,,, which gives a contradiction.
Thus every straight line lying in a plane contains at most two points
of the set QY. Now suppose that there exist straight lines containing less
than two points of the set Q and let S, denote the first term of the se-
quence {Sz}s<g with this property. ;
From the definition of the lines S, it easily follows that fa, < Pa,
for a, <a,<g. But q,¢ Sg,, and, as we already know, no straight line
contains more than two points of the set @. Hence it follows that 6,, =
= Ba, = Pa, 18 impossible for a, < a, < a, <q, whence we conclude that
Ba < Pate for a <<, whence it easily follows that there exists an ordinal
number a<q such that p, > y. Since S, is the first term of the se-
quence {Sz}s<, that is a straight line not belonging to 7, in view of
y <P, we must have S,« T,, whence it follows that the line 8, has at
least two points in common with the set Q, and thus in view of Q, CQ
with the set Q, which contradicts the definition of the line S,. We have
proved that every straight line lying in a plane contains two points of
the set Q.
Theorem 1 is thus proved. We are unable to prove it without the
aid of the axiom of choice. Neither can we define effectively a plane set
which has at least one and at most two points in common with every
straight line lying in a plane. But it is easy to give an example of a plane
set which every straight line intersects at a denumerable aggregate of
points: such a set is for instance the sum of the circumferences of all
circles «2+ y? = n? where n = 1, 2,...
448 XVII. Applications of Zermelo’s theorem
transfinite induction for all terms of the sequence {wg}s<o,, 7. e. for all
elements of the set M, and
(3.1) g(a) Afeas) for E<A<ay.
Now let y = f(x) be any curve belonging to family F. Thus it is one
of the terms of the sequence {C;}:<, and therefore there exists an ordinal
number uw < w, such that f(a”) = f,(x) for 7 « M. By (3.1) we have g(a) ¢
AF ful.) for w<A<o,; thus the equality g(a) = f,(a,) may hold at
most only for ordinal numbers 4 < yw whose set is of power <p<m
(since u < @,). Elements (x, y) of the set P such that y = g(#) and y =
= f,(v) form a set of power <m, which proves that the curves C, and
y = g(x) are almost disjoint. The curve y = g(a) is almost disjoint with
every curve of family #. Our lemma is thus proved.
Let us return to the proof of Theorem 1. We shall define a trans-
finite sequence of type +1 of curves {Ke}ecw,,, a8 follows. Let fe(v) = xe
for €< a, ce M. The curves Ky = f,(x)], where ¢< @,, are of course
disjoint and their family is of power m. Now let 4 denote an ordinal num-
ber such that w, <A < mg41 and suppose that we have already defined
all curves K,; for & <A; their set F, is of power A4=s,=™ since a, <
<A < @g41. Therefore we can apply our lemma, which gives a curve K,;,
almost disjoint with every curve Ke for §<A. The curves K; where
€ < Wg+1 are thus defined by transfinite induction and it can easily be
seen that each two of them are almost disjoint; their sum (even the sum
») Kz) gives the set P. Theorem 1 is thus proved.
$<
It will be observed that the proof of Theorem 1 for m= xs, may
be carried out without the use of the axiom of choice, as has been done
in Theorem 18 of IV.14. .
Given two denumerable sets, A and B, we say that the set B is
almost contained in the set A if the set B—A is finite. The sets A and B
are said to be essentially different if the set (A—B)+(B—A) is infinite.
With the aid of the axiom of choice we can prove the following
theorem:
There exists a transfinite sequence {Ngsecq of type 2 of infinite sets
of natural numbers such that, for a<B <Q, the set Ng is almost: con-
tained in the set N, and essentially different from N, (Sierpinski [30], p. 9).
With the aid of the continuum hypothesis we can prove the follow-
ing theorem:
There exists a transfinite sequence {Nelsco of type 2 of infinite sets
of natural numbers such that the sets N,—N: are finite for <7 <2 and
that there exists no infinite set of natural numbers A such that the sets A—N;
are finite for € <Q (Sierpinski [46], p. 148, [62], p. 99).
Cardinal and ordinal numbers 99
450 XVII. Applications of Zermelo’s theorem
G)= S'U(A
AeG
(where U(A) Bae the family of all subsets of the set A). It is easy to
verify that f(G@) is (for GCF) a hereditary family formed of subsets of
the set M.
Now suppose that G,CF, G,CF, G,~G,. We shall prove that
f(G,) #7 (G,). Since G, #~G,, we have either G,—G, 40 or G,—G, 4 0.
Suppose for instance that G,—G, #40; then there exists a set A, such
that A, « G,—G,. In view of A, « G, we have A, « f(@). If we had A,« f(G
there would exist a set A « G, such that A,« U(A), t.e. A,yCA, and in
view of Aye G,—G,, AeG we have A, #4A. The sets A, and A, both
belonging to family F (since G,C F and G@, CF), would thus be two dif-
ferent subsets of the set M, the first of them being a subset of the second,
which er sa the property of family F. Therefore A, ¢f(@,) and
thus A, «/f(G,)—f(G,). We have proved that if G,CF, G,C F- and
G,—G, 4 0, hr f(G,)—f(G,) #0. Hence it follows that to different
subfamilies G of family # correspond different hereditary families f(G@)
made up of subsets of the set M. Since there are 2?" different subfamilies
§ 4. Some theorems on families of subsets 453
where X «Ff. We shall have of course p(A)c M for Ace the family @ is formed of
subsets of the set M.
If A ¢«F,BeF, A ~ B, then, in view of the property of familyF,we have A -BS 0;
thus there exists an element a¢« A—B such that ae A and a€ B, therefore a ¢ b for
454 XVII. Applications of Zermelo’s theorem
i. €. f(a): p(B) = 0, and since, in view of ae A, we have f(a) cp(A) and p(A) ¥ (Bb).
Thus p(A) ~ g(B) for A «Ff, Be F, A ~ B, and since family F is of power 2™, the family
of all sets p(A) for A « F, i. e. family ® is of power 2™. Suppose now that X «®, ¥ «9,
X = JY. From the definition of family @ it follows that there exist sets A ef and Bel’
such that X = ~(A), Y = y(B). In view of X 4 Y we have A + B, therefore 4—B A 0
and there exists an element ae«A—B, whence, as above, we obtain f(a) Ccp(A) = X
and f(a): g(B) = 0, i.e. f(a)¥ = 0, whence f(a) c X—Y, and since f(a) =m (because
ae M), we have X¥—Y >m. On the other hand, in view of X—Y c M we have X-—Y<
<m. Therefore XY = miforex «®, Ye@®, X~AY. Thus family © satisfies the re-
quired conditions.
2. Let us divide all subsets of an infinite set J of power m into classes, assigning
to the same class two subsets 4 and B of the set M if and only if dA—B<m and B—A <
<m. How many such classes are there?
Answer. There are 2™ of them. On one hand, our classes are disjoint; they
are subsets of the set of all subsets of the set M of power m, the number of which is 2™;
if a set of power 2™ is decomposed into disjoint subsets, then, by Theorem 6 of VI.5
the number of those subsets is <2™. Thus the number of our classes is <2™. On the other
hand, let ® denote a family of subsets of the set M, satisfying the conditions of Exer-
cise 1. Clearly, different subsets of the set M belonging to family ® belong to different
classes. Therefore the number of our classes is >® = 2™. Thus the number of our clas-
ses. is 2") +g. end,
Remark. Without the aid of the axiom of choice we are unable not only to solve
Ixercise 2 for infinite sets of arbitrary power but also to prove that if we define, for
sets of real numbers, a relation Aeb by the condition that for two sets of real num-
bers A and B we have AgB if and only if each of the sets 4—B and B—A is of less
power than the continuum, then the relation @ will be transitive. But we can prove
without resorting to the axiom of choice that the set of all real numbers can be de-
composed into disjoint classes in such a manner that two sets, 4 and B, belong to the
same class if and only if the sets 4 —B and A—A are finite, and we can prove that there
are 2? such classes. We are also able without the aid of the axiom of choice to solve
Exercise 2 for m = &,.
3. Prove that the total number of different famihes of disjoint sets which can
be formed of the subsets of a given infinite set of power m is 2™.
Proof. Every family of disjoint sets which can be formed from the subsets of
a given set of power m is obviously of power < mand it is easy to prove with the aid of the
axiom of choice that the total number of families of power <m formed of the subsets
of a given infinite set of power m is (2")™ — Qm’ _ 9m. Thus the total number of different
families of disjoint sets which can be formed of the subsets of a given infinite set of
power m is <2™. That their number is >2™ follows from the fact that each subset of
a set of power m (there are 2™ of them) determines a family of one-element disjoint
sets. different subsets determining different families.
Cf. also Exercise 6 of IV.11.
4. Let X denote!an arbitrary infinite set of real numbers and D the straight line
y =. Prove with the aid of Zermelo’s theorem that the set 7 = (X x X)—D contains
§ 4. Some theorems on families of subsets 455
a subset symmetrical with respect to the line D and such that for every number a ¢« X
the line x = a intersects the set Z exactly at one point.
5. Prove with the aid of Zermelo’s theorem the proposition of.P. Erdos that if
all straight lines lying in a plane P are divided into two disjoint classes L, and L,, then
there exists a decomposition of the plane into two sets P = S,+8, such that, for 1 =
= 1, 2, each line of class Z, has fewer than 2*° points in common with the set S,.
Proof. Let my denote the least ordinal number of power 2*° and {de}ec, a trans-
finite sequence of type y formed of all straight lines of the plane P. For i= 1, 2 and
a<~y let us denote by 7, the sum of all lines d. where € < a belonging to the class L,.
For a < let HE, = d,(T?—T!) if d, «Ll, and #, = d(T. —T:) if d,«L,. For i=1, 2,
let H denote the sum of all sets H, where a<g and d,¢«L,. Let 8,= E+ (P—T’).
We easily prove that the sets S, and S, satisfy the required bonaiacn
In this way the inequality 6, < 6B, (where B, <a, < my, Be < & < w,)
always implies the inequality qj 4 q, which proves that the sets Rg,
and Rs, are disjoint for 6, < py < ay,.
Since there are m sets Rs where 6 < w,, in order to prove Theorem 2
it remains to show that each of them has m elements in common with
each of the sets A of family F.
Therefore let A denote a given set of family /. In virtue of the de-
finition of the sequence tAghe<o, there exist m different ordinal numbers
a<o, such that A= A,; thus also for every given ordinal number
6 <o, there exist m different ordinal numbers a such that B <a<ao,
and A = A,. For each of those numbers a the element qj is a term of the
sequence {gzte<, and therefore, in virtue of the definition of that se-
quence, it belongs to the set A,; on the other hand qj belongs to the set Rg
(in virtue of the definition of that set). Thus we have qj « A,Rs = ARg
for m different ordinal numbers a satisfying the condition 6 <a < a,.
But from the definitions of the sets Q, and the terms qg it follows that
fOr hay G),68-< 0; <0, 0b as0, We. Nave gs «Oo uaheds te 4s, Gs
Qa,+1C Q., therefore q;' ~ q;. The set AR, contains m different elements
and of course no more (since 4R;C M and M =m).
Theorem 2 is thus proved. We easily deduce from it the following
COROLLARY. If {Xs}e<w, is a transfinite sequence of type ow, of sets of
power S$,, then there exists a transfinite sequence {Yehe<w, of disjoint sets
of power s, such that Y;C Xe for E<a,.
Without the use of transfinite numbers we could express this corol-
lary in the following way:
If we have m > sy sets of power m, then with each of them we can as-
sociate a subset of it of power m such that all those subsets are disjoint.
We shall also mention, without proof, the following two theorems,
which can be proved with the aid of Zermelo’s theorem:
If M is an infinite set of power m and F a family of power m of fune-
tions defined for the elements of the set M (whose values do not necessarily
belong to M), then there exists a family © of power 2™ of different subsets
of the set M such that f(H) AH for Hed, He®, HAH, fe F').
If M is an infinite set of power m and F a family of power m of func-
tions defined for the elements of the set M and assuming each value from
the set M at most once (assuming besides, for the elements of the set M,
values not belonging to M), then there exists a family ® formed of 2™ dif-
ferent subsets of the set M such that f(H)—H 40 for He®@, He®,
EH, f«F (Sierpinski, Fund. Math. 34(1947), p. 33).
1) This theorem was given without proof by A. Lindenbaum [1], p. 185; I gave
the proof in Fund. Math. 34(1947), p. 29-33.
§ 5. Set of all order types of a given power 457
ry
5. The power of the set of all order types of a given power. We shall prove
THEOREM 1. Jf m ts a non-finite cardinal number, then the set of all
order types of power m is of power 2™.
Proof. Let m denote a non-finite cardinal number and / the set
of all order types of power m. As has been proved in XI.1, for a set M of
power m there exist 2™ different relations between its elements, then not
more than 2™ different order arrangements of the set M. Thus if we
divide the order arrangements of the set M into classes, assigning two
order arrangements of the set M to the same class if and only if they
determine the same order type, then, by Theorem 6 of VI.5, the set of
those classes will be of power <2™’, hence by formula (2.2) of Chapter XVI,
of power <2". Thus T < 2™.
On the other hand, if a cardinal number is not finite, then there
exists an ordinal number uw such that m=x,. Let s= {As}e<w, denote
an arbitrary transfinite sequence of type w, with the terms 0 or 1. Let
2. Give an example of an ordered set whose all non-empty segments are equa-
but all remainders different.
Answer. A set of type w*. More generally, a set of type o* where g is an ordinal
number >1 and a prime component.
3. Prove that if m is a non-finite cardinal number, then the set Z of all order
types gm of power m such that an ordered set of type y will not change its power if we
delete from it an arbitrary finite set of elements is of power 2™.
Proof (Chajoth [1], p. 132). If m is an infinite type of power m, then the type wp
is of power $)Mm, 7. e., by formula(2.1) of Chapter XVI, of power m, and moreover, as
can easily be seen, a set of type wy will not change its type if we remove from it an ar-
bitrary finite set of elements. Further, it is easy to prove that if y, Ag. then wg, ~
A wy, (Chajoth [1], p. 132). Hence it follows that the set Z is of no less power than the
set T of all order types of power m, whence Z > 7, and since of course Zc T, whence
HM, SKE T; therefore Z — 7, and thus in virtues of Theorem 1, Z = 2", qu Cad:
Remark. The set Ss, satisfying the conditions of our lemma, has
a first and a last element but it is not continuous because it has jumps.
H.g. the sequences a = {dag}:<9 where a,=0 and a=1 for 0<&<%
and the sequence b= {be}scg where 6, = 1 and bs: = 0 for 0< €<@% can
easily be shown to be successive elements of the set Sy (7. e. there is no
intermediate element between them).
In the case where ? is a number of the 2-nd kind we can obtain
a continuous set from the set Ss by removing from it all those sequences
for which there exists such an ordinal number yw that all their terms with
indices > are equal to 0.
Proof of Theorem 2. Let M denote an ordered set of power x,
and let (Ug}e<o, Genote a transfinite sequence of type w,, formed of all
different elements of the set M. We shall define by transfinite induc-
tion a double transfinite sequence {a5}, where & < o,, 1 < , formed of
numbers 0 and 1, as follows.
Let aj = a,= 0. Let a denote a given ordinal number such that
0 <a<o, and suppose that we have already defined all numbers Can
and. Bsn for é<aand y <a. If there is no element wu: of the set M such
that €< a and uz: S u,, then let az, = a3,+1 for 7 <a. If such elements uz
exist in the set M, then let A,, denote the set of all sequences ony ees
where € is an ordinal number such that é<a and u:<u,. In virtue
of our lemma in the set S,, exists the first sequence that does not precede
any sequence of the set A»a,; let {a,},<2q be that sequence. (If in the set Seq
there were no sequences succeeding each sequence of the set Aa, we
should have of course a, = 1 for 7 < 2a). Further, for <a let On=
= Oi1= 0 if ug Ua, ANd Ag= G5a1i1 if ue—<us. Finally let ag,=0
and 43,11= 1. Numbers as, and Bint are thus defined for <a and
H< a.
Thus we have defined numbers A, for <q, and 7 <a, by trans-
finite induction.
Let a® = {On ln<e, for §<, and T'= {a*}e<, and let us order the
set 7 according to the principle of first differences. We shall prove that
the sets M and 7 will be ordered similarly. For this purpose it is of course
sufficient to prove that
(6.2) Ti aes 8 oy, eer endwe << ig, then: | 10% ar:
(6.3) ff oxi =o, ieand “w—=2w,, then “a? ae?
Suppose that a< 6 < ,, U.< Ug. According to the definition of
the double sequence {a;}, the sequence {an in<ee in the set Sg, does not
precede the sequence {a;},<2,: thus we have either at*<a’ or
(6.4) a,—a for <28.
462a XVII. Applications of Zermelo’s theorem
From the definition of the double sequence {az} it follows that either
we have A, == Ose = 0 for 7 < 6, and in that case we of course have
(in the set So,)
Proof. Let 4 = a+1 and let us denote by H, the set of all sequences
{ds}e<oy belonging to the set S,,, for which there exists an ordinal number
(dependent on the set in question) 2 < w, such that
jes NAS
pAb)
cep
aie aN ne
AS eames, ai,
i<@
which proves that 2 < wg11 = o,).
\
464 XVII. Applications of Zermelo’s theorem
Let
dge= On LOY Sen} aya
Since the sets Q, and Q, are of power <x,, we can set Q, = {a5}r<9,,
where 9, <a, Qo = {D*}c<q, Where g, <%,. Since Q,C H, and 9,C H,,
we shall obtain, as above, an ordinal number 4 < , such that
(6.21) HO eT
o<o,, |
But in. view of 0 < w, = @ai1, we-have 3 <x,, and j= Sati < 2°;
thus we have 75 = 2 < 2% and formula (6.21) gives
Sa =’ N 8
UE EO a icy ali WR
Cardinal and ordinal numbers 30
466 XVII. Applications of Zermelo’s theorem
It will be observed that we can prove that for every ordinal num-
ber a the least power of the y,.1-sets is 2°*, whence it follows that for
a given ordinal number a an 4,41-Set of power s,+41 exists if and only if
s,
Zi ee Na+1-
For a= 0, we obtain from Theorem 4 and from the properties of
Hy- Sets
EXERCISE. Prove with the aid of Zermelo’s theorem that every real function
of a real variable is the sum of two (1-1)-mapping functions*).
Proof. Let m denote the least ordinal number of the power of the continuum.
From Zermelo’s theorem it follows that there exists a transfinite sequence of type ,
Wehccg: formed of all different real numbers. Let f(x) denote a given real function
of a real variable. Let f,(”,) = f(x), f2(z,) = 0. Let a denote an ordinal number such
that 1 << a< gq and suppose that we have already defined all values fil@s) and f(%z)
for § < a. We shall define number /,(7,) as the first term of the sequence {x,} E<p that is
different from each of the numbers
Of numbers (*) there are not more than a+ a, 7. e. fewer than 2“ since if a+ a>
=> 2*0 — 2%0 + 20 we should have @ > 2*° = », which is impossible because m denotes
the least ordinal number of power 2°. Thus there exist terms of the sequence {w,}.— é
that are different from each of the numbers (*).
Finally let 7.(a,) = f(~,)—fi(w,). In this manner the functions f,(#) and /,(x) are
defined for real x» by transfinite induction and we always have f (az) = flv) + fo(ae)
for § <p; it follows from the definition of numbers f,(~,) that the function f,(x) is
(1-1)-mapping and that f,(~,) A f(x) — f(x) + fil@e) for §<a<p@, and thus f(x,)—
— fila) A f(%s)—fi(wg), 0. e. falXq) F fale) for §<a<q. whence it follows that the
function f,(#) is (1-1) mapping.
Thus we have proved that for real # the function f(x) is the sum of two (1-1) mapp-
ing functions f.(”)+ f(z), q.e. d.
30*
APPENDIX
Aigner, A.
|1] Der multiplikative Aufbau beliebiger unendlicher Ordnungszahlen, Monats-
hefte f. Math. 55 (1951), p. 297-300.
Alexiewicz, A. and Orlicz, W.
[1] Remarque sur Véquation fonctionnelle f(x+y) = f(w)+ fly), Fund. Math. 33
(1945), p. 314-315.
Aronszajn, N.
[1] Characterization of types of order satisfying aj+ a, = a,+ a), Fund. Math. 39
(1952), p. 65-96.
Bagemihl, F.
{l] ‘A theorem on intersections of prescribed cardinality, Annals of Math. 55 (1952),
p. 34.
Banach, Ss.
[1] Sur Péquation fonctionnelle f (a+ y) = f(a)+ f(y), Fund. Math. 1 (1920), p. 123.
[2] Un théoréme sur les transformations biunivoques, ibidem 6 (1924). p. 236-239.
Bernays, P.
[1] A system of axiomatic set theory, Journal of Symbolic Logie 8 (1934), p. 89-106.
[2] Sur le Platonisme dans les Mathématiques, Enseignement Math. 34 (1935),
p. 52-69.
Bernstein, F.
[1] Untersuchungen aus der Mengenlehre, Math. Annalen 61 (1905), p. 117-155.
[2] Inaugural Dissertation, Halle 1909.
Bieberbach, L.
[1] Bemerkung zum dreizehnten Hilbertschen Problem, Journal f. reine u. angew.
Math. 165 (1931), p. 89-92.
Birkhoff, G.
[1] Lattice theory, New York 1948.
Blumberg, H.
[1] Methods in point sets and theory of real numbers, Bull. Amer. Math. Soe. 36
(1930), p. 809.
Borel, E.
[1] Legons sur la théorie des fonctions, Paris 1914.
[2] Sur les ensembles effectivement énumérables et sur les définitions effectives,
Rend. Acad. dei Lincei 5. 28, sem. II (1919), p. 163-165.
470 Bibliography
Bourbaki, M.
[1] Bléments de Mathématique, Partie I-Livre I (Théorie des Hnsembles, fascicule
de résultats), Actualités Scientifiques et Ind. 843 (1939), p. 37.
Cantor, G.
[1] Gesammelte Abhandlungen, Berlin 1932.
[2] Uber eine Higenschaft des Inbegriffes aller reellen algebraischen Zahlen, Journal tf.
reine u. angew. Math. 77 (1874), p. 258-262.
Carruth, Ph. W.
[1] Roots and factors of ordinals, Proc. Amer. Math. Soc. 1 (1950), p. 470-480.
Cavaillés, I.
[1] Sur la dewxiéme definition des ensembles finis donnée par Dedekind, Fund.
Math. 19 (1932), p. 143-184.
Chajoth, Z.
[1] Beitrdge zur Theorie der geordneten Mengen, Bull. Se. Math. 2 (1897), p. 257.
[2] Fund. Math. 16 (1930), p. 132-136.
Church, A.
[1] Alternatives to Zermelo’s assumption, Trans. Amer. Math. Soc. 29 (1927),
p. 178-208.
[2] Introduction to mathematical logic, Ann. of. Math. Studies 13, Princeton 1944.
Cippola, M.
[1] Sur postulate di Zermelo e la teoria dei limiti delle funzioni, Atti Acad. Gioenia
Se. Nat. in Catania 6. V (1913). i
[2] Sut fondamenti logict delle’ Matematica, Annali di Matematica IV. 1 (1924),
Deer 9): ge :
Cooper, R.
[1] The converses of the Cauchy -Hoélder inequality and the solutions of the inequality
g(a+y) <g(x)+ gy), Proc. London Math. Soc. 2.26 (1927), p. 415-432.
Couturat, L.
1) Les principes des Mathématiques, Paris 1905.
Cuesta, N.
[1] Notas sobre unos trabajos de Sierpinski, Revista Mat. Hisp.-Amer. 4 (1947),
p. 130-131.
[2] Ibidem 9 (1949), p. 132.
Dayis, A. C.
[1] Cancellation theorems for products of order types, Bull. Amer. Math. Soc: 58
(1952), p. 63. f
Davis, A. C. and Sierpinski, W.
[1] Sur les types Wordre distincts dont les carrés sont éqaux, C. R. Acad. Se.
Parise Zoom (L952) p.ss00:
Bibliography 471
Dedekind, R.
[1] Stetigkett und irrationale Zahlen, Braunschweig 1872.
[2] Was sind und was sollen die Zahlen, Braunschweig 1888.
Denjoy, A.
[1] L’énumération transfinie (Livre I, La notion du rang), Paris 1946.
[2] Lordination des ensembles, C. R. Acad. Sc. Paris 236 (1953), p. 1393-1396.
Fraenkel, A.
[1] Hinleitung in die Mengenlehre, Berlin 1919.
[2] Uber den Begriff ,,definit “und die Unabhdngigkeit des Auswahlaxioms, Sitzungs-
ber. Preuss. Akad. Wiss. Phis.-Math. (1922), p. 253-257.
[3] Zehn Vorlesungen uber die Grundlagen der Mengenlehre, Leipzig und Berlin 1927.
[4] Sur la notion dexistence dans les mathématiques, Enseignement Math, 34 (1935),
De LS=a2"
[5] Uber eine abgeschwichte Form des Auswahlaxioms, J. Symbolic Logie 2 (1937),
p-. 1-25.
[6] Zu den Grundlagen der Cantor-Zermeloschen Mengenlehre, Math. Ann. 86
(1922), p. 230-237. .
[7] Laxiome du choix, Revue Phil. de Louvain 50 (1950), p. 431.
Fraissé, R.
[1] Sur la comparaison des types de relations, C. R. Acad. Sc. Paris 226 (1948),
Ds GSE
Fréchet, M.
[1] Pri la funkeia ekvacio, Enseignement Mathématique 15 (1923), p. 390-393.
472 Bibliography
Gillman, L.
[1] Remarque sur les ensembles 7,,.C. R. Acad. Sc. Paris 241 (1955), p. 12-13.
[2] Some remarks on 1,-sets, Fund. Math. 43 (1956), p. 77-82.
Godel, K.
[1] What is Cantor continuum problem, Amer. Math. Monthly 54 (1947), p. 515-525.
[2] Proc. Nat. Acad. Se. 24 (1938), p. 556-557.
[3] The consistency of the axiom of choice and the generalized continuum hypothesis,
Ann. Math. Stud. 3 (1940).
Hahn, H.
[1] Uber die Abbildung einer Strecke auf ein Quadrat, Ann. di Matematica
3.21 (1913), p. 33-35. ;
Hamel, G.
[1] Hine Basis aller Zahlen und die unstetige Losungen der Funktionalgleichung
f(@+y) = f(x)+ fly), Math. Ann. 60 (1905), p. 459-462.
Haperin, [.
[1] Non-measurable sets and the equation f (a+ y) = f(#)+ f(y), Proc. Amer. Math.
Soc. 2 (1951), p. 221-224.
Hardy, G. H.
[1] A theorem concerning the infinite cardinal numbers,’ Journal of pure and appl.
Math. 35 (1903), p. 87-94.
‘ Hartogs, F.
[1] Uber das Problem der Wohlordnung, Math. Ann. 76 (1914), p. 438-443.
Hausdorff, F.
[1] Der Potenzbegriff in der Mengenlehre, Jahresber. der deutschen Math. Ver. 13
(1904), p. 569-571.
[2] Grundziige der Mengenlehre, Leipzig 1914.
[3] Uber zwei Sdtze von G. Fichtenholtz wnd L. Kantoroviteh, Studia Math. 6 (1936),
p. 18-19.
Hessenberg, G.
[1] Grundbegriffe der Mengenlehre, Gottingen 1906.
Hilbert, D.
[1] Uber die stetige Abbildung einer Linie auf ein Flichenstiick, Math. Ann. 38
(1891), p. 459-460.
[2] Die logischen Grundlagen der Mathematik, Math. Ann. 88 (1923), p. 151-165.
[3] Grundlagen der Geometrie, re 1923.
Hoborski,A
[1] Une remarque sur la limite des nombres ordinaux, Fund. Math. 2 (1921), p. 193-198.
Hobson, E. W.
[1] Theory of functions of real variable, Cambridge 1907.
Inagaki, T.
[1] Le probleme de Souslin dans les espaces SONS Journal Fac. Se. Hokkaido
Univ. 1. 8 (1939-1940), p. 25-46.
[2] Ibidem 1. 9 (1940), p. 145-201.
Bibliography 473
Jacobsthal, E.
[1] Vertauschbarkeit transfiniter Ordnungszahlen, Math. Ann. 64 (1907), p. 475-488.
[2] Uber den Aufbau der transfiniter Arithmetik, ibidem 66 (1909), p. 145-194.
Kac, M.
[1] Une remarque sur les équations fonetionnelles, Comm. Math. Helv. 9 (1936/37),
p. 170.
Kamke, E.
[1] Allgemeine Mengenlehre, Encyklop. der Math. Wiss., § 13.
Kestelman, H.
[1] Automorphisms of the field of complex numbers, Proc. London Math. Soe. 2, 53
(1951), p. 1-12.
Khintchine, A.
[1] Sur les suites de fonctions analytiques bornées dans leur ensemble, Fund.
Math. 4 (1924), p. 72-75.
Kiyoshi Iséki
[1] Journ. Osaka Inst. Se. Technol. 2 (1950), p. 109.
[2] Sur les ensembles finis, C. R. Acad. Sc. Paris 231 (1950), p. 1396-1397.
Knaster, B. et Kuratowski, C.
[1] Sur les ensembles connexes, Fund. Math. 2 (1921), p. 206-259.
Kneser, H.
{1] Hine direkte Ableitung des Zornschen Lemmas aus dem Auswahlaxiom, Math.
Zeistschrift 53 (1950), p. 110-122.
K6nig, D.
[1] Zur
Theorie der Médchtigkeiten, Rend. Palermo 26 (1908), p. 339-342.
[2] Uber Graphen wnd ihre Anwendung auf Determinantentheorie und Mengenlehre,
Math. Ann. 77 (1916), p. 453-465.
[3] Sur les correspondances multivoques des ensembles, Fund. Math. 8 (1926),
p. 114-134.
Kuratowski, K.
[1] Sur la notion Wensemble fini, Fund. Math. 1 (1920), p. 129-131.
[2] Une méthode @ elimination des nombres transfinis des raisonnements mathéma-
tiques, ibidem 3 (1922), p. 77-108.
[3] Sur Vétat actuel de Vaxiomatique de la théorie des ensembles, Ann. Soe. Pol.
Math. 3 (1924), p. 146-147.
[4] Topologie I, Warszawa-Wroclaw 1948.
Kurepa, G.
[1] Ensembles ordonnés et ramifiés, Publ. Math. Univ. Belgrade 4 (1935), p. 1-138.
[2] Le probléme de Souslin et les espaces abstraits, C. R. Acad. Se. Paris 203 (1936),
p. 1049-1051.
[3] L’hypothése de ramification, ibidem 202 (1936), p. 185-187.
ATA Bibliography
Lebesgue, H.
[1] Sur les fonctions représentables analytiquement, Journal de Math. 1 (1905),
p. 213.
[2] Atti Acad. Se. di Torino 42 (1907), p. 532.
[3] Sur les correspondances entre les points de deux espaces, Fund. Math. 2 (1921),
p. 256-286.
[4] Bull. Sc. Math. 2. 46 (1922).
[5] Sur la mesure des grandeurs, Enseign. Math. 32 (1933), p. 23-51.
[6] Les controverses de la théorie des ensembles et la question des fondaments, Les
entretiens de Zurich sur les fondaments et les méthodes des sciences mathématiques, 8-9,
Décembre 1938 — Exposés et discussions publiés par le président des débats F. Gonseth,
Zurich ,1941, p. 109-122.
[7] Legons sur les constructions géométriques, Paris 1950.
Levi, B.
[1] Intorno alla teoria degli aggregati, Rend. R. Sct. Lomb. di Se. e Lett. Il. 35
(1902), p. 863-868.
Lindemann, F.
[1] Uber die Zahl x, Ann. Math. 20 (1882), p. 213-225.
Lindenbaum, A.
[1] Sur le nombre des invariants des familles de transformations arbitraires, Ann.
Soc. Pol. Math. 15.(1936), p. 185.
[2] Sur quelques propriétés des fonctions de variable réelle, ibidem 6 (1927),
p. 129-130.
[3] Sur les constructions non-effectives dans lV Arithmetique élémentaire, ibidem 10
(1931), p. 118-119.
[4] Sur les ensembles dans lesquels towtes les équations dune famille donnée on
un nombre de solutions fixé @avance, Fund. Math. 20 (1933), p. 1-29.
Lindenbaum, A. und Mostowski, A.
[1] Uber die Unabhéngigkeit des Auswahlarioms wnd einiger seiner Folgerungen,
C. R. Soc. Sc. et Lett. de Varsovie, Cl. III. 31 (1938), p. 27-32.
Lindenbaum, A. et Tarski, A.
[1] Communication sur les recherches de la théorie des ensembles, C. R. Soc. Se.
et Lett. de Varsovie, Cl. III. 19 (1926), p. 299-330.
Liouville, J.
[1] Sur des classes trés entendues de quantités dont la valeur west ni algébrique,
ni méme reductive a des irrationnelles algébriques, Jour. Math. 16 (1851).
Lusin, N.
[1] Sur les ensembles analytiques, Fund. Math. 10 (1926), p. 1-95.
Bibliography AT
[2] Sur les ensembles analytiques, Recueil Math. Moscou 33 (1926), p. 237-289.
[3] Legons sur les ensembles analytiques, Paris 1930.
Maccaferri, E.
[1] Per. di Matem. IV (1939), p. 92-101.
Malchair, H.
[1] Bull. Soc. Se. Liége 1.
[2] Ibidem 3, p. 133-134.
Maraham, D.
[1] Set functions and Souslin’s Hypothesis, Bull. Amer. Math. Soe. 54 (1948),
p. 587-590.
Marczewski, E.
[1] Sur deux propriétés des classes Wensembles, Fund. Math. 33 (1945), p. 303-307.
[2] Ensembles indépendants et leurs applications a la théorie de la mesure, ibidem 35
(1948), p. 13-28.
Mazurkiewicz, S.
[1] C. R. Soc. Sc. et Lettres de Varsovie 7 (1914), p. 322-383.
[2] Sur la décomposition du plan en courbes, Fund. Math. 21 (1933), p. 43-45.
Menger, K.
[1] Der Intuitionismus, Blatter f. deutsche Phil. 4 (1930), p. 314.
Milgram, A. P .
[1] Rep. Math. Coll. II. 1 (1939), p. 18-30.
Mostowski, A.
[1] Bericht aus einem Satz von Gédel viber die Widerspruchsfreiheit des Auswahl-
axioms, Ann. Soc. Pol. Math. 17 (1938), p. 115.
[2] On the concept of a finite set, Sprawozd. z posiedzen Warsz. Tow. Mat. 31
(1938), p. 13-20.
[3] Axiom of choice for finite sets, Fund. Math. 33 (1945), p. 137-168.
[4] Logika Matematycena, Warszawa-Wroclaw 1948.
[5] On the principle of dependent choices, Fund. Math. 35 (1948), p. 127-130.
[6] Uber die Unabhdngigkeit des Wohlordnungssatzes von Ordnungsprinzip, ibidem
32 (1939), p. 201-232.
Miller, E. W.
[1] A note of Souslin’s problem, Amer. J. Math. 65 (1943), p. 673-678.
y. Neumann, J.
[1] Zur Hinfihrung der transfiniten Zahlen, Acta Litt. Acad. Sc. Szeged X. 1 (1923),
p. 199-208.
Neumer, W.
[1] Uber den Aufbaw der Ordnwngszahlen, Math. Zeitschr. 53 (1951), p. 59-69.
476 Bibliography
Novak, J.
[1] A paradoxical theorem, Fund. Math. 37 (1950), p. 77-83.
_ Novikoff, P.
[1] Bull. Acad. Se. URSS 1939, p. 35-39.
Ostrowski, A.
[1] Uber die Funktionalgleichung der Haponentialfunktion, Jahresber. d. deutschen
Math. Ver. 38 (1929).
Peano, G.
[1] Sur une courbe qui remplit toute une aire plane, Math. Ann. 36 (1890), p. 157-160.
[2] Additione, Revista di Mat. III. 8 (1906), p. 143-155.
Poincaré, H.
[1] Science et méthode, Paris 1912.
Ramsey, F. P.
[1] On a problem in formal logic, Proc. London Math. Soc. 2. 30 (1930), p. 264-286.
Rothberger, F.
[1] On some problems of Hausdorff and of Sierpinski, Fund. Math. 35 (1948),
p-. 29-46. ;
Russell, B.
[1] On some difficulties in the theory of transfinite numbers and order types, Proc.
London Math. Soc. 4 (1906), p. 29-53.
[2] Sur les axiomes de Vinfini et du transfini, C. R. Soc. Math. de France, Séance
du 22 Mars 1911, p. 22-35.
Schonberg, I. J.
_ [1] On the Peano curve of Lebesgue, Bull. Amer. Math. Soc. 44 (1938), p. 519.
Schonflies, A.
[1] Entwickelung der Mengenlehre, Leipzig und Berlin 1913.
Segre, B.
fll Atti Acad. Naz. Lincei (1947), p. 414.
Sehrmann, S.
[1] Some new properties of transfinite ordinals, Bull. Amer. Math. Soc. 47 (1941),
p. 111-116.
Sieczka, F.
[1] Sur Punicité de la decomposition de nombres ordinaux en facteurs irréductibles,
Fund. Math. 5 (1924), p. 172-176.
Sierpinski, W.
[1] Sur Vensemble des points angulaires dune courbe y = f(x), Bull. Acad. Se. de
Cracovie 1912, p. 850-855.
[2] O kreywych wypetniajacych kwadrat, Prace Mat.-Fiz. 23 (1912), p. 193-219.
[3] C. R. Acad. Se. de Paris 163 (1916), p. 688.
Bibliography AT 7
[71] Sur Véquation 2 = 7?+1 pour les nombres ordinaus transfinis, Fund. Math.
43 (1956), p. 1-2.
Sierpinski, W. et Davis, A. C.
[1] Sur les types dordre distinets dont les carrés sont égaux, C. R. Acad. Sc. Paris 235
(1952), p. 850.
Sierpinski, W. et Tarski, A.
[1] Sur une propriété caractéristique des nombres inaccessibles, Fund. Math. 15
(1930), p. 292.
Skolem, S.
[1] Logisch-kombinatorische Untersuchung, Skrifer utgit av Videnskap. Kristiania
I KI. (1920), p. 4.
Stackel, P.
[1] Zu H. Webers elementarer Mengenlehre, Jahresber. deutsch. Math. Ver. 16
(1907), p. 425-428.
Steckel, S.
[1] Remarques sur classe Wensembles ordonnés, Fund. Math. 11 (1928), p. 285-287.
Souslin, M.
[1] Fund. Math. 1 (1920), p. 223, Probléme 3.
Szele, T.
[1] On Zorn’s Lemma, Publ. Math. 1 (1950), p. 254-256.
Szmielew, W.
[1] On choices from finite sets, Fund. Math. 34 (1946), ps 75-80.
Szpilrajn, E.
[1] Sur
Vextension de Vordre partiel, Fund. Math. 16 (1930), p. 386-390.
[2] The characteristic function of a sequence of sets and some of tts applications,
ibidem 31 (1938), p. 207-223.
Tarski, A.
[1] Sur quelques théoremes qui équivalent a Vaxiome de choiz, Fund. Math. 5
p. 147-154.
[2] Sur les ensembles finis, ibidem 6 (1924), p. 45-95.
[3] Quelques théoremes sur les alephs, ibidem 7 (1925), p. 1-13.
[4] Remarques concernant Varithmeéetique des nombres cardinaux, Ann. Soc. Pol.
Math. 5 (1926), p. 106-107.
[5] C. R. Soc. Se. et Lett. Varsovie III. 19 (1926), p. 308-309.
[6] Sur les ensembles définissables des nombres réels I, Fund. Math. 17 (1931),
p- 210-240.
[7] Der Wahrhettsbegriff in den Sprachen der deductiven Disziplinen, Anzeiger Akad.
Wien 69 (1932), p. 23-25.
8] Pojecie prawdy w jezykach nauk dedukeyjnych, Prace Tow. Nauk. Warsz.
Ii. 34 (1933).
[9] Der Wahrheitsbegriff in formalisierten Sprachen, Stud. Phil. 1 (1935), p. 261-406,
‘ 110] C. R. Soc. Se. et Lett. Varsovie 30 (1937).
[11] Axiomatic and algebraic aspects of two theorems on sums of cardinals, Fund.
Math. 35 (1948), p. 79-104.
480 Bibliography as.
[3] Uber die Grundlagen der Arithmetik, Atti de! TV Congr. Intern. dei Math. 2. J,
Roma 1909.
[4] Sur les ensembles finis et le principe de Vinduction complete, Acta Math. 32
(1909), p. 185-193.
[5] Uber Grenzzahlen und Mengenbereiche, Fund. Math. 16 (1930), p. 29-47.
[6] Untersuchungen tiber die Grundlagen der Mengenlehre I, Math. Ann. 65 (1907),
p. 261-281.
Zorn, M.
{1] A remark on method in transfinite algebra, Bull. Amer. Math. Soc. 41 (1935),
p- 667-670.
Sets. .
. Elements of a ae
. Symbols « and ¢ . Or
Ol
ON
. Equality of sets
. Sets of sets
. Subset of a set.
. Sum of sets .
. Difference of sets. .
. Product of sets.
. Disjoint sums
. Complement of a set .
. Ordered pairs :
. Correspondence. Tinneuion :
. One-to-one correspondence. .
. Cartesian product of sets
. Exponentiation of sets.
Page
1. Sets of the power of the continuum and sets effectively of the power of the
CONMULMUTLUTE sane ne ; PL NE Rae CHe NAL Ot Pei ec 53
2. Non-denumerability is ae ee of Sal Stan ee eths eines 53
3. Removing a denumerable set from a set of the power of he Rone Ah geen
4. Set of real numbers of an arbitrary interval ........4:~... 5. 56
5. Sum of two sets of the power of the continuum .. . kos
6. Cartesian product of a denumerable set and a set of the power of tie convene 59
7. Set of all infinite sequences of natural numbers... . te 5 ee oO
°8. Cartesian product of two sets of the power of the count Bae Ar 61
9. Impossibility of a continuous (1-1) mapping of a plane on a Siralewe a 66
LOSContinuoustcurye: fillies up, aesquares.: s-y -nle) -nie olen ce) ten en SS
11. Set of all infinite sequences of real numbers ... . oats "a SE deen aeRO
12. Continuous curve filling up a denumerably iaenone) ene bE) de aw Re oe eT
13. Set of all continuous functions .. . 75
14. Decomposition of a set of natural Panbers tc a; coatnnimn of mirtioet disjoueh
SOLS u.hye: rove deelia RSe Mennee ease eeVga Lo) My, Twn eh roa ericfetes Sue ea 0 rr
. Cardinal numbers
. Sum of cardinal numbers
. Product of two cardinal numbers
Rm
=wh. Exponentiation of cardinal numbers ‘
. Power of the set of all subsets of a given Neate
. Ordered sets . as
. Partially ordered sets . pao
. Lattices . . . bau o kite bsp. M lade oa uth
. Similarity of oaks : :
oO
Rm
WD . First and last element of an brdered aot Guise i;umps. Denes nad continuity
of a set eo
Finite ordered sets
. Sets of type w .
. Sets of type 7
. Dense
oor ordered sets as ibsets of Peenupad baie
10. Séts of type A
186 Contents
Page
. Order types
ed. Sum of two praered Cane
. Product of two order types .
mw. Sum of an infinite series of order oes : Ww
ve
wwSOW
by
- Power of the set of all order types of the power of the pour : 240
. Sum of an arbitrary ordered set of order types . tonN w
. Infinite products of order types : 244
oN. Segments and remainders of order types
coro 247
10. Divisors of order types 250
ee Comparison of order types. . 254
S
>»
I
a
Owat
. Infinite series of ordinal numbers and their sums
. Product of ordinal numbers .
. Properties of the product of ordinal ber ; ©
bo
Chapter XVI. Zermelo’s theorem and other theorems equivalent to the axiom of chcice
sets . Pe gs F 448
4. Some AN sece,on Pratuies of acces. of sats re given power . 450
5. The power of the set of all order types of a given power 457
6. Applications of Zermelo’s theorem to the theory of ordered sets 458
Appendix 468
Bibliography 469
Index 482
iF . 1 iene
nie. bike f ni ed
; A a iy} | ‘
' ra) wih te
hive. th 3
1 tt erh
eg ney |
int Saw ae
JOURNAUX
FUNDAMENTA MATHEMATICAE I-XLV.1.
STUDIA MATHEMATICA I-XVI.
COLLOQUIUM MATHEMATICUM I-V.1.
ZASTOSOWANIA MATEMATYKI I-III.
ROZPRAWY MATEMATYCZNE I-XIV.
ANNALES POLONICI MATHEMATICI I-IV.1.
MONOGRAFIE MATEMATYCZNE